Sie sind auf Seite 1von 304

DG3SM586_ 5th06fm.

qxd 8/2/06 6:45 PM Page i

Discovering

Geometry An Investigative Approach

Solutions Manual
DG3SM586_ 6th07fm.qxd 1/3/07 4:34 PM Page ii

Teacher’s Materials Project Editor: Joan Lewis


Editor: Heather Dever
Project Manager: Erin Gray
Writers: Abby Tanenbaum, Stacey Miceli
Editorial Assistants: Lori Dixon, Karen Rodriguez’G
Accuracy Checker: Dudley Brooks
Production Editors: Jacqueline Gamble, Angela Chen
Copyeditor: Margaret Moore
Editorial Production Manager: Deborah Cogan
Production Director: Diana Jean Parks
Production Coordinator: Ann Rothenbuhler
Text Designers: Jenny Somerville, Garry Harman
Art Editor: Jason Luz
Composition, Technical Art: Interactive Composition Corporation
Art and Design Coordinator: Caroline Ayres
Cover Designer: Jill Kongabel
Printer, Prepress: Data Reproductions

Executive Editor: Casey FitzSimons


Publisher: Steven Rasmussen

©2003 by Key Curriculum Press. All rights reserved.


Cover Photo Credits: Background image: Doug Wilson/Westlight/Corbis.
Construction site image: Sonda Dawes/The Image Works. All other images:
Ken Karp Photography.

Limited Reproduction Permission


The publisher grants the teacher who purchases Discovering Geometry: An
Investigative Approach, Solutions Manual the right to reproduce material for use in
his or her own classroom. Unauthorized copying of Discovering Geometry: An
Investigative Approach, Solutions Manual constitutes copyright infringement and is
a violation of federal law.
®Key Curriculum Press is a registered trademark of Key Curriculum Press.
All registered trademarks and trademarks in this book are the property of
their respective holders.
Key Curriculum Press
1150 65th Street
Emeryville, CA 94608
510-595-7000
editorial@keypress.com
http://www.keypress.com

Printed in the United States of America


10 9 8 7 6 09 08 07 ISBN 978-1-55953-586-1
DG3SM586_ 5th06fm.qxd 8/2/06 6:45 PM Page iii

Introduction
Contents
Introduction . . . . . . . . . . . . . . . . . . . . . . . . . . . . . . . . . . . . . . . . . . . . . . . . . . . . . . . vii

Chapter 0
Lesson 0.1: Geometry in Nature and in Art . . . . . . . . . . . . . . . . . . . . . . . . 1
Lesson 0.2: Line Designs . . . . . . . . . . . . . . . . . . . . . . . . . . . . . . . . . . . . . . . . . 1
Lesson 0.3: Circle Designs . . . . . . . . . . . . . . . . . . . . . . . . . . . . . . . . . . . . . . . . 2
Lesson 0.4: Op Art . . . . . . . . . . . . . . . . . . . . . . . . . . . . . . . . . . . . . . . . . . . . . . . 4
Lesson 0.5: Knot Designs . . . . . . . . . . . . . . . . . . . . . . . . . . . . . . . . . . . . . . . . . 4
Lesson 0.6: Islamic Tile Designs . . . . . . . . . . . . . . . . . . . . . . . . . . . . . . . . . . . 5
Chapter 0 Review . . . . . . . . . . . . . . . . . . . . . . . . . . . . . . . . . . . . . . . . . . . . . . . . 6
Chapter 1
Lesson 1.1: Building Blocks of Geometry . . . . . . . . . . . . . . . . . . . . . . . . . . 7
Using Your Algebra Skills 1: Midpoint . . . . . . . . . . . . . . . . . . . . . . . . . . . . . 8

Lesson 1.2: Poolroom Math . . . . . . . . . . . . . . . . . . . . . . . . . . . . . . . . . . . . . . . 9

Lesson 1.3: What’s a Widget? . . . . . . . . . . . . . . . . . . . . . . . . . . . . . . . . . . . . 11


Lesson 1.4: Polygons . . . . . . . . . . . . . . . . . . . . . . . . . . . . . . . . . . . . . . . . . . . . 13
Lesson 1.5: Triangles and Special Quadrilaterals . . . . . . . . . . . . . . . . . . . 15
Lesson 1.6: Circles . . . . . . . . . . . . . . . . . . . . . . . . . . . . . . . . . . . . . . . . . . . . . . 17
Lesson 1.7: A Picture Is Worth a Thousand Words . . . . . . . . . . . . . . . . . 19
Lesson 1.8: Space Geometry . . . . . . . . . . . . . . . . . . . . . . . . . . . . . . . . . . . . . 21
Chapter 1 Review . . . . . . . . . . . . . . . . . . . . . . . . . . . . . . . . . . . . . . . . . . . . . . . 24
Chapter 2
Lesson 2.1: Inductive Reasoning . . . . . . . . . . . . . . . . . . . . . . . . . . . . . . . . . 25
Lesson 2.2: Deductive Reasoning . . . . . . . . . . . . . . . . . . . . . . . . . . . . . . . . 27
Lesson 2.3: Finding the nth Term . . . . . . . . . . . . . . . . . . . . . . . . . . . . . . . . 29
Lesson 2.4: Mathematical Modeling . . . . . . . . . . . . . . . . . . . . . . . . . . . . . . 30
Lesson 2.5: Angle Relationships . . . . . . . . . . . . . . . . . . . . . . . . . . . . . . . . . . 32
Lesson 2.6: Special Angles on Parallel Lines . . . . . . . . . . . . . . . . . . . . . . . 34
Using Your Algebra Skills 2: Slope . . . . . . . . . . . . . . . . . . . . . . . . . . . . . . . . 35
Chapter 2 Review . . . . . . . . . . . . . . . . . . . . . . . . . . . . . . . . . . . . . . . . . . . . . . . 36
Chapter 3
Lesson 3.1: Duplicating Segments and Angles . . . . . . . . . . . . . . . . . . . . 37
Lesson 3.2: Constructing Perpendicular Bisectors . . . . . . . . . . . . . . . . . . 39
Lesson 3.3: Constructing Perpendiculars to a Line . . . . . . . . . . . . . . . . . 41
Lesson 3.4: Constructing Angle Bisectors . . . . . . . . . . . . . . . . . . . . . . . . . 43
Lesson 3.5: Constructing Parallel Lines . . . . . . . . . . . . . . . . . . . . . . . . . . . 45
Using Your Algebra Skills 3: Slopes of Parallel
and Perpendicular Lines . . . . . . . . . . . . . . . . . . . . . . . . . . . . . . . . . . . . . . . 46

iii
DG3SM586_ 5th06fm.qxd 8/2/06 6:45 PM Page iv

Lesson 3.6: Construction Problems . . . . . . . . . . . . . . . . . . . . . . . . . . . . . . . 47


Exploration: Perspective Drawing . . . . . . . . . . . . . . . . . . . . . . . . . . . . . . . . 50
Lesson 3.7: Constructing Points of Concurrency . . . . . . . . . . . . . . . . . . . 50
Lesson 3.8: The Centroid . . . . . . . . . . . . . . . . . . . . . . . . . . . . . . . . . . . . . . . . 52
Exploration: The Euler Line . . . . . . . . . . . . . . . . . . . . . . . . . . . . . . . . . . . . . . 54
Chapter 3 Review . . . . . . . . . . . . . . . . . . . . . . . . . . . . . . . . . . . . . . . . . . . . . . . 54
Chapter 4
Lesson 4.1: Triangle Sum Conjecture . . . . . . . . . . . . . . . . . . . . . . . . . . . . . 56
Lesson 4.2: Properties of Special Triangles . . . . . . . . . . . . . . . . . . . . . . . . 58
Using Your Algebra Skills 4: Writing Linear Equations . . . . . . . . . . . . . . 61
Lesson 4.3: Triangle Inequalities . . . . . . . . . . . . . . . . . . . . . . . . . . . . . . . . . 63
Lesson 4.4: Are There Congruence Shortcuts? . . . . . . . . . . . . . . . . . . . . . 64
Lesson 4.5: Are There Other Congruence Shortcuts? . . . . . . . . . . . . . . . 66
Lesson 4.6: Corresponding Parts of Congruent Triangles . . . . . . . . . . . 68
Lesson 4.7: Flowchart Thinking . . . . . . . . . . . . . . . . . . . . . . . . . . . . . . . . . . 69
Lesson 4.8: Proving Isosceles Triangle Conjectures . . . . . . . . . . . . . . . . . 71
Exploration: Napoleon’s Theorem . . . . . . . . . . . . . . . . . . . . . . . . . . . . . . . . 72
Chapter 4 Review . . . . . . . . . . . . . . . . . . . . . . . . . . . . . . . . . . . . . . . . . . . . . . . 72
Take Another Look . . . . . . . . . . . . . . . . . . . . . . . . . . . . . . . . . . . . . . . . . . . . . . 74
Chapter 5
Lesson 5.1: Polygon Sum Conjecture . . . . . . . . . . . . . . . . . . . . . . . . . . . . . 75
Lesson 5.2: Exterior Angles of a Polygon . . . . . . . . . . . . . . . . . . . . . . . . . 78
Lesson 5.3: Kite and Trapezoid Properties . . . . . . . . . . . . . . . . . . . . . . . . 79
Lesson 5.4: Properties of Midsegments . . . . . . . . . . . . . . . . . . . . . . . . . . . 81
Lesson 5.5: Properties of Parallelograms . . . . . . . . . . . . . . . . . . . . . . . . . . 84
Using Your Algebra Skills 5: Solving Systems of Linear Equations . . . 86
Lesson 5.6: Properties of Special Parallelograms . . . . . . . . . . . . . . . . . . 88
Lesson 5.7: Proving Quadrilateral Properties . . . . . . . . . . . . . . . . . . . . . . 91
Chapter 5 Review . . . . . . . . . . . . . . . . . . . . . . . . . . . . . . . . . . . . . . . . . . . . . . . 93
Take Another Look . . . . . . . . . . . . . . . . . . . . . . . . . . . . . . . . . . . . . . . . . . . . . . 96
Chapter 6
Lesson 6.1: Chord Properties . . . . . . . . . . . . . . . . . . . . . . . . . . . . . . . . . . . . 97
Lesson 6.2: Tangent Properties . . . . . . . . . . . . . . . . . . . . . . . . . . . . . . . . . . 99

Lesson 6.3: Arcs and Angles . . . . . . . . . . . . . . . . . . . . . . . . . . . . . . . . . . . . 102


Lesson 6.4: Proving Circle Conjectures . . . . . . . . . . . . . . . . . . . . . . . . . . . 105
Using Your Algebra Skills 6: Finding the Circumcenter . . . . . . . . . . . . 107
Lesson 6.5: The Circumference/Diameter Ratio . . . . . . . . . . . . . . . . . . . 110
Lesson 6.6: Around the World . . . . . . . . . . . . . . . . . . . . . . . . . . . . . . . . . . 112
Lesson 6.7: Arc Length . . . . . . . . . . . . . . . . . . . . . . . . . . . . . . . . . . . . . . . . . 113
Exploration: Cycloids . . . . . . . . . . . . . . . . . . . . . . . . . . . . . . . . . . . . . . . . . . . 115
Chapter 6 Review . . . . . . . . . . . . . . . . . . . . . . . . . . . . . . . . . . . . . . . . . . . . . . 116
Take Another Look . . . . . . . . . . . . . . . . . . . . . . . . . . . . . . . . . . . . . . . . . . . . . 120

iv
DG3SM586_ 5th06fm.qxd 8/2/06 6:45 PM Page v

Chapter 7
Lesson 7.1: Transformations and Symmetry . . . . . . . . . . . . . . . . . . . . . . 120
Lesson 7.2: Properties of Isometries . . . . . . . . . . . . . . . . . . . . . . . . . . . . . 122
Lesson 7.3: Compositions of Transformations . . . . . . . . . . . . . . . . . . . . 125
Lesson 7.4: Tessellations with Regular Polygons . . . . . . . . . . . . . . . . . . 127
Lesson 7.5: Tessellations with Nonregular Polygons . . . . . . . . . . . . . . 129
Lesson 7.6: Tessellations Using Only Translations . . . . . . . . . . . . . . . . . 130
Lesson 7.7: Tessellations That Use Rotations . . . . . . . . . . . . . . . . . . . . . 130
Lesson 7.8: Tessellations That Use Glide Reflections . . . . . . . . . . . . . . 131
Using Your Algebra Skills 7: Finding the Orthocenter
and Centroid . . . . . . . . . . . . . . . . . . . . . . . . . . . . . . . . . . . . . . . . . . . . . . . . . 132
Chapter 7 Review . . . . . . . . . . . . . . . . . . . . . . . . . . . . . . . . . . . . . . . . . . . . . . 134
Chapter 8
Lesson 8.1: Areas of Rectangles and Parallelograms . . . . . . . . . . . . . . 136
Lesson 8.2: Areas of Triangles, Trapezoids, and Kites . . . . . . . . . . . . . . 138
Lesson 8.3: Area Problems . . . . . . . . . . . . . . . . . . . . . . . . . . . . . . . . . . . . . . 142
Lesson 8.4: Areas of Regular Polygons . . . . . . . . . . . . . . . . . . . . . . . . . . . 144
Lesson 8.5: Areas of Circles . . . . . . . . . . . . . . . . . . . . . . . . . . . . . . . . . . . . . 146
Lesson 8.6: Any Way You Slice It . . . . . . . . . . . . . . . . . . . . . . . . . . . . . . . . 147
Exploration: Geometric Probability II . . . . . . . . . . . . . . . . . . . . . . . . . . . . 149
Lesson 8.7: Surface Area . . . . . . . . . . . . . . . . . . . . . . . . . . . . . . . . . . . . . . . 149
Exploration: Alternative Area Formulas . . . . . . . . . . . . . . . . . . . . . . . . . . 151
Chapter 8 Review . . . . . . . . . . . . . . . . . . . . . . . . . . . . . . . . . . . . . . . . . . . . . . 152
Take Another Look . . . . . . . . . . . . . . . . . . . . . . . . . . . . . . . . . . . . . . . . . . . . . 155
Chapter 9
Lesson 9.1: The Theorem of Pythagoras . . . . . . . . . . . . . . . . . . . . . . . . . 156
Lesson 9.2: The Converse of the Pythagorean Theorem . . . . . . . . . . . 159
Using Your Algebra Skills 8: Radical Expressions . . . . . . . . . . . . . . . . . . 161
Lesson 9.3: Two Special Right Triangles . . . . . . . . . . . . . . . . . . . . . . . . . . 162
Lesson 9.4: Story Problems . . . . . . . . . . . . . . . . . . . . . . . . . . . . . . . . . . . . . 165
Lesson 9.5: Distance in Coordinate Geometry . . . . . . . . . . . . . . . . . . . . 168
Lesson 9.6: Circles and the Pythagorean Theorem . . . . . . . . . . . . . . . . 171
Chapter 9 Review . . . . . . . . . . . . . . . . . . . . . . . . . . . . . . . . . . . . . . . . . . . . . . 175
Take Another Look . . . . . . . . . . . . . . . . . . . . . . . . . . . . . . . . . . . . . . . . . . . . . 181
Chapter 10
Lesson 10.1: The Geometry of Solids . . . . . . . . . . . . . . . . . . . . . . . . . . . . 182
Lesson 10.2: Volume of Prisms and Cylinders . . . . . . . . . . . . . . . . . . . . 183
Lesson 10.3: Volume of Pyramids and Cones . . . . . . . . . . . . . . . . . . . . . 186
Lesson 10.4: Volume Problems . . . . . . . . . . . . . . . . . . . . . . . . . . . . . . . . . . 189
Lesson 10.5: Displacement and Density . . . . . . . . . . . . . . . . . . . . . . . . . 191
Lesson 10.6: Volume of a Sphere . . . . . . . . . . . . . . . . . . . . . . . . . . . . . . . . 194
Lesson 10.7: Surface Area of a Sphere . . . . . . . . . . . . . . . . . . . . . . . . . . . 197

v
DG3SM586_ 5th06fm.qxd 8/2/06 6:45 PM Page vi

Chapter 10 Review . . . . . . . . . . . . . . . . . . . . . . . . . . . . . . . . . . . . . . . . . . . . . 199


Take Another Look . . . . . . . . . . . . . . . . . . . . . . . . . . . . . . . . . . . . . . . . . . . . . 203
Chapter 11
Using Your Algebra Skills 9: Proportion and Reasoning . . . . . . . . . . . 203
Lesson 11.1: Similar Polygons . . . . . . . . . . . . . . . . . . . . . . . . . . . . . . . . . . 205
Lesson 11.2: Similar Triangles . . . . . . . . . . . . . . . . . . . . . . . . . . . . . . . . . . . 208
Lesson 11.3: Indirect Measurement with Similar Triangles . . . . . . . . . 212
Lesson 11.4: Corresponding Parts of Similar Triangles . . . . . . . . . . . . 215
Lesson 11.5: Proportions with Area and Volume . . . . . . . . . . . . . . . . . . 219
Exploration: Why Elephants Have Big Ears . . . . . . . . . . . . . . . . . . . . . . . 222
Lesson 11.6: Proportional Segments Between Parallel Lines . . . . . . . 222
Exploration: Two More Forms of Valid Reasoning . . . . . . . . . . . . . . . . . 226
Chapter 11 Review . . . . . . . . . . . . . . . . . . . . . . . . . . . . . . . . . . . . . . . . . . . . . 226
Take Another Look . . . . . . . . . . . . . . . . . . . . . . . . . . . . . . . . . . . . . . . . . . . . . 228
Chapter 12
Lesson 12.1: Trigonometric Ratios . . . . . . . . . . . . . . . . . . . . . . . . . . . . . . . 229
Lesson 12.2: Problem Solving with Right Triangles . . . . . . . . . . . . . . . 231
Exploration: Indirect Measurement . . . . . . . . . . . . . . . . . . . . . . . . . . . . . . 234
Lesson 12.3: The Law of Sines . . . . . . . . . . . . . . . . . . . . . . . . . . . . . . . . . . 234
Lesson 12.4: The Law of Cosines . . . . . . . . . . . . . . . . . . . . . . . . . . . . . . . . 237
Lesson 12.5: Problem Solving with Trigonometry . . . . . . . . . . . . . . . . . 241
Exploration: Trigonometric Ratios and the Unit Circle . . . . . . . . . . . . . 244
Chapter 12 Review . . . . . . . . . . . . . . . . . . . . . . . . . . . . . . . . . . . . . . . . . . . . . 245
Take Another Look . . . . . . . . . . . . . . . . . . . . . . . . . . . . . . . . . . . . . . . . . . . . . 251
Chapter 13
Lesson 13.1: The Premises of Geometry . . . . . . . . . . . . . . . . . . . . . . . . . 252
Lesson 13.2: Planning a Geometry Proof . . . . . . . . . . . . . . . . . . . . . . . . . 254
Lesson 13.3: Triangle Proofs . . . . . . . . . . . . . . . . . . . . . . . . . . . . . . . . . . . 257
Lesson 13.4: Quadrilateral Proofs . . . . . . . . . . . . . . . . . . . . . . . . . . . . . . . 264
Exploration: Proof as Challenge and Discovery . . . . . . . . . . . . . . . . . . . 272
Lesson 13.5: Indirect Proof . . . . . . . . . . . . . . . . . . . . . . . . . . . . . . . . . . . . . 272
Lesson 13.6: Circle Proofs . . . . . . . . . . . . . . . . . . . . . . . . . . . . . . . . . . . . . . 273
Lesson 13.7: Similarity Proofs . . . . . . . . . . . . . . . . . . . . . . . . . . . . . . . . . . . 278
Using Your Algebra Skills 10: Coordinate Proof . . . . . . . . . . . . . . . . . . . 286
Chapter 13 Review . . . . . . . . . . . . . . . . . . . . . . . . . . . . . . . . . . . . . . . . . . . . . 291

vi
DG3SM586_ 5th06fm.qxd 8/2/06 6:45 PM Page vii

Introduction
The Solutions Manual for Discovering Geometry: An Investigative Approach
contains solutions to the exercises at the end of each lesson and to the
Extensions, Improving Your Reasoning Skills, Improving Your Visual
Thinking Skills, Improving Your Algebra Skills, Projects, and Take Another
Look activities. You can find solutions for the Investigations and Explorations
in the Teacher’s Edition.
The solutions in this Solutions Manual are more complete than those offered
as annotations in the Teacher’s Edition. Although complete solutions for the
problems are provided here, keep in mind that often there is more than one
method students might use to solve a particular problem. Also, the answers will
vary for some problems, depending on assumptions that students make. For
problems that could have many different answers, a sample solution is given.
In order to save space, the size of constructions has been reduced in the Solutions
Manual. Keep this in mind as you check student constructions.
Refer to these solutions when your students have difficulty solving a problem
and need some assistance in determining a possible approach toward solving it.
You might also want to provide a copy of certain solutions for students who
have been absent for an extended period of time.

vii
DG3SM586_ 5th06fm.qxd 8/2/06 6:45 PM Page viii
DG3SM586_001-024.qxd_k1 8/2/06 5:14 PM Page 1

sections parallel to the table are circular (and


CHAPTER 0 therefore have both reflectional and rotational
symmetry). Cross sections perpendicular to the
LESSON 0.1 table are fairly round for an orange, but not for an
apple or a pear. For all three fruits, these perpendi-
EXERCISES cular cross sections have approximate reflectional
1. Possible answers: snowflakes and crystals; flowers symmetry, but only the orange has rotational
and starfish symmetry. You can slice a banana to get a round
2. Possible answers: to create shapes and patterns, cross section that has both reflectional and
to show perspective and proportions, to create rotational symmetry. You can also slice it to create
optical illusions a long, skinny cross section that may or may not
have reflectional symmetry, depending on the
3. Possible answers: a butterfly, a valentine heart, a shape of the banana.
person; bilateral symmetry
B. Possible answer: Bilateral symmetry gives humans
4. A, B, C, and F. Students might also include D in balance, which allows them to make smooth, even
their answer if they don’t notice that some of the movements. Symmetry also makes it easier to lift
rounded corners of the triangular sections are more and catch objects.
rounded than others.
5. A, B, D, and E LESSON 0.2
6. 4 of diamonds; none EXERCISES
7. The line of symmetry is a line along the surface of 1. Compass and straightedge
the lake. 2. Answer will be the Astrid or 8-pointed star, possibly
8. There is a vertical line of symmetry through the with variations.
middle of the Taj Mahal, and a horizontal line of 3. Answer will be a design from among the three
symmetry between the Taj Mahal and the reflecting choices.
pool. The pool reflects the building, giving the scene
more reflectional symmetry than the building Steps for creating the first design:
itself has. Draw a 12-by-12 frame. Then draw half an Astrid
9. Designs will vary, but all should have 2-fold (see the instructions on page 8 of the student book)
rotational symmetry. It is impossible to have two on the left side of the frame. Draw three more half-
intersecting lines of reflectional symmetry without Astrids, on the top, right, and bottom of the frame.
rotational symmetry.

2 lines of reflectional
symmetry

Steps for creating the second design:


10. Answers will vary.
Draw a triangle. Mark and connect the midpoints
11. Answers will vary. of the sides. Then mark and connect the midpoints
IMPROVING YOUR VISUAL THINKING SKILLS of the sides of the three outer triangles. Then color
the smallest “point-up” triangles to create the final
Stick j is on top, so visually remove it. Then determine
design.
which stick is next. It appears to be stick f. Visually
remove that stick and figure out which stick is next.
Continue this process until all the sticks have been
“removed.” The order of removal is j, f, g, b, i, k, h, c,
e, a, d.
EXTENSIONS
A. Possible answer: If you set an apple, a pear, or
an orange on its “bottom” on a table, then cross

Discovering Geometry Solutions Manual CHAPTER 0 1


©2003 Key Curriculum Press
DG3SM586_001-024.qxd_k1 8/2/06 5:14 PM Page 2

Steps for creating the third design: IMPROVING YOUR ALGEBRA SKILLS
Draw a regular hexagon. Move clockwise around the a  20, b  36, c  24, d  17. The easiest way to find
hexagon. On each side, mark the point one-third the solution is to write and solve systems of equations.
of the way from the first endpoint to the second For example, using the information in the pyramid, you
endpoint. Connect the points in the order you mark can write these equations:
them. Then follow the procedure described in the a  b  56
previous step with the new, inside hexagon. Repeat
the process several more times, each time marking a  d  37
points on the innermost hexagon, until the design Subtracting the second equation from the first gives
looks like the one in the student book. Then color b  d  19.
the design.
From the pyramid, you also know that b  d  53. So,
to find the value of b, you can add these equations:
b  d  19
b  d  53
The sum of the equations is 2b  72, or b  36. Once
you know the value of b, you can easily find the other
values: b  c  60, so c  24; b  d  53, so d  17;
4. The first design has 4-fold rotational symmetry and b  a  56, so a  20.
four lines of reflectional symmetry (the horizontal
line through the center, the vertical line through the EXTENSIONS
center, and the two diagonals of the square). A. Answers will vary.
The second design has 3-fold rotational symmetry B. Sample designs:
and three lines of reflectional symmetry (from each
vertex of the large triangle to the midpoint of the
opposite side).
The third design has 6-fold rotational symmetry if
you ignore color and 2-fold rotational symmetry if
you don’t.
5. Answers will vary. The given design may be placed
in any corner. Once that design is in place, the rest
of the pattern is determined by the symmetry. Here
are two possible answers: LESSON 0.3

2 lines of
EXERCISES
reflectional 1. Create the grid of circles by using the technique
symmetry described on page 10 of the student book. The grid
should look like the one shown below. The way the
grid is colored will vary, but the final design should
have reflectional symmetry.
6. Possible answer:

7. 3 lines of reflectional
symmetry 2. Using compass and straightedge: Draw an equilat-
eral equiangular hexagon with side length 2. Each
vertex will be the center of a circle with radius 1.
3-fold rotational
Fit the seventh small circle inside the other six.
symmetry: rotated
120°, 240°, 360°

2 CHAPTER 0 Discovering Geometry Solutions Manual


©2003 Key Curriculum Press
DG3SM586_001-024.qxd_k1 8/2/06 5:14 PM Page 3

The large circle has the same center as the seventh Next, make arcs centered at the two points where
circle and has radius 3. this arc intersects the circle. Continue this process
until the daisy is complete. The shape of the final
daisy should look like the one below. The way the
daisy is colored may vary; the result should have
reflectional symmetry, but not rotational symmetry.
Possible answer:

Using isometric dot paper: Start by drawing a circle


centered at one of the dots and passing through
the surrounding six dots. Then, draw a circle 5. To create the central hexagon, use a technique
of the same radius centered at each of the six similar to the one described on page 11 of the
dots surrounding the original circle. Draw the student book, but draw entire circles instead of arcs.
large surrounding circle and color the result. The
result should look like the design below. The way
the design is colored will vary, but it should have
rotational symmetry.

3. Create the daisy by using the technique described


on page 11 of the student book. The way the daisy Then, without changing the compass setting,
is colored will vary. The final design should have draw circles centered at the intersection points
rotational symmetry, but not reflectional symmetry. of the outer circles. (Each of these new circles
Possible answer: should intersect the endpoints of one of the
hexagon’s sides.)

4. To create the 12-petal daisy, first draw a 6-petal


daisy using the technique described on page 11
of the student book. Then, without changing the
compass setting, make an arc centered on the
outer circle, halfway between two of the petals.

Use the technique described on page 11 to create a


hexagon inside each of these new circles.
IMPROVING YOUR ALGEBRA SKILLS
The diagonal from the lower left corner to the upper
right corner is complete, so add the numbers. The sum
is 90, so the numbers in each row, column, and diagonal
must add to 90.

Discovering Geometry Solutions Manual CHAPTER 0 3


©2003 Key Curriculum Press
DG3SM586_001-024.qxd_k1 8/2/06 5:14 PM Page 4

The four numbers in the other diagonal have sum 73, so 2. Possible answer: When zebras group together,
the fifth number must be 90  73, or 17. their stripes make it hard for predators to see
individual zebras.
20 8 14

19 25 26 3. Designs will vary.


12 18 30 4. Designs will vary, but should have reflectional
29 10 11 17 symmetry and not rotational symmetry.
22 15 16 5. Possible answers: Two-dimensional: rectangle,
triangle, trapezoid; three-dimensional: cylinder,
Once that number is filled in, you can find the last cone, prism. The palace facade has one line of
number in the fourth row (23) and the third number in reflectional symmetry (bilateral symmetry).
the fourth column (24). Continue looking for rows or
columns with a single blank and filling in that blank.
IMPROVING YOUR REASONING SKILLS
When you have done all you can, the magic square Here is one way to reason through Game 1: Clue 1
should look like this. indicates that the digits 1, 2, and 3 are not digits of the
number. Clue 2 indicates that the number includes one
20 8 14 of the three digits 4, 5, and 6. Clue 3 indicates that it
13 19 25 26 7 includes one of the three digits 7, 8, and 9. The third
6 12 18 24 30 digit must be 0, because it is the only digit left. In
29 10 11 17 23
Clue 4, there are two correct digits. You know one of
the correct digits must be 0, so the other must be either
22 15 16
7 or 5. Clue 5 contains only one correct digit, which
must be 0. Therefore, the number cannot contain 7 or 8.
The three numbers in the first row have sum 42, so Combining Clues 5 and 4, you can conclude that the
the numbers in the two blank squares must have sum number contains 5, and combining Clues 5 and 7, you
90  42, or 48. The only two numbers on the list with can conclude that the number contains 9. So, the three
sum 48 are 21 and 27. Try putting 21 in the first blank digits are 0, 5, and 9. Clue 5 indicates that 0 cannot be
square and 27 in the second. the first digit. Combining this information with Clue 4,
20 21 27 8 14 you can conclude that 5 is the last digit. Therefore, 0
13 19 25 26 7
must be the second digit, so the number is 905.
6 12 18 24 30 Here is one way to reason through Game 2: Clue 1
29 10 11 17 23
indicates that 9, 0, and 8 are not digits of the number.
Clue 3 indicates that two of the digits 3, 8, and 7 are
22 15 16
digits of the number. Because 8 was eliminated in Clue
1, 3 and 7 must be digits of the number. Therefore, in
The four numbers in the second column have sum 62. Clue 2, the one correct digit must be 3, so 1 and 4 are
So, the fifth number must be 90  62, or 28. The four eliminated as possible digits. In Clue 3, either 3 or 7
numbers in the third column have sum 81. So, the fifth must be in the correct position. Using Clue 5, we can
number must be 9. As a check, add the numbers in the conclude that 3 is in the correct position. Because 7
fifth row. The sum is 90, which is what it should be. cannot be in the third or first position (3 is in the first
20 21 27 8 14 position), it must be in the second position. Thus, the
13 19 25 26 7
number is of the form 37X. In Clue 4, the digit in the
correct position must be 6, so the number is 376.
6 12 18 24 30

29 10 11 17 23
LESSON 0.5
22 28 9 15 16
EXERCISES
1. Possible answers: Scotland, Nigeria
LESSON 0.4
2. Possible answer:
EXERCISES
1. Possible answer: The designs appear to go in and
out of the page (appear to be three-dimensional).
The squares appear to spiral (although there are no
curves in the drawing). The spiral appears to go
down as if into a hole. Lines where curves meet
look wavy. Bigger squares appear to bulge out.

4 CHAPTER 0 Discovering Geometry Solutions Manual


©2003 Key Curriculum Press
DG3SM586_001-024.qxd_k1 8/2/06 5:14 PM Page 5

3. Possible answer: Extra credit


● Paragraph about etching includes how the shapes are
related and mentions that rectangles and triangles are
not exact because of perspective and because they
are bent or broken.
EXTENSIONS
A. Encourage students to experiment with shapes
that can be deformed into a circle. Remind them
that any one ring must be completely above or
completely below any other ring. This is a difficult
problem, but students can find plenty of material
on the Internet. The Borromean Rings are an
4. Possible answer: (You would cut open the example of Brunnian links, which are configurations
middle ring.) of any number of rings that are linked but cease to
be linked if any one of them is removed. Solutions
for four and five rings are shown.

5. 4 rings

6. Answer will be a lusona from among the


three choices.
7. It means to solve a problem boldly and decisively or
in a creative way not considered by others.
8. The square knot has reflectional symmetry across a 5 rings
horizontal line. The less secure granny knot has B. Research results will vary.
2-fold rotational symmetry.
9. The result is a regular pentagon. LESSON 0.6
EXERCISES
1. Possible answers: Morocco, Iran, Spain, Malaysia
2. The Alhambra Palace
3. The shortest translation is a vertical or horizontal
PROJECT slide of 2.1 cm.
Project should satisfy the following criteria:
● Descriptions of the events depicted are tied to 2.1 cm

the etching.
● Geometric shapes mentioned include triangles
2.1 cm
and rectangles.
● Student story is clearly tied to his or her art.

Discovering Geometry Solutions Manual CHAPTER 0 5


©2003 Key Curriculum Press
DG3SM586_001-024.qxd_k1 8/2/06 5:14 PM Page 6

4. The tiles will match up 4 times around some points, 3. Compass: A geometry tool used to construct circles.
and 2 times around others. In terms of the original
Straightedge: A geometry tool used to construct
square tile: 4-fold symmetry in the center (center of
straight lines.
orange) or corner (center of white); 2-fold
symmetry in the midpoint of the edge of the 4. To create the 12-petal daisy, first draw a 6-petal
tile (where two orange shapes meet) daisy using the technique described on page 11 of
the student book. Then, without changing the
4-fold symmetry
about these points
compass setting, make an arc centered on the outer
circle, halfway between two of the petals. Next,
make arcs centered at the two points where this arc
2-fold symmetry
about this point intersects the circle. Continue this process until the
daisy is complete. The shape of the final daisy
should look like the one below.

5. To create the Petronas Towers design, first construct


the 8-pointed star using the instructions on page 21
of the student book, but don’t erase the interior
lines of the squares. Then make arcs centered at the
dents in the star and just touching the large circle.

5.

6. Possible answers: hexagon: honeycomb, snowflake;


pentagon: starfish, flower
6. Designs will vary. 7. Designs will vary.
7. Answers will vary but should be some form of an
8. Tessellations will vary. interweaving design.
PROJECT 8. Wheel A has four lines of reflectional symmetry,
Project should satisfy the following criteria: and Wheel C has five lines of reflectional symmetry.
Wheels B and D do not have reflectional symmetry.
● Includes photographs with captions or a videotape
with commentary.
● The captions or the commentary mentions geometry
terms from this chapter.
● The use of terms is accurate.
EXTENSION
Research results will vary.
9. Wheels B and D have only rotational symmetry.
CHAPTER 0 REVIEW Wheel B has 4-fold rotational symmetry, and
Wheel D has 3-fold rotational symmetry. Wheels
EXERCISES A and C have both reflectional and rotational
1. Possible answers: Islamic, Hindu, Celtic, Japanese, symmetry (A: 4-fold; C: 5-fold).
American 10. Designs will vary but should contain concentric
2. Depending on how you look at them, the large circles and symmetry in some of the rings.
hexagons can appear as blocks with a corner 11. Designs will vary but should contain concentric
removed or as corners with small cubes nestled circles and symmetry in some of the rings.
in them.
12. Designs will vary but should use techniques from
Islamic art, be a knot design, and have optical effects.

6 CHAPTER 0 Discovering Geometry Solutions Manual


©2003 Key Curriculum Press
DG3SM586_001-024.qxd_k1 8/2/06 5:14 PM Page 7

13. a. The flag of Puerto Rico is not symmetric, because  or QP


9. PQ 
of the star and the colors.
 or RT
10. TR , RI
 or IR
, and TI
 or IT

11. A B

12. y
S
11

Puerto Rico

b. The flag of Kenya does not have rotational R


symmetry, because of the colors and the
spearheads. x

  14.3 cm
13. mAB   6.7 cm
14. mCD
15–17. Check the length of each segment to see if it
is correct. Refer to each exercise for the correct
measurements.

Kenya
. X is the midpoint of WY
18. R is the midpoint of PQ .
. No midpoints are shown
Y is the midpoint of XZ
c. Possible answers: in ABC. (Note: Even though AD and DC appear
to be the same length, you cannot assume they are
congruent without the markings.)
19. Possible answers:
A B C D E
or
 ⬵ CE
AC 
Japan Nigeria
 
BC ⬵ CD

 with midpoint M. Add


20. First, draw a segment ST
CHAPTER 1 congruence marks.
T
M
S
LESSON 1.1
Next, add point P so that P
EXERCISES SP  PT. Draw SP and PT
1. Possible answers: point: balls, where lines cross; and add congruence marks.
segment: lines of the parachute, court markings;
collinear: points along a line of the court structure;
coplanar: each boy’s hand, navel, and kneecap, M
T
S
points along two strings
, TP
  to
Finally, extend PT P
2. PT
twice its length. Mark the
3. The name can have any two of the point names—A, endpoint Q. Because
R, or T—in either order. So, the correct answer can PT  TQ, mark TQ 
, RA
be any two of the following: AR , AT
, TA
, with the same
, TR
RT . congruence mark M
T

, MS
4. Any two of the following: MA , AS
, AM
, SA
, SM
 .
used for PT
S

5. A B
6. K L
Q
7. y  or CA
8. AC 
3 21. The first point listed must be the endpoint, A; the
D other point can be either B or C. So, the two names
–3 3
x  and AC
are AB .
–3 E

Discovering Geometry Solutions Manual CHAPTER 1 7


©2003 Key Curriculum Press
DG3SM586_001-024.qxd_k1 8/2/06 5:14 PM Page 8

, PN
22. PM  , XZ
23. XY  Possible answer:
y
24. The first point listed must be the endpoint of
the ray. Q
N
A
B x
P
25. Y 26. M N

X
34. A
R
27.
C T
G
D Y
A B
PROJECT
28. First, list all the segments with endpoint A: AB, AC
, Project should satisfy the following criteria:
, AE
AD . Next, list the segments with endpoint B,
● Uses concentric circles whose radii increase by a
but don’t include AB  because you already listed it:
constant amount and segments that divide the circles
, BD
BC , BE . Next, list all the segments with
into equal sectors.
endpoint C, but don’t include AC  or BC
 because
you already listed them: CD , CE. The only segment ● The decoration of the design emphasizes the spirals.
with endpoint D that hasn’t been listed yet is DE.
All the segments with endpoint E have been listed. USING YOUR ALGEBRA SKILLS 1
, AC
In all, there are 10 possible segments: AB , AD
,
, BC
AE , BD , BE , CD
, CE
, DE. EXERCISES
x x 12  (6) y y
29–31. y 1. x  
1
2    3 and y  
2 2
1
2
2
7  15
 2  4, so the midpoint is (3, 4).
5
x x 17  (1) y y
2. x  
1
2    9 and y  
2 2
1
2
2
M
8  11
C
x  2  1.5, so the midpoint is (9, 1.5).
–5 O A (4, 0) x x 14  (3) y y
3. x  
1
2    5.5 and y  
2 2
1
2
2
D
(7)  18
–5  2  5.5, so the midpoint is (5.5, 5.5)
B
4. Let (x, y) represent the other endpoint, then
12  x 8  y
3  2 and 18  2. Here are the
32. Change the signs of the coordinates to get P(6, 2),
Q(5, 2), R(4, 6). step-by-step solutions to these equations:
12  x
y 3  2
R
6  12  x Multiply both sides by 2.
Q P⬘ 6  x Subtract 12 from both sides.
x 8  y
18  2
Q⬘
P
36  8  y Multiply both sides by 2.

R⬘
44  y Add 8 to both sides.

The endpoint is (6, 44).


rise
Because the slope  run  between any two points is

4
, the points are collinear.
5. Yes. The coordinates of the midpoint of a segment
1
with endpoints (a, b) and (c, d) are found by taking
33. Plot point N. It will be easiest to determine the ac
the average of the x-coordinates,  
2 , and the
lengths if you draw a horizontal or vertical segment. bd
average of the y-coordinates, 2. Thus, the
To locate P and Q, mark points the same number of ac bd
units above and below (or to the left and the right midpoint is  2 , 2 .
 
of) point N. 6. (3, 2) and (6, 4). To get the first point of trisection,
add the coordinates of points A and B to get (9, 6),
then multiply those coordinates by 13 to get (3, 2).

8 CHAPTER 1 Discovering Geometry Solutions Manual


©2003 Key Curriculum Press
DG3SM586_001-024.qxd_k1 8/2/06 5:14 PM Page 9

To get the second point of trisection, add the 7. 90°


coordinates of points A and B to get (9, 6), then  passes through 60° on one scale and 120°
8. Side QC
multiply those coordinates by 23 to get (6, 4). This
on the other. Because AQC is larger than a 90°
works because the coordinates of the first point
angle, 120° must be the correct measure.
are (0, 0).
9. 45° 10. 135° 11. 45° 12. 135°
7. Possible method: Find the midpoint of the segment,
and then find the midpoint of each half of the 13. One way to find mCQB is to find mAQC and
segment. mAQB and subtract. mAQC  120° and
mAQB  90°, so mCQB  30°.
8. a. For Figure 1, both midpoints are (5.5, 6.5). For
Figure 2, both midpoints are (16, 6.75). For 14. To find mXQY, find mAQY and mAQX and
Figure 3, both midpoints are (29.75, 5.5). subtract. mAQY  135° and mAQX  45°, so
b. For these figures, the midpoints of the two mXQY  90°.
diagonals are the same point. This means the 15. 69°. Place the center mark of the protractor on
diagonals intersect each other at their midpoints. point A. Rotate the zero-edge of the protractor to
. AM
line up with side AC  passes through 69° on
LESSON 1.2 one scale and 111° on another. Because MAC is
smaller than a 90° angle, mMAC  69°.
EXERCISES
1. When three letters are used to name an angle, the 150 16
130 140 0
middle letter must be the vertex. Also, a single letter 10
120 170
1 50 40 30 20 18
0
60 10
can name an angle only if there is no confusion 1 00 M 70 0
about which angle it is referring to. For example, 80

90
90
there is only one angle with vertex E, so it can be

80
100
named E. However, there are two angles with

0
C

0 130 120 110


50 60 7
vertex O, so the name O is ambiguous. TEN,
A
NET, E; FOU, UOF, 1; ROU,
UOR, 2
4
40

1
150
30

In Exercises 2–4, the angles may be any size, but the 0


16
20

0
17
vertex labels should be the same as those in the 0
10

18

answers below. 0

2. N 3.
G B
16. 110° 17. 40° 18. 125° 19. 55°
A T
20. SML has the greater measure because
mSML  30° and mBIG  20°. The measure of
I an angle is the amount of rotation from one side to
the other. It has nothing to do with the lengths of
4. S
the angle’s sides.
M
21. Draw a ray with endpoint A.
L A

5. In the first figure, S, P, R, Q. The other Place the center mark of the protractor on point A
angles all have the same vertex, T, so none of them and line up the zero-edge with the ray.
can be described with just one letter. In the second
figure, all the angles have the same vertex, D, so 80 90 100 1
70 10
none of them can be described with just one letter. 60 100 90 80 70
120
110 13
50 60 0
120
6. Possible answer: 13
0 50
14
40

0
0

40
14

150
30
150

30

160
20

B
60

20
180 170 1

170 180
10

10

C A
0

A
D

Discovering Geometry Solutions Manual CHAPTER 1 9


©2003 Key Curriculum Press
DG3SM586_001-024.qxd_k1 8/2/06 5:14 PM Page 10

Mark a point next to the 44° mark of the 27. 28.


protractor. (Use the scale that will give you an angle
smaller than 90°.)

80 90 100
70 110
120
60 100 90 80
0 1 10 70 13
0
5
0
120
60
5 0
29. Possible answer: 4:00
13

14
40

0
0

40
14

30.

150
30

H
150

30

160
20

60

90°

20
180 170 1

170 180
10

6 8

10
A 44
0

0
T O
Remove the protractor, and draw a ray from
point A through the point you marked. 31. T H

44° R A S
A

32. G T
22. 23.
C
135°
D E
A
90°
B
I N
24. Align your protractor as if you were going to
33. W
measure A. Mark a point at the 22° mark (half
the measure of A). Remove the protractor, and
draw a ray from point A through the point you B O
T
marked. Add markings to show that the two angles
formed are congruent.
I

 and MY
34. MI  have the same congruence markings, so
22° 67.5°  and
they are the same length. That is, MI  MY. IC
67.5°
22° 
CK have the same markings, so IC  CK. M and
A C
I have the same markings, so mM  mI.
25. D and Y are not the same point. 35. SEU; EUO; MO
C
36. A quarter of a rotation is 90°. So,
Y
D x  90°  (15°  21°)  54°.
37. Half of a rotation is 180°. So,
N
y  180°  (41°  37°)  102°.
A 38. A full rotation has measure 360°. Both angles
marked as congruent have measure z, so
26. Don’t forget that at half past the hour, the hour 2z  360°  (87°  135°  74°)  64°. Therefore,
hand will be halfway between 3 and 4. z  32°.
39. The incoming angle has measure 50°. Draw an
outgoing angle of measure 50°. The side of the

10 CHAPTER 1 Discovering Geometry Solutions Manual


©2003 Key Curriculum Press
DG3SM586_001-024.qxd_k1 8/2/06 5:14 PM Page 11

angle does not pass through the pocket, so the ball 1. The steps are shown below. (Read across the rows
will not go in. rather than down the columns.)

50° 50° 10
8 7
4
15

6 2. PDDPPPDDDPPPDDP

LESSON 1.3

40. The incoming angle measures 73°. The ray through EXERCISES
point B forms a 73° angle with the side of the table, 1. Possible answer: 2. Possible answer:
so the ball will pass through point B. D T E

41. Yes

45° R
O G

3. Possible answer:  must be


4. Note that DG
a segment and MS
must be a line.
B E
G
D

I G S
M
42. 12 cm
 must be
 must be a segment and AR
5. Note that PE
6 cm 6 cm
a ray.
43. The wire has length 4.36 cm. The center of gravity A
is the midpoint of the wire, which is 4.36 divided
by 2, or 2.18 cm from the end of the wire.
4.36 cm
2.18 cm 2.18 cm
P E
44. Possible answer: MS  DG means that the distance R

between M and S equals the distance between D


and G. MS  DG
 means that segment MS is 6. A D
congruent to segment DG. The first statement
B
equates two numbers. The second statement C
E
concerns the congruence between two geometric
figures. However, both statements convey the same 7. A and B are complementary, so their measures
information and are marked the same way on must add to 90°. Because mA  40°, mB  50°.
a diagram.
IMPROVING YOUR VISUAL THINKING SKILLS
40°
In problems of this kind, it’s a good strategy to move A
the coins so that the middle ones form an alternating
sequence. Note that due to the symmetry of the
50°
problems, switching the P’s and D’s is also a solution. B

Discovering Geometry Solutions Manual CHAPTER 1 11


©2003 Key Curriculum Press
DG3SM586_001-024.qxd_k1 8/2/06 5:14 PM Page 12

8. C and D are supplementary, so their measures 19. False. Possible counterexample:


must add to 180°. Because mD  40°,
mC  140°.

140°
C
140° 40°
C D 40°
D

9. B is a Zoid. A Zoid is a creature 20. False. Possible counterexample:


whose interior contains a small A
triangle surrounding a black dot.
10. A good definition places an object in a class and
also differentiates it from other objects in that class.
A good definition has no counterexamples. C T

11. True 21. If P is not between A and B or not between C


12. True 13. True and D, then the definition is not true. Here is
a counterexample:

P
C
B D

A
14. False

 and CD
The following is a correct definition: If AB 
intersect at point P so that P is between A and B
15. False. Think in three dimensions. There are an and P is between C and D, then APC and BPD
infinite number of planes that contain the given are a pair of vertical angles.
line, and each one contains a line perpendicular
22. Point P must be on the line through T and S. The
through the given point.  is 69, or 23. One way to find another
slope of TS
point on the line is to start at point T and move up
2 units and right 3 units. This would give you
point (3, 0). You can continue moving up and
over to find more possible points. Possible answers:
(3, 0), (0, 2), or (6, 6).
  TS
23. For QR , the lines must have the same slope.
The slope of TS is 69, or 23. One way to find
16. False. Possible counterexample:  is 23 is to start at
point Q so that the slope of QR
point R and move up 2 units and right 3 units. You
50° will be at point (2, 3). You can continue moving
up and over to find more possible points. Possible
answers: (2, 3) or (5, 1).
50° 80° 24. Just as in pool, the measure of the incoming angle
is equal to the measure of the outgoing angle. The
17. True. (Parallel lines must be in the same plane.) incoming angle has measure 46°, so draw an
outgoing angle with measure 46°.
A

46°
18. False. Possible counterexample:
Reflected
C T segment
46°

B
Mirror
A

12 CHAPTER 1 Discovering Geometry Solutions Manual


©2003 Key Curriculum Press
DG3SM586_001-024.qxd_k1 8/2/06 5:14 PM Page 13

You should find that the reflected ray and the ray 29.
that passes through (called the “refracted” ray) are 0 pts.
mirror images of each other. They form congruent
angles with the mirror. 1 pt.

25. This problem is best solved by trial and error. Here


, so
is a possible method. Point C has to lie on TR 2 pts.
choose that point first. Put it far enough to the left
of point S so that you have room for points A and 3 pts.

B. Because BAC will be a right angle, you know


30.
points A and B will be on a segment perpendicular 1 pt. 2 pts.
. Because ABS will be obtuse, B needs to
to AC
be to the right of A and to the left of S. BAT
needs to be an acute angle, so place segment AB 3 pts. 4 pts.
somewhere such that ABS is obtuse and BAT
is acute. Possible answer: A(8, 8), B(4.5, 6.5),
C(11, 1). 5 pts. 6 pts.
y

A Infinitely
B many points
6

S
120° 1 2
31. a. , or  is missing, so  is left.
C 360° 3 3
x 60° 1
–12 –6 b. , or  is missing.
360° 6
T 360°
R c.   40°
9
IMPROVING YOUR VISUAL THINKING SKILLS
26. Make a sketch showing the given information. The other four tetrominoes:
, AC  6 cm.
Because D is the midpoint of AC

Because C is the midpoint of AB , AB  12 cm.
3 cm
A D C B
Note that if two tetrominoes are congruent, they
are considered to be the same. For example, these
27. Make a sketch showing the given information. tetrominoes are all the same:
 is the bisector of ABC,
Because BD
mABD  24°. Because BE  is the bisector
of ABD, mEBD  12°. Therefore,
mEBC  12°  24°, or 36°.
LESSON 1.4
A
E EXERCISES
D 1. Octagon 2. Rectangle or square
3. Hexagon 4. Triangle
24°
B C 5. Heptagon 6. Pentagon
7. Undecagon 8. Dodecagon
28. Possible answer:
9. Possible answer: 10. Possible answer:

Discovering Geometry Solutions Manual CHAPTER 1 13


©2003 Key Curriculum Press
DG3SM586_001-024.qxd_k1 8/2/06 5:14 PM Page 14

11. Possible answer:   FI


21. PA  and IVE  ANC.
22. At right is an equilateral concave
pentagon. It is impossible to
draw an equiangular concave
pentagon because an angle where
there is a “dent” must be larger than
12. To name the figure, start with any vertex, and list an angle where there is no “dent.”
the letters in order as you move clockwise or
counterclockwise around the figure. A few possible 23. (5, 6). Here is one way to reason through this
names are FIVER, VERFI, REVIF, IVERF. problem: Point P is 5 units right of point C and
1 unit up. Point E is 5 units right of point A
13. Possible name: quadrilateral FOUR and 1 unit up. So, point T could be 5 units right
14. Possible name: equilateral quadrilateral BLOC and 1 unit up from point R. Counting squares,
this gives one possibility for T as (5, 6).
15. Consecutive angles are angles whose vertices are
endpoints of the same side. For example, C and 24. S(9, 0), I(4, 2). Use reasoning similar to that
Y are consecutive angles. Consecutive sides are described in the solution to Exercise 23.
 and YN
sides that share a vertex. For example, CY 
25. S(3, 0), I(1, 5)
are consecutive sides.
26. A(7, 6), N(5, 9) or A(5, 2), N(7, 1)
16. Nine. A concave hexagon is a hexagon with at least
one diagonal outside the polygon. The hexagon on 27. Draw two sides of the octagon by connecting A, B,
the left below is a convex hexagon. On the right, and C. Get two more sides by reflecting these sides
the hexagon’s nine diagonals have been drawn. over the x-axis. Get the remaining sides by reflecting
those four sides over the y-axis. The octagon is not
regular because its angles are not all the same size.
y

C (0, 5)
B (4, 4)
, AD
17. Make a sketch. The diagonals are AC , BD
,
, and CE
BE .
x
A (0, 5)
A E

B
D

28. Possible answer: 29. Possible answer:


P N G
C

18. When stating that two triangles are congruent, you E A


H A
need to list the vertices in the order that shows the
correspondences. Because T corresponds to E,
I corresponds to A, and N corresponds to R, N T
E X
EAR  TIN.
19. WEN 30. Possible answer: 31. Possible answer:
Q U
 corresponds to TH
20. a. Because PO , they must be the
same length. Therefore, PO  28. Using similar
reasoning, PR  34, EW  20, RE  58, and
WO  44. D A
b. Because P and T correspond, mT  mP
 87°. Because W and I correspond, mI
 mW  165°.

14 CHAPTER 1 Discovering Geometry Solutions Manual


©2003 Key Curriculum Press
DG3SM586_001-024.qxd_k1 8/2/06 5:14 PM Page 15

32. Possible answer: 35. Complementary angles: AOS and SOC; vertical
angles: OCT and ECR, or TCE and RCO
5 cm
5 cm
36. 37. Possible answer:
E
A B 5

20

33. Use your compass to draw a circle. Because a C D


regular pentagon has five equal sides, you want to F

divide the circle into five equal pieces. Draw five 72°
angles (360°  5  72°) with vertices at the center 38. All are possible except two points.
of the circle as shown below. Then, connect the
points where the sides of the angles intersect
the circle. 0 pts. 1 pt.

3 pts. 4 pts.
72° 72°
72° 72°
72°

5 pts. 6 pts.

34. 90 cm. Let l and w represent l l IMPROVING YOUR VISUAL THINKING SKILLS
the length and the width of w w Here is one way to organize your work to make sure
one of the small rectangles. you have found all the possibilities. Keep in mind that
Then, as the diagram at right l l congruent pentominoes are considered to be the same.
illustrates, the perimeter of
w w w
the large rectangle is 4l  5w.
So, 4l  5w  198. You can
also see from the diagram Pentominoes with a Pentominoes with a row
row of five squares of four squares
that 2l  3w.
Now, do a little algebra.
4l  5w  198 The equation for the perimeter
of the large rectangle.
Pentominoes with a row of three squares and both other
2(2l)  5w  198 Rewrite 4l as 2(2l). squares on the same side of that row

2(3w)  5w  198 Substitute 3w for 2l.

11w  198 Simplify.

w  18 Solve for w.
Pentominoes with a row of three squares and the other
To find l, go back to the equation 2l  3w. squares on opposite sides of that row

2l  3w
LESSON 1.5
2l  3(18) Substitute 18 for w.
EXERCISES
2l  54 Multiply.   BC
 and D is a right angle.
1. First figure: AD
l  27 Solve for l.   HG  and EF
  FG , or EFGH
Second figure: EH
So, each small rectangle has length 27 cm and width is a kite. (Note: EFGH is drawn so that it doesn’t
18 cm. Therefore, the perimeter of a small rectangle look like a kite, but the markings indicate it is
is 2(27  18)  90 cm. a kite.)

Discovering Geometry Solutions Manual CHAPTER 1 15


©2003 Key Curriculum Press
DG3SM586_001-024.qxd_k1 8/2/06 5:14 PM Page 16

Third figure: I and J are right angles. 18. Possible answer:


  MN
Fourth figure: QP  and QM
  PN
, or MNPQ A
6 cm
B
40°
is a parallelogram.
2. Figure A. A scalene right triangle has three sides of 6 cm
C A B
40°
different lengths and one right angle.
3. Figure E. An isosceles right triangle has two sides of
the same length and one right angle. C

4. Figure D. An isosceles obtuse triangle has two sides 19. To have two outside diagonals, the hexagon must be
of the same length and one obtuse angle. concave with two “dents.”
5. Figure B. A trapezoid is a quadrilateral with exactly
one pair of parallel sides.
6. Figure F. A rhombus is a quadrilateral with four
congruent sides.
7. Figure I. A rectangle is a parallelogram with four 20. A regular quadrilateral has four congruent sides and
congruent angles. four congruent angles. It is a square.
8. Figure C. A kite is a quadrilateral with two distinct
pairs of consecutive congruent sides.
9. Figures F, G, and I. A parallelogram is a quadrilat-
eral with two pairs of parallel sides.
10. C 11. C
21. There are three possibilities for the other two
vertices: (2, 6) and (2, 4); (6, 2) and (2, 4);
(3, 1) and (1, 3). In two cases, the given vertices
are the endpoints of a side. In the third case, they
A T are the endpoints of a diagonal.
S M L
y
12. A 13. D I (2, 6)

Z O (–2, 4)
(1, 3)
(4, 2)

x
R C (0, 0)
(3, –1)
(6, –2)
14. N 15. L
E

(2, –4)
A
E U
B
F 22. There are an infinite number of possible answers.
Here are just a few possibilities: (1, 1), (1, 0),
Q (4, 3).
16. T H 23. There are six possible answers: (3, 4), (11, 4),
I
(3, 3), (11, 3), (0.5, 0.5), or (7.5, 0.5).
R G The triangle must have one right angle and two
congruent sides. To find the third vertex of an
17. Possible answer: isosceles right triangle with right angle at vertex
M, draw a horizontal line through M. Because
 is 7 units long, count over 7 units in either
ME
3a 3a
2a 2a direction to locate point O. You will get (3, 4)
b b – 2a or (11, 4).

16 CHAPTER 1 Discovering Geometry Solutions Manual


©2003 Key Curriculum Press
DG3SM586_001-024.qxd_k1 8/2/06 5:14 PM Page 17

y 31. Find the midpoint of each rod. All the midpoints lie
O M O on the same line; place the edge of a ruler under
4
this line.
2
PROJECT
x
–8 –6 –4 –2 2 Project should satisfy the following criteria:
–2
● Both of the given “impossible” objects are reproduced.
E –4
● Students created their own “impossible” object.
You can use a similar method to find the third Extra credit
vertex if the right angle is at E. You will get (3, 3) ● The objects are drawn in creative ways that accentuate
or (11, 3). their impossibility.
 and EO
If the right angle is at O, then MO  will be ● Students include analyses of Escher’s Waterfall and
the congruent sides. Visually, you can see that vertex Belvedere.
O must be located on the horizontal line y  0.5
(that is, the horizontal line through the midpoint of EXTENSION
). For O to be a right angle, the slopes of MO
ME  Answers will vary.
 must be negative reciprocals. The two
and EO
points that satisfy these criteria are (0.5, 0.5) LESSON 1.6
or (7.5, 0.5).
EXERCISES
24. Possible answers: (3, 1), (1, 9), (2, 2), (4, 8),
(0, 1), or (1, 6). If the right angle is at C, then , BD
1. Three of the following: AB , EC
, EF

 will be 5, the negative reciprocal
the slope of CR 
2. EC
of the slope of CL. Draw a line through point C
with slope 5 and locate point R so that CR  CL. , EP
3. AP , FP
, BP
, CP

You can use similar reasoning to locate point R if 4. Five of the following: EF, AE
, AB, BC
, CD
, DF
,
the right angle is at L. If the right angle is at R, the        
EB , ED , FC , AC , DB , AF , AD , BF
problem is a little trickier. You can see that in order  (or EDC ) and EBC  (or EAC )
for RC to equal RL, point R will have to be on the 5. EFC
perpendicular line through the midpoint of CL . The , ECF
6. Two of the following: ECD , EDB, EDA, CBF
,

midpoint of CL is (0.5, 3.5). Draw a line with        
CBD , CDA , CDB , FEC , FED , FCA , DEC , DCF ,
slope 5 through this point. Then, try to locate a , BAC
BCA 
point R so that the slopes of RC  and RL are nega-
tive reciprocals. , HB
7. FG 
25. True 26. True 8. Either F or B

27. False. A diagonal connects nonconsecutive vertices. 9. Possible answers: cars, trains, motorcycles; washing
machines, dishwashers, vacuum cleaners; tape
28. False. An angle bisector divides an angle into two players, compact disc players, record players, car
congruent angles. racing, Ferris wheel
29. True  is the measure of the central angle
10. mPQ
30. (4, 1) → (3, 2); (1, 1) → (2, 2); whose sides pass through points P and Q. So,
  110°. Because PQ  and PRQ make up a
(2, 4) → (3, 1); (3, 5) → (2, 2). Yes, the mPQ
quadrilaterals are congruent. full circle, and a full circle has measure 360°,
  360°  110°, or 250°.
mPRQ
y
11. To make the 65° arc, first draw a circle. Then, draw
(–3, 5)
(2, 4) a 65° angle with its vertex at the center of the circle.
The minor arc with endpoints at the intersection
(–4, 1) points has measure 65°. To make the 215° arc, first
(1, 1) x
draw a circle. Then, draw a 145° angle with its
vertex at the center of the circle. The major arc with

Discovering Geometry Solutions Manual CHAPTER 1 17


©2003 Key Curriculum Press
DG3SM586_001-024.qxd_k1 8/2/06 5:14 PM Page 18

endpoints at the intersection points has measure 17.


360°  145°, or 215°.
A
C
65°
145°
B E

215°

D
  18. Yes 19. No
mAB  65° mCDE  215°
y y
12. Possible answers: concentric rings on cross sections
of trees (annual rings), bull’s-eye or target, ripples 5 5
from a rock falling into a pond
13. x x
–5 5 –5 5

–5 –5

20. No y
8
14.

P Q x
8 16

15. Equilateral quadrilateral (The figure is actually a


21.
rhombus, but students have not yet learned the
properties needed to conclude that the sides
are parallel.)
B
22. A scalene triangle has no congruent sides and an
P Q isosceles triangle has at least two congruent sides.
Therefore, it is not possible to draw a triangle that
A
is both isosceles and scalene.
16. From the way the hexagon was constructed, you 23. Possible answer: 24. Possible answer:
know that the length of each side is s, the same as B A
the radius of the circle. Therefore, the triangles are
equilateral. The perimeter of the hexagon is 6s and C B
the diameter of the circle is 2s, so the ratio is 6s to C A
2s, or 3 to 1. 25. Possible answer: 26. Possible answer:
T P A

A
100°
s

T R

18 CHAPTER 1 Discovering Geometry Solutions Manual


©2003 Key Curriculum Press
DG3SM586_001-024.qxd_k1 8/2/06 5:14 PM Page 19

27. Possible answer: must be a two-way street. Because it is an even-


120° 60° numbered street, 14th Street must run one-way east.
60° 60° The finished diagram indicates that a car traveling
south on S Street could make a legal left turn
120° 120° 60° 120°
onto 14th.
28. An equilateral triangle has three equal sides and N

three equal angles, so it cannot have one right W E


angle. Therefore, it is not possible to draw a S Street
(2 ways) S
triangle that is both equilateral and right.
29. Possible answer:
14th Street
R (1-way east)

3. 28 days. You can draw a vertical number line


T G representing the well. Mark the number line to
show Freddie’s location at the end of each day
12a  6b
30. Each side must have length ,
3 or 4a  2b. (D1, D2, and so on) and the end of each night
(N1, N2, and so on). After marking the results for
several days, you should begin to see a pattern: At
the end of Day n, Freddie is n  2 feet from the
bottom of the well; at the end of Night n, he is
4a + 2b
n feet from the bottom of the well. So, Freddie will
31. Possible answer: 32. Possible answer: first reach the top of the 30-foot well on Day 28.
Top 30
I 2p U
29
2p 28
2p
27
55°
50° 70° 50° 70° E Q 26
2p
25
33. Possible answer: 24
23
E
22
120°
21
K T 20
19
I 18
17
IMPROVING YOUR REASONING SKILLS
Height (ft)

16
In both solutions, the third steps must be third, but the 15
first two steps can be interchanged. 14
13
1. B1 to B3, A1 to A3, A3 to C3; or A2 to C2, A1 to 12
C1, C1 to C3 11
10
2. A4 to C4, A3 to C3, C4 to C2, A1 to C1, C1 to C3,
9
A2 to C2, C3 to C1 8 D6
7 D5
LESSON 1.7 6 D4
5 D3 N5
EXERCISES 4 D2 N4
3 D1 N3
1. Sample answer: Furniture movers might visualize
2 N2
how to rotate a couch to get it up a narrow
1 N1
staircase. Bottom
of well
2. Draw a diagram. S Street is a lettered street, so it
runs north-south. Because S is not a vowel, S Street

Discovering Geometry Solutions Manual CHAPTER 1 19


©2003 Key Curriculum Press
DG3SM586_001-024.qxd_k1 8/2/06 5:14 PM Page 20

4. Draw a diagram. She will need 28 posts. 9. The locus of all points in the plane that are 3 cm
5 ft
 is two lines, one on either side of AB
from AB , each
5 ft 3 cm from AB . The locus of all points in space that
5 ft
5 ft
are 3 cm from AB  is a cylinder of radius 3 cm with
25 ft .
axis AB

A B 3 cm
45 ft
3 cm
5. Make a number line on which each tick mark
represents 5 meters. Mark the given information 3 cm
on the number line. The final diagram will show A B
that Nadine is ahead.
W O M E N
10. No. The diagram below shows Beth’s route. The
6. 0 feet. When the cable hangs, it is bent in half, with interval between two dots represents 1 km. The time
each half measuring 15 feet. However, the vertical is labeled every 4 km. The diagram shows that Beth
distance from the lowest point on the cable to the will not reach Birnam Woods Road until 8:00, which
horizontal line level with the top of the poles is is not before sunset.
also 15 feet. Therefore, the cable must be hanging Start
vertically. For this to happen, the poles must be 3:00
touching. That is, the distance between the poles Birnam 1 km
Woods
must be 0 feet. Road 1 km 1 km
Cable N
4:00
For these two
15 lengths to be equal, 5:00
20 15 15 20 the cable must be
hanging vertically. 6:00
5 8:00 7:00

7. The locus of all points in the plane that are equally


distant from A and B is the line that passes through 11. A(0, 0), B(0, 5), C(2, 3), D(2, 1). First, rotate
 and is perpendicular to AB
the midpoint of AB . the horizontal segment AB . The rotated segment
The locus of all points in space that are equally will be vertical with point A at (0, 0) and point B at
distant from points A and B is the plane that passes (0, 5). Once you draw AB , you can locate the other
through the midpoint of AB  and is perpendicular vertices relative to points A and B. (You could also

to AB . copy trapezoid ABCD onto patty paper or tracing
paper, and then rotate the tracing paper 90°, or a
1
 turn, counterclockwise.)
4
B
12. C(3, 1), Y(4, 1), N(0, 3); If CYN were
A B reflected over the y-axis, the y-coordinates would
A stay the same, but the x-coordinates would be the
opposite of what they are now (the points would
be the same distance from the y-axis, but on the
opposite side).
8. The locus of all points that are the same distance
from the two sides of A is the bisector of A. 13. ABCD was slid right 3 units and up 2 units. So,
the rule is (x, y) → (x  3, y  2).

20 CHAPTER 1 Discovering Geometry Solutions Manual


©2003 Key Curriculum Press
DG3SM586_001-024.qxd_k1 8/2/06 5:14 PM Page 21

  CP
14. AB , EF
  GH
; i  k, j  k (Just because two 34. They bisect each other and are perpendicular.
lines look parallel or perpendicular, you cannot A
assume that they are parallel or perpendicular.)
15. The perimeter of the first figure is 6, and then the P Q
perimeter increases by 4 with each subsequent
B
figure. So, the perimeter of the second figure
is 6  4(1), the perimeter of the third figure is IMPROVING YOUR VISUAL THINKING SKILLS
6  4(2), and the perimeter of the fourth figure
is 6  4(3). Continuing this pattern, the perimeter
of the eighth figure would be 6  4(7), or 34 cm.

16. Triangles

Acute Right EXTENSIONS


Obtuse
A. Answers will vary. B. Answers will vary.
Scalene Isosceles
Scalene Isosceles
Isosceles Scalene
Equilateral LESSON 1.8

17. EXERCISES
Parallelograms
1. 2.
Rhombuses Squares Rectangles

18. B, D. Net A, when folded, would have too many


sides; in net C the two top squares would coincide;
net E would be missing a side; and net F would 3. 4.
have too many sides in the horizontal direction and
would be missing one in the vertical direction. Only
nets B and D would fold into a cube.
19. B 20. C
21. D 22. A
5. 6.
23. point, line, plane 
24. AB

25. AB 26. vertex

27. AB   CD
28. AB 
29. protractor 30. ABC
31. AB 
  CD 32. congruent to
7. Possible answer:
33. The distance is two times the radius.

r r
P Q

PQ = 2r

Discovering Geometry Solutions Manual CHAPTER 1 21


©2003 Key Curriculum Press
DG3SM586_001-024.qxd_k1 8/2/06 5:14 PM Page 22

8. Follow Steps 1–4 below. The 3 m-by-4 m face is 15. Pyramid with square base
biggest, so it should be on the “bottom.”

16. Below are sketches of the top and front views. From
the top view, you can see that 17  x  13  45,
so x  15. From the front view, you can see that
Step 1 Step 2 y  18  45, so y  27.
45 17 13
x
y
2m 45

3m 4m 18
x
17 13
Step 3 Step 4
Top Front
9. There are 3  4, or 12, boxes in the base layer, and
17. From this sketch of the front view, you can see
there are 5 layers, so there are 12  5, or 60, boxes.
that x  3  7  2, so x  12. You can also
see that y  2  2  8, so y  4.
3

5 y
7
8
2 2
2
3 4
2 2

10. 11. 3 7 2
x

18. To visualize the solid, cut out a rectangle and tape it


to your pencil. Rotate your pencil to see what shape
the rotating rectangle forms.

12.

19.
13. Pyramid with hexagonal base

20. To visualize the section, imagine slicing an orange.


What shape is revealed?
14. Prism with hexagonal base

22 CHAPTER 1 Discovering Geometry Solutions Manual


©2003 Key Curriculum Press
DG3SM586_001-024.qxd_k1 8/2/06 5:14 PM Page 23

21. 22. True 29. True

23. True

30. (3, 1)
31. Perimeter  20.5 cm; m(largest angle)  100°.
24. False. The two lines are not necessarily in the same
plane, so they might be skew. 32.

8 cm

120°

13 cm

IMPROVING YOUR VISUAL THINKING SKILLS


25. True
First, draw A, B, and C. Because AB  BC  AC,
points A, B, and C are vertices of an equilateral triangle.
C

A B

Now, where does point D go? If it is in the same plane


as A, B, and C, then it can be the same distance from
26. True any two points, but not from all three points. Try
thinking in three dimensions. Imagine placing D above
your paper so that ABD, BCD, and ACD are
equilateral triangles. If you could connect all four points,
you would get a tetrahedron, a three-dimensional figure
with four faces that are equilateral triangles. So, A, B, C,
27. True and D are vertices of a tetrahedron.
D

A C

EXTENSIONS
28. False. They divide space into eight parts. A. Here is one way to divide the solids.
Solids with all straight edges: prism, pyramid
Solids with curved edges: cylinder, cone,
sphere, hemisphere
B. To draw a cube truncated to edge midpoints, first
draw a cube. Then, mark the midpoints of each side
and connect the midpoints surrounding each face to
make triangular wedges at each corner. Finally, erase

Discovering Geometry Solutions Manual CHAPTER 1 23


©2003 Key Curriculum Press
DG3SM586_001-024.qxd_k1 8/2/06 5:14 PM Page 24

the corners of the cube (you will erase all the cube’s 27. S
lines) to make the truncated cube. This is also called 5 3
a cuboctahedron. T P
O
7

28. A 29. R T

N Y E
R
A P

I E

Draw the truncated tetrahedron in a similar way.


G
A tetrahedron truncated to edge midpoints is an
octahedron. 30. 31. A

32. A
33.

C. Answers will vary. D. Results will vary.

CHAPTER 1 REVIEW
EXERCISES C B D

1. True .
2. False. It is written as QP
34. 35.
3. True 4. False. The vertex is point D.
2 in.
5. True 6. True 125°

7. False. The measure of an acute angle is less than 90°. 5 in.


3 in.
8. False. Here are two possible counterexamples:
D C
36.
A
D
A 40°
P
P B
C

⬔APD and ⬔APC ⬔APD and ⬔APC 37. Here is one possible method. Draw a circle and
are linear angles. are the same angle. one diameter. Draw another diameter perpendicular
to the first. Draw two more diameters so that eight
9. True 10. True
45° angles are formed. Draw the regular octagon
11. False. They are supplementary. formed by connecting the endpoints of the
diameters.
12. True 13. True 14. True
15. False. A pentagon has five diagonals.
16. True 17. E 18. G 19. L 45° 45°
45° 45°
20. J 21. C 22. I 23. No match 45° 45°
24. A 25. No match 45° 45°

26. T

E Y

24 CHAPTER 1 Discovering Geometry Solutions Manual


©2003 Key Curriculum Press
DG3SM586_025-055_k4.qxd 8/2/06 5:21 PM Page 25

38. 114° 39. x  2, y  1 Shed


Shed
40. x  12, y  4 41. x  4, y  2.5
Path of flashlight
42. Find y first. y  12  20, so y  8. x  8  18,
so x  10.
43. The diagram below illustrates the given information.
Ground
3z  12, so z  4 cm. Therefore, AB  4z  16 cm.
12 cm 51. It will take 46 to 50 minutes, depending on whether
A z D z C 2z B
he can stop when he hits his house or must wait
until he is blown back to his house.
44. The diagram below illustrates the given  will be
52. If the triangle is rotated 90° clockwise, AB
 is the bisector of
information. Because BE
a vertical segment. Point B will have the same
DBC, mDBC  64°. Because BD  is the
location as it does now, and point A will be
bisector of ABC, mABD  64°. So,
at (2, 3).
mEBA  mDBE  mABD  32°  64°  96°.
53. 54.
A

E
32°

B
C
55. 56.
45. Drawing radii from each hour mark to the center of
a clock forms 12 central angles, each with measure
360°
30° 12 . The angle formed by the hands at 2:30

includes 312 of those central angles. (At 2:30, the
hour hand will be halfway between 2 and 3.) So, 57.
the angle has measure 3.5(30°), or 105°.
360°
46. 1
2  30°
47. Quadrilateral Quadrilateral

Rectangle Square Rhombus or Rectangle Rhombus Trapezoid

Square
Trapezoid
CHAPTER 2
48. The top and the bottom each need one 9-inch strip
and one 14-inch strip. The four sides each need one LESSON 2.1
5-inch strip. So, the total length needed is 2(9  14)
EXERCISES
 4(5)  66 inches.
1. “All rocks sink.” Stony needs to find one rock that
49. Draw a number line on which each tick mark will not sink.
represents 3 feet. Label the line to show each boy’s
position. The completed diagram shows that Paul 2. Possible conjecture: If two angles are formed by
was 3 feet ahead of George. drawing a ray from a line, then their measures add
up to 180°.
J R G P
3. 10,000, 100,000. Each term is 10 times the previous
50. The dashed segments in the following diagram term.
represent the ladder in various positions as it slides 4. 56, 1. Written with the common denominator of 6,
down the wall. The midpoint of each segment is the sequence becomes 16, 26, 36, 46, 56, 66, . . . .
marked. The path traced by the midpoint is an arc
of a circle or a quarter-circle if the ladder slides all 5. 17, 21. Four is subtracted from each term to get
the way from the vertical to the horizontal. the next term.

Discovering Geometry Solutions Manual CHAPTER 2 25


©2003 Key Curriculum Press
DG3SM586_025-055_k4.qxd 8/2/06 5:21 PM Page 26

6. 28, 36. To get from term to term, you add 2, then 16. The nth figure is an n-by-n grid of squares with a
add 3, then add 4, and so on. shaded circle inscribed in each square.
7. 21, 34. To find each term, you add the two previous
terms.
8. 49, 64. The terms are the squares of consecutive
whole numbers: 12, 22, 32, 42, 52, 62, . . . . The next
two terms are 72  49 and 82  64.
9. 10, 24. To get from term to term, you subtract
2, then subtract 4, then subtract 6, and so on. 17. The first term is 3(1)  2, the second term is
3(2)  2, the third term is 3(3)  2, and so on.
10. 64, 128. Each term is double the previous term.
So, the first five terms are 1, 4, 7, 10, 13.
11. Each figure has one more point than the previous 5(5  1)
18. The fifth term is 2, the sixth term is
figure. Each point in a figure is connected to each of 6(6  1)
the other points. , and so on. So, the next five terms
2
are 15, 21, 28, 36, 45.
19. Sample answer: 1, 2, 4, 8, 16, . . . . Each term
is 2 times the previous term.
20. Sample answers: 3, 6, 12, 24, 48, . . . and
12. To get the next figure, increase the number of rows 4, 8, 12, 16, 20, . . . .
by 2 and the number of columns by 1. 21. Sample answer: I learned by trial and error that
you turn wood screws clockwise to screw them into
wood and counterclockwise to remove them. This is
inductive reasoning because it involves making
generalizations from patterns.
22. The conjecture is false. Sample counterexample:
142  196 but 412  1681.
13. To get the next figure, add one row to the bottom
and one column to the right of the previous figure, 23. 24.
and then shade all the rectangles in the bottom row
but the rightmost one.

25. 26.

14. To create each figure, add two branches to each of


the new branches from the previous figure.

27. Turn your book so that the red line is vertical.


Imagine rotating the figure so that you can see faces
on both sides of the red line. Possible answers:

15. To create each figure, connect the midpoints of the


sides of each shaded triangle of the previous figure,
dividing it into four triangles, and then color the
middle triangles white.

28. Sample answer:

26 CHAPTER 2 Discovering Geometry Solutions Manual


©2003 Key Curriculum Press
DG3SM586_025-055_k4.qxd 8/2/06 5:21 PM Page 27

29. collinear 30. isosceles of the list. To generate the next term, multiply the 3
(the second term you used to find the previous
31. protractor 32. radius
product) and the next term, 6. Write the digits of
33. diagonal 34. dodecagon the product, 1 and 8, as separate terms at the end of
the list. To get the next term, multiply the 6 (the
35. parallel 36. regular
second term you used to find the previous product)
37. 90° 38. perpendicular by the next term, 1, and write the product’s digits
(in this case, just 6) at the end of the list. Continue
39. Sample answer: 40. Sample answer:
this process to get the remaining terms.
I
2, 3 → 2  3  6 → 2, 3, 6
A G 2, 3, 6 → 3  6  18 → 2, 3, 6, 1, 8
2, 3, 6, 1, 8 → 6  1  6 → 2, 3, 6, 1, 8, 6
2, 3, 6, 1, 8, 6 → 1  8  8 → 2, 3, 6, 1, 8, 6, 8
2, 3, 6, 1, 8, 6, 8 → 8  6  48 → 2, 3, 6, 1, 8, 6, 8,
N T

41. Sample answer: 42. Possible answer: 4, 8

A 
Clearly AC
2, 3, 6, 1, 8, 6, 8, 4, 8 → 6  8  48 → 2, 3, 6, 1, 8,
C
does not
6, 8, 4, 8, 4, 8
bisect A
or C.
2, 3, 6, 1, 8, 6, 8, 4, 8 → 8  4  32 → 2, 3, 6, 1, 8,
6, 8, 4, 8, 4, 8, 3, 2
2, 3, 6, 1, 8, 6, 8, 4, 8, 4, 8, 3, 2 → 4  8  32
IMPROVING YOUR REASONING SKILLS → 2, 3, 6, 1, 8, 6, 8, 4, 8, 4, 8, 3, 2, 3, 2
1. 9. The sequence is the perfect squares, in reverse
2, 3, 6, 1, 8, 6, 8, 4, 8, 4, 8, 3, 2, 3, 2 → 8  4  32
order, written backward. That is, the first term is 92,
→ 2, 3, 6, 1, 8, 6, 8, 4, 8, 4, 8, 3, 2, 3, 2, 3, 2
or 81, written backward, the second term is 82, or
64, written backward, and so on. Because the last 8. The letters in the top row are formed from straight
term listed is 42, or 16, written backward, the next segments only. The letters in the bottom row have
term must be 32, or 9, written backward. But 9 curves. Because X, Y, and Z have only straight
written backward is just 9, so the next term is 9. segments, they belong in the top row.
2. T. The terms are the first letters in the words One, EXTENSION
Two, Three, Four, and so on. The last term listed is Discussions will vary.
the first letter in Nine, so the next term must be T,
the first letter in Ten.
LESSON 2.2
3. 64. The sequence can be rewritten as 1, 22, 3, 42, 5,
62, 7. The next term must be 82, or 64. EXERCISES
1. inductive; deductive
4. 8671. To find each term, you double the previous
term and then write the digits in the reverse order. 2. mB  65°. This problem could be solved
To find the next term, double 884 to get 1768 and inductively, by drawing and measuring, or
then reverse the digits to get 8671. deductively, by subtracting 25° from 90°.
5. 18. The numbers added to or subtracted from each 3. Each term includes a figure and a number. Each
term to get the next term are consecutive primes. figure is a regular polygon with one more side than
The pattern alternates between addition and the previous polygon and contains two fewer dots
subtraction. The next term is 1  17, or 18. than the number of sides. The numbers are squares
6 8 5 10 3 14 1
of consecutive integers. The next two terms are
shown below. This solution involves observing
2 3 5 7 11 13
patterns and making generalizations, which is
6. 2. Each number is the number of “open ends” on inductive reasoning.
the previous letter. The letter G has two open ends.
25
7. 2. The steps below show how the sequence is gener- 36
ated. To get the third term, multiply the first two
terms, 2 and 3, and write the product, 6, at the end

Discovering Geometry Solutions Manual CHAPTER 2 27


©2003 Key Curriculum Press
DG3SM586_025-055_k4.qxd 8/2/06 5:21 PM Page 28

4. DG  258 cm; deductive. TD  DG  GT  756. 13. 75, 91. Add 10, then add 11, then add 12, and so on.
Let x be the length of DG. Then x is also the length 4
14. 5, 12. The terms alternate between fractions and
of TD. So, x  x  240  2x  240  756. Therefore,
integers. The integers are consecutive. The denomi-
2x  516, so x  258 cm. This solution involves
nators of the fractions are consecutive integers, and
deductive reasoning because it uses a sequence of
each numerator is one less than its denominator.
logical statements that are based on agreed-upon
assumptions and proven facts. 15. The first term is one line, the second term is two
intersecting lines, and the third term is three lines,
5. The sum of the measures of the marked angles in
each of which intersects the other two. So the fourth
star D is 180°. If this pattern continues, the sum
term is four lines, each intersecting the other three.
of the marked angles in star E will be 180°. This
solution involves inductive reasoning.
6. LNDA is a parallelogram. This conclusion involves
deductive reasoning because it is based on agreed-
upon assumptions. 16. Each term has one more row and one more column
7. a. CD  AB  3 than the previous term. The shading alternates
b. By the overlapping segments property, between “left half shaded” and “right half shaded.”
BD  AC  10.
c. By the overlapping segments property,
AC  BD  4  3  7.
8. Just over 45°. The smallest possible obtuse angle is 17. In the next term, the number of rows will increase
just over 90°. So, the smallest possible acute angle by 2 and the number of columns will increase by 1.
formed when an obtuse angle is bisected is just over In each corner, a 3-by-3 square will be shaded.
1
(90°), or 45°.
2
9. In each case, the middle digit in the product is the
number of 1’s in each factor. The digits to the left
of the middle digit are consecutive integers, from 1
up to the middle digit. The digits to the right of the
middle digit are the same as the digits on the left,
but in reverse order. So 18. Each term contains polygons with one more side
11,111  11,111  123,454,321 than the previous term, so the next term must be
hexagons. In each term the midpoints of the outer
111,111  111,111  12,345,654,321 polygon are connected, and then the midpoints of
However, in the tenth line, 1,111,111,111 is that polygon are connected to form a third polygon.
multiplied by itself. The digits can’t go up
to 10, so, carrying the 1, you get
1,111,111,111  1,111,111,111
 1,234,567,900,987,654,321
10. Sample answer: I wanted to buy a CD that cost 19. Sample answer: My friend is on the basketball team.
$15.89. I had $17, and I wasn’t sure it would be She has taken two tests the day after a game and
enough. The sales tax is 5%. I know that 10% of received A’s on both of them. She concluded that
$16 is $1.60, so 5% of $16 would be half of that, or she will get an A on any test given the day after a
$0.80. So, if the CD had cost $16, the total price game. This is an incorrect use of inductive
would have been $16.80. Because the actual cost was reasoning. The fact that the two A tests were taken
$15.89, I knew the total price would have to be less the day after a game is probably just a coincidence.
than that. So, I figured out that I did have enough The A’s also involved a lot of work and studying by
money. This is an example of deductive reasoning my friend. Using her reasoning, you would conclude
because it involves a series of logical steps, each of that no matter how much she studies for a test,
which is based on facts. even if she doesn’t study at all, she will get an A.
This is probably not true.
11. The pattern cannot be generalized because once the
river is straight, it cannot get any shorter. 20. L (Note: K looks like a kite, but without information
about the lengths of the other two sides, you cannot
12. 900, 1080. Each term is 180 more than previous term.
conclude that it is a kite.)

28 CHAPTER 2 Discovering Geometry Solutions Manual


©2003 Key Curriculum Press
DG3SM586_025-055_k4.qxd 8/2/06 5:21 PM Page 29

21. M this pattern for terms 5 and 6. The rule is 8n  c


for some number c. Because the first term is 8,
22. A (Note: O looks like a trapezoid, but without infor-
8(1)  c  8, and so c  0. Thus, the rule is 8n.
mation about whether the sides are parallel, you
The number of tiles in the 200th figure must be
cannot conclude that it is a trapezoid.)
8(200), or 1600.
23. B 24. E 25. C 26. G
Figure 1 2 3 4 5 6 n 200
27. D 28. H number
29. I (Note: The segment in J looks like an angle Number 8 16 24 32 40 48 8n 1600
bisector, but without information about whether the of tiles
angles formed are congruent, you cannot conclude
that it is an angle bisector.) 5. 4n  3; 797. Possible method: Fill in the table for
the first four terms. The difference is always 4.
30. W 31. B
Continue this pattern for terms 5 and 6. The rule is
I E 4n  c for some number c. Because the first term
140°
O
is 1, 4(1)  c  1, and so c  3. Thus, the rule is
G
L D 4n  3. The number of tiles in the 200th figure
must be 4(200)  3, or 797.
32. C 33. N
Figure 1 2 3 4 5 6 n 200
K
T number
O D
Number 1 5 9 13 17 21 4n  3 797
D I
of tiles
IMPROVING YOUR VISUAL THINKING SKILLS
6. Number of matchsticks: 4n  1; 801. Possible
Each gear will rotate in the opposite direction of the method: Fill in the table for the first four terms.
gear to its left. Gear E will rotate counterclockwise. The difference of the numbers in the “number of
matchsticks” row is always 4. Use the pattern to fill
B
in the table for Figures 5 and 6. The rule is in the
D E
F form 4n  c. Using the first term, 4(1)  c  5.
A C
Therefore, c  1, so the rule for the number of
matchsticks is 4n  1. The 200th figure has
4(200)  1, or 801, matchsticks.
EXTENSION
Discussions will vary. Matchsticks in perimeter: 3n  2; 602. Possible
method: The difference of the numbers in the
“number of matchsticks in perimeter of figure” row
LESSON 2.3
is always 3. Use this fact to fill in the numbers for
EXERCISES Figures 5 and 6. The rule is in the form 3n  c.
Using the first term, 3(1)  c  5. Therefore, c  2,
1. 6n  3; 117. Possible method: The difference between
so the rule for the number of matchsticks in the
terms is always 6, so the rule is 6n  “something.”
perimeter is 3n  2. The 200th figure has
Let c stand for the unknown “something.” The first
3(200)  2, or 602, matchsticks in its perimeter.
term, f(1), is 3, so 6(1)  c  3. Solving this
equation gives c  3. So, the rule is f(n)  6n  3. Figure number 1 2 3 4 5 6 n 200
To find the 20th term, substitute 20 for n:
Number of 5 9 13 17 21 25 4n  1 801
f(20)  6(20)  3  120  3  117. matchsticks
2. 3n  4; 56. Possible method: The difference Number of 5 8 11 14 17 20 3n  2 602
between the terms is always 3, so the rule is matchsticks
3n  c for some number c. To find c, use the in perimeter
fact that the value of the first term, f(1), is 1. So, of figure
3(1)  c  1. Solving this equation gives c  4, so
the rule is f(n)  3n  4. To find the 20th term, 7. triangles; 33
substitute 20 for n: f(20)  3(20)  4  56. Number of sides 3 4 5 6 n 35
3. 8n  12; 148 Number of 1 2 3 4 n2 33
triangles formed
4. 8n; 1600. Possible method: Fill in the table for the
first four terms. The difference is always 8. Continue

Discovering Geometry Solutions Manual CHAPTER 2 29


©2003 Key Curriculum Press
DG3SM586_025-055_k4.qxd 8/2/06 5:21 PM Page 30

8. The points for 8n (Exercise 4) lie on the steepest Extra credit


line. The coefficient of n gives a measure of the ● Student chooses one relationship where it is expected
steepness. (It is the slope of the line.) that the points will fall in a straight line, and another
f (n) where there is some scatter.
8n
● A geometric pattern is explored such as data on width
50 and area of figures, or number of figures in a sequence
40
4n  1
and figure perimeter.
30 4n  3
● A slider is used in Fathom to fit a curve to nonlinear
20 3n  2 data.
10

2 4 6 8
n LESSON 2.4

9. y  32x  3. Possible reasoning: The slope of the line EXERCISES


96 3
through the points is  
4  2 , or
, and the y-intercept 1. 2n; 70. The diagrams below show the results for one
3 2
is 3. So the equation is y  2x  3. line through four lines. Notice that the number of
regions is always twice the number of lines. So, the
x 4 2 0 2 5 function rule is 2n, and 35 concurrent lines divide
y 3 0 3 6 9 the plane into 70 regions.

10. L 11. O 2 2 3 8 1
1 4 7 2
M 1 2 1 3 6 3
4 6 5 54
T
L Y
1 line 2 lines 3 lines 4 lines
E Q 2 regions 4 regions 6 regions 8 regions

12. R E 13. 2 Lines 1 2 3 4 5 n 35


2 1 1
1 1 Regions 2 4 6 8 10 2n 70
T C
3 3
2. n  1; 36. The diagrams below show the results for
1 1
2 one parallel line through four parallel lines. Notice
1 1
2
that the number of regions is always one more than
the number of lines. Therefore, the rule is n  1,
14. Possible answer: Márisol should point out that and 35 parallel lines divide the plane into 36 regions.
although all the triangles José drew were isosceles,
1
it is possible to draw a triangle with no two 1 2
sides congruent. She should then show José a 1 2 3
counterexample. 1 2 3 4
2 3 4 5
; deductive.
15. M is the midpoint of AY 1 line 2 lines 3 lines 4 lines
2 regions 3 regions 4 regions 5 regions
16. She could try eating one food at a time; inductive.
17. mCPB  48°; mAPD  17°; mCPB  62°. Lines 1 2 3 4 5 n 35
Conjecture: If points C and D lie in the interior of
Regions 2 3 4 5 6 n1 36
APB, and mAPC  mDPB, then mAPD 
mCPB.
3. n  3; 32. From each vertex, you can
PROJECT draw a diagonal to any vertex except
Project should satisfy the following criteria: the two adjacent vertices and the
vertex itself. Therefore, if a polygon
● Topic, relationship, data, and source are clearly
has n sides, you can draw n  3
presented.
diagonals from each vertex. So, if a polygon
● Accurate graph shows a line of best fit. has 35 sides, you can draw 32 diagonals.
● Writing shows an understanding of how good or bad Sides 3 4 5 6 7 n 35
the line of fit is, and explains why.
Diagonals 0 1 2 3 4 n–3 32

30 CHAPTER 2 Discovering Geometry Solutions Manual


©2003 Key Curriculum Press
DG3SM586_025-055_k4.qxd 8/2/06 5:21 PM Page 31

n(n  3) n(n  3)
4. 2; 560. For an n-sided polygon, you can draw of diagonals of the n-sided polygon, 2, plus
n  3 diagonals from each vertex (see Exercise 3) the number of sides, n. Thus, the total number
for a total of n(n  3) diagonals. However, this of segments connecting n random points is
counts each diagonal twice. Therefore, the total n(n  3) 2n n(n  3)  2n
    
number of diagonals for an n-sided polygon is 2 2 2
n(n  3) n2  3n  2n n2  n
. So, if a polygon has 35 sides, you can
2  2  2
35(35  3)
draw 2, or 560, diagonals. n(n  1)
 2
8. 780. If each house must be connected to each of the
other houses by a direct line, then this situation is
similar to Exercise 5 (and to the handshake
n(n  1)
problem). The formula for n houses is 2. So,
Sides 3 4 5 6 7 n 35
for 40 houses, 780 lines are needed. It is more
n(n  3)
Diagonals 0 2 5 9 14  560 practical to have a central hub with a line to each
2

n(n  1) house, so that only 40 lines are needed. The


5. 2; 595. This is essentially the same as the
diagrams show the direct-line solution and the
handshake problem in the investigation. If there
practical solution for six houses.
are n points, you need to draw n  1 segments
(each point must be connected to each point but
itself) for a total of n(n  1) segments. However,
this counts each segment twice, so you must divide
by 2. Therefore, for n points, you need to draw
n(n  1) 9. 180 games are played. Possible model: First suppose
 segments. So, for 35 points, you need to
2
35(35  1) each team plays each of the other teams once. Use
draw 2, or 595, segments.
points to represent the teams and segments to
represent the games. This is the same model used in
Exercise 5 (and the handshake problem). If n teams
n(n  1)
each play each other once, 2 games are
played. If they play each other four times,
n(n  1)
Points 1 2 3 4 5 n 35
4  2, or 2n(n  1), games are played. For a
n(n  1)
Segments 0 1 3 6 10 
2 595 ten-team league, 2(10)(10  1), or 180, games are
n(n  1) played.
6. 2; 595. If there are n lines, then each line n(n  1)
10. 12. If 2  66, then n(n  1)  132. Find two
intersects each of the other n  1 lines for a total of
consecutive integers whose product is equal to 132.
n(n  1) intersection points. However, this counts Because 12  11  132, n must be 12. Thus, there
each intersection point twice, so you must divide were 12 people at the party.
n(n  1)
by 2. Therefore, for n lines, there are 2
11. True
intersection points. So, for 35 lines, there are
35(35  1) 12. True
, or 595, intersection points.
2
13. False. An isosceles right triangle has two congruent
sides.
14. False. AED and BED are not a linear pair.

E
Lines 1 2 3 4 5 n 35
n(n  1)
Intersections 0 1 3 6 10  595 B C
2
A D

7. Visualize the points in Exercises 5 and 6 as vertices 15. False. They are parallel.
of a polygon, as in Exercise 4. The total number of
16. True
segments connecting n random points is the number

Discovering Geometry Solutions Manual CHAPTER 2 31


©2003 Key Curriculum Press
DG3SM586_025-055_k4.qxd 8/2/06 5:21 PM Page 32

17. False. A rectangle is a parallelogram with all of its LESSON 2.5


angles congruent.
EXERCISES
18. False. A diagonal connects any two nonconsecutive
In Exercises 1–5, there is more than one way to find
vertices.
each answer. One possible method is described.
19. True
1. By the Vertical Angles Conjecture, a  60°. By the
20. C2H2n2. Each carbon atom has one hydrogen atom Linear Pair Conjecture, b  120° and c  120°.
above it and one hydrogen below it, plus there is
2. By the Linear Pair Conjecture, a  90° and b  90°.
always one hydrogen atom on each end of the
By the Vertical Angles Conjecture, 40°  c  90°,
chain. So, if there are n carbon atoms, there are
so c  50°.
2n  2 hydrogen atoms.
H H H H H H H H 3. By the Linear Pair Conjecture, 51°  (a  52°)
 180°, so a  77°. By the Vertical Angles Conjec-
H C C C C C C C C H
ture, b  52°, c  77°, and d  51°.
H H H H H H H H
Octane (C8H18) 4. By the Vertical Angles Conjecture, a  60°. By the
Linear Pair Conjecture, b  120° and c  120°.
IMPROVING YOUR VISUAL THINKING SKILLS Similar reasoning can be used to find the other
angles: d  115°, e  65°, f  115°, g  125°,
h  55°, i  125°.
5. By the Linear Pair Conjecture, a  90°. By the
Vertical Angles Conjecture, b  163°. By the Linear
Pair Conjecture, c  17°. By the Linear Pair Conjec-
ture, d  110° and e  70°.
EXTENSION 6. The measures of the linear pair of angles add to
Below are several examples of quadratic functions. In 170°. They should add to 180°.
each case the constant second difference is twice the
coefficient of the squared term. 7. The angles must be the same size and their meas-
ures must add to 90°, so he should cut the ends at a
y  3x 2  10 45° angle.
x 1 2 3 4 5 6 7 8. Greatest: 120°; smallest: 60°. Possible explanation:
y 7 2 17 38 65 98 137
The tree is perpendicular to the horizontal. The
angle of the hill measures 30°. The smaller angle
9 15 21 27 33 39 and the angle between the hill and the horizontal
form a pair of complementary angles, so the
6 6 6 6 6 measure of the smaller angle is 90°  30°, or 60°.
y  x 2  2 The smaller angle and the larger angle form a linear
pair, so the measure of the larger angle is
x 1 2 3 4 5 6 7 180°  60°, or 120°.
y 1 2 7 14 23 34 47 9. The converse is not true. Possible counterexample:

3 5 7 9 11 13

2 2 2 2 2 55° 125°
A
B
y  2x 2  3x  1
10. each must be a right angle
x 1 2 3 4 5 6 7

y 0 3 10 21 36 55 78

3 7 11 15 19 23

4 4 4 4 4

32 CHAPTER 2 Discovering Geometry Solutions Manual


©2003 Key Curriculum Press
DG3SM586_025-055_k4.qxd 8/2/06 5:22 PM Page 33

11. Let the measure of each of the congruent angles be is 4n  6. The perimeter of the 200th rectangle is
x. The angles are supplementary, so x  x  180°. 4(200)  6, or 806.
Therefore 2x  180°, so x  90°. Thus each angle is
a right angle. Rectangle 1 2 3 4 5 6 n 200
Perimeter of 10 14 18 22 26 30 4n  6 806
12. The ratio is always 1. The ratio does not change as rectangle
long as the lines don’t coincide. The demonstration
may convince students that the Vertical Angles 23. 3160. Possible method: This is essentially the same
Conjecture is true, but it does not explain why it n(n  1)
as the handshake problem, so use the rule 2.
is true. 80(79)
For 80 students, 2, or 3160, pieces of string are
13. P
needed.

3 24. 3160. Possible method: If there are 80 lines, then


each intersects 79 other lines, for a total of 80(79)
A 5 T
intersections. However, this counts each intersection
80(79)
14. 15. A twice, so there are actually 2, or 3160,
intersections.
O
25. 760. Possible method: There are 40 people and each
T shakes hands with 38 other people. However, simply
16. 17. multiplying 40  38 counts each handshake twice,
so you must divide by 2. Therefore the number of
40(38)
handshakes is 2, or 760, handshakes.
6 26. 21; 252. Possible method: 21 diagonals can be
drawn from each of the 24 vertices for a total of
21  24 diagonals. However, this counts each diag-
3 4 onal twice. So, the actual number of diagonals is
24(21)
18. Possible answer: All the cards look exactly as they , or 252.
2
did, so it must be the 4 of diamonds because it has 27. 35. Possible method: Let n be the number of diago-
rotational symmetry while the others do not. n(n  3)
nals, then 2  560. So, n(n  3)  1120. You
360°
19. 1
6  22.5° 20. can solve this problem by guess and check or by
rewriting the equation as n2  3n  1120  0 and
then factoring or using the quadratic formula. The
solution is n  35, so the polygon has 35 diagonals.
IMPROVING YOUR ALGEBRA SKILLS
1. The length of the first segment is 3(x  3)  20.
21. CnH2n. The number of hydrogen atoms is The length of the second segment is 2(2x  23)  30.
always twice the number of carbon atoms. Because the segments are the same length, set these
So, if there are n carbon atoms, there are expressions equal and solve.
2n hydrogen atoms. 3(x  3)  20  2(2x  23)  30
H H H H H H H
3x  9  20  4x  46  30 Apply the distributive
H C C C C C C C C property.
H H H H H H H H
3x  11  4x  16 Combine the constant
22. 4n  6; 806. Possible method: Fill in the table for terms.
the fourth rectangle. Notice that the perimeter
11  x  16 Subtract 3x from
increases by 4 with each rectangle. Use this pattern
both sides.
to fill in the values for 5 and 6. The rule is in the
form 4n  c for some number c. Using the data for 27  x Add 16 to both sides.
case 1, 4(1)  c  10, so c  6. Therefore, the rule x3 x3
2. 14

x4 x4 x4 11

Discovering Geometry Solutions Manual CHAPTER 2 33


©2003 Key Curriculum Press
DG3SM586_025-055_k4.qxd 8/2/06 5:22 PM Page 34

LESSON 2.6 7. a  64°, b 116°, c  116°, d  64°, e  108°,


f  72°, g  108°, h  72°, i  108°, j  108°,
EXERCISES k  108°, m  105°, n  79°, p  90°, q  116°,
1. By the Alternate Interior Angles Conjecture, s  72°, and t  119. a  64° by the Vertical Angles
w  63°. Conjecture, d  64° by the Alternate Exterior Angles
Conjecture, b  116° by the Linear Pair Conjecture,
2. By the Corresponding Angles Conjecture, x  90°.
c  116° by the Alternate Exterior Angles Conjecture,
3. No. By the Linear Pair Conjecture, the angle above e  108° by the Vertical Angles Conjecture, g  108°
the 122° angle has measure 58°. Because the corre- by the Alternate Exterior Angles Conjecture, i 108°
sponding angles indicated below are not congruent, by the Corresponding Angles Conjecture, j  108° by
the lines are not parallel. the Vertical Angles Conjecture, k  108° by the
Alternate Exterior Angles Conjecture, f  72° by
68°
k Corresponding the Linear Pair Conjecture, h  72° by the Alternate
angles are not Exterior Angles Conjecture, s  72° by the Linear
58° congruent.
122° ᐉ Pair Conjecture, m  105° by the Linear Pair Conjec-
ture, n  79° by the Vertical Angles Conjecture,
 to the left and label a point Q on the
4. Extend AN p  90° by the Linear Pair Conjecture, q  116° by
extension. Because AN and TU
 are parallel, the the Corresponding Angles Conjecture, and t  119°
alternate interior angles T and QAT are by the Linear Pair Conjecture.
congruent. Thus, mQAT  57°. Because AT  and 8. Possible answer: a  b  180°, so they can’t both be

NU are parallel, the corresponding angles N and right angles. Therefore, the shaded portion is not
QAT are congruent. Thus, y  mN  57°. rectangular. Students might also analyze height of
Q A N people compared with the height of the wall in the
57° y
photo.
9. By the Linear Pair Conjecture,
the angle labeled 1 at right has 56°
1
57° measure 124°. For the lines to be
T U 114°
parallel, this angle would have to
5. Yes. Extend FI to the left and label a point G have measure 114°. Therefore, the
on the extension. By the Linear Pair Conjecture, lines are not really parallel.
mHFG  65.° Because the alternate interior angles
 and FI are 10. By the Linear Pair Conjecture, the angle labeled 2
SHF and HFG are congruent, HS
below has measure 125°. For the lines marked with
parallel by the Converse of the Parallel Lines
double arrowheads to be parallel, this angle would
Conjecture. Because the corresponding angles
have to have measure 45°. Therefore, those lines are
HFG and I are congruent, the same conjecture
 and HF
 are parallel. Because quadri- not really parallel. By a similar argument, the lines
tells us that SI
marked with single arrowheads are not parallel.
lateral FISH has two pairs of parallel opposite sides,
it is a parallelogram.
45°
H S
65°
2
55° 55°

G 65° 115° 65°


F I

6. In the diagram below, some of the angles have been 11. The incoming and outgoing 45°
numbered. Because m  n, m1  67° by the Alter- angles measure 45°. The 90°
surfaces of the mirror are 45°
nate Interior Angles Conjecture. Thus, m2 is also
67°. Because 2 and 3 are alternate interior angles, parallel. Possible explanation:
m3  67°. By the Linear Pair Conjecture, z  113°. The alternate interior angles
are congruent and thus, by
the Converse of the Parallel
45°
z 3 m Lines Conjecture, the 90°
1 45°
mirrors are parallel.

67° 2 n

34 CHAPTER 2 Discovering Geometry Solutions Manual


©2003 Key Curriculum Press
DG3SM586_025-055_k4.qxd 8/2/06 5:22 PM Page 35

12. Isosceles triangles (which include equilateral PROJECT


triangles) Project should satisfy the following criteria:
13. Parallelograms that are not rectangles, squares, ● Correct equations are given for the lines:
or rhombuses y  17x  1, y   26x  2, y  35x  3,
14. 18 cm y  44x  4, y  53x  5, y  62x  6, and
15. 39° y  –7x  7.

16. The triangle moved to the left one unit. The new Extra credit
triangle is congruent to the original. ● Student creates another line design and states equa-
y tions that match the lines.
5
EXTENSIONS
A. Results will vary.
x
5 B. The angle properties of transversals crossing parallel
lines are true on a cylinder, because the cylinder can
be mapped without distortion onto the plane. To
visualize this, draw long parallel lines and a trans-
17. The quadrilateral was reflected across both axes, or versal on a sheet of paper and wrap the paper into
rotated 180° about the origin. The new quadrilateral a cylinder, making the parallel lines become circles
is congruent to the original. or helixes.
y
4

x
4

18. The pentagon was reflected across the line y  x.


The new pentagon is congruent to the original.
y

N
O yx USING YOUR ALGEBRA SKILLS 2
L
O
M EXERCISES
x
N y2  y1 80 8
1. m   
x x   
12  16  4  2
E 2 1
E L y2  y1 8  (4) 12
M 2. m   
x 2  x1   
16  (3)  13

y2  y1
3. m    1.5   8.2 9.7
x 2  x1   
0.7  5.3  4.6  2.1
19. Each of the 84 phones is connected to 83 other
phones, for a total of 83(84) lines. However, this
y2
counts each line twice, so the actual number of lines 4.  
2  (5)  3 The slope of the line through (5, 2)
83(84) and (2, y) is 3.
is 2, or 3486.
y2
20. 30 squares (one 4-by-4, four 3-by-3, nine 2-by-2,   3 Simplify the denominator.
7
and sixteen 1-by-1) y  2  21 Multiply both sides by 7.
21. (See table at bottom of page.) y  23 Add 2 to both sides.

Lesson 2.6, Exercise 21


Figure number 1 2 3 4 5 6 n 35
Number of yellow squares 2 3 4 5 6 7 n1 36
Number of blue squares 3 5 7 9 11 13 2n  1 71
Total number of squares 5 8 11 14 17 20 3n  2 107

Discovering Geometry Solutions Manual CHAPTER 2 35


©2003 Key Curriculum Press
DG3SM586_025-055_k4.qxd 8/2/06 5:22 PM Page 36

92 7
5.   
7x  3 The slope of the line through 4. 91, 105. The pattern could be rewritten as 7(1),
(x, 2) and 7, 9) is 73. 7(3), 7(5), 7(7), 7(9), 7(11). The next two terms
7 7 would be 7(13), or 91, and 7(15), or 105.
   Simplify the numerator.
7x 3
5. V, 16. The pattern alternates between numbers and
7(3)  7(7  x) Multiply both sides by 3(7  x).
letters. The letters are in reverse alphabetical order.
21  49  7x Use the distributive property. Each number is double the one before.
28  7x Subtract 49 from both sides. 6. 19, 30. Each term is the difference of the two
previous terms. The next two terms are 8  (11),
4x Divide both sides by 7.
or 19, and 11  (19), or 30.
6. The slope of the line through (0, 0) and (3, 4) is
7. S, 36. The pattern alternates between numbers and
43. One way to locate another point on the line is
letters. The letters start with A and skip 2 letters,
to start at (3, 4) and move down 4 units and right
then 3 letters, then 4 letters, and so on. The
3 units. This gives (6, 8). To find other points, you
numbers are the squares of consecutive integers,
can continue this pattern of moving down 4 units
starting with 22.
and right 3 units. Possible answer: (6, 8),
(9, 12), and (12, 16) (Any correct point will be 8. 2, 5, 10, 17, 26, 37. Here’s how to calculate the first
of the form (3p, 4p).) three terms:
7. The speed is the slope of the line. Using the 12  1  2
400  200 200
points marked, m   6  3  3  66.7 mi/hr.
 
22  1  5
8. At 6 m/sec, Skater 1 is 4 m/sec faster than Skater 2
32  1  10
at 2 m/sec.
9. 1, 2, 4, 8, 16, 32. Here’s how to calculate the first
9. A 100% grade has a slope of 1. It has an inclination
three terms:
of 45°, so you probably could not drive up it. You
might be able to walk up it. Grades higher than 211  20  1
100% are possible, but the angle of inclination
221  21  2
would be greater than 45°.
231  22  4
10. The slope of the adobe house flat roof is approxi-
mately 0. The Connecticut roof is steeper to shed 10. Look at the pattern on each face separately. The top
the snow. face appears to be rotating 90° with each term. On
the left face, the shaded square alternates between
CHAPTER 2 REVIEW the first and second row and moves down the
columns. On the right face, the figures in the four
EXERCISES squares seem to be rotating in a clockwise direction
1. Diana is using poor inductive reasoning, but she and alternating between a solid triangle and an
was probably just joking. outlined triangle.

2. Sample situation: One night my sister arrived home


15 minutes after her curfew and did not get in
trouble. She concluded that she would never
get punished as long as she wasn’t more than
15 minutes late. A few days later, she got home 11. The figures alternate between a net for a pyramid
10 minutes late and was punished. My sister used and a pyramid. The number of sides in the base
poor inductive reasoning because she based her increases by 1 with each term. Because the last
conclusion on only one observation. figure shown is a pyramid with a hexagonal base,
3. Sample situation: When Leslie found out her party the next figure will be the net for a pyramid with a
was on the same night as a home football game, heptagonal base.
she said, “Half the people I invited won’t even
show up!” Because she invited 20 people, her
brother concluded that only 10 people would show
up. On the night of the party, 18 guests arrived.
This is incorrect deductive reasoning because it is
based on a faulty assumption (that half the people
would not show up).

36 CHAPTER 2 Discovering Geometry Solutions Manual


©2003 Key Curriculum Press
DG3SM586_025-055_k4.qxd 8/2/06 5:22 PM Page 37

12. 3n  5; 55. Possible method: The difference vertex, dividing the interior into n  2 regions. So,
between f(n) values is always 3, so the rule is in in a 54-sided polygon, the diagonals would divide
the form 3n  c for some number c. Using the the interior into 52 regions.
first term, 3(1)  c  2, so c  5. Therefore,
the nth term is 3n  5 and the 20th term is
3(20)  5  55.
13. n 2, 400
4 sides 5 sides 6 sides 7 sides
n(n  1) 1 diagonal 2 diagonals 3 diagonals 4 diagonals
14. 2, 190 (This is the pattern from the hand-
2 regions 3 regions 4 regions 5 regions
shake problem from Lesson 2.4.)
n(n  1) 25. 56. From Exercise 24, you know that the diagonals
15. 2, 210
from one vertex of an n-sided polygon divide the
16. 900. Look for a pattern in the sums: interior into n  2 regions. In this case, n  2  54,
1  1  12 First 1 odd number
so the polygon has 56 sides.

134 22 First 2 odd numbers


26. a  38°, b  38°, c  142°, d  38°, e  50°,
f  65°, g  106°, and h  74°. a  38° by the
1  3  5  9  32 First 3 odd numbers Linear Pair Conjecture, b  38° by the Alternate
1  3  5  7  16  42 First 4 odd numbers
Interior Angles Conjecture, c  142° because it
forms a linear pair with the alternate interior
1  3  5  7  9  25  52 First 5 odd numbers angle of b, d  38° by the Alternate Exterior Angles
Based on this pattern, the sum of the first 30 odd Conjecture, f  65° because it is half of the angle
whole numbers is 302, or 900. corresponding to the 130° angle, e  50° by the
Linear Pair Conjecture, g  106° by the Corre-
17. 930. Look for a pattern in the sums: sponding Angles Conjecture, and h  74° by the
2  1(2) First 1 even number Linear Pair Conjecture.

2  4  6  2(3) First 2 even numbers


2  4  6  12  3(4) First 3 even numbers
CHAPTER 3
2  4  6  8  20  4(5) First 4 even numbers
2  4  6  8  10  30  5(6) First 5 even numbers LESSON 3.1
Based on this pattern, the sum of the first 30 even EXERCISES
whole numbers is 30(31), or 930. , draw a ray that is clearly longer
1. To duplicate AB
. Label the endpoint A. Open your compass
than AB
18. n 2;900. Possible method: The number of blocks in
 in the book. Then, without
to the length of AB
a stack n blocks high is the sum of the first n odd
whole numbers. From Exercise 16, the sum is n 2. changing the compass opening, place the sharp end
A stack 30 blocks high would require 302, or 900, of your compass on point A of your ray and draw
blocks. an arc that intersects the ray. Label the point of
 and EF
intersection B. Duplicate CD  in the
19. 5050. Possible method: The number of bricks same way.
in stacks like those shown is determined B
n(n  1) A
by the formula 2. A stack 100 bricks high C
D
100(101)
would have 2, or 5050, bricks. E
F

20. It passes through (10  101) squares, or 2. Duplicate segment AB  as described in the solution
111 squares. It passes through (10  100)  so that
to Exercise 1. Then, duplicate segment CD
segments, or 110 interior segments. the left endpoint is the right endpoint of the
n(n  1)
21. 2  741. Therefore, n  39. previous segment.
n(n  1) 28(27)
22. Use the formula 2. 2  378, so there is a AB CD

maximum of 378 possible intersection points. AB  CD

n(n 1)
23. 2  2926. Therefore, n  77. .
3. Draw a long ray and copy AB
24. 52. These diagrams show that for an n-sided AB

polygon, you can draw n  3 diagonals from each

Discovering Geometry Solutions Manual CHAPTER 3 37


©2003 Key Curriculum Press
DG3SM586_025-055_k4.qxd 8/2/06 5:22 PM Page 38

 twice so that the left endpoint of each


Copy EF Step 6 Draw a ray from the endpoint of the orig-
segment is the right endpoint of the previous inal ray you drew through the point where the arcs
segment. intersect.
AB EF EF

 so that its right endpoint is the right


Copy CD
endpoint of the previous segment. (This essentially
subtracts CD from the combined length of the
previous segments.)
AB EF EF

CD

The segment from the left endpoint of the first 5. Possible answer:
segment copied to the left endpoint of the last
segment copied has length AB  2EF  CD. L

AB EF EF G E

AB  2EF  CD CD
copy
4.

6. Possible answer:

1 2

Follow these steps to copy each angle:


Step 1 Draw a ray.
2

Step 2 Go back to the angle in the book. Open 1


3
your compass so that it reaches from the vertex of
that angle to the arc. 7. Possible answer: C

Step 3 Without changing the opening of your


compass, place the sharp end on the endpoint of AC BC
the ray you drew and draw a large arc.
A B
AB

Follow these steps to duplicate ABC:


Step 1 Duplicate A.
Step 4 Go back to the angle in the book. Notice Step 2 Duplicate AB  on one side of A, and
that the arc intersects the angle in two points. Open  on the other side.
duplicate AC
your compass so that it reaches from one intersec- Step 3 Connect the endpoints of the segments
tion point to the other. from the previous step. Use a compass to verify
Step 5 Go back to your drawing. Without adjusting that the resulting segment is the same length
the compass, place the sharp end on the point .
as BC
where the arc intersects the ray, and draw a small
arc that intersects the large arc. 8. Here are the steps for constructing the triangle.
Step 1 Duplicate the segment.

38 CHAPTER 3 Discovering Geometry Solutions Manual


©2003 Key Curriculum Press
DG3SM586_025-055_k4.qxd 8/2/06 5:22 PM Page 39

Step 2 From each endpoint of the duplicated numbered streets, which are all parallel, so the
segment, make an arc of the same length as lettered streets must also be parallel. The fact that P
the segment so that the arcs intersect above the is the northernmost street indicates that the lettered
segment. streets run west to east and are in reverse alphabet-
ical order from north to south. Therefore, K is
north of J. If Hyacinth is facing 24th Street with
Avenue J to her left, she must be facing west.
N
24th
W E

S
K

Step 3 Connect the endpoints of the segment with


the intersection point of the arcs to form a triangle.
J

14. An isosceles triangle is a triangle that has at least


one line of reflectional symmetry. Yes, all equilateral
triangles are isosceles.
15. New coordinates: A(0, 0), Y(5, 0), D(0, 2)
9. For Exercise 7, trace the triangle. For Exercise 8, 16.
trace the segment onto three separate pieces of patty
paper. Lay them on top of each other, and slide
them around until the segments join at the
endpoints and form a triangle.
10. Quadrilaterals and methods will vary. Possible IMPROVING YOUR ALGEBRA SKILLS
method: Draw DU . Copy Q and construct a  32, b  56, c  46, d  73. Possible method: From
COY  QUD. Duplicate DUA at point O. the diagram, b  d  129 and b  c  102. Subtracting
Construct OYP  DAU. the second equation from the first gives d  c  27.
U O
From the diagram, d  c  119. Adding the equations
A P d  c  27 and d  c  119 gives 2d  146, or d  73.
Once you know the value of d, it is easy to find the
Q D C Y
values of the other variables.

11. Possible method: Construct a daisy design using LESSON 3.2


the Circle tool. Make sure the outer circles are
connected to the center of the middle circle. EXERCISES
Connect every other intersection point to form 1. Draw AB. Open your compass to more than half
a triangle. . With the sharp end of your
the length of AB
12. a  50°, b  130°, c  50°, d  130°, e  50°, compass on one endpoint of AB, make arcs above
f  50°, g  130°, h  130°, k  155°, l  90°, and below the segment. Repeat for the other
m  115°, and n  65°. a  c  50° by the Linear endpoint. Draw a line through the points where
Pair Conjecture, b  130° by the Vertical Angles the arcs intersect.
Conjecture, d  130° by the Corresponding Angles
Conjecture, e  50° by the Linear Pair Conjecture,
f  50° by the Corresponding Angles Conjecture, A B

g  130° by the Linear Pair Conjecture, h  130° by


the Corresponding Angles Conjecture, k  155° by
the Linear Pair Conjecture, l  90° by definition,
m  115° by the Corresponding Angles Conjecture,  and then bisect
2. Bisect QD
and n  65° by the Linear Pair Conjecture. each half.
Q D
13. West. Use the given information to make a sketch.
The lettered streets are perpendicular to the

Discovering Geometry Solutions Manual CHAPTER 3 39


©2003 Key Curriculum Press
DG3SM586_025-055_k4.qxd 8/2/06 5:22 PM Page 40

3. Use one compass setting to construct two inter- Step 9 Unfold and draw a line in the crease.
secting arcs, one arc centered at each endpoint Step 10 Label the point of intersection C.
of the segment. Then, repeat the process with a
different compass setting. (All the arcs will be on For Exercise 3:
the same side of the segment.) Draw a line through This is the same as Investigation 1.
the two intersection points that intersects the
For Exercise 4:
segment.
Step 1 Do Investigation 1 to get 12CD.
Edge of the paper
Step 2 On a second piece of patty paper, trace
Original segment  two times so that the two segments form a
AB
segment of length 2AB.
Step 3 Lay the first piece of patty paper on top of
the second so that the endpoints coincide and the
shorter segment is on top of the longer segment.
 to create two segments of length
4. First, bisect CD Step 4 Trace the rest of the longer segment with
1
CD. Second, construct a segment with length 2AB. a different colored pen or pencil. That will be
2
Then, duplicate a segment with length 12CD so that the answer.
the right endpoint of the segment is the right
For Exercise 5:
endpoint of the previous segment.
Step 1 Trace AB and CD
 so that the two segments
form a segment of length AB  CD.
C D
1 CD 1 CD Step 2 Fold your patty paper so that points A and
2 2 D coincide. Crease along the fold.
AB AB Step 3 Unfold and draw a line in the crease.
2AB – 1 CD 1 CD
2 2 7. The perpendicular bisectors all intersect in one point.
5. The average of two lengths is half the sum of I
the lengths. So, construct a segment of length
AB  CD, and then bisect it.

A L

AB CD
M N

MN = 1 (AB + CD) 8. The medians all intersect in one point.


2

6. For Exercise 1: N

This is the same as Investigation 1. A


M
For Exercise 2:
L
Step 1 Draw a segment on patty paper. Label
.
it QD B

Step 2 Fold your patty paper so that endpoints Q  appears to be parallel to EF


9. Possible answer: GH ,
and D coincide. Crease along the fold. .
and its length is half the length of EF
Step 3 Unfold and draw a line in the crease.
F
Step 4 Label the point of intersection A.
Step 5 Fold your patty paper so that endpoints Q G
and A coincide. Crease along the fold.
Step 6 Unfold and draw a line in the crease.
Step 7 Label the point of intersection B. D H E

Step 8 Fold your patty paper so that endpoints A


and D coincide. Crease along the fold.

40 CHAPTER 3 Discovering Geometry Solutions Manual


©2003 Key Curriculum Press
DG3SM586_025-055_k4.qxd 8/2/06 5:22 PM Page 41

10. The quadrilateral appears to be a rhombus. 21. Possible answer: 22. Possible answer:

5 cm
E V O

R C
9 cm

D I S 5 cm

9 cm
11. Any point on the perpendicular bisector would be
equidistant from the two post offices. Therefore, any 23. C, D, E, H, I, O, X (B and K are symmetric in some
point on one side of the perpendicular bisector fonts, though not the one used).
would be closer to the post office on that side.
IMPROVING YOUR VISUAL THINKING SKILLS
1. B 2. C 3. A
Ness Station
LESSON 3.3
Umsar
Station EXERCISES
1. The answer depends on the angle drawn and the
12. The quadrilateral appears to be a parallelogram. placement of P. Possible construction:

B
F L
P

I G
T A

13. The triangles are not necessarily congruent, but 2. Construct a perpendicular from C
their areas are equal. To balance a cardboard point C through AB . Point D
triangle on the edge of a ruler, line the ruler up will be the intersection of the
with the median of the triangle. perpendicular and AB . Possible
14. One way to balance it is along the median. The two construction:
D
A B
halves weigh the same. Sample figure:
C

3. Extend OT and construct a B


perpendicular from point B
. Point U will be the
to OT
D
intersection point of the
perpendicular with OT. U
Possible construction: O T

Two altitudes fall outside


A B Ruler
the triangle (the altitudes
15. F 16. E 17. B 18. A from the vertices with acute
angles) and one falls inside.
19. D 20. C

Discovering Geometry Solutions Manual CHAPTER 3 41


©2003 Key Curriculum Press
DG3SM586_025-055_k4.qxd 8/2/06 5:22 PM Page 42

4. From the point, swing arcs on the line to end of your compass on the points where the arc
construct a segment whose midpoint is that intersects the horizontal line of the angle, and make
point. Then construct the perpendicular bisector an arc to the upper right. Put the sharp end of the
of the segment. compass on the other place the arc intersects the
angle, and make a small arc intersecting the last arc
you made. Draw the line through this intersection
and A. This will be the diagonal of your square.
Copy AL onto the diagonal. Construct the perpen-
dicular to the horizontal line through L. Label this
point B. Construct the perpendicular to the vertical
line through L and label this point E.

5. Construct perpendiculars to E L
 from points Q and R.
QR Q R
 and RE
Mark off QS 
. Connect
congruent to QR
A B
points S and E.
S E .
11. Draw AB
A B
6. To construct the perpendicular
through Q, fold the line onto P Construct perpendiculars at points A and B.
itself so that Q is on the crease.
If point Q is on the perpendicular Q
through P, then the two folds will
be the same. Otherwise, they will
be parallel. The diagram at right A B
shows the latter case.
. Label the midpoint M.
Bisect AB
7. Fold the patty paper through the point so that two
perpendiculars coincide to see the side closest to the
point. Fold again using the perpendicular of the
side closest to the point and the third perpendi-
cular; compare those sides. A M B

8. To construct the triangle: Draw a line. Mark two


points on it, and label them A and C. Construct
 congruent to
a perpendicular at C. Mark off CB  on the perpendiculars to create AD
Duplicate AM 

CA . Connect points A and B. . Connect points D and C.
and BC
: Fold
To construct altitude CD A
 onto itself so that point C is
AB D C
D
on the crease. Label the point

where the crease intersects AB C A B
point D. Draw CD. The altitude B M

 is also the perpendicular


CD
 (or, the altitude
bisector of AB
 is also the median to side AB
CD ). 12. Duplicate A. Then extend one side of A in the
9. Extend side OT. Fold OT
 onto T
other direction. Construct the perpendicular to that
itself so that point B is on the side through point A.
crease. Label the point where the
 point U.
crease intersects OT
O B

Draw BU . U Complement
of A

10. Draw a horizontal line and label a point on the line


A A
A. Construct the perpendicular to the line through
point A. Draw an arc through A. Put the sharp

42 CHAPTER 3 Discovering Geometry Solutions Manual


©2003 Key Curriculum Press
DG3SM586_025-055_k4.qxd 8/2/06 5:22 PM Page 43

13. (2n  1)(n  1); 2484 (see table at bottom of page). to form a hexagon, and draw three diagonals
connecting alternating vertices to form an
Possible method: In each term, the number of
equilateral triangle. Connect the other three
shaded triangles in each row is the same. So, the
vertices to form another triangle.
total number of triangles is the number of rows
times the number of shaded triangles in each row.

Term 1 1 row of 2
Term 2 3 rows of 3
Term 3 5 rows of 4
Term 4 7 rows of 5 18. The triangles are not congruent.

Look for a pattern. The number of rows in term 6 cm


n is 2n  1. The number of shaded triangles in 6 cm

each row of term n is n  1. So, the nth term has 40° 40°
8 cm 8 cm
(2n  1)(n  1) shaded triangles. Therefore, the
35th term has 69(36), or 2484, shaded triangles. IMPROVING YOUR VISUAL THINKING SKILLS
14. 15. F Sample answer:

I T

16. A E

EXTENSION
C D Put the straightedge on the line segment so that one end
of the line segment touches one side of the straightedge
and the other end touches the other side. Draw lines.
F
Repeat with the sides touching opposite ends.
B

17. To draw the pentagon, start with a circle. Draw


360°
five central angles measuring  
5 , or 72°. Connect
the points where the sides of the angle intersect
the circle.

LESSON 3.4
EXERCISES
1. D 2. F 3. A 4. C
To construct the hexagon, start by drawing a circle. 5. E
Without changing the setting on your compass,
mark a point on the circle and copy the length 6. Construct a perpendicular at one endpoint of the
of the radius five times around the circle. (This segment. Put the sharp end of your compass on that
is similar to the daisy designs constructed in endpoint and mark length z on the perpendicular.
Chapter 0.) Then, connect the points on the circle

Lesson 3.3, Exercise 13


Rectangle 1 2 3 4 5 6 n 35
Number of 2 9 20 35 54 77 (2n  1)(n  1) 2484
shaded triangles

Discovering Geometry Solutions Manual CHAPTER 3 43


©2003 Key Curriculum Press
DG3SM586_025-055_k4.qxd 8/2/06 5:22 PM Page 44

Connect the endpoints of the original segment and 10.


Angle
the perpendicular segment to form a triangle. Altitude bisector

z
Median
z

z 11. Preferences and explanations


. Draw an arc
7. To construct the triangle: Copy RP will vary.
with length RA centered at R. Draw an arc with
length AP centered at P. The intersection point is
vertex A. Connect points R and P to A to complete
the triangle. 12. The angle bisectors are perpendicular. Possible
: Construct the midpoint
To construct median PM explanation: The sum of the angle measures of the
, and label it M. Connect points P and M.
of AR linear pair is 180°. Therefore, the sum of half of
each angle measure must be 90°.

B
M

R P

13. If a point is equally distant from the sides of an


A P
R A angle, then it is on the bisector of an angle. This is
R P true for points in the plane of the angle. In three

8. To construct the triangle: Copy M. Duplicate ME dimensions, these points lie in a plane through the
on one side of M and MS  on the other side. angle bisector perpendicular to the plane of the
Connect the endpoints of these segments. angle. Opinions about the truth of this statement
may vary. (See the note for Exercise 13 in the
: Construct the midpoint of MS
To construct OU , Helping with the Exercises section on page 159 of
, and
and label it O. Construct the midpoint of SE the Teacher’s Edition.)
label it U. Connect points O and U.
14. Stop
E
M
S
M
M

E 15. Construct two perpendicular lines to form four 90°


U angles. Bisect each angle to form eight 45° angles.
Construct a circle centered at the vertex of the
M O S angles. Connect the points where the angles inter-
sect the circle.
9. a., b. Constructing a perpen-
dicular to a line creates G
two 90° angles. To create a N I
45° angle, bisect one of 45°
these angles. 90° 45°

c. A 90° angle and a 45° A S

angle combined form a


135° angle. L O
T
135°
45°

44 CHAPTER 3 Discovering Geometry Solutions Manual


©2003 Key Curriculum Press
DG3SM586_025-055_k4.qxd 8/2/06 5:22 PM Page 45

16. The triangles do not look congruent. To draw the LESSON 3.5
first triangle, draw an 8 cm segment with a 40°
angle on one end and a 60° angle on the other. EXERCISES
Extend the angle sides until they meet to form a 1. Draw a line intersecting the original line through
triangle. Drawing the second triangle is trickier. the point. Duplicate the angle formed by the two
You can use the fact that the angle measures of a lines to create a pair of congruent alternate interior
triangle add to 180°. Therefore, the triangle has a angles as shown below. Extend the side of the new
40° angle and an 80° angle, and the side between angle to form a line.
these angles measures 8 cm. So, draw an 8 cm
segment with a 40° angle on one end and an 80°
angle on the other. Extend the sides of the angles
until they intersect.

60°

40° 60° 40°


8 cm
2. Draw a line intersecting the original line through
8 cm
the point. Duplicate the angle formed by the two
17. lines to create a pair of congruent corresponding
angles as shown below. Extend the side of the new
A
angle to form a line.

B C
Parabola

3. Construct a segment with length z. Bisect the


a. A web of lines fills most of the plane, except a segment to get a segment of length 2z. Bisect again
U-shaped region and a V-shaped region. (The to get a segment with length 4z. Construct a square
U-shaped region is actually bounded by a section with each side of length 4z.
 were
of a parabola and straight lines. If AB
, the U would be a complete
extended to AB
parabola.)
 and half the length
b. A line segment parallel to AB
 (The segment is actually the midsegment
of AB 1z
of ABD.) 4 1z
2
z
IMPROVING YOUR VISUAL THINKING SKILLS
PDDPPPDDDDPPPPDDDDPPPDDP or
DPPDDDPPPPDDDDPPPPDDDPPD 4. Duplicate A. Mark off a segment of length x on
EXTENSION each side of the angle. This creates two sides of the
rhombus. From the endpoint of each of these sides,
Align one edge of the straightedge with one side of
draw an arc of length x. The intersection point of
the angle and draw a line along the other edge of the
these arcs is the fourth vertex of the rhombus.
straightedge. Repeat this process with the other side
x
of the angle. The point where the lines intersect is A
equidistant from the sides of the angle. The ray from x
the angle vertex through this intersection point is the x
angle bisector. x
A x

Discovering Geometry Solutions Manual CHAPTER 3 45


©2003 Key Curriculum Press
DG3SM586_025-055_k4.qxd 8/2/06 5:22 PM Page 46

5. Trapezoids and construction methods will vary. Eliminate the rays beyond where the bisectors inter-
The following method was used to construct the sect. A point within any region will be closest to the
trapezoid below. fire station in that region.
Step 1 Duplicate TR . 11. a  72°, b  108°, c  108°, d  108°, e  72°,
Step 2 Draw a perpendicular to TR  through R. f 108°, g  108°, h  72°, j  90°, k  18°, l  90°,
 on the perpendicular, placing point P
Duplicate AP m  54°, n  62°, p  62°, q  59°, and r  118°.
on point R. Label the other point A. a  72° by the Linear Pair Conjecture, b  108° by
 through point the Vertical Angles Conjecture, c  108° by the
Step 3 Draw a perpendicular to RA
 are both perpendicular Alternate Interior Angles Conjecture, d  108° by
A. (Note: This line and TR
, so they are parallel to each other.) the Corresponding Angles Conjecture, e  72° by
to RA
the Linear Pair Conjecture, f  108° by the Corre-
Step 4 Duplicate AP  on the perpendicular drawn sponding Angles Conjecture, g  108° by the Corre-
in the previous step. sponding Angles Conjecture, h  72° by the
Step 5 Connect points P and T. Corresponding Angles Conjecture, j  90° by the
Corresponding Angles Conjecture, k  18° by the
Linear Pair Conjecture, l  90° by the Alternate
P A Interior Angles Conjecture, m  54° by the Vertical
Angles Conjecture, n  62° by the Linear Pair
Conjecture, p  62° by the Corresponding Angles
Conjecture, q  59° by the Linear Pair Conjecture,
and r  118° by the Vertical Angles Conjecture.
T R
12. Z O 13. B M
6. Draw a line and construct ML perpendicular to it. R T
Swing an arc of length RA from M, and label the
D I 60°
point where it intersects the line G. From point G, R O
swing an arc to construct RG. Finish the parallelo-
gram by swinging an arc of length RA from R and 14. R E
W
swinging an arc of length GR from M. There is only
one possible parallelogram. K C
RC = KE = 8 cm
M A
IMPROVING YOUR VISUAL THINKING SKILLS
1. C 2. C 3. B

G L R USING YOUR ALGEBRA SKILLS 3


7. 1  S and 2  U by the Alternate Interior EXERCISES
Angles Conjecture. 42
   2
3  1  2  1; slope of
1. Slope of AB  
  
2  4  2
5  3  2  1. The slopes, 1 and 1,
8. The ratios appear to be the same. BC  

9. A parallelogram. In the diagram below, DAB are negative reciprocals of one another, so the
 ABC. Therefore, by the Converse of the Parallel lines are perpendicular.
  BC
Lines Conjecture, AD . Because the quadrilateral   1; slope of CD  32 1
8  5  3 . The
2. Slope of AB  
has two pairs of parallel sides, it is a parallelogram. 1
slopes, 1 and 3, are neither equal nor negative recip-
rocals of one another, so the lines are neither
D B
parallel nor perpendicular.
3. Perpendicular
32
   1   58
8  5  3 ; slope of EF  6  3
4. Slope of CD   
A C 3 1
   . The slopes are equal, so the lines
9 3
are parallel.
10. Construct the perpendicular 5. Possible answer: (8, 7) and (2, 5). The slope of
bisector of each of the three 3  (3)
  
AB  6
5  0  5 . Let (x, y) represent the coordi-

segments connecting the fire 1  y
nates of point P. The slope of PQ   
3  x . The
stations.
 and AB
slopes of PQ  must be equal. That is,

46 CHAPTER 3 Discovering Geometry Solutions Manual


©2003 Key Curriculum Press
DG3SM586_025-055_k4.qxd 8/2/06 5:22 PM Page 47

1  y
  65. To find one possible solution, set the   midpoint AD
b. Midpoint HN   1, 3. The
3  x 2
denominators equal and set the numerators equal: diagonals of a rectangle bisect each other.
3  x  5 1  y  6 11. a. Yes, the diagonals are perpendicular. Slope
  1, slope VR
OE   1.
x  8 y  7
b. Midpoint VR  midpoint OE   (2, 4). The
So, point P could be at (8, 7). To find another diagonals of OVER bisect each other.
6
point, write 65 in an equivalent form, say  
5 . c. OVER appears to be a rhombus.
1  y 6
So,   . This gives two new equations:
3  x 5
12. a. Both slopes equal 12.
3  x  5 1  y  6
b. The segments are not parallel because they
x2 y5 are coincident.
So, P could also be at (2, 5). c. Distinct

6. Possible answer: (1, 5) and (2, 12). The 13. (3, 6). Let (x, y) represent the coordinates of point
4  (1) 3 D. Because AB and CD  are parallel, their slopes are
  
slope of CD 5  (2)  7 . If (x, y)
  y  (3) 3
equal. That is,  x  7  4 . Using some algebra,
 
represents the coordinates of point P, then the slope
2y you can rewrite this equation as 3x  4y  33.
  
of PQ  
4  x . The slopes of PQ must equal the  and AD
Because AB  are perpendicular, the slope
, so
negative reciprocal of the slope of CD  .
2y 7
of AD is the negative reciprocal of the slope of AB
  3. To find one possible solution, set the y2 4
x  (3)   3 . You can rewrite this as
That is, 
4x  
denominators equal and set the numerators equal: 4x  3y  6. To find the values of x and y, solve
4x3 2y7 the system
3x  4y  33
x1 y  5
4x  3y  6
So, point P could be at (1, 5). To find another
point, write 73 in an equivalent form, say 164 . To do this, multiply the first equation by 4 and the
2 y 14 second equation by 3 and subtract:
4  x  6 . This gives two new equations:
So,   
12x  16y  132
4x6 2  y  14
12x  9y  18
x  2 y  12
25y  150
So, P could also be at (2, 12).
  1, slope IM
  4, slope TI
  4, slope y  6
7. Slope TE 3 5

EM  1; ordinary quadrilateral. No two sides have Substitute 6 for y in either equation to get x  3.
the same slope, so no sides are parallel (although So, point D has coordinates (3, 6).
  EM
TE  because their slopes are negative
reciprocals). LESSON 3.6
  1, slope AP
8. Slope TE   3, slope TA
  4, slope
4 8 EXERCISES
  ; ordinary quadrilateral. No two sides have
EP 7
2
the same slope, so no two sides are parallel. 1. Sample description: Duplicate one of the segments,
and mark arcs of the correct length from the
  1, slope RO
9. Slope KC   1, slope KR
 is endpoints. Draw sides to where those arcs meet.
3 13

undefined, slope CO  3; trapezoid. KC  and RO


have the same slope, so they are parallel. KR and M A

 have different slopes, so they are not parallel.


CO
Because quadrilateral KROC has exactly one pair
of parallel sides, it is a trapezoid.
  slope ND
10. a. Slope HA   1; slope HD  slope
6
NA  6. The opposite sides have the same
slope, so they are parallel. The adjacent sides have M A
S
slopes that are negative reciprocals of one A S
another, so they are perpendicular. Therefore, M S
quadrilateral HAND is a rectangle.

Discovering Geometry Solutions Manual CHAPTER 3 47


©2003 Key Curriculum Press
DG3SM586_025-055_k4.qxd 8/2/06 5:22 PM Page 48

2. Sample description: Duplicate O. Mark off 6. Sample description: The sum of the lengths of all
distances OD and OT on the sides of the angle. the sides of the triangle is y. Duplicate the segment
Connect D and T. of length y. Mark off a segment of length x on the
segment to account for the base of the triangle.
O D
The length of each of the other sides is half of the
yx
length that remains that is, 2. Bisect the
remaining length to find the length of each side.
Now, copy the segment of length x. From each end
yx
O
of this segment, make an arc of length 2. The
D O T point where the arcs intersect is the third vertex of
O T the triangle.
C
3. Sample description: Construct IY . Duplicate I at
A
point I and Y at point Y. Extend the sides of the y x
____
x 2
angles until they intersect. The intersection point is y x
____
point G. 2 T
x
y

I Y
7. Sample description: Draw an angle and mark off
congruent segments on the sides of the angle. Then
use a different compass setting to draw intersecting
G I
Y
arcs from the ends of those segments. The intersec-
I Y tion is the fourth vertex of the kite.

4. There are infinitely many triangles with the


same three angles. Sample description: Draw
one side with a different length than the lengths in
the book. Copy one of the angles from the given
triangle at one endpoint of the segment and
another angle at the other endpoint. Extend
the sides of the angles until they intersect. The
8. Sample description: Draw an angle and mark
intersection point is the third vertex of the
off segments of different lengths on each side
triangle.
of the angle. At the other endpoint of the longer
segment (the endpoint that is not the vertex of
the angle), swing an arc with the same length as
the shorter segment. From the other end of the
shorter segment, swing an arc the length of the
longer segment. Connect the endpoints of the
5. Sample description: Construct A and mark off the
segments to the intersection point of the arcs to
distance AB on one side of the angle. From B swing
form a quadrilateral.
an arc of length BC that intersects the other side of
A at two points. Each intersection point gives a
different triangle.
Two intersection C
points

9. Sample description: Draw a segment and draw an


angle with its vertex at an endpoint of the segment.
Mark off a distance equal to the length of the orig-
C inal segment on the other side of the angle. Draw
an angle at that point and mark off the same

A B

48 CHAPTER 3 Discovering Geometry Solutions Manual


©2003 Key Curriculum Press
DG3SM586_025-055_k4.qxd 8/2/06 5:22 PM Page 49

distance. Connect that point to the other end of 13. New coordinates: E(4, 6), A(7, 0), T(1, 2)
the original segment. y

–5
E
T

A
x
10. Sample description: Draw an angle and mark off –5

equal segments on the two sides. From the other A

endpoint of each segment (the endpoint that is not –5 T


the vertex of the angle), draw arcs with lengths E
equal to the length of the segments. Connect the
endpoints of the segments to the intersection point
14. Half a cylinder
of the arcs to form a quadrilateral.

15. Use the formula from Exercise 3, Lesson 2.4:


diagonals  n  3.
11. Answers will vary. The angle bisector lies between n  503
the median and the altitude. The order of the IMPROVING YOUR REASONING SKILLS
points is either M, R, S or S, R, M. One possible
One way to solve this problem is to draw 13 blanks to
conjecture: In a scalene obtuse triangle the angle
represent the 13 face-down cards in the pile. The top
bisector is always between the median and the
card is represented by the blank on the left, and the
altitude.
bottom card by the blank on the right. To start, spell the
mABC = 111° word “ACE,” moving one blank to the right each time
B you say a letter (moving to the right models moving the
cards to the bottom of the pile). After you have finished
R Median spelling the name of the card, write “A” (for ace) in the
A S M C next blank (in this case the fourth blank). Note that now
Altitude Angle the ace has been placed on the table, so it is no longer
bisector part of the deck.
A
12. Reflectional Rotational A C E
Figure symmetries symmetries
Now, start with the next empty blank (which represents
Trapezoid 0 0
the card that is now at the top of the pile), and spell
Kite 1 0 “TWO,” moving one empty blank for each letter, and
Parallelogram 0 2 write “2” in the next empty blank.
A 2
Rhombus 2 2
T W O
Rectangle 2 2
Continue this process being careful to count the empty
blanks only. (Once a card has been turned over, it is no
longer part of the deck.) Here are the steps. The final

Discovering Geometry Solutions Manual CHAPTER 3 49


©2003 Key Curriculum Press
DG3SM586_025-055_k4.qxd 8/2/06 5:22 PM Page 50

step shows the way the cards should be ordered for the 2. The largest circle that fits in a given triangle is the
trick to work: 3, 8, 7, A, Q, 6, 4, 2, J, K, 10, 9, 5. inscribed circle. The center of the inscribed circle is
3 A 2 the incenter of the triangle.
T H R E E

3 A 4 2
Countertop
F O U R
Sink
3 A 4 2 5 Incenter
F I V E

3 A 6 4 2 5
S I X
3. The refrigerator, stove, and sink are the vertices of a
3 7 A 6 4 2 5
triangle. The point that is equidistant from these
N S E V E three items is the circumcenter of the triangle.
Refrigerator
3 8 7 A 6 4 2 5
E I G H T

3 8 7 A 6 4 2 9 5 Circumcenter
N I N E

3 8 7 A 6 4 2 10 9 5 Stove Sink
T E N
4. The circumcenter is the center of the circumscribed
3 8 7 A 6 4 2 J 10 9 5 circle.
J A C
K

3 8 7 A Q 6 4 2 J 10 9 5
U Q
E E
N
3 8 7 A Q 6 4 2 J K 10 9 5

EXTENSION
Because the sum of the angle measures of a triangle is
180°, the measure of the third angle can easily be deter- Circumcenter

mined. Therefore, you know the measures of two angles


and an included side. These measures (ASA) determine
a triangle, so SAA must determine a triangle as well. 5. The table should be located at the circumcenter
of the triangle with vertices at the three points
EXPLORATION • PERSPECTIVE DRAWING where the classes are positioned. The planners
should find the perpendicular bisectors of two of
EXTENSIONS the sides of the triangle and locate the pie table
A., B. Drawings will vary. where these two lines intersect.
6. Construct bisectors of two of the angles. The
LESSON 3.7 intersection of the bisectors is the incenter.
Construct a perpendicular from the incenter to
EXERCISES one of the sides of the triangle. Construct a circle
1. The first-aid center must be equidistant from the centered at the incenter and with a radius equal to
three bike paths, so it should be at the incenter of the length of the perpendicular from the incenter
the triangle formed by the paths. to the triangle’s side.
Path 1
Incenter

Path 2

Path 3

50 CHAPTER 3 Discovering Geometry Solutions Manual


©2003 Key Curriculum Press
DG3SM586_025-055_k4.qxd 8/2/06 5:22 PM Page 51

7. Construct perpendicular bisectors of two of the 13. Duplicate MT. Duplicate T at point T. Duplicate
sides. The intersection point is the circumcenter. 
AT on the other side of T. Construct a line
Construct a circle centered at the circumcenter and  through point A. Make an arc of
parallel to MT
passing through one of the vertices of the triangle. length AT centered at point M. Label the point
where the arc intersects the parallel line O.
Complete trapezoid MOAT.
A O

T M

14. The measure of A is 90°. The angle inscribed in a


8. Yes, any circle with a larger radius would not fit semicircle appears to be a right angle.
within the triangle. To get a circle with a larger
A
radius tangent to two of the sides would force the
circle to pass through the third side twice.
M T

15. Possible answer: The diagonals appear to be perpen-


dicular bisectors of each other.
9. The circumscribed circle of an acute triangle does A
create the smallest circular region that contains the
triangle. However, for an obtuse triangle, the circle
with the largest side of the triangle as the diameter
T
of the circle creates the smallest circular region that
contains the triangle. 16. The points satisfy the equation x  y  9, or
y  x  9. This is a line with slope 1 and
y-intercept 9.
y

9
x+y=9

x
9

10. The circumcenter of a right triangle lies on the 17. Construct the incenter by bisecting
midpoint of the hypotenuse. the two angles shown.
11. The orthocenter of a right triangle lies on the vertex The incenter is one point on the
of the right angle. bisector of the third angle. Now,
you need to find another point.
12. The length of the midsegment is half the length of From the incenter, make arcs of
. (The midsegment also appears to be parallel
MA equal radius on the sides to the
.)
to MA left and right of the incenter.
These points are equidistant from
A
the incenter.

S
Make arcs centered at each of the points you just
constructed. The point where these arcs intersect

M H T

Discovering Geometry Solutions Manual CHAPTER 3 51


©2003 Key Curriculum Press
DG3SM586_025-055_k4.qxd 8/2/06 5:22 PM Page 52

is equidistant from the two points, concurrency is the Nagel Point. Point X below is
so it is on the angle bisector. To the Nagel Point.
create the angle bisector, draw a
line through this intersection point
and the incenter.
18. Answers should describe the process of discovering A
that the midpoints of the altitudes are collinear for
an isosceles right triangle. F E

X
19. An isosceles triangle B C
D

B. Other points of concurrency include the Gergonne


20. E 21. A 22. B 23. C Point and the Fermat Point. To construct the
Gergonne Point of a triangle, inscribe a circle.
24. D
Then, draw segments connecting each vertex with
IMPROVING YOUR VISUAL THINKING SKILLS the point where the inscribed circle is tangent to
There are several possible solutions. Here is one: the opposite side. The intersection of these
segments is the Gergonne Point.
Coin Placed on Slid to
C
Gergonne Point
First 1 → 4
D
Second 2 → 7
Third 5 → 2 E

Fourth 6 → 1
Fifth 3 → 6 A F
B

Sixth 8 → 3
To construct the Fermat Point for ABC, construct
Seventh 5 → 8 equilateral ABC on AB, equilateral ACB on
Eighth 5 , and equilateral BCA on BC
AC . Then, connect
A to A, B to B, and C to C. The intersection of
EXTENSIONS these segments is the Fermat Point.
A. The incenter of the original triangle is the ortho- C
center of the triangle formed by connecting the
excenters, called the excentral triangle.
A
B
Excenter A
Excenter

B C

Fermat Point
C B

Incenter of A
ABC

LESSON 3.8

Excenter
EXERCISES
1. The center of gravity is the centroid. She needs to
If you draw lines from each vertex of a triangle locate the incenter to create the largest circle within
to the point of tangency of the opposite excircle, the triangle.
the three lines will be concurrent. The point of

52 CHAPTER 3 Discovering Geometry Solutions Manual


©2003 Key Curriculum Press
DG3SM586_025-055_k4.qxd 8/2/06 5:22 PM Page 53

2. AM  20; SM  7; TM  14; UM  8. By the To find the center of gravity for an ordinary quadri-
Centroid Conjecture, AM  2(MO)  2(10)  20. lateral, make a copy of the quadrilateral. Draw a
SM  TM  TS  21. By the Centroid Conjecture, diagonal on one copy, and draw a different diagonal
TM  2(SM). Therefore, SM  2(SM)  3(SM)  on the other copy. On each copy, find the centroids
21. So SM  7. TM  2(SM)  14. By the Centroid of the two triangles formed by the diagonals, and
Conjecture, CM  2(UM). Because CM  16, construct a segment connecting those centroids.
UM  8. Place the two quadrilaterals on top of each other,
matching the congruent segments and angles. The
3. BG  24; IG  12. EG  GR  ER  36. By the
centroid of the quadrilateral is the point where the
Centroid Conjecture, EG  2(GR). So 2(GR) 
two segments connecting centroids of the triangles
GR  3(GR)  36. Therefore, GR  12. It is given
intersect.
that GN  GR, so GN  12. By the Centroid
C⬘
Conjecture, BG  2(GN)  24. It is given that C

IG  GR, so IG  12. D D⬘
M2 M4
4. RH  42; TE  45. By the Centroid Conjecture,
AZ  2(CZ)  28. Because RZ  AZ, RZ  28. By M1
M3
the Centroid Conjecture, RZ  2(HZ), so HZ  14.
A B A⬘ B⬘
RH  RZ  HZ  28  14  42. By the Centroid
Conjecture, TZ  30  2(EZ), so EZ  15. 9. Circumcenter
Therefore, TE  TZ  ZE  30  15  45.
10. The longest chord through P
5. The points of concurrency are the is the diameter through P.
same point for equilateral triangles The shortest chord through
because the segments are the same. P is a segment perpendicular O

6. The points of concurrency are to the diameter through P.


P
collinear. The order is circum-
center, centroid, incenter, orthocenter.
11.

A B

Circumcenter C

Centroid

Incenter B

Orthocenter
12. 2n  2. Possible method: The number of hydrogen
atoms appears to increase by 2 each time the
7. Orthocenter, incenter, centroid, circumcenter. The number of carbon atoms increases by 1, so the
order changes when the triangle becomes equilat- rule is in the form 2n  c. Using the first term,
eral, which is when the points become one. 2(2)  c  2, so c  2. Thus, the rule is 2n  2.
H H H H H H
Orthocenter
H C C C C C C C C H

H H H H H H

Incenter
13.
Centroid

Circumcenter

8. For a square, rectangle, and rhombus, the center of


gravity is at the point where the diagonals intersect.

Discovering Geometry Solutions Manual CHAPTER 3 53


©2003 Key Curriculum Press
DG3SM586_025-055_k4.qxd 8/2/06 5:22 PM Page 54

14. a  128°, b  52°, c  128°, d  128°, e  52°, Extra credit


f  128°, g  52°, h  38°, k  52°, m  38°, ● Conjecture: If in any given set of four points, one of
n  71°, and p  38°. a  128° by the Vertical the points is the orthocenter of the other three, then
Angles Conjecture, b  52° by the Corresponding each of the four points is the orthocenter of the other
Angles Conjecture and the Linear Pair Conjecture, three.
c  128° by the Corresponding Angles Conjecture
and the Linear Pair Conjecture, d  128° by the
CHAPTER 3 REVIEW
Corresponding Angles Conjecture, e  52° by the
Linear Pair Conjecture, f  128° by the Corre- EXERCISES
sponding Angles Conjecture and the Linear Pair
1. False. You use a straightedge and a compass.
Conjecture, g  52° by the Corresponding Angles
Conjecture and the Linear Pair Conjecture, h  38° 2. False. A diagonal connects any two nonconsecutive
because g and h are complementary, k  52° by the vertices.
Alternate Interior Angles Conjecture, m  38° by
3. True
the Corresponding Angles Conjecture, n  71° by
the Linear Pair Conjecture, and p  38° by the 4. True
Vertical Angles Conjecture.
5. False. Possible counterexample:
15. Construct altitudes from the two accessible vertices B
to locate the orthocenter. Through the orthocenter,
construct a line perpendicular to the southern
boundary of the property. This method will divide A C
the property equally only if the southern boundary
is the base of an isosceles triangle. 6. False. The set of all points in the plane that are
a given distance from the segment is a pair of
segments parallel to the given segment and a pair
Altitude to of semicircles connecting them. The lines can’t be a
missing vertex
given distance from a segment, because the segment
has finite length and the lines are infinite.
7. False. Possible counterexample:
D C

59(60)
16. 2  40  1730 greetings A B

IMPROVING YOUR REASONING SKILLS 8. True 9. True


Start dealing from the bottom of the deck in the opposite 10. False. The incenter does not always lie inside
direction, starting with yourself, until all cards are dealt. the triangle.
EXTENSION 11. A 12. B or K
Research results will vary. See the solutions to Extensions
13. I 14. H
A and B in Lesson 3.7 for information about the Nagel
Point and the Fermat Point. 15. G 16. D
17. J 18. C
EXPLORATION • THE EULER LINE
19. 20.
PROJECT
Project should satisfy the following criteria:
Copy
● The orthocenter of an acute triangle is inside the
triangle, that of a right triangle is on the vertex of the
right angle, and that of an obtuse triangle is outside
21. 22.
the triangle.
● Given a triangle and its orthocenter, any triangle
formed by the orthocenter and two vertices of the
original triangle has as its orthocenter the third vertex
of the original triangle.

54 CHAPTER 3 Discovering Geometry Solutions Manual


©2003 Key Curriculum Press
DG3SM586_025-055_k4.qxd 8/2/06 5:22 PM Page 55

23. Construct a 90° angle and bisect it twice. 31. 4x


A B

y y

45° D F
90° 22.5°
32.

y
24. 25. Incenter

x T
R

33. Rotational symmetry 34. Neither


26. Dakota Davis should locate the circumcenter of the 35. Both 36. Reflectional symmetry
triangular region formed by the three stones, which
is the location equidistant from the stones. 37. D 38. A
27. B 39. C 40. B
41. False. An isosceles triangle has two congruent sides.
42. True
A z C 43. False. Any non-acute triangle is a counterexample.

28. 44. False. The orthocenter is the point of intersection of


the three altitudes.
45. True
_1 z
2
46. False. Any linear pair of angles is a counterexample.

y y x
47. False. Each side is adjacent to one congruent side
_1 z and one noncongruent side, so two consecutive
Segment 2
sides may not be congruent.
48. False. Possible counterexample:
29. 5x
P R

3x 4x
49. False. The measure of an arc is equal to the measure
of its central angle.
Q 50. False. TD  2DR
30. mA  mD. 2y 51. False. A radius is not a chord.
B D
You must first
find B. mB 52. True
 180°  2(mA) 53. False. Inductive reasoning is the process of
observing data, recognizing patterns, and making
2y generalizations about those patterns.
54. This is a paradox. If you answer true, you will make
D the answer false because there will then be four true
B A statements in Exercises 41–53. If you answer false,
A then the statement will be true.

Discovering Geometry Solutions Manual CHAPTER 3 55


©2003 Key Curriculum Press
DG3SM586_056-096.qxd 8/2/06 5:48 PM Page 56

55. a. 2 and 6 or 3 and 5 b. Measures of the acute angles change, but you
b. 1 and 5 may notice or deduce that their sum remains 90°.
c. 138° 2. 73°. x  52°  55°  180°, so x  107°  180°, and
x  73°.
56. 55. To generate successive terms, you subtract 3,
then you subtract 6, then 9, then 12, and so on. 3. 60°. All three angles of the triangle are congruent,
so 3v  180°. Therefore, v  60°.
57. Possible answer:
4. 110°. Ignore the 100° angle and the line that
58. a. Yes intersects the large triangle. Find the angle measures
b. If the month has 31 days, then the month for the large triangle: The supplement of the 120°
is October. angle measures 60°, and the supplement of the 130°
c. No angle measures 50° (Linear Pair Conjecture). Then,
by the Triangle Sum Conjecture, the measure of the
59. third angle is 180°  (60°  50°)  70°. Because
z  70°  180°, z  110°.
Q
5. 24°. Let x represent the measure of each of the
marked congruent angles. Then x  x  48°  180°,
so 2x  132°, and x  66°. Now, to find w, look at
the right triangle in which the measures of the
acute angles are x and w.
w  x  90°  180°
w  66°  90°  180°
Q
w  156°  180°
60. 3n  4; 56 w  24°
61. n2  1; 399 6. 900°. At each vertex of the triangle, four angles are
62. a  38°, b  38°, c  142°, d  38°, e  50°, formed, the sum of whose measures is 360°. The
f  65°, g  106°, and h  74°. a  38° by the three (interior) angles of the triangle are unmarked,
Linear Pair Conjecture, b  38° by the Alternate so the sum of their measures, which is 180°, must
Interior Angles Conjecture, c  142° by the Linear be subtracted from the total. Therefore, the sum of
Pair Conjecture, d  38° by the Alternate Exterior the measures of the marked angles is 3  360° 
Angles Conjecture, f  65° by the Corresponding 180°  900°.
Angles Conjecture, e  50° by the Linear Pair 7. 360°. At each vertex of the triangle, a linear pair is
Conjecture, g  106° by the Corresponding formed by a marked angle and an interior angle of
Angles Conjecture, and h  74° by the Linear the triangle. Because the sum of the measures of the
Pair Conjecture. angles in a linear pair is 180°, the sum of the meas-
63. Triangles will vary. Check that the triangle is ures of the marked angles is 3  180°  180° 
scalene and that at least two angle bisectors have 540°  180°  360°.
been constructed. 8. a  69°, b  47°, c  116°, d  93°, and e  86°.
64. mFAD  30° so mADC  30°, but its vertical First look at the large triangle that contains angles
angle has measure 26°. This is a contradiction. with measures a, 40°, and 71°. By the Triangle Sum
Conjecture, a  40°  71°  180°, so a  69°. The
65. Minimum: 101 regions by 100 parallel lines; angle with measure b forms a linear pair with the
maximum: 5051 regions by 100 intersecting, 133° angle, so b  133°  180°, and b  47°. Next
nonconcurrent lines. look at the triangle that includes angles with meas-
ures a and b. By the Triangle Sum Conjecture, the
measure of the unmarked angle in this triangle is
CHAPTER 4 180°  a  b  180°  69°  47°  64°. The angle
with measure c forms a linear pair with the 64°
LESSON 4.1 angle, so c  180°  64°  116° (Linear Pair
Conjecture). Now look at the triangle that includes
EXERCISES
angles with measures d and 47°. The unmarked
1. a. The angle measures change, but the sum remains angle in this triangle forms a linear pair with the
180°.

56 CHAPTER 4 Discovering Geometry Solutions Manual


©2003 Key Curriculum Press
DG3SM586_056-096.qxd 8/2/06 5:48 PM Page 57

140° angle, so its measure is 180°  140°  40°. a linear pair with the angle of measure t has
Therefore, d  47°  40°  180° (Triangle Sum measure 180°  n  68°  180°  50°  68°  62°,
Conjecture), so d  93°. Next look at the triangle so t  180°  62°  118°. Now look at the triangle
containing the 40° angle. One of its angles is a that includes the angle with measure s and an angle
vertical angle of the angle with measure d, so that forms a linear pair with the angle of measure t.
the measure of this angle is 93° (Vertical Angles Because s  78° and 180°  t  62°, the third angle
Conjecture). The measure of the third angle of this of this triangle measures 180°  78°  62°  40°.
triangle is 180°  40°  93°  47° (Triangle Sum Finally, look at the right triangle that includes the
Conjecture). Finally, look at the small triangle at angle with measure u. The other acute angle in this
the top of the figure that includes the angle with triangle measures 40° (Vertical Angles Conjecture),
measure e. The angle in the lower left measures 47° so 90°  40°  u  180°, and u  50°.
(vertical angle of angle of measure b or supplement
10. Draw a line with a straightedge and copy A with
of 133° angle), and the angle in the lower right
one side along this line. Copy R so that angles
measures 47° (vertical angle of angle found in
R and A have a common vertex and a common
the triangle containing the 40° angle). Therefore,
side that is not along the original line. The side
e  47°  47°  180° (Triangle Sum Conjecture),
of R that is not common with A and the ray
so e  86°.
opposite the one that forms the first side of A will
9. m  30°, n  50°, p  82°, q  28°, r  32°, form M because mM  mR  mA  180°.
s  78°, t  118°, and u  50°. First look at the
right triangle that contains a 60° angle. Here,
R
m  60°  90°  180° (Triangle Sum Conjecture),
so m  30°. Now look at the right triangle that
contains a 40° angle. Here, by the Triangle Sum M A
Conjecture, the unmarked angle measures 180° 
90°  40°  50°. Because this angle and the angle This construction could also be done by
with measure n are vertical angles, n  b (Vertical constructing the given angles at the ends of a line
Angles Conjecture), so n  50°. Now look at the segment; then M is the third angle formed where
small triangle with three unmarked angles, which is the sides of the angles meet.
below the triangle that includes the angle with
measure m. In this triangle, one angle measures 11. Draw a line with a straightedge and copy L with
180°  112°  68° (Linear Pair Conjecture), and one side along this line. Bisect the angle that forms
another measures 30° because it is a vertical angle a linear pair with L. Either of the two congruent
with the angle of measure m (Vertical Angles Conjec- angles formed can be used as G.
ture). Therefore, the third angle in the small triangle
G
measures 180°  68°  30°  82°. Because this
angle is a vertical angle with the angle of measure p, L
p  82° (Vertical Angles Conjecture). Next look at
the triangle that includes angles with measures p 12. First construct E, using the method used in Exer-
and q. The unmarked angle in this triangle meas- cise 10. Then copy AE and A. Extend the sides of
ures 70° (Vertical Angles Conjecture), so 82°  q  A and E that are not along AE  until they inter-
70°  180° (Triangle Sum Conjecture), and q  28°. sect. The intersection point of these two rays will be
Now look at the triangle that contains the angle R, the third vertex of the triangle.
with measure n, the angle that forms a linear pair E
with the angle of measure p, and the vertical angle
of the angle with measure r. The angle measures in
this triangle are 50°, 180°  p  98°, and r. 50°  A R
98°  r  180°, so r  32°. Next look at the large
13. For Exercise 10: Use the method described for Exer-
triangle that includes the angles with measures n
cise 10, but copy A and R by tracing them onto
and q. In this triangle, the measure of the unmarked
patty paper rather than using compass and straight-
angle is 180°  n  q  180°  50°  28°  102°
edge. No folding is necessary.
(Triangle Sum Conjecture), so s  78° (Linear Pair
Conjecture). Now look at the triangle that includes
the angle with measure n and the angles that form R
linear pairs with the 112° angle and the angle with
measure t. The supplement of the 112° angle meas- M A
ures 68°. Thus, the angle in this triangle that forms

Discovering Geometry Solutions Manual CHAPTER 4 57


©2003 Key Curriculum Press
DG3SM586_056-096.qxd 8/2/06 5:48 PM Page 58

For Exercise 11: Use the method described for Exer- 20. False. The triangles will be the same shape, but may
cise 11, but copy L by tracing it onto patty paper not be the same size.
and then bisect the angle that forms a linear pair
with L by folding the patty paper.

Fold
21. True
G
L
22. Use inductive reasoning to find a pattern.
5 8

For Exercise 12: Use the method described for Exer- 2, 7, 15

cise 12, but copy the side and two angles by tracing
The number of cards added when a new story is
them onto patty paper. No folding is necessary.
added to the tower increases by 3 each time, so a
four-story house will need 15  11  26 cards, and
R E a five-story house will need 26  14  40 cards.
Complete the table, stopping with the largest
E A
A R number of cards that is less than 2  52  104.
Number of
14. By the Triangle Sum Conjecture, mA  mS  stories 1 2 3 4 5 6 7 8
mM  180°. Because M is a right angle, Number of
mM  90°. By substitution, mA  mS  cards 2 7 15 26 40 57 77 100
90°  180°. By subtraction, mA  mS  90°.
So two wrongs make a right! Thus, the tallest house you can build with two
15. You know from the Triangle Sum Conjecture that 52-card decks is eight stories. It will take 100 cards
mA  mB  mC  180°, and mD  to build this house.
mE  mF  180°. Thus, mA  mB  x  IMPROVING YOUR VISUAL THINKING SKILLS
180° and mD  mE  y  180°, so by the tran-
1. If you are having difficulty, see if you can divide the
sitive property, mA  mB  x  mD 
hexagon into two congruent parts. Each part is a
mE  y. Because the figures show that mA 
right trapezoid.
mD and that mB  mE, subtracting equal
terms from both sides results in the equation x  y,
that is, mC  mF.
16. For any triangle, the sum of the angle measures is
180° by the Triangle Sum Conjecture. Because the
triangle is equiangular, each angle has the same
2. Start by dividing the hexagon into two congruent
measure, say x. So x  x  x  180°, and x  60°.
parts; then divide each part into four congruent
17. False. The angles between the parts. Each part is a trapezoid with three congruent
two given sides may differ, sides.
which will result in triangles
that differ in both size and
shape.
18. False. The triangles will be the same shape, but may
not be the same size.
LESSON 4.2
EXERCISES
1. 79°. Because mH  mO  mT  180° and
19. False. The other sides and angles of the triangle mT  22°, mH  mO  180°  22°  158°.
may differ. By the Isosceles Triangle Conjecture, mH  mO.
Therefore, 2mH  158°, and mH  79°.
2. 54°. By the Isosceles Triangle Conjecture,
mD  mO, so mD  63°. Then
mG  180°  (2  63°)  54°.

58 CHAPTER 4 Discovering Geometry Solutions Manual


©2003 Key Curriculum Press
DG3SM586_056-096.qxd 8/2/06 5:48 PM Page 59

3. 107.5°. By the Triangle Sum Conjecture, mS  triangle containing angles with measures b and c is
mSLO  35°  180°. By the Isosceles Triangle also 72°. Thus, b  c  72°  180° or 36°  c 
Conjecture, mS  mSLO. Therefore, 2mSLO  72°  180°, so c  72°. Therefore, this triangle has
35°  180°, so 2mSLO  145° and mSLO  two congruent angles and is thus also isosceles. The
72.5°. SLO and OLE form a linear pair, so, by angle with measure d forms a linear pair with a 72°
the Linear Pair Conjecture, mOLE  180°  angle, so d  108°. The measure of the third angle
72.5°  107.5°. in the triangle containing the angles of measures d
and e is 180°  72°  c  180°  72°  72°  36°.
4. mR  44°, RM  35 cm. Use the Triangle Sum Therefore, e  180°  36°  d  180°  36° 
Conjecture to find mR  180°  (2  68°)  44°. 108°  36°. Thus, this triangle has two 36° angles
By the Converse of the Isosceles Triangle and is also isosceles.
Conjecture, ARM is isosceles with RA  RM
.
Therefore, RM  RA  35 cm. Every triangle in the design is isosceles. (In other
words, none of the triangles is not isosceles.)
5. mY  76°, RD  3.5 cm. mY  mR  180° 
28°  152° and mR  mY, so mY  76°. 9. a. Yes. Two sides are radii of a circle. Radii must
RD  YD  3.5 cm. be congruent; therefore, each triangle must be
isosceles.
6. mD  72°, MD  10 cm. mM  mD 
180°  36°  144° and mM  mD, so b. 60°. If the vertex angle of an isosceles triangle
mD  72°. UD  UM  14 cm, so MD  measures 60°, each of the base angles will
38 cm  (2  14 cm)  10 cm. measure 12(180°  60°)  12(120°)  60°. In
other words, this isosceles triangle is equilateral.
7. a  124°, b  56°, c  56°, d  38°, e  38°,
f  76°, g  66°, h  104°, k  76°, n  86°, and 10. GEA  NCA. From the information given in
p  38°. a  56°  180° by the Linear Pair Conjec- the figure, the correct correspondence of vertices is
ture, so a  124°. b  56° by the Vertical Angles G to N, E to C, and A to A, and all pairs of corre-
Conjecture. The angle with measure c and the angle sponding parts are congruent, so GEA  NCA.
marked 56° are alternate interior angles formed 11. JAN  IEC. Notice that mN  40° and
when the parallel lines are cut by a transversal, so mI  50°. From the information given in the
c  56° by the AIA Conjecture. Now look at the figure, the correct correspondence of vertices is J
large triangle with angle measures 66°, d, and 2e. to I, A to E, and N to C, and all pairs of corre-
Here, 66°  d  2e  180°, but d  e, so 66°  sponding parts are congruent, so JAN  IEC.
3d  180°, and d  e  38°. Now look at the tall
triangle on the left: 66°  38°  f  180°, so f  12. Possible answer: The three angles F

76°. Next look at the isosceles triangle containing of ABC will be congruent to the
the angle with measure d. Because d  38°, the corresponding angles of DEF, D E

other base angle (not labeled) also measures 38°. but the corresponding sides should C

Then 76°  38°  g  180°, so g  66°. Also, from not be congruent. To construct
the isosceles triangle, h  180°  (2  38°)  104°. ABC, copy D to make A, and A B

The angles with measures k and h form a linear copy E to make B, but make
pair, so k  180°  104°  76°. d  c  n  180°, AB  DE.
so n  180°  38°  56°  86°. d and p are the 13. Possible answer: Copy MN to make
K2
measures of corresponding angles, so p  d by the 
GH and copy M to make G. K1
CA Conjecture; thus, p  38°. Draw an arc with center at H,
8. a  36°, b  36°, c  72°, d  108°, e  36°; none. intersecting the ray that forms the G H

There are 10 congruent central angles at the center side of G that does not lie along
. This arc will intersect the ray in two points,
GH
of the star decagon, and a is the measure of one of
these angles, so a   360° giving two possible positions for K, which are
10  36°. Because b  a, b 

36°, and the triangle containing the angles with labeled as K1 and K2.
measures a and b is isosceles by the Converse of the GHK2  MNP, so choose K1 P
Isosceles Triangle Conjecture. Now look at the for K, and complete the triangle.
smaller isosceles triangle in which the angle of M N
measure a is the vertex. (You also know that this K
triangle is isosceles by the Converse of the Isosceles
Triangle Conjecture.) Here, the measure of each G H
base angle is 12(180°  36°)  12(144°)  72°. This
tells you that the measure of the third angle in the

Discovering Geometry Solutions Manual CHAPTER 4 59


©2003 Key Curriculum Press
DG3SM586_056-096.qxd 8/2/06 5:48 PM Page 60

14. Possible answer: Construct a Fold 1 Fold 3 21. Move each point of the original triangle to the right
60° angle (one angle of an Fold 2
5 units and down 3 units to obtain the new
equilateral triangle) and Fold 4
triangle. Original triangle: (1, 0), (3, 2), (2, 3).
a 45° angle (bisect a right 105° 60° New triangle: (6, 3), (2, 5), (3, 0). The original
angle). 60°  45°  105°, so 45° triangle was translated (moved to the right and
you obtain a 105° angle. downward), but its size and shape were not
changed, so the two triangles are congruent.
15. Perpendicular
y
  0  3 3 3
  
Slope AB 6  1  5  5
  2  3 5 5
  
Slope CD 1  4  3  3
(–2, 3)

The slopes, 5 and 53, are negative reciprocals of


3 (3, 0)
x
each other, so by the perpendicular slope property, (1, 0)
 and CD
AB  are perpendicular. (–3, –2) (6, –3)

16. Parallel (2, –5)

   61 5
 
Slope FG 1  (4)  3 22. Reflect each point of the original triangle over the
  5.
From Exercise 15, slope CD x-axis to obtain the new triangle. Original triangle:
3
The slopes are equal, so by the parallel slope (3, 3), (3, 1), (1, 5). New triangle: (3, 3),
 and CD
property, FG  are parallel. (3, 1), (1, 5). The original triangle has been
flipped over the x-axis, but its size and shape were
 is a vertical line because the
17. Parallel. AD not changed, so the two triangles are congruent.
x-coordinates of points A and D are equal. CH  is
y
also a vertical line because the x-coordinates of
points C and H are equal. Any two vertical lines (–1, 5)
 and CH
are parallel, so AD  are parallel. (Note that (3, 3)
slopes cannot be used here because the slope of a (–3, 1)
vertical line is undefined.) x
(–3, –1)
18. Neither (3, –3)

8  (2)
   10 5
  
3  1  4  2
Slope DE (–1, –5)

  4  6 10


 
Slope GH 4  (1)  5  2 IMPROVING YOUR REASONING SKILLS
The slopes are neither equal nor negative First notice which digits are missing, and find what
 and GH
reciprocals, so DE  are neither parallel number they have to equal when combined.
nor perpendicular.
1. 1  2  3  4  5  6  78  9  100
 and CD
19. Parallelogram. In quadrilateral FGCD, FG 
 and FD
are opposite sides, and GC  are opposite 2. 1  2  3  4  5  6  7  8(9)  100
  CD
sides. From Exercise 16, FG . Find the slopes
3. 1  2  ((3)(4)(5)  6)  78  9  100
of the other two sides:
36 3 3 4. ((1  2  3  4)  5)  6  7  89  100
   
Slope GC 4  (1)  5  5 5. 1  23  4  56  7  8  9  100
 2  1 3 3
  
Slope FD 1  (4)  5  5 Many other identities using all nine digits in order are
  FD
Because their slopes are equal, GC . possible, including 12  34  (5)(6)  7  8  9  100,
123  4  5  6  7  8  9  100, and 1  2 
Both pairs of opposite sides of FGCD are parallel,
3  4(5)(6)  7  8  9  100.
so FGCD is a parallelogram.
20. 40. From the figure, you can see that point P will
EXTENSIONS
land at 20 on the number line after one cycle and at A. The conjecture is false; any scalene triangle is a
40 after two cycles. counterexample. It is possible to divide any triangle
into a kite and two triangles, but the triangles will
be isosceles only if the original triangle is isosceles.
P
(Recall that equilateral triangles are isosceles.)
0 8 20 28 40

60 CHAPTER 4 Discovering Geometry Solutions Manual


©2003 Key Curriculum Press
DG3SM586_056-096.qxd 8/2/06 5:48 PM Page 61

This extension can be explored with geometry soft- 2. The line with equation y  43x  4 is in the form
ware or with compass-and-straightedge or patty- y  mx  b with m  43 and b  4, so its graph
paper constructions. Here is one possible approach is the line with slope 43 and y-intercept 4. To graph
using compass and straightedge: Starting with this line, first use the y-intercept to plot the point
PQR, to construct the kite, make an arc to find (0, 4). From this point, use the slope to move to
two points, S and T, one on each side of P, that the right 3 units and up 4 units to reach the point
are equidistant from point P. Connect these two (3, 8). Draw the line through these two points.
points with ST and then construct the perpendi- y
. U is the point where the
cular bisector of ST
(3, 8)
perpendicular bisector intersects the third side of
. Draw SU
the triangle, QR  and TU . By construc-
(0, 4)
tion, PS  PT and SU  TU, so PSUT is a kite.
The figures below show this construction for an x
isosceles triangle (using the vertex angle of the
triangle as one vertex of the kite) and a scalene 3. To find the slope and y-intercept of the line, rewrite
triangle. Observe that the two smaller triangles the given equation in the form y  mx  b by
that are formed are each isosceles when the solving it for y.
original triangle is isosceles, but are scalene
when the original triangle is scalene. 2y  3x  12
2y  3x  12
3
P y  2x  6
The slope is 32 and the y-intercept y
is 6. To graph this line, first use the
y-intercept to plot the point (0, 6). (2, 9)
S T
From this point, use the slope to (0, 6)
move to the right 2 units and up
3 units to reach the point (2, 9).
U Draw a line through these two x
Q R
points.
4. y  x  2. Find the y-intercept and then the
Q
U
R slope. From the point (0, 2), the y-intercept is 2.
The second point shown is (4, 2), so the slope
2  2 4
4  0  4  1. Therefore, the equation of
S
P
T is   
the line in point-slope form is y  x  2.
5. y  163 x  7143 . First use the two points shown
B. See the solutions to Take Another Look activities 2 on the graph to find the slope.
and 3 on page 74. 28 6 6
Slope    
8  (5)  13  13

USING YOUR ALGEBRA SKILLS 4 The slope between any point (x, y) and either of
the given points must also be 163 . Choose the
EXERCISES point (8, 2).
1. The equation y  1  2x is in the form y  a  bx y2 6 6
with a  1 and b  2, so its graph is the line with     
x8 13 13
slope 2 and y-intercept 1. To graph this line, first 13(y  2)  6(x  8)
use the y-intercept to plot the point (0, 1). From
this point, use the slope to move to the right 1 unit 13y  26  6x  48
and down 2 units to reach the point (1, 1). Draw 13y  6x  74
the line through these two points. 6 74
y
y  1x 
3  13
Once you find the slope, an alternative method for
(0, 1) finding the equation is to use the point-slope form,
(1, –1)
x y  y1  m(x  x1), with either of the two points.

Discovering Geometry Solutions Manual CHAPTER 4 61


©2003 Key Curriculum Press
DG3SM586_056-096.qxd 8/2/06 5:48 PM Page 62

As before, choose the point (8, 2) so that x1  8 3  11 4  6


coordinates of the midpoint are  2 , 2 
 
and y1  2. (7, 5). Now find an equation of the line with
6 slope 3 that passes through the point (7, 5).
y  2  1(x
3  8)
6 48 y5
y  2  1x    3
3  13 x7
6
y  1x
48 26
  y  5  3(x  7)
3  13  13
6 74 y  5  3x  21
y  1x 
3  13
y  3x  26
6. y  x  1. First find the slope of the line through
42 2 11. y  14x  3. The slope of the line y  4x  5 is
3  1  2  1.
the points (1, 2) and (3, 4), which is   
The slope between any point (x, y) and either 4, so the slope of any line perpendicular to it will
of the given points must also be 1. Choose the be the negative reciprocal of 4, which is 14. The
point (1, 2). required line passes through the point (0, 3),
so its y-intercept is 3. Thus, the equation is
y 2
  1 y  14x  3.
x1
y  2  1(x  1) .
12. y  65x. O is the midpoint of HY
y2x1 y
10 Y(2, 9)
yx1
O(5, 6)
7. y  3x  5. First find the slope of the line
through the two points (1, 2) and (3, 4), which H(8, 3)
4  2 6
is 3  1  2  3. The slope between any
 
x
point (x, y) and either of the given points must W(0, 0) 10

also be 3. Choose the point (1, 2). Find the coordinates of O:  28 93
2 , 2   (5, 6).
 
y 2 Now find the equation of the line containing the
  3
x1 median WO , which is the line through (0, 0) and
y  2  3(x  1) 60 6
5  0  5 . Because
(5, 6). The slope of this line is   

y  2  3x  3 this line passes through the origin, the y-intercept


is 0. Therefore, the equation of the line is y  65x.
y  3x  5
 passes
13. y  x  1. The perpendicular bisector of HY
8. y   85. First find the slope of the line through
2
x
5 
through the midpoint of HY and is perpendicular
the given points (1, 2) and (6, 4), which is  is O,
to it. From Exercise 12, the midpoint of HY
4  (2) 2 2 which has coordinates (5, 6).
    . Use the point-slope form,
6  (1) 5 5
y  y1  m(x  x1), with m  25, x1  1, and y
10 Y(2, 9)
y1  2.
2
y  2  5(x (1)) O(5, 6)

2
y  2  5(x  1) H(8, 3)

2 2 x
y  2  5x  5 W(0, 0) 10

2 8  is  93 6
y  5x  5 2  8  6  1, so the
The slope of HY  
 is the negative
slope of a line perpendicular to HY
9. y  80  4x. The set-up fee is a fixed cost, so if this reciprocal of 1, which is 1. Find the equation of
equation were graphed, 80 would be the y-intercept. the line with slope 1 that passes through (5, 6).
The cost of the T-shirts varies with the number of y6
T-shirts, so 4 is the slope.   1
x5
10. y  3x  26. First find the midpoint of the y6x5
segment with endpoints (3, 4) and (11, 6). The yx1

62 CHAPTER 4 Discovering Geometry Solutions Manual


©2003 Key Curriculum Press
DG3SM586_056-096.qxd 8/2/06 5:48 PM Page 63

 is
14. y  29x  493 . The line containing altitude HT 7. b, a, c. The lengths of the sides, from longest to
the altitude to YW .  shortest, are 12 cm, 9 cm, 5 cm, so the order of the
y
angles opposite those sides is the same: b, a, c.
10 Y(2, 9) 8. a, c, b. The lengths of the sides, from longest to
shortest, are 28 in., 17 in., 15 in., so the order of the
T angles opposite those sides is the same: a, c, b.
H(8, 3) 9. a, b, c. There are two isosceles triangles in the
x figure. In the larger triangle, the vertex angle meas-
W(0, 0) 10
ures 30°, so each base angle measures 12(180° 
 is  90 9 30°)  75°. You can see by referring to the isosceles
2  0  2 , so the slope of a
The slope of YW  
 is 2. triangle on the right that the length of the base
line perpendicular to YW 9 of the triangle on the left is b. By the Side-Angle
Find an equation for the line passing through Inequality Conjecture, you know that a  b because
H(8, 3) with slope 29. 75° > 30°. In the smaller triangle, one base angle
y3 2 2 measures 72°, so the other base angle must also
    
x8 9 9 measure 72°, and therefore the vertex angle meas-
9(y  3)  2(x  8) ures 180°  72°  72°  36°. Apply the same
conjecture to the triangle on the right: Because
9y  27  2x  16 72°  36°, you know that b  c. Thus, a  b  c,
2 43
y  9x  9 so the correct order is a, b, c.
10. v, z, y, w, x. Apply the Side-Angle Inequality Conjec-
IMPROVING YOUR REASONING SKILLS
ture first to the larger triangle and then to the
One of many possible solutions: Fill the 4-liter smaller one, noticing that the two triangles share a
container. Dump into the 9-liter container. Repeat. side. In the larger triangle, the third angle measures
Then fill the 4-liter container and dump into the 9-liter 122°. Therefore, the angle measures in this triangle,
container until the 9-liter container is full (1 liter). from largest to smallest, are 122°, 30°, 28°, and the
Three liters will be left in the 4-liter container. order of the side lengths, from largest to smallest, is
v, z, y. Now look at the smaller triangle. Here, the
LESSON 4.3 third angle measures 104°, so the angle measures
from largest to smallest are 104°, 42°, 34°, and the
EXERCISES order of the side lengths from largest to smallest is
1. Yes. The sum of the lengths of any two sides is y, w, x. Putting together the results from the two
greater than the length of the third side. triangles, the order of all the side lengths in the
figure is v, z, y, w, x.
2. No. 4  5  9, so the sum of the two shorter sides
is not greater than the length of the longest side. 11. 6  length  102. The third side must be greater
4 5 than 54  48  6 and less than 54  48  102, so
9 that the sum of the lengths of any two sides will be
greater than the length of the third side.
3. No. 5  6  11  12
12. By the Triangle Inequality Conjecture, the sum of
5 6
12
11 cm and 25 cm should be greater than 48 cm,
but 11  25  36 cm.
4. Yes. The sum of the lengths of any two sides is
13. Label the figure as follows:
greater than the length of the third side.
A
5. a, b, c. The measure of the third angle is 180°  x
70°  35°  75° (Triangle Sum Conjecture), so the
angle measures from greatest to least are 75°, 70°, D E
35°. By the Side-Angle Inequality Conjecture, the
order of the sides opposite those angles is the same: B C
a, b, c.
Notice that ABC and ADE are both isosceles
6. c, b, a. The measure of the third angle is 57°, so the triangles that share the same vertex angle. The
angle measures from greatest to least are 68°, 57°, measure of the vertex angle determines the
55°, and the order of the sides opposite those angles
is the same: c, b, a.

Discovering Geometry Solutions Manual CHAPTER 4 63


©2003 Key Curriculum Press
DG3SM586_056-096.qxd 8/2/06 5:48 PM Page 64

measures of the base angles: If the measure of (Linear Pair Conjecture), so g  124°. Finally, the
the vertex angle is x, then by the Triangle Sum angles with measures g and h are alternate interior
Conjecture, the measure of each base angle will angles, so h  g  124° (AIA Conjecture).
be 12(180°  x). Thus, it is impossible for two
21. BAR  ABE. All pairs of corresponding parts
isosceles triangles to share the same vertex angle
are congruent. Two pairs of congruent sides are
but have base angles of different measures, as the
marked and the shared side is congruent to itself.
figure indicates.
Two pairs of angles are congruent by the AIA
14. 135°. By the Triangle Exterior Angle Conjecture, Conjecture, and the third pair of angles must be
t  p  135°. congruent by the Triangle Sum Conjecture.
15. 72°. By the Triangle Exterior Angle Conjecture, 22. FAR  FNK. All pairs of corresponding parts
r  58°  130°, so r  72°. are congruent. Two pairs of congruent sides are
marked. Applying the Converse of the Isosceles
16. 72°. The triangle is isosceles with base angles of
Triangle Conjecture to ANF shows that AF   NF,
measure x. By the Triangle Exterior Angle Conjec-
which gives the third pair of congruent sides.
ture, x  x  144°, or 2x  144°, so x  72°.
23. Cannot be determined. Only one pair of congruent
17. To show that the Triangle Exterior Angle Conjecture
sides and one pair of congruent angles are given, so
is true, we must show that x  a  b. By the
it is not possible to determine congruence of the
Triangle Sum Conjecture, a  b  c  180°.
triangles.
Because BCA and BCD are a linear pair,
x  c  180°. Then, by substitution, x  c  PROJECT
a  b  c. Subtracting c from both sides gives Project should satisfy the following criteria:
x  a  b.
● Presentation of data is organized and clear.
18. 45°. When the perpendicular distance is measured,
the bearing will be 90°. Because this is double the ● Explanations of predictions and descriptions of the
bow angle when you begin recording, that angle results are consistent.
must measure 45°. ● For students cutting straws, lengths probably won’t be
19. a  52°, b  38°, c  110°, and d  35°. First random. If lengths are generated using a graphing
a  38°  90°  180° (Triangle Sum Conjecture), calculator, Fathom, or another random-length gener-
so a  52°. The angle with measure b and the 38° ator, the experimental probability for large samples
angle are alternate interior angles, so b  38° (AIA will be around 25%.
Conjecture). Then, in the triangle with angles of Extra credit
measure c and 32°, the third angle also measures
● A graph of the sample space uses shading to show cut
38°, so c  32°  38°  180°, and c  110° (Triangle
combinations that do produce a triangle. Sample graph:
Sum Conjecture). In the small isosceles triangle at
the top of the diagram, the vertex angle measures y

110° (Vertical Angles Conjecture) and the third


1
angle measures d (Isosceles Triangle Conjecture), so
2d  110°  180° (Triangle Sum Conjecture), and
1/2
thus d  35°.
20. a  90°, b  68°, c  112°, d  112°, e  68°, x
0
f  56°, g  124°, and h  124°. First, the angle 1/2 1

with measure a and the angle marked as a right EXTENSIONS


angle are corresponding angles formed when two
parallel lines are cut by a transversal, so a  90° A. Research results will vary.
(CA Conjecture). a  b  22°  180° (Triangle Sum B. See the solution to Take Another Look activity 4 on
Conjecture), so b  68°. b  c  180° (Linear Pair page 75.
Conjecture), so c  112°. The angles with measures
c and d are alternate interior angles, so d  c  112° LESSON 4.4
by the AIA Conjecture. The angles with measures b
and e are alternate exterior angles, so e  b  68°. EXERCISES
In the isosceles triangle containing the angles with 1. SAS. Notice that if one of the triangles is rotated
measures e and f, the angle with measure e is the 180° in either direction, it will coincide with the
vertex angle, so 2f  e  180° (Triangle Sum other, with L coinciding with I, U with D, and Z
Conjecture); therefore, f  56°. Then 56°  g  180° with A, which confirms the congruence.

64 CHAPTER 4 Discovering Geometry Solutions Manual


©2003 Key Curriculum Press
DG3SM586_056-096.qxd 8/2/06 5:48 PM Page 65

, as the third pair of


2. SSS. Use the shared side, FD 18. Draw a baseline with your straightedge and copy
congruent sides. Every side is congruent to itself. the length of one side along this baseline to form
one side of the new triangle. Place the point of your
3. Cannot be determined. Matching congruent sides
compass at one endpoint of this segment and
and angles give COT  NAP, rather than
draw an arc with radius equal to the length of a
COT  NPA.
second side of the original triangle. Then place your
, as the third pair of
4. SSS. Use the shared side, CV compass point at the other endpoint of the same
congruent sides. Every side is congruent to itself. segment and draw an arc using a radius equal to the
length of the third side of the original triangle. The
, for the second pair of
5. SAS. Use the shared side, KA
intersection point of the two arcs will be the third
congruent sides.
vertex of the new triangle.
  RY
6. SSS. Because Y is a midpoint, AY , and by the
Converse of the Isosceles Triangle Conjecture,
  YN
YB .
7. Possible answer: Boards nailed diagonally in the
corners of the gate form triangles in those corners.
Triangles are rigid, so the triangles in the gate’s
corners will increase the stability of those corners 19. Copy one side of the triangle.
and keep them from changing shape. Using the resulting segment
as one side of the angle, copy
8. b  55°, but 55°  130°  180°, which is impossible an angle adjacent to this side.
by the Triangle Sum Conjecture. Another way to see Then copy a second side of the
that this situation is impossible is to apply the original triangle so that the
Triangle Exterior Angle Conjecture. The exterior copied angle will be included
angle shown in the figure measures 125°, which must between the two copied sides.
be the sum of the two remote interior angles. But
one of the remote interior angles measures 130° and 20. a  37°, b  143°, c  37°, d  58°, e  37°,
a must be a positive number, so this is impossible. f  53°, g  48°, h  84°, k  96°, m  26°,
p  69°, r  111°, and s  69°. a  143°  180°
9. ANT  FLE by SSS. Match congruent sides. (Linear Pair Conjecture), so a  37°. b  143°
10. Cannot be determined. SSA is not a congruence (Vertical Angles Conjecture). c  a  37° (AIA
conjecture. Conjecture). c  d  85°  180° (Triangle Sum
Conjecture), so d  58°. e  c  37° (Vertical
11. Cannot be determined. Parts do not correspond. Angles Conjecture). e  f  90°  180° (Triangle
Notice that the shared side is included between the Sum Conjecture), so f  53°. g  48° (Isosceles
two marked angles in WOM, but not in WTO. Triangle Conjecture). g  h  48°  180° (Triangle
12. Cannot be determined. Parts do not correspond. Sum Conjecture), so h  84°. h  k  180° (Linear
Notice that the marked side is included between the Pair Conjecture), so k  96°. d  k  m  180°
two marked angles in BOY, but not in MAN. (Triangle Sum Conjecture), so m  26°. In the
triangle containing angles with measures p and 85°,
13. SAT  SAO by SAS. Use the shared side for one the third angle has measure m, so p  m  85° 
pair of congruent sides. 180° (Triangle Sum Conjecture), and p  69°.
14. GIT  AIN by SSS. Or use the vertical angles as p  r  180° (Linear Pair Conjecture), so r  111°.
the pair of included congruent angles for SAS. Finally, s  p  69° (CA Conjecture).

15. SUN  RAY by SAS. UN  AY  4, 21. 3 cm  third side  19 cm. The length of the third
SU  RA  3, and mU  mA  90°. side must be greater than 11  8  3 cm and less
than 11  8  19 cm.
16. DRO  SPO by SAS. The midpoint of both SD 
 is (0, 0), giving two pairs of congruent
and PR 22. Side length 1 2 3 4 5 n 20
sides. Use the vertical angles as the pair of included Elbows 4 4 4 4 4 4 4
congruent angles.
T’s 0 4 8 12 16 4n  4 76
17. Because the LEV is marking out two triangles that
Crosses 0 1 4 9 16 (n  1)2 361
are congruent by SAS, measuring the distance x will
also approximate the diameter of the crater.

Discovering Geometry Solutions Manual CHAPTER 4 65


©2003 Key Curriculum Press
DG3SM586_056-096.qxd 8/2/06 5:48 PM Page 66

There is one elbow at each corner, so the number IMPROVING YOUR REASONING SKILLS
of elbows is always 4. The T’s are used around the Fill the cylinder twice to the 150 mL mark and once
edges; there are (n  1) of these at each edge and to the 50 mL mark. Or fill the small container twice
4 edges, so the number of T’s is 4(n  1)  4n  4. to the 250 mL mark, pouring the contents into the
A cross is used at each interior intersection point; large container each time. Pour 150 mL from the large
there are (n  1)(n  1), or (n  1)2 such intersec- container back into the small container. It is not possible
tions, so there are (n  1)2 crosses. in fewer than three steps.
23. (12, 7). Graph both equations on the same EXTENSION
coordinate system. To graph y  23x  1, use the
See the solution to Take Another Look activity 9 on
y-intercept, 1, to plot the point (0, 1). Then
page 75.
move up 2 units and to the right 3 units to reach
the point (3, 1); draw a line through these two
points. To graph the line 3x  4y  8, first rewrite LESSON 4.5
the equation in the form y  mx  b by solving the
equation for y : 4y  3x  8, so y  34x  2. Use
EXERCISES
the y-intercept, 2, to plot the point (0, 2), and 1. AMD  RMC by ASA. Use the vertical angles at
then move up 3 units and to the right 4 units to M as one pair of congruent angles.
reach the point (4, 1). Draw the line through these 2. BOX  CAR by ASA.
two points.
y
3. GAS  IOL by SAA. To confirm the congruence
10 and correspondence of vertices, flip over one of the
(12, 7)
triangles and slide it to coincide with the other.
y  _2 x  1
3
4. Cannot be determined. There is only one pair of
congruent sides given, and the only pair of angles
(3, 1) 3x  4y  8 that you know are congruent are the vertical angles.
(4, 1)
x
(0, –1) 10 15 This is not enough information to determine
(0, –2) congruence.
5. Cannot be determined. Using the shared side as a
The graph shows that the two lines intersect at the pair of congruent sides, you have SSA, which is not
point (12, 7). a congruence shortcut.
The intersection point can also be found from a 6. Cannot be determined. The parallel lines and two
calculator graph. Enter Y1  (23)X1 and Y2  transversals give two pairs of congruent angles, and
(34)X2 and find the intersection point of the the vertical angles at T are congruent, but you don’t
two lines. have any information about the sides. AAA is not a
congruence shortcut.
24. (5, 3). The orthocenter of a triangle is the
point of concurrency of the three altitudes. In a 7. FAD  FED by SSS. Use the shared side as the
right triangle, two of the altitudes are sides of the third pair of congruent sides.
triangle, so, because you are told that ABC is a 8. WHO  WTA by ASA or SAA. HT  and OA
 are
right triangle, the orthocenter will be the right-angle transversals of the parallel sides, so H  T and
vertex of this triangle. To identify this vertex, find O  A by the AIA Conjecture. Also, HWO 
the slopes of the three sides: TWA by the Vertical Angles Conjecture. If you use
 3  2 5 5
  
5  (8)  3  3
Slope AB one pair of alternate interior angles and the pair of
vertical angles, the congruence is true by ASA, or if
0  (3)
    3
Slope BC you use both pairs of alternate interior angles, the
0  (5) 5
congruence is true by SAA.
 02 2 1
  
Slope AC 0  (8)  8  4 9. LAT  SAT by SAS. Because AT is an angle
,
bisector, LAT  SAT, and the shared side, AT
Because 53 3

and are negative reciprocals,
5
  BC
AB , so B(5, 3) is the vertex of the right is congruent to itself.
angle. Therefore, the orthocenter has coordinates 10. POE  PRN by ASA or SAS, SON  SRE
(5, 3). by ASA. For POE and PRN, use the shared right

66 CHAPTER 4 Discovering Geometry Solutions Manual


©2003 Key Curriculum Press
DG3SM586_056-096.qxd 8/2/06 5:48 PM Page 67

angle, P, as one pair of congruent angles. Every 18. The construction is the
angle is congruent to itself. same as the construction
in Exercise 17 once you
11. Cannot be determined. Parts do not correspond. find the third angle, which
 but C is
Note that A is adjacent to AM is given here.
.
opposite RM
19. Draw any triangle. Copy any two angles of the orig-
12. RMF  MRA by SAS. FMR  ARM inal triangle, but make the included side a different
because both are right angles, and the shared side, length from the one in the original triangle. By the
, is congruent to itself.
MR Third Angle Conjecture, the two triangles will have
13. Cannot be determined. Using alternate interior three pairs of congruent angles. The new triangle
angles formed by the two transversals to the parallel will have the same shape, but not the same size, as
lines and the vertical angles, you have three pairs of the original. Because the sides of the new triangle
congruent angles, but no information about the are not congruent to those of the original, the
sides. AAA does not guarantee congruence. triangles are not congruent. Possible answer:
14. LAW  WKL by ASA. Use the shared side, the E

right angles, and the other marked angles. B

15. Yes, three exterior triangles are congruent by SAS.


Because the triangle is equilateral, SN  NL  LS. A C D F
The figure shows that TN  EL  IS, so by subtrac-
20. Draw a line segment. Construct a perpendicular.
tion, ST  NE  LI. Therefore, you have two sets of
  EL Bisect the right angle. Construct a triangle with
congruent sides in the three triangles: TN
 and NE   LI   ST . Also, because SLN is equi- two congruent sides and with a vertex that
IS
measures 135°.
lateral, each of its angles measures 60°, so N 
L  S. Therefore, TNE  ELI  IST by 135°
SAS. Because all corresponding parts of congruent
triangles are congruent, TE  EI   IT
, which
means that TIE is equilateral.
16. ABC  CDA by SAA. Find the slopes of the
  slope
sides of quadrilateral ABCD: slope AB
  
CD  3 and slope BC  slope DA  13. Therefore,
  BC
AB  and CD   DA , which tells us that B 21. 125. Look for a pattern. Two concurrent lines
and D are both right angles, so they are divide the plane into 4 parts, 3 concurrent lines
  DA
congruent. Also, from the slopes, BC , so divide it into 6 parts, and 4 concurrent lines divide
BCA  CDA by the AIA Conjecture. Using it into 8 parts. Each time another line is drawn,
these two pairs of congruent angles and the shared two more parts are added. Thus, n concurrent
, ABC  CDA by SAA.
side, AC lines divide the plane into 2n parts; if 2n  250,
then n  125.
17. Copy one side of the original triangle. Then copy
the two angles that include that side. Use one 22. False. One possible counterexample is a kite.
endpoint of the new segment as the vertex of one
of these angles and the other endpoint of the
segment as the vertex of the second angle. Find the
intersection of the rays of the two angles that you
have copied that are not along the segment that
you copied.

23. None. Because the triangle is isosceles, both legs will


have length KM. By SAS, the two pairs of congruent
Because a side and the angles that include that sides and the included right angles will guarantee
side in the new triangle are congruent to the
corresponding parts of the original triangle, the
second triangle is congruent to the first by SAS.

Discovering Geometry Solutions Manual CHAPTER 4 67


©2003 Key Curriculum Press
DG3SM586_056-096.qxd 8/2/06 5:48 PM Page 68

that all isosceles right triangles with a given length 3. Cannot be determined. The given congruent parts
for one of the legs will be congruent.   CH
and CH  lead to SSA for triangles CHS and
L HCR, but SSA does not prove triangle congruence.
4. Yes. S  I, G  A (given), and TS  IT

(definition of midpoint), so ATI  GTS by SSA.
  IA
Therefore, SG  by CPCTC.
K M
 to form FOU and FRU. Then
5. Yes. Draw UF
  FR
FO  and UO
  UR (given), and UF
  UF

24. (same segment), so FOU  FRU by SSS.
Therefore, O  R by CPCTC.
  MA
6. Yes. MN  (given), ME
  MR
 (given), and
M  M (same angle), so EMA  RMN by
SAS. Therefore, E  R by CPCTC.
 to form triangles TUB and UTE.
7. Yes. Draw UT
25. a. About 100 km southeast of San Francisco   EU
BT  (given), BU
  ET  (given), and UT

b. Yes. No, two towns would narrow it down to  (same segment), so TUB  UTE by SSS.
UT
two locations. The third circle narrows it down Therefore, B  E by CPCTC.
to one. 8. Cannot be determined. HLF  LHA by ASA
EXTENSIONS (using two pairs of alternate interior angles and the
 and HF
shared side), but HA  are not corresponding
A. Given that any two corresponding angles of two
sides.
triangles are congruent, the remaining correspond-
ing angles are also congruent by the Third Angle 9. Cannot be determined. AAA does not guarantee
Conjecture. Thus, ASA implies SAA, and SAA congruence. There is no information given about
implies ASA. the sides.
B. See the solutions to Take Another Look activities 10. Yes. RO  GE  2, FO  TE  4, and the included
5–8 on page 75. angles are congruent because they are both right
angles. Therefore, FRO  TGE by SAS and
C. See the solution to Take Another Look activity 9 on
  GT
FR  by CPCTC.
page 75.
11. Yes. DNO  RCO by SAS, and OND 
IMPROVING YOUR ALGEBRA SKILLS
OCR by CPCTC.
The next terms in the sequences are 21x  23y,
34x  37y; and 64x  23y, 128x  28y.  and DF
12. Draw AC  to form ABC and DEF.
  
AB  CB  DE  FE  because all were drawn
In the first sequence, add the coefficients of any two   DF
with the same radius. AC  for the same
x-terms to get the coefficient of the next x-term, and reason. ABC  DEF by SSS. Therefore,
add the coefficients of any two y-terms to get the coeffi- B  E by CPCTC.
cient of the next y-term. In the second sequence, double
the coefficient of each x-term to find the coefficient of 13. Cannot be determined. You have two pairs of
the next x-term, and subtract 5 from the coefficient for   PM
congruent sides (CM  from the median and
any y-term to get the coefficient of the next y-term.   AM
AM ), but that is not enough information to
show that the triangles are congruent.
LESSON 4.6 14. HEI  KEI by ASA. The shared side is included
between the two pairs of given congruent angles.
EXERCISES
  BD
 (same segment), A  C (given), 15. ULF  UTE by SAS. Because U is the midpoint
1. Yes. BD
, UF
of FE   UE , and because U is also the
and ABD  CBD (given), so DBA  DBC
midpoint of LT, UL
  UT. The included angles are
  CB
by SAA. Therefore, AB  by CPCTC.
vertical angles.
  WN
2. Yes. CN  and C  W (given), and
16. Copy one side of the original triangle, then one of
RNC  ONW (Vertical Angles Conjecture),
  ON  the angles adjacent to that side using an endpoint
so CRN  WON by ASA. Therefore, RN
by CPCTC.

68 CHAPTER 4 Discovering Geometry Solutions Manual


©2003 Key Curriculum Press
DG3SM586_056-096.qxd 8/2/06 5:48 PM Page 69

of the segment as a vertex. Then use a compass to 22. Values c and d always decrease. The distance from
mark off an arc whose length is equal to the other point P to point C decreases until PB  ; then the
side of the original triangle that is adjacent to the distance begins to increase. The distance from C to
same angle. Connect the endpoints of the two  remains constant; it is always half the distance
AB
segments you have constructed that are not the between the two parallel lines. Because AP is
vertex of the angle between them to form the third increasing and AB is not changing, the ratio AA
B
P is
side of the new triangle. decreasing. Because AP is increasing faster than AC,
the difference is always decreasing.
23. x  3, y  10. Add or subtract given lengths:
x  5  2  3 and y  3  4  3  10.

17. Make the included angles PROJECT


different. One way to understand the answer of 26 regions is to
see that the number of parts of space added at each step
is the number of regions into which the new plane is
divided by the lines that are its intersections with previ-
ously existing planes. For example, the fourth plane
18. a  112°, b  68°, c  44°, d  44°, e  136°, intersects the three existing planes in three lines, and
f  68°, g  68°, h  56°, k  68°, l  56°, and m  three lines divide a plane into seven parts, so seven new
124°. a  68°  180° (Linear Pair Conjecture), so regions are added by the third plane.
a  112°. b  68° (AIA Conjecture). 68°  68° 
Project should satisfy the following criteria:
c  180° (Linear Pair Conjecture), so c  44°. d 
c  68° (AIA Conjecture). d  e  180° (Linear Pair ● The answer is correct: 26 regions.
Conjecture), so e  136°. In the triangle with angle
● The student has looked for analogies to the cases in
measures f and g, the unmarked angle measures 44°
other dimensions.
(Vertical Angles Conjecture). This is an isosceles
triangle with base angles of measures f and g, so ● Tables are used to look for patterns across dimensions.
2f  44°  180°, therefore, f  68° and g  68°. In
Extra credit
the isosceles triangle with base angle of measure h,
the measure of the vertex angle is 68° (Vertical ● The problem is solved in more than one way.

Angles Conjecture), so 2h  68°  180° and h  ● The student considered bounded and unbounded
56°. k  68° (AIA Conjecture). k  h  l  180°, regions.
so l  56°. Finally, m  k  h (AIA Conjecture),
so m  124°. ● The student used finite differences to find the cubic
formula for the number of parts into which n
19. ASA. The “long segment in the sand” is a shared random planes divide space: f(n)  16n3  56n  1.
side of both triangles.
20. (4, 1). In Lesson 3.7, Exercise 10, you discovered LESSON 4.7
that the circumcenter of a right triangle is the
midpoint of the hypotenuse. Using slopes, you can EXERCISES
  BC
determine that AB , so B is the right angle 1. The angle bisector does not go to the midpoint of

and AC is the hypotenuse. Therefore, the circum- the opposite side in every triangle, only in isosceles
, and its coordinates
center is the midpoint of AC triangles.
are (4, 1).
2. 1. Given; 2. Given; 3. Vertical Angles Conjecture;
21. Number of sides 3 4 5 6 7 12 n 4. ESM  USO; 5. CPCTC
Number of struts 3. 2. Given; 4. Definition of midpoint; 5. Vertical
needed to make Angles Conjecture; 6. CIL  MIB by SAS;
polygon rigid 0 1 2 3 4 9 n3   MB
7. CL 

The number of struts needed to make a polygon 4. 5. Given; 6. WSN  ESN by SSS; 7. W  E
with n sides rigid is n  3. This is the number of by CPCTC
diagonals needed to divide the polygon into 5. 2. 1  2, Definition of angle bisector; 3. Given;
triangles by connecting one of the n vertices to all 4. Same segment; 5. WNS  ENS by SAA;
of the other vertices that are not adjacent to it. 6. CPCTC; 7. Definition of isosceles triangle

Discovering Geometry Solutions Manual CHAPTER 4 69


©2003 Key Curriculum Press
DG3SM586_056-096.qxd 8/2/06 5:48 PM Page 70

6. 1. Given; 2. Given; 4. 1  2 by AIA Conjecture; 14. The circumcenter is equidistant from all three
6. ESN  ANS by ASA; 7. SA   NE by vertices because it is on the perpendicular bisector of
CPCTC. This proof shows that in a parallelogram, each side. The incenter is equidistant from all three
opposite sides are congruent. sides because it is on the angle bisector of each side.
7. Flowchart Proof 15. ASA. The fishing pole forms the side. “Perpendic-
ular to the ground” forms one angle. “Same angle
1 2 3
BA  BC AD  CD BD  BD on her line of sight” forms the other angle.
Given Given Same segment
10
16. 
35  27. Divide the number of shortest diagonals
4
ABD  CBD by the total number of diagonals. Each of the
SSS 10 vertices can be connected by a diagonal to each
5
1  2 6  bisects ABC
BD
of the 7 non-adjacent diagonals, but this counts
each diagonal twice, once from each endpoint. The
CPCTC Definition of angle 10  7
bisector total number of diagonals is  2  35. A shortest
. Apply the Side-Angle Inequality Conjecture diagonal occurs whenever a vertex is connected to
8. NE
 is a side of both triangles.
twice, noticing that EA another vertex that is two vertices away. Each of the

First in EAL, EA is the shortest side because 10 vertices can be connected to 2 other vertices in
it is opposite the smallest angle. Now look at this way, but again, each diagonal would be counted
NAE. The measure of the unmarked angle is twice. Therefore, the number of shortest diagonals is
10  7
180°  61°  58°  60°. In this triangle, EAN is  2  10.
the smallest angle, so the side opposite this angle,
, is the shortest side. Because EA
which is NE  is also
17.
a side of this triangle, you are able to compare the
side lengths of the two triangles and conclude that
 is the shortest segment in the figure.
NE
9. The triangles are congruent by SSS, so the two
central angles cannot have different measures.
18. The vertices of the original rectangle are B(1, 2),
10. POE  PRN by ASA, SON  SRE by ASA.
O(3, 2), X(3, 3), and Y(1, 3). The vertices of the
For the first pair of triangles, use the shared right
rotated rectangle are B(2, 1), O(2, 3),
angle and ignore the marks for OS   RS . For the
X(3, 3), and Y(3, 1). Under this rotation, each
second pair, use the vertical angles and ignore the
point (x, y) is replaced by the point (y, x).
right-angle marks and the given statement,
  PR
PO . y
4 Y X
11. Cannot be determined. The parts do not
B O
correspond. x
B' Y'4
 is a
12. RCK  ACK by SSS. KA  KR because CK
O' X'

median, and CK is the shared side.
13. a  72°, b  36°, c  144°, d  36°, e  144°, IMPROVING YOUR REASONING SKILLS
f  18°, g  162°, h  144°, j  36°, k  54°, and
In effect, the problem states that there is a unique solu-
m  126°. a  72° (AIA Conjecture). 2a  b  180°,
tion, so finding any solution is sufficient. If you are
so b  36°. b  c  180°, so c  144°. 72°  72° 
stuck, systematically try all possible locations of, for
d  180° (Triangle Sum Conjecture), so d  36°.
example, two jacks. You can use symmetry to diminish
d  e  180°, so e  144°. The triangle with the
the number of possibilities; that is, for every placement
angles with measures e and f is isosceles, so 2f 
of the jacks, you can rotate or reflect the entire square to
e  180°, and f  18°. g  18°  180°, so g  162°
account for other possibilities. (There are eight different
(Linear Pair Conjecture). h  e  144° (CA Conjec-
placements of two identical cards in a 3-by-3 array.)
ture). j  b  36° (CA Conjecture). The unmarked
angle in the right triangle containing the angle with Solution: An ace is in the center.
measure k is 180°  c  36°, so k  36°  90° 
180°, and therefore k  54°. Finally, in the isosceles K A K
triangle with vertex angle of measure a, the measure
of each base angle is 12(180°  72°)  54°, so m 
Q A Q

54°  180° and m  126°. J K J

70 CHAPTER 4 Discovering Geometry Solutions Manual


©2003 Key Curriculum Press
DG3SM586_056-096.qxd 8/2/06 5:48 PM Page 71

LESSON 4.8 student book, but in reverse, and using the


Converse of the Isosceles Triangle Conjecture in
EXERCISES place of the Isosceles Triangle Conjecture.
1. 6. The perimeter of ABC is 48, so AB  BC 
10. First construct a 60° angle
AC  48. Because AC  BC  18, AB  (2  18) 
by drawing arcs that
48, so AB  12. By the Vertex Angle Bisector
determine an equilateral
 is also a median, so
Conjecture, altitude CD
triangle. Then bisect the
AD  12(AB)  6.
60° angle to form two 30°
2. 90°. By the Vertex Angle Bisector Conjecture, the 30° angles.
angle bisector of the vertex angle of isosceles
triangle ABC is also the altitude to the base, so 11. a  128°, b  128°, c  52°, d  76°, e  104°,
mADC  90°. (Notice that it is not necessary to f  104°, g  76°, h  52°, j  70°, k  70°, l  40°,
know that mABC  72°.) m  110°, and n  58°. a  52°  180° (Linear Pair
3. 45°. Because DB  DA, CD  is the median from the Conjecture), so a  128°. b  128° (AIA Conjecture
vertex of the isosceles triangle, so it is also an with 1 and 2). b  c  180° (Linear Pair Conjec-
altitude. Therefore, mCDA  90°. CAD and ture), so c  52°. d  180°  2c (Triangle Sum
CAB are the same angle, so mCAD  45°. Thus, Conjecture), so d  76°. The angle directly below
mACD  180°  90°  45°  45°. the angle with measure d has measure 180°  d
(Linear Pair Conjecture), so its measure is 104°.
4. 2. 1  2; 5. SAS Then, e  104° (AIA Conjecture with 1 and 2).
 is
5. 4. CPCTC; 7. Definition of perpendicular; 8. CD Next, f  e  104° (AIA Conjecture with 3 and 4).
an altitude g  d  76° (CA Conjecture with 3 and 4).
h  12(180°  g)  52°. j  70° (marked congruent
6. 1
Isosceles ABC 2
ADC  BDC to other 70° angle). Look at the triangle containing
with AC  BC Conjecture A the angle with measure n. The unmarked angle in
and CD bisects C (Exercise 4)
this triangle measures 70° (CA Conjecture with 1
Given
and 2). Then, k  70° (Vertical Angles Conjecture).
3
AD  BD 4
CD is a median l  k  70°  180° (Triangle Sum Conjecture), so
CPCTC Def. of median l  40°. m  k  180° (Linear Pair Conjecture), so
m  110°. Finally, n  70°  52°  180° (Triangle
7. 1
AC  BC 2
CD  CD 3
D is Sum Conjecture), so n  58°.
Given Same segment midpoint
of AB 12. Between 16 and 17 minutes. First find a five-minute
Given interval during which this happens, and then
5
ADC  BDC 4
AD  BD narrow it down to a one-minute interval. At 3:15,
SSS Def. of midpoint
the hands have not yet crossed each other because
the minute hand is at the 3, but the hour hand has
6 7
ACD  DCB CD is angle moved one-fourth of the way between the 3 and the
bisector of ACB
CPCTC 4. At 3:20, the hands have already crossed because
Def. of angle bisector
8 the minute hand is on the 4, but the hour hand is
CD is altitude
of ABC
9
CD  AB only one-third of the way from the 3 to the 4. To
Conjecture B Def. of altitude get closer to the time where the hands overlap, try
(Exercise 5) some times between 3:15 and 3:20. At 3:17, the
hour hand has gone 1670 of the way from the 3 to
8. Drawing the vertex angle bisector as an auxiliary the 4, while the minute hand has gone 25 of the
segment, you have two triangles. You can show them way from the 3 to the 4. Compare these fractions
to be congruent by SAS, as you did in Exercise 4. by changing them to decimals: 1670  0.283 and
Then, A  B by CPCTC. Therefore, base 2 2 17
  0.4, so   . This means that at 3:17, the
5 5 60
angles of an isosceles triangle are congruent. minute hand has already crossed the hour hand, so
C try 3:16. At 3:16, the hour hand has gone 1660 of the
way from the 3 to the 4, while the minute hand
has gone 15 of the way from the 3 to the 4. Again,
compare the fractions by changing them to decimals:
A D B
16
  0.26  and 1  0.2. 16  1, so at 3:16, the
60 5 60 5
minute hand has not yet crossed the hour hand.
9. The proof is similar to the proof of the Equilateral Therefore, the hands overlap sometime between 3:16
Triangle Conjecture on pages 242–243 of the and 3:17, or between 16 and 17 minutes after 3:00.

Discovering Geometry Solutions Manual CHAPTER 4 71


©2003 Key Curriculum Press
DG3SM586_056-096.qxd 8/2/06 5:48 PM Page 72

2  3
: 
13. y  53x  16. First find the slope of AB 41 
 EXTENSION
5 5
  . Parallel lines have equal slopes, so find If any two segments coincide, all three will. (This
3 3
the equation in the form y  mx  b of the line happens when the three segments are drawn from the
through C(6, 6) with slope 53. vertex angle of an isosceles triangle or from any vertex
y  6 5 of an equilateral triangle.)
  
x6 3
3(y  6)  5(x  6)
EXPLORATION • NAPOLEON’S THEOREM
3y  18  5x  30
PROJECT
3y  5x  48
1. Any pair that has the same y-intercepts and slopes
5
y  3x  16 with the same value but opposite signs.
14. 120. n concurrent lines divide the plane into 2n 2. Any pair that has the same x-intercepts and slopes
parts; if n  60, 2n  120. (See solution for with the same value but opposite signs.
Lesson 4.5, Exercise 21.)
3. y  mx  b; y  mx  b
15. Any point at which the x-coordinate is either 1 or 7
4. The lines form an isosceles triangle with the base on
and the y-coordinate does not equal 3, or the points
the line y  x and with line of symmetry the line
(4, 6) or (4, 0).
y  x. Other such triangles are formed on the base
16. Hugo and Duane can locate the site of the fireworks y  x by lines with equations in the form y 
by creating a diagram using SSS. mx  m and y  m 1
x  1.
Fireworks
CHAPTER 4 REVIEW

340 m/sec 3 sec

340 m/sec 5 sec = 1.02 km
EXERCISES
= 1.7 km
Hugo
1. Their rigidity gives strength.
1.5 km
2. The Triangle Sum Conjecture states that the sum of
Duane the measures of the angles in every triangle is 180°.
Possible answers: It applies to all triangles; many
17. H
H H
other conjectures rely on it.
H
H C H
C C 3. Possible answer: The angle bisector of the vertex
H H angle is also the median and the altitude.
C C
H H
C C
H H
4. The distance between A and B is along the segment
H H connecting them. The distance from A to C to B
can’t be shorter than the distance from A to B.
Because “hept” means 7, cycloheptane has 7 C’s.
Therefore, AC  CB  AB. Points A, B, and C
There are 2 H’s branching off each C, so there are
form a triangle. Therefore, the sum of the lengths
14 C’s. In general, a cycloparaffin has n C’s and 2n
of any two sides is greater than the length of the
H’s, so the general rule for a cycloparaffin is CnH2n.
third side.
IMPROVING YOUR ALGEBRA SKILLS
5. SSS, SAS, ASA, or SAA
Let x represent the month you were born, and let y
represent the day. 6. In some cases, two different triangles can be
constructed using the same two sides and non-
2x included angle.
2x  16 7. Cannot be determined. SSA does not guarantee
5(2x  16)  10x  80 congruence.

10x  80  100  10x  20 8. TOP  ZAP by SAA.

10x  20  20  10x 9. MSE  OSU by SSS.

10x  10  100x 10. Cannot be determined. SSA does not guarantee


congruence.
100x  y
11. TRP  APR by SAS.

72 CHAPTER 4 Discovering Geometry Solutions Manual


©2003 Key Curriculum Press
DG3SM586_056-096.qxd 8/2/06 5:48 PM Page 73

12. Cannot be determined. Parts don’t correspond. 24. PTS  TPO by ASA or SAA; yes. Use the shared
Notice that the shared side is not opposite side to show that the triangles are congruent. By the
congruent angles. Converse of the Parallel Lines Conjecture, you can
  TO
show that PS  and PO   ST, so STOP is a
13. CGH  NGI by SAS. Use the Converse of the
  IG
Isosceles Triangle Conjecture to get HG , and parallelogram.
use the vertical angles. 25. ANG is isosceles, so A  G. Then, mA 
14. ABE  DCE by SAA or ASA. mN  mG  188°. The sum of the measures of
the three angles of a triangle must be 180°, so an
15. ACN  RBO  OBR by SAS. In a regular isosceles triangle with the given angle measures is
polygon, all sides are congruent and all interior impossible.
angles are congruent.
26. ROW  NOG by ASA, implying that OW  
  UT
16. AMD  UMT by SAS; AD  by CPCTC. 
OG . However, the two segments shown are not the
17. Cannot be determined. AAA does not guarantee same length.
congruence. 27. c is the longest segment, and a and g are the
18. Cannot be determined. SSA does not guarantee shortest. Apply the Side-Angle Inequality Conjecture
congruence. to all three triangles in succession. First, in the
triangles with side lengths a and g, the third angle
19. TRI  ALS by SAA; TR   AL by CPCTC. Use measure is 30°, so this triangle is isosceles with a 
alternate interior angles to show the triangles are g  f. Now look at the triangle with side lengths d,
congruent. e, and f. The angle opposite the side with length e
20. SVE  NIK by SSS; EI   KV
 by the overlap- measures 60°, because the measure of this angle 
ping segments property. EV  VI  EI and KI  30°  90°  180°. So the angle opposite the side
IV  KV. Because EV  KI and VI  IV, EI  KV with length f must also measure 60°. Thus, this
  KV
and thus EI . triangle is equilateral, and f  d  e. Finally, look at
the right triangle. The unmarked angle measures
21. Cannot be determined. You have two pairs of 45°, so this is an isosceles right triangle with b  d.
congruent sides, but there is no information In this triangle, c (the hypotenuse) is the longest
about the third pair of sides or about any of the side, so b  d  f. Putting the inequalities from the
pairs of angles. three triangles together, you have a  g  f  d 
22. Cannot be determined. There is not sufficient infor- e  b  c, so c is the longest segment and a and g
mation to determine that the triangles are are the shortest.
congruent. By the AIA Conjecture, MNT  28. x  20°. Apply the Triangle Sum Conjecture to both
CNT. With the shared side, this gives SSA, which the large triangle and the triangle containing the
does not guarantee that the triangles are congruent. angle marked as 100°. From the large triangle,
You are not able to determine whether MT   NC
, so 2a  2b  x  180°, and from the triangle with the
you cannot tell whether quadrilateral NCTM has 100° angle, a  b  100°  180, or a  b  80°,
one pair of opposite parallel sides or two pairs, and which is equivalent to 2a  2b  160°. Substituting
thus cannot determine whether it is a parallelogram 160° for 2a  2b in the equation 2a  2b  x 
or a trapezoid. 180° gives x  20°.
23. LAZ  IAR by ASA, LRI  IZL by ASA, 29. Yes
and LRD  IZD by ASA. There are actually two
  AE
Given: RE , S  T, ERL  EAL
isosceles triangles in the figure, LAI and LDI,
and there are three pairs of congruent triangles. For   TR
Show: SA 
LAZ  IAR, use the shared angle, A. For Flowchart Proof
LRI  IZL, use the common side, LI . Also,
1 2 3
mRLI  mZIL (Isosceles Triangle Conjecture), RE  AE S  T ERL  EAL

and mRLZ  mZIR (given), so by subtraction, Given Given Given

mZLI  mRIL, and thus, ZLI  RIL. Thus, 4


SAE  TRE
the triangles are congruent by ASA. Because
SAA
ZLI  RIL, DI   DL  by the Converse of the
Isosceles Triangle Conjecture. Using this pair of 5
SA  TR
sides, the marked angles, and the vertical angles, CPCTC
LRD  IZD by ASA.

Discovering Geometry Solutions Manual CHAPTER 4 73


©2003 Key Curriculum Press
DG3SM586_056-096.qxd 8/2/06 5:48 PM Page 74

30. Yes   MR
midpoint; 4. IM , Definition of midpoint;
Given: A  M, AF  FR, MR
  FR
 5. TMI  EMR, SAS; 6. T  E or
  RE
R  I, CPCTC; 7. TI , Converse of
Show: FRD is isosceles AIA Conjecture.
Flowchart Proof
36. Given three sides, only one triangle is possible;
1 2
AF  FR MR  FR therefore, the shelves on the right hold their shape.
Given Given The shelves on the left have no triangles and move
3 4
freely as a parallelogram.
AFR is a right angle MRF is a right angle
Definition of Definition of 37. Possible method: With a compass and straightedge,
perpendicular perpendicular construct an equilateral triangle, and bisect one
5
AFR  MRF 6
FR  RF 7
A  M
angle to obtain two 30° angles. Adjacent to that
angle, construct a right angle, and bisect it to obtain
All right angles Same segment Given
are congruent two 45° angles. The 30° angle and 45° angles that
8
FRM  RFA share a common side form a 75° angle.
SAS

9
RFM  FRA
CPCTC

11 10
FRD is isosceles RD  FD
Definition of Converse of Isosceles
isosceles triangle Triangle Conjecture 30°
45°
75°
31. x  48°. The sum of the measures of the interior
angles of any quadrilateral is 360° because it can be
subdivided into two triangles. Therefore, x  90° 
132°  90°  180°, so x  48°.
For a patty-paper construction, see the solution for
32. The legs form two triangles that are congruent by Lesson 4.2, Exercise 14, which shows the construc-
SAS, using the vertical angles as the included angles. tion of a 105° angle. If you use just the lower half
Alternate interior angles are congruent by CPCTC, of the 60° angle shown there, you will have a 30°
so by the Converse of the Parallel Lines Conjecture, angle adjacent to a 45° angle, which makes a
the seat must be parallel to the floor. 75° angle.
33. Construct P and A to be adjacent. The angle Fold 1 Fold 3

that forms a linear pair with the conjunction of Fold 2 Fold 4


P and A is L. Construct A. Mark off the
length AL on one ray. Construct L. Extend the 30°
75° 45°
unconnected sides of the angles until they meet.
Label the point of intersection P.
P
TAKE ANOTHER LOOK
1. On a sphere or a globe, angles are measured along
A
L P tangent lines. It is indeed possible to draw a triangle
with two or more obtuse angles or with three right
angles. The sum of the interior angle measures of
any triangle drawn on a sphere is always greater
than 180°.

A y L

34. Construct P. Mark off S2


the length PB on one ray.
From point B, mark off S1 x
the two segments that x 2. Both conjectures are true on a sphere, but the
intersect the other ray P z B
angles in an equilateral triangle no longer
of P at distance x. measure 60°.
 and IR
35. 1. M is midpoint of TE , Given; 2. TMI  3. A triangle with the measure of one angle twice
  ME
RME, Vertical angles; 3. TM , Definition of that of another can be divided into two isosceles

74 CHAPTER 4 Discovering Geometry Solutions Manual


©2003 Key Curriculum Press
DG3SM586_056-096.qxd 8/2/06 5:48 PM Page 75

triangles for certain only if the smallest angle meas- 8. Use this construction of the perpendicular to a line
ures less than 45°. When the angle with twice the through point P: With a compass, mark off points A
measure is acute, it works, but when this angle and B on the line, equidistant from point P. Choose
measures, for example, 100°, it doesn’t work. a longer radius for the compass, and draw arcs with
centers at points A and B and meeting at point Q.
80°
100° Draw PQ. PQ is perpendicular to the line.
  BP
Proof: AP , AQ
  BQ , and PQ
  PQ. So,
40° 50° APQ  BPQ by SSS. Therefore, APQ 
BPQ, and these angles form a linear pair, so they
4. The claim must always be true. Let a and b be the   AB
are right angles. Therefore PQ .
measures of interior angles A and B, respectively,
and let x be the measure of the exterior angle to 9. Neither SSSS nor SSSD guarantee congruence,
C, with x  2a. Then 2a  a  b, so a  b. but SSSDD does.

5. The measure of an exterior angle of a convex


quadrilateral is 180° less than the sum of the meas-
ures of the remote interior angles. (In a concave
quadrilateral, with appropriate interpretation, the Draw diagonals and consider triangle congruence.
exterior angle at an angle with measure more than Four sides and an angle will guarantee congruence
180° is the supplement of the sum of the measures of quadrilaterals, and there are other possibilities.
of the remote interior angles.) SASA does not guarantee congruence, but SASAS
6. Two possible answers: (1) The sum of the lengths of does. SASA is similar to the SSA case for triangles.
any three sides of a quadrilateral is greater than the
length of the remaining side. (2) The sum of the 
lengths of any two consecutive sides of a quadrilat-
eral is greater than the length of the diagonal
joining their endpoints.

7. Use this construction to bisect AB: From points A
and B, draw arcs that have the same radius and
have length greater than 12AB, intersecting at points
 is the perpendicular bisector, bisecting
C and D. CD

AB at point E. CHAPTER 5
C
LESSON 5.1
E
A B EXERCISES
D 1. (See table at bottom of page.)
  BC
Proof: AC   AD  BD and CD   CD, For each given value of n (the number of sides of
so ACD  BCD by SSS. Therefore, ACE  the polygon), calculate 180°(n  2) (the sum of the
  CE
BCE. CE , so ACE  BCE by SAS. AE  measures of the interior angles).
 by CPCTC, so AB
 BE . AEC
 is bisected by CD
2. (See table at bottom of page.)
 BEC. AEC and BEC form a linear pair.
Therefore, AEC and BEC are right angles. For each given value of n (number of sides), calcu-
  AB
Therefore, CD . late 180°(n  2) to find the sum of the measures of

Lesson 5.1, Exercises 1, 2


1. Number of sides of polygon 7 8 9 10 11 20 55 100
Sum of measures of angles 900° 1080° 1260° 1440° 1620° 3240° 9540° 17,640°

2. Number of sides of equiangular polygon 5 6 7 8 9 10 12 16 100

Measures of each angle of equiangular polygon 108° 120° 12847° 135° 140° 144° 150° 15712° 17625°

Discovering Geometry Solutions Manual CHAPTER 5 75


©2003 Key Curriculum Press
DG3SM586_056-096.qxd 8/2/06 5:48 PM Page 76

the interior angles, and then divide by n to find the be 540°. Three exterior angles of the pentagon
measure of each interior angle. In an equiangular each measure 49°, so the interior angles at these
polygon, all interior angles have the same measure. three vertices each measure 131°. Another exterior
angle measures 154°, so the exterior angle at that
3. 122°. In the quadrilateral, the unmarked angle
vertex measures 26°. Thus, the sum of the interior
measures 90° (Linear Pair Conjecture). Then, by
angle measures would be 3  131°  135°  26° 
the Quadrilateral Sum Conjecture, a  72°  76° 
554°, which is impossible.
90°  360°, so a  122°.
11. 18 sides. The figure shows an equilateral triangle,
4. 136°. The two unmarked angles in the hexagon
measure 112° and 110° (Linear Pair Conjecture), a regular nonagon, and a third regular polygon
and the sum of the interior angles of a hexagon is meeting at point A. The sum of the measures of the
180°  4  720° (Polygon Sum Conjecture). There- angles sharing A as a common vertex must be 360°.
fore, 2b  110°  112°  116°  110°  720°, so The measure of each interior angle of an equilateral
2b  272° and b  136°. triangle is 60°, and the measure of each interior
180°  7
5. e  108°, f  36°. In the equiangular pentagon, angle of a regular nonagon is  9
1260°
 9 

5e  540° (Pentagon Sum Conjecture), so e  108°. 140°. Therefore, the measure of each interior angle
The triangle is isosceles with base angles of measure of the third regular polygon is 360°  60°  140° 
72° (Linear Pair Conjecture), so f  2  72°  180° 160°. Because all angles of a regular polygon have
and f  36°. the same measure, the measure of each interior
180°(n  2)
6. c  108°, d  106°. One of the unmarked angles is angle is n. Use this expression to find n
a vertical angle of the angle marked as 44°, so its when each interior angle measures 160°.
measure is also 44°, and the other unmarked angle
180°(n  2)
of the quadrilateral measures 102° (Linear Pair   160°
n
Conjecture). Look at the triangle containing the
180°(n  2)  160°n
angle with measure d. Here, d  44°  30°  180°
(Triangle Sum Conjecture), so d  106°. Then 180°n  360°  160°n
apply the Quadrilateral Sum Conjecture:
20°n  360°
c  d  102°  44°  360°, so c  108°.
n  18
7. g  105°, h  82°. In the pentagon, 3g  108° 
117°  540°, so g  13(540°  (117°  108°))  Therefore, the largest polygon has 18 sides.
105°. Now look at the vertex on the right side of
12. a  116°, b  64°, c  90°, d  82°, e  99°,
the figure that is shared by the pentagon and the
f  88°, g  150°, h  56°, j  106°, k  74°,
quadrilateral. The sum of the measures of the three
m  136°, n  118°, and p  99°. a  116° (Vertical
angles at this vertex is 360°, so the unmarked angle
Angles Conjecture). b  64° (CA or AIA Conjecture,
in the quadrilateral measures 122°. In the quadrilat-
and Linear Pair Conjecture). c  90° (CA or AIA
eral, h  90°  66°  122°  360°, so h  82°.
Conjecture, and Linear Pair Conjecture). a  b 
8. j  120°, k  38°. The hexagon is equiangular, so d  98°  360° (Quadrilateral Sum Conjecture), so
6j  180°  4  720°, and j  120°. Now look at the d  82°. Now look at the quadrilateral containing
parallel lines to see that k  38° (AIA Conjecture). angles with measures e and p. Here, p  e, and the
unmarked angles measure 98° and 64° (Linear Pair
9. The sum of the interior angle measures of this
Conjecture), so 2e  98°  64°  360°. Therefore,
quadrilateral is 358°, but it should be 360°. Use
2e  198° and e  99°. Next look at the pentagon in
vertical angles to find that three of the angle
the lower left. By the Linear Pair Conjecture, the
measures in the quadrilateral are 102°, 76°, and
two unmarked angles measure 82° and 116°. The
82°. Using the CA Conjecture and the Linear Pair
sum of the measures of the interior angles of a
Conjecture shows that the fourth angle of the
pentagon is 540°, so f  138°  116° 
quadrilateral is 180°  82°  98°. Then the sum
82°  116°  540°, and f  88°. f  g  122° 
of the four interior angles of the quadrilateral
360°, so g  150°. Next look at the quadrilateral that
would be 76°  102°  82°  98°  358°, which
includes the angles with measures g and h. By the
is impossible.
Linear Pair Conjecture, the unmarked angles
10. The sum of the measures of the interior angles is measure 64° and 90°, so g  h  64°  90°  360°,
554°. However, the figure is a pentagon, so the and thus h  56°. Now look at the quadrilateral in the
sum of the measures of its interior angles should lower right that includes the angle with measure j.

76 CHAPTER 5 Discovering Geometry Solutions Manual


©2003 Key Curriculum Press
DG3SM586_056-096.qxd 8/2/06 5:48 PM Page 77

The unmarked angle is 90° (vertical angle with a  b  c  d  e  f  g  h  i  540°


angle of measure c), so j  90°  77°  87°  360°, by the addition property of equality. Therefore,
and thus j  106°. Either of the angles that form a the sum of the measures of the angles of a
linear pair with the angle of measure j measures pentagon is 540°.
74°, so k  74° (AIA Conjecture or CA Conjecture).
18. x  120°. The sum of the angle measures at a vertex
Next look at the quadrilateral in the upper right
of the tiling must be 360°, so 90°  60°  90° 
that includes the angle with measure m. Here, the
x  360°, and x  120°.
unmarked angles measure 90° and 74° (Linear Pair
Conjecture and Vertical Angles Conjecture), so 19. The segments joining the opposite midpoints of a
m  60°  74°  90°  360°, and thus m  136°. quadrilateral always bisect each other.
n  m  106°  360°, so n  118°. Finally, p  e, 20. y  89x  896 , or 8x  9y  86. The perpendicular
so p  99°. bisector of the segment with endpoints (12, 15)
13. 17 sides. Find the value of n for which and (4, 3) is the line through the midpoint of
180°(n  2)  2700°. Solve this equation. this segment and perpendicular to it. First find
the midpoint:
180°(n  2)  2700°
180°  n  360°  2700°  ,   ,   (4, 6)
12  4 15  (3)
2 2
8 12
2 2
180°  n  3060° The slope of this segment is
3  15 18 9
n  17     
4  (12) 16 8
Therefore, if the sum of the measures of the interior So, its perpendicular bisector will have slope 98. Find
angles is 2700°, the polygon has 17 sides. an equation of the line with m  98 through (4, 6).
14. 15 sides. The measure of each interior angle of y6 8
180°(n  2)   
an equiangular polygon is n, so x  (4) 9
180°(n  2) y6 8
solve the equation n  156°.   
x4 9
180°(n  2) 9(y  6)  8(x  4)
  156°
n 9y  54  8x  32
180°(n  2)  156°n 8 86
y  9x  9, or 8x  9y  86
180°n  360°  156°n
21. y  170 x  152 , or 7x  10y  24. The median to
24°n  360°  goes through C(8, 8) and the midpoint of AB ,
AB
n  15 which is (2, 1). Find an equation of the line that
contains (8, 8) and (2, 1). First find the slope.
Therefore, an equiangular polygon in which each
interior angle measures 156° has 15 sides. 8  (1) 7
m 
8  (2)  10

15. The twelfth century. Compare the measures of Now use the slope and either point to find the
each interior angle in a regular 16-gon to those equation of the line. Here, the point (2, 1)
in a regular 18-gon. Each interior angle of a is used.
180°  14
16-gon measures  16  157.7°, while each y  (1) 7
180°  16    
interior angle of an 18-gon measures  18  x  (2) 10
160°. Therefore, the plate is probably from the y  1 7
  
twelfth century. x2 10
10(y  1)  7(x  2)
16. The angles of the trapezoid
measure 67.5° and 112.5°. 67.5° 135° 10y  10  7x  14
is half the value of each angle of 7 12
67.5° 7x  10y  24, or y  1x 
0  5
a regular octagon, and 112.5° is
half the value of 360°  135°. 22. D. As vertex P moves from left to right: The
distance PA decreases and then soon increases,
17. mU  b  c, mI  d  e, mT  g  h  i,
the perimeter of the triangle decreases before it
so mQ  mU  mI  mN  mT 
increases, and the measure of APB increases for
a  b  c  d  e  f  g  h  i.
a while, but then decreases.
a  b  i  180°, c  d  h  180°, and e  f  g 
180° by the Triangle Sum Conjecture.

Discovering Geometry Solutions Manual CHAPTER 5 77


©2003 Key Curriculum Press
DG3SM586_056-096.qxd 8/2/06 5:48 PM Page 78

IMPROVING YOUR VISUAL THINKING SKILLS look at the common vertex of the heptagon and the
two quadrilaterals. The sum of the angle measures
DOT at this vertex is 360°, so 2d  12847°  360°. There-
fore, 2d  23137°, and d  11557°.
7. e  72°, f  45°, g  117°, and h  126°. Use the
DOT Equiangular Polygon Conjecture. In the regular
pentagon, the measure of each interior angle
3  180°
is  5  108°, so e  180°  108°  72°. In
EXTENSIONS the regular octagon, the measure of each interior
6  180°
A. The angles of a quadrilateral completely surround a angle is  8  135°, so f  180°  135°  45°.
point, so the sum of their measures is 360°. The g is the measure of one of three angles whose sum
angles of a pentagon will begin to overlap and will is 360°. Because one of these angles is an interior
surround a point 112 times (for a sum of 540°).
angle of the pentagon and another is an interior
B. Results will vary. angle of the octagon, g  108°  135°  360°,
C. See the solutions to Take Another Look so g  117°. Finally, h  e  f  g  360°
activities 1 and 2 on page 96. (Quadrilateral Sum Conjecture), so h  234° 
360°, and h  126°.
LESSON 5.2 8. a  30°, b  30°, c  106°, and d  136°. First,
a  56°  94°  180° (Triangle Sum Conjecture),
EXERCISES so a  30°. Next, b  a (AIA Conjecture), so
1. 1. Supplementary angles; 2. Supplementary angles; b  30°. From the triangle on the left, b  c  44° 
3. Supplementary angles; 4. 540°; 5. 180°, Triangle 180°, so c  106°. Finally, look at the quadrilateral
Sum Conjecture; 6. 360° that contains angles with measures 56°, 94°, and d,
as well as an unmarked angle. The measure of the
2. 360°. By the Exterior Angle Sum Conjecture, the unmarked angle is 180°  c  74° (Linear Pair
sum of the measures of a set of exterior angles for Conjecture), so d  94°  56°  74°  360°
any polygon is 360°. (Quadrilateral Sum Conjecture), and d  136°.
3. 72°; 60°. In an equiangular polygon, all interior 9. a  162°, b  83°, c  102°, d  39°, e  129°,
angles are congruent, so all exterior angles are also f  51°, g  55°, h  97°, and k  83°.
congruent. For a polygon with n sides, the measure a  180°  18°  162°. To find b, look at the
360°
of each exterior angle is  
n , so the measure of exterior angles of the large pentagon (which is
each exterior angle of an equiangular pentagon is subdivided into a triangle, a quadrilateral, and a
360°
  72°, and the measure of each exterior angle pentagon). The unmarked exterior angle at the
5 360°
of an equilateral hexagon is  6  60°.
 lower right forms a linear pair with an angle that
4. a  108°. Use the Exterior Angle Sum Conjecture. is marked as congruent to the angle with measure
The figure shows a set of exterior angles (one at h, so the measure of this exterior angle is b. The
each vertex) for a pentagon. The measure of the Exterior Angle Sum Conjecture says that the sum
unmarked exterior angle is 180°  140°  40° of the measures of a set of exterior angles (one at
(Linear Pair Conjecture). Therefore, a  68°  each vertex) of any polygon is 360°. Therefore,
84°  60°  40°  360°, so a  108°. 86°  b  18°  b  90°  360°, or 2b  194° 
360°, and b  83°. Next look at the isosceles
5. b  4513°. Use the Exterior Angle Sum Conjecture. triangle. Here, d  39° (Isosceles Triangle Conjec-
The measure of the unmarked exterior angle is ture), and 2  39°  c  180° (Triangle Sum
180°  68°  112° (Linear Pair Conjecture). There Conjecture), so c  102°. Next look at the vertex of
are three exterior angles with measure b. Therefore, the triangle with measure c. Here, 2e  c  360°, so
3b  69°  43°  112°  360°, so 3b  136°, and 2e  258° and e  129°. Now look at the upper-
b  4513°. right corner of the figure. Here, d  f  90°  180°,
6. c  5137°, d  11557°. By the Equiangular Polygon so f  51°. Next look at the upper-left corner of the
Conjecture, the measure of an interior angle of an figure. Here, 86°  g  39°  180°, so g  55°. Now
5  180° look at the lower-left corner. h  b  180°, so h 
equiangular heptagon is  7  12847°. Then, c 
97°. Finally, look at the quadrilateral. The angle in
180°  12847°  5137° (Linear Pair Conjecture). Now
the lower-right corner of the quadrilateral is

78 CHAPTER 5 Discovering Geometry Solutions Manual


©2003 Key Curriculum Press
DG3SM586_056-096.qxd 8/2/06 5:48 PM Page 79

congruent to the angle with measure h, so its IMPROVING YOUR VISUAL THINKING SKILLS
measure is also 97°. By the Quadrilateral Sum Eleven possible answers are shown.
Conjecture, k  97°  f  129°  360°, or k 
277°  360°, so k  83°.
10. 15 sides. The measure of each exterior angle of a
360°
regular polygon is  
n . (See solution for Exercise 3.)
If n  24°, 360°  24°  n, so n  15. Thus, if
360°

each exterior angle of a regular polygon measures


24°, the polygon has 15 sides.
11. 43 sides. Use the Polygon Sum Conjecture. Find n if
180°(n  2)  7380°.
180°(n  2)  7380°
180°  n  360°  7380°
180°  n  7740°
n  43 EXTENSIONS
Thus, if the sum of the measures of the interior A. Because it is easy to construct angles with measures
angles of a polygon is 7380°, the polygon has 60° and 90° with compass and straightedge and
43 sides. these angles may be bisected to obtain angles with
measures 30° and 45°, you can construct regular
12. Yes, the maximum number of obtuse exterior angles polygons that have any of these four interior angle
that a polygon can have is three. This is because the measures or angle measures that can be found by
measure of an obtuse angle is greater than 90°, and adding or subtracting these angles. This includes
if there were four or more exterior angles that each equilateral triangles (interior angle measure  60°),
measured more than 90°, their sum would be squares (interior angle measure  90°), regular
greater than 360°. That is impossible because the hexagons (interior angle measure  120°  2  60°),
sum of all the exterior angles (one at each vertex) regular octagons (interior angle measure  135° 
is 360° for any polygon. 90°  45°), and dodecagons (interior angle
The minimum number of acute interior angles that measure  150°  2  60°  30°). It is also
a polygon must have is 0. It’s possible for a polygon possible (but difficult) to construct regular polygons
to have no acute interior angles. A simple example with 10, 16, 17, and 20 sides.
of this situation is a rectangle, which has four right B. See the solutions to Take Another Look activities 3,
angles. 4, and 5 on page 96.
13. Regular polygons: triangle and dodecagon. Angle
measures: 60°, 150°, and 150°. Point A is LESSON 5.3
surrounded by an interior angle of a triangle and
an interior angle of two dodecagons. The measure EXERCISES
of each interior angle of an equilateral triangle is 1. 64 cm. The kite has two sides of length 12 cm and
60°, while the measure of each interior angle of a two sides of length 20 cm, so the perimeter is
10  180°
regular dodecagon is  12  150°. 2  12  2  20  64 cm.
14. Regular polygons: square, hexagon, and dodecagon. 2. x  21°, y  146°. By the Kite Angles Conjecture,
Angle measures: 90°, 120°, and 150°. y  146°. Then, by the Quadrilateral Sum Conjec-
ture, x  2  146°  47°  360°, so x  21°.
  CR
15. Yes. RAC  DCA by SAS. AD  by
CPCTC. 3. x  52°, y  128°. By the Isosceles Trapezoid
Conjecture, y  128°. Also, by the Isosceles Triangle
16. Yes. DAT  RAT by SSS. D  R by CPCTC.
Conjecture, the fourth angle of the trapezoid has
 to form the two triangles.)
(Draw AT
measure x. By the Quadrilateral Sum Conjecture,
2x  2  128°  360°, so 2x  104° and x  52°.

Discovering Geometry Solutions Manual CHAPTER 5 79


©2003 Key Curriculum Press
DG3SM586_056-096.qxd 8/2/06 5:48 PM Page 80

4. 15 cm. The perimeter of the trapezoid is 85 cm, so and E. Connect the four vertices B, E, N, and F in
2x  18  37  85. Thus, 2x  30 and x  15 cm. order, forming the kite.
5. x  72°, y  61°. The Kite Diagonals Conjecture F
says that the diagonals of a kite are perpendicular, N
so the two diagonals form four right triangles. Also, B

the diagonal that connects the vertices of the E


nonvertex angles divides the kite into two isosceles
triangles. Look at the small right triangle in the Only one kite is possible because three sides
upper right. The smallest angle in this triangle determine a triangle. (Note: The two points labeled
measures 18° because this is one of the base angles E and F are interchangeable, so the figure can be
of the smaller isosceles triangle (Isosceles Triangle labeled in two ways, but there is only one
Conjecture). Then, x  90°  18°  180° (Triangle possible figure.)
Sum Conjecture), so x  72°. In the right triangle in 11. Copy WI. Copy I and W, using the endpoints of
the lower right, the acute angles are 29° and 90°  
WI as their vertices. Copy IS along the side of I
29°  61°. The 61° angle in this triangle is a base 
that does not lie on WI . Construct a line through
angle of the larger isosceles triangle, so y  61° . Label the point where this
point S parallel to WI
(Isosceles Triangle Conjecture). line intersects the side of W that does not lie
6. x  99°, y  38 cm. Notice that the figure is an  as H. WISH is the required trapezoid.
along WI
isosceles trapezoid. First look at the unmarked angle
S H
at the lower-right corner of the trapezoid. By the
Trapezoid Consecutive Angles Conjecture, the
measure of this angle is 180°  81°  99°. By the I W
Isosceles Triangle Conjecture, this angle is congruent
to x, so x  99°. The perimeter of this isosceles . Use congruent
12. Possible construction: Copy NE
trapezoid is 164 cm, so y  2(y  12)  (y  12)  corresponding angles to construct a line parallel to
164. Solve this equation. , and copy the length BO along this line.
NE
y  2y  24  y  12  164 Connect the vertices B, O, N, and E to form
a trapezoid.
4y  12  164
4y  152
B O
y  38
7. E  I I E
N

T K Infinitely many trapezoids can be drawn. Notice


that none of the angles of the trapezoid, nor the
E lengths of either of the nonparallel sides, are given.
In the construction shown above, the angle that was
. Q and
8. The other base is ZI Z I
used to construct the parallel lines is arbitrary as is
U are a pair of base angles. Z
the position of point O along one of the rays of
and I are a pair of base angles.
Q U that angle.
 is the other base. S and H
9. OW W O 13. The definition of a kite says that I

are a pair of base angles. O and   IT


KI  and TE  EK . So point I
W are a pair of base angles. is equidistant from points K and T. K T
  HO
SW . S H
Likewise, point E is equidistant from
points K and T. Therefore, by the
10. Start by copying BN, which will form one diagonal
Converse of the Perpendicular Bisector
of the kite. Next, draw an arc centered at N with Conjecture, both I and E lie on the
radius NE and a second arc centered at B with . So
perpendicular bisector of KT E
radius BE. Draw these two arcs large enough so that 
diagonal IE is the perpendicular
.
they will intersect twice on opposite sides of BN .
bisector of diagonal KT
Label the two points of intersection of the arcs as F

80 CHAPTER 5 Discovering Geometry Solutions Manual


©2003 Key Curriculum Press
DG3SM586_056-096.qxd 8/2/06 5:48 PM Page 81

  BN
14. 3. BN ; 4. BEN  BYN by SSS; 5. 1  the lower left that contains the angle with measure
 bisects B,
2 and 3  4 by CPCTC; 6. BN h. The measure of the other acute angle of this

BN bisects N triangle is d (Vertical Angles Conjecture), so
h  d  90° and h  70°. Then g  180°  h
15. 80°, 80°, 100°, 100°. Look at one of the common (Linear Pair Conjecture), so g  110°. Finally, by
vertices where two of the trapezoids and the 18-gon the CA Conjecture, m  g, so m  110°, and
meet. The sum of the three angles that meet at this similarly n  a  100°.
vertex must be 360°. Let x represent the measure of
one of the base angles of the trapezoid along the PROJECT
inner edge of the arch. Each interior angle of a If the steps were infinitely small, the two parametric
(18  2)  180° 16  180°
regular 18-gon measures  18  18  equations would generate a circle.
160°. Therefore, 2x  160°  360°, so x  100°. The
two base angles of the isosceles trapezoid along the Project should satisfy the following criteria:
inner edge of the arch each measure 100°, and the ● The polygons are regular. (Use a friendly window,
two base angles along the outer edge of the arch perhaps from the zoom menu. Multiply the minimum
each measure 80°. and maximum values of x and y by a common factor
Notice that the exercise asks you to draw the arch, to make the window bigger.)
not to construct it, so you may use a protractor to ● The measure of the central angle of each polygon is
measure angles. given, and answers to the questions are clear.
To draw the arch, begin by drawing two concentric ● Rotated polygons are shown.
circles with a compass. Draw a diameter to form
four semicircles and erase the bottom two. To form Extra credit
half of the regular 18-gons inscribed in the semicir- ● Star polygons are created and explained.
180°
cles, draw central angles of measure  9  20°. On

EXTENSION
each semicircle, draw line segments connecting
consecutive intersections of the semicircles with the See the solution to Take Another Look activity 4 on
rays of the central angles. Then draw segments page 96.
connecting the two semicircles at those points.
LESSON 5.4
EXERCISES
1. A triangle has three midsegments, one parallel to
each of its sides, while a trapezoid has just one,
160° 100° which is parallel to the two bases.
20° 80° 2. 28. By the Triangle Midsegment Conjecture,
PO  12(RA)  10. From the figure, PT  PR  8,
16. Because ABCD is an isosceles trapezoid, A  B.
and OT  OA  10, so the perimeter of TOP is
AGF  BHE by SAA. Thus, AG   BH by
8  10  10  28.
CPCTC.
3. x  60°, y  140°. By the Triangle Midsegment
17. a  80°, b  20°, c  160°, d  20°, e  80°,
Conjecture, the midsegment shown is parallel to the
f  80°, g  110°, h  70°, m  110°, and n  100°.
third side, so x  60° (AIA Conjecture). Also, the
First a  100°  180° (Linear Pair Conjecture), so
angle that forms a linear pair with the angle of
a  80°. In the isosceles triangle that contains the
measure y measures 40°, so y  180°  40°  140°.
angles with measures a and b, notice that b is the
measure of the vertex angle. Therefore b  2a  4. 65°. By the Three Midsegments Conjecture, the
180° (Triangle Sum Conjecture), so b  20°. c  midsegments divide the large triangle into four
b  180° (Linear Pair Conjecture), so c  160°. smaller congruent triangles. The three angles of the
The angles with measures b and d are correspond- large triangle measure 65°, 42°, and 180°  (65° 
ing angles, so d  b (CA Conjecture), and d  20°. 42°)  73°. Because all four triangles are congruent,
Next 2e  d  180°, so e  80°. The angle that each must have angle measures of 65°, 42°, and 73°.
forms a linear pair with the angle of measure f has By the Triangle Midsegment Conjecture, each of the
the same measure as the angle formed by three midsegments is parallel to the third side. By
combining the angles of measures e and d, so its using the CA Conjecture, you can find the angle
measure is 80°  20°  100°, and thus f  180°  measures in the three outer triangles. (Notice that
100°  80°. Now look at the small right triangle in

Discovering Geometry Solutions Manual CHAPTER 5 81


©2003 Key Curriculum Press
DG3SM586_056-096.qxd 8/2/06 5:48 PM Page 82

all three of these triangles are oriented the same 12.


way, while the inner triangle is oriented differently.)
From these angle measures, you can see that 42°  80 cm
z  73°  180°, so z  65°. 60 cm

5. 23. By the Triangle Midsegment Conjecture, each


40 cm 60 cm
side of TEN is half the length of the parallel side
of UPA: TN  12(PA)  9, TE  12(UA)  8, and 13. If a quadrilateral is a kite, then exactly one diagonal
NE  12(UP)  6. Therefore, the perimeter of TEN bisects a pair of opposite angles. Yes, both the orig-
is 9  8  6  23. inal and converse statements are true.
6. m  129°, n  73°, p  42 cm. By the Trapezoid 14. a  54°, b  72°, c  108°, d  72°, e  162°,
Midsegment Conjecture, the midsegment is f  18°, g  81°, h  49.5°, i  130.5°, k  49.5°,
parallel to the two bases, so n  73° (CA Conjec- m  162°, and n  99°. a  54° (AIA Conjecture).
ture). By the Interior Supplements Conjecture, 2a  b  180°, so b  180°  2  54°  72°. The
m  180°  51°  129°. By the Trapezoid angle with measure c is a remote exterior angle of
Midsegment Conjecture, p  12(36 cm  48 cm)  base angles in the isosceles triangle whose base
1
(84 cm)  42 cm. angles measure 54°, so c  54°  54°  108°
2
(Triangle Exterior Angle Conjecture). The angles
7. 35. By the Trapezoid Midsegment Conjecture, 24 
1
(13  q), so 48  13  q and q  35. Another way
with measures b and d are corresponding angles
2 formed when the parallel lines are cut by a trans-
to find q is to notice that the length of the shorter
versal, so d  b, and d  72° (CA Conjecture).
base is 11 less than the length of the midsegment,
(Also, d  180°  b  180°  108°  72° by the
so the length of the longer base must be 11 more
Linear Pair Conjecture.) Next look at the right
than the midsegment: q  24  11  35.
triangle in which the angle with measure d is
  RD
8. 3. Triangle Midsegment Conjecture; 4. OA ; one of the acute angles. The other acute angle
  RD
5. LN  of this triangle measures 90°  72°  18°. Then,
e  180°  18°  162° (Linear Pair Conjecture),
9. Parallelogram. Draw a diagonal of the original
and f  18° (Vertical Angles Conjecture). The
quadrilateral. The diagonal forms two triangles.
triangle containing angles with measures f and g
Each of the two midsegments is parallel to the diag-
is isosceles with f the measure of the vertex angle,
onal, and thus the midsegments are parallel to each
so 2g  f  180°, and g  81°. Now look at the
other. Now draw the other diagonal of the original
isosceles triangle that contains the angle with
quadrilateral. By the same reasoning, the second
measure h as one of its base angles. In this triangle,
pair of midsegments is parallel. Therefore, the
the vertex angle measures 81° (Vertical Angles
quadrilateral formed by joining the midpoints is a
Conjecture), so 2h  81°  180°, and therefore,
parallelogram.
2h  99°, so h  49.5°. The angle with measure i
10. The length of the edge of the top base is 30 m forms a linear pair with one of the base angles of
by the Trapezoid Midsegment Conjecture. that isosceles triangle, so i  180°  49.5°  130.5°.
If x represents the length of the edge of the top, The angle with measure k and the angle of the
52  x
  41, so 52  x  82, and x  30. You can small isosceles triangle that forms a linear pair with
2
also figure this out by noticing that the bottom base the angle of measure i are corresponding angles
is 11 meters longer than the midsegment, so the top formed when the parallel lines are cut by a trans-
edge must be 11 meters shorter than the midseg- versal, so k  49.5°. Next look at the pentagon that
ment: 41  11  30. contains angles of measures i, k, and m. The two
unmarked angles each measure 99° because each of
11. Ladie drives a stake into the ground to create a
them forms a linear pair with one of the base angles
triangle for which the trees are the other two
vertices. She finds the midpoint from the stake to of the isosceles triangle that contains the angles of
each tree. The distance between these midpoints is measures f and g, and in that triangle, it was found
half the distance between the trees. earlier that the base angles each measure 81°. By the
Pentagon Sum Conjecture, the sum of the measures
of the interior angles of a pentagon is 540°, so k 
Cabin
i  2  99°  m  540°, or 49.5°  130.5°  198° 
m  360°, so m  162°. Finally, look at the
pentagon that contains angles of measures b, c, e,
and n. The unmarked angle in this pentagon forms

82 CHAPTER 5 Discovering Geometry Solutions Manual


©2003 Key Curriculum Press
DG3SM586_056-096.qxd 8/2/06 5:48 PM Page 83

a linear pair with the angle of measure g, so its ● The diagram includes the measure of one voussoir and
measure is 180°  81°  99°. Then, by the Polygon the number in the arch.
Sum Conjecture, b  c  e  99°  n  3  180°, or
● Measurement of each angle is determined from the
72°  108°  162°  99°  n  540°. Therefore,
number of sides of the circle that contains the
441°  n  540°, and n  99°.
arch’s arc.
  CA
15. (3, 8). Find the y-coordinate first. Because TR ,
Extra credit
 is a horizontal segment, so the y-coordinate of
TR
● There is an estimate of the rise and the span.
T must be 8. Because of the symmetry of an
 and RA
isosceles trapezoid, the slopes of TC  EXTENSION
must be opposite signs. Because the slope of RA  is Triangle: Place a triangle on a coordinate system with
80 8 8  is . Let x
8 one vertex at the origin and one side along the x-axis.
    , the slope of TC
12  15 3 3 3
80 8 Draw the midsegment that connects the endpoints of
x  0  3,
represent the x-coordinate of T. Then   
the two sides of the triangle that don’t lie on the x-axis.
so x  3, and the coordinates of T are (3, 8). Assign coordinates to the three vertices of the triangle,
16. (0, 8). By the Kite Diagonals Conjecture, HS  is the and use the midpoint formula to find the coordinates of

perpendicular bisector of ER . The diagonals of this the endpoints of the midsegment.
kite intersect at the origin, so (0, 0) is the midpoint y
. Therefore, because E is 8 units directly above
of ER (b, c)
the origin, R must be 8 units directly below the
origin. This means that the coordinates of R are a  b , _c
(0, 8).  _b2 , _2c  _____
2 2

17. Coordinates: E(2, 3.5), Z(6, 5); both slopes are 38.
E is the midpoint of RY , so its coordinates are (0, 0) (a, 0)
x
40 70
2, 2  (2, 3.5), and Z is the midpoint of Trapezoid: Place a trapezoid on a coordinate system with
, so its coordinates are  48 73
2 , 2   (6, 5).
RT  
one vertex at the origin and one side along the x-axis.
  5  3 .5 1 .5 3
6  2  4  8 , and the slope
The slope of EZ    Draw the midsegment of the trapezoid, which connects
  30 3
8  0  8.
of YT   the midsegments of the two nonparallel sides. Assign
coordinates to the four coordinates of the trapezoid.
18. Possible construction: Copy FN,
The two vertices that do not lie on the x-axis will lie
which will form one of the R
on the same horizontal line and therefore have the same
diagonals of the kite, and then
y-coordinate because the two bases of a trapezoid are
construct its perpendicular
F N parallel. Use the midpoint formula to find the coordi-
bisector. Because the diagonals
nates of the endpoints of the midsegment.
of a kite are perpendicular, the
, will lie
other diagonal, RK
y
(b, c) (d, c)
along this perpendicular bisector,
but you must locate the points R
a  d , _c
and K. To do this, draw an arc _____
2 2
centered at N with radius NK. K
 _b2 , _2c 
Label the point where this arc (0, 0) (a, 0)
x
intersects the perpendicular bisector as K. Now use
your compass to mark off a distance equal to RK In both the triangle and the trapezoid, the two
along the perpendicular bisector to locate K. endpoints of the midsegment have the same
Connect the vertices F, R, N, and K to form the kite. y-coordinates, so each of the midsegments lies on
a horizontal line (or has slope 0) and is therefore
There is only one kite, but more than one way to
parallel to the x-axis. Therefore, the midsegment of the
construct it.
triangle is parallel to the third side, and the midsegment
PROJECT of the trapezoid is parallel to the two bases.
Project should satisfy the following criteria:
● The project shows knowledge of the properties of
isosceles trapezoids.

Discovering Geometry Solutions Manual CHAPTER 5 83


©2003 Key Curriculum Press
DG3SM586_056-096.qxd 8/2/06 5:48 PM Page 84

LESSON 5.5 the circle) with radius 12PR. Connect the vertices D,
R, O, and P to form the parallelogram.
EXERCISES
1. c  34 cm, d  27 cm. Use the Parallelogram
Opposite Sides Conjecture. R

2. a  132°, b  48°. By the Parallelogram Consecu- D O


P R
tive Angles Conjecture, a  180°  48°  132°, P
and by the Parallelogram Opposite Angles
Conjecture, b  48°.
3. g  16 in., h  14 in. Use the Parallelogram 9. Complete the parallelogram with 
Vh
Opposite Sides Conjecture to find g and the the given vectors as sides. The
Parallelogram Diagonals Conjecture to find h. h  Vw
V
resultant vector is the diagonal
4. 63 m. Use the Parallelogram Diagonals Conjecture of the parallelogram.
to find VN  12(VF)  18 m and NI  12(EI)  
Vw
21 m, and use the Parallelogram Opposite Sides 10. Follow the same procedure as
Conjecture to find that VI  EF  24 m. Then, in Exercise 9.
the perimeter of NVI  18  21  24  63 m.

Vb
5. 80. By the Parallelogram Opposite Sides Conjecture,
x  3  17, so x  20, and therefore, x  3  23.
Therefore, the perimeter of the parallelogram is
2  17  2  23  34  46  80. 
Vc
 c
Vb  V
6. e  63°, f  78°. By the definition of a parallelo-
gram, both pairs of opposite sides are parallel, so
e  63° by the AIA Conjecture, and f  78° by the 11. (b  a, c). Use the definition of a parallelogram
Parallelogram Opposite Angles Conjecture. and slopes. By the definition of a parallelogram,
  PR
MA ; because PR is a horizontal segment (and
; then, copy L
7. Possible construction: First, copy LA , and therefore, the
has a slope of 0), so is MA
. Mark off an arc of
with one of its sides along LA
y-coordinate of M is c. Also, by the definition of a
length AS along the other side of L, and label the
  RA
parallelogram, PM , so PM and RA  have equal
intersection point of this arc with the ray that forms c0
 is 
slopes. The slope of RA c
  . The coor-
the second side of the angle as T. Draw an arc of ba ba
length AS centered at A and an arc of length LA dinates of P are (0, 0), so if you let x represent the
c0
 is  c
x  0  x.
centered at T. Label the intersection mark of these x-coordinate of M, then the slope of PM  
two arcs as S. Connect the vertices in order to form Equating the slopes of RA and PM  gives   .
c c
ba x
parallelogram LAST. Notice that you used two pairs Therefore, x  b  a and the coordinates of M are
of parallel sides in this construction. (b  a, c).
T S
12. Possible answer:
a b d
d c a
b c
L A

13. 1. Given; 2. Definition of parallelogram; 4. EAL 


8. Possible construction: Recall that the diagonals of
NLA; 5. AE  LN ; 6. AET  LNT; 8. AT 
a parallelogram bisect each other (Parallelogram
; 9. EN
LT  and LA  bisect each other
Diagonals Conjecture). First copy DO , which will
form the longer diagonal of the parallelogram. 14. The parallelogram linkage is used for the sewing
 to locate its midpoint. Also, bisect the
Bisect DO box so that the drawers remain parallel to each
given segment PR , and then open your compass to other (and to the ground) and the contents cannot
1
a radius of 2PR. Draw arcs (or a complete circle) fall out.
with centers at the midpoint of DO  and radius 1PR.
2
Also, draw arcs with centers at D and O and with
lengths DR. Label as R the point where the arc
centered at D intersects one of the arcs (or the
circle) with radius 12PR. Label as P the point where
the arc centered at O intersects one of the arcs (or

84 CHAPTER 5 Discovering Geometry Solutions Manual


©2003 Key Curriculum Press
DG3SM586_056-096.qxd 8/2/06 5:48 PM Page 85

15. a  135°, b  90°. a is the measure of an interior common vertex along the inside edge of the arch
180°  6
angle of a regular octagon, so a   8  135° where a base angle from each of two isosceles trape-
(Equiangular Polygon Conjecture), and b is the zoids meets an interior angle of the 30-gon. Here,
measure of an interior angle of a square, so b  90°. 2b  168°  360°, so b  96°. By the Trapezoid
16. a  120°, b  108°, c  90°, d  42°, and e  69°. Consecutive Angles Conjecture, a  b  180°, so
Start by applying the Equiangular Polygon Conjec- a  84°.
ture to the three equiangular polygons in the figure. 19. No. The congruent angles and the common side do
Recall that the Equiangular Polygon Conjecture not correspond. Notice that the common side of the
says that the measure of each interior angle of an , is opposite the 83° angle in
two triangles, WY
(n  2)180°
equiangular n-gon is n. a is the measure WYZ, but is opposite the 58° angle in XYW.
of an interior angle of an equiangular hexagon, so
20. The section is an isosceles trapezoid.
180°  4
a    120°
6
b is the measure of an interior angle of an equian- 21. Parallelogram. Possible construction: Construct a
gular pentagon, so segment, AC, of any length, and bisect it to find its
180°  3 midpoint, M. Construct any line that passes through
b    108°
5 M; call this line p. Open your compass to a radius
c is the measure of an interior angle of an equian- that is not equal to 12AC. Place your compass point
gular quadrilateral, so at M, and use this new radius to mark off arcs on p
. Label the two points where
on either side of AC
180°  2
c
 4  90°
these arcs intersect p as B and D. Connect the
vertices A, B, C, and D to form a parallelogram.
The four polygons share a common vertex, so a  Notice that M is the midpoint of both diagonals,
b  c  d  360°, and d  360°  120°  108°   and BD
AC , so the diagonals bisect each other
90°  42°. Now look at the isosceles triangle. Here, as required.
d is the measure of the vertex angle, so d  2e  B C
180° (Triangle Sum Conjecture). Therefore,
2e  138°, and e  69°. M

17. x  104°, y  98°. The quadrilaterals on the left


and right sides are kites, and the nonvertex angles
A D
of a kite are congruent (Kite Angles Conjecture).
p
The quadrilateral at the bottom of the figure is an
isosceles trapezoid. Base angles are congruent To explain why the quadrilateral you have
(Isosceles Trapezoid Conjecture), and consecutive constructed is a parallelogram, look at the figure
angles are supplementary (Trapezoid Consecutive with the construction marks removed. The marks
Angles Conjecture). Look at the three angles that on the figure show that the diagonals bisect each
share a common vertex on the left side of the other.
figure, one of which has angle measure x. The other
B C
two angles measure 154° (Kite Angles Conjecture) 2
3
and 180°  78°  102° (Trapezoid Consecutive
Angles Conjecture). Therefore, x  154°  102° 
360°, and x  104°. Now look at the three angles M

that share a common vertex on the right side of the 4


1
A D
figure, one of which has angle measure y. The other
two angles measure 160° (Kite Angles Conjecture)   MD  and MA  MC (diagonals
Because MB
and 102° (Isosceles Trapezoid Conjecture), so y 
bisect each other) and also BMC  DMA
102°  160°  360°, and y  98°.
(vertical angles), BMC  DMA by SAS. Then,
18. a  84°, b  96°. The arch is semicircular, so 1  2 by CPCTC, and because 1 and 2 are
the complete regular polygon on which the arch   AD
alternate interior angles, BC  by the Converse
is constructed has 30 sides, and each of its interior of the Parallel Lines Conjecture. Likewise, using the
(30  2)  180° same sides and vertical angles AMB and CMD,
angles measures  30  168°. Look at a
AMB  CMD by SAS. Then, 3  4 by
CPCTC, and because 3 and 4 are alternate

Discovering Geometry Solutions Manual CHAPTER 5 85


©2003 Key Curriculum Press
DG3SM586_056-096.qxd 8/2/06 5:48 PM Page 86

  DC
interior angles, AB . Therefore, both pairs side that lies on the x-axis must also lie on a
of opposite sides are parallel, and ABCD is a horizontal line (slope  0), so the y-coordinates of
parallelogram by the definition of a parallelogram. its endpoints will be equal.
y
22. Kite, dart, or rhombus. Construct two intersecting
(b, c) (d, c)
circles.

x
(0, 0) (a, 0)

The Parallelogram Diagonals Conjecture states that


the diagonals of a parallelogram bisect each other,
so that is what you need to show for this figure.
The figure is a kite or a dart because all radii of a Two segments bisect each other if they have the
given circle are congruent, which gives the two pairs same midpoint, so if you can show that the two
of consecutive congruent sides. It is only a rhombus diagonals of the parallelogram have the same
if the two circles have the same radius, so the midpoint, you will have verified the conjecture.
quadrilateral has four congruent sides. Here it is a
kite. If the center of the smaller circle was inside the The midpoint of the diagonal with endpoints
0d 0c
larger circle, the figure would be a dart. (0, 0) and (d, c) is  2 , 2    2 , 2 , and the
 
d c
 

IMPROVING YOUR VISUAL THINKING SKILLS midpoint of the diagonal with endpoints (a, 0) and
ab c0 ab c
The matching quilt squares are (A1, B3), (B1, D3), (d, c) is  2 , 2    2 , 2 . Now look at the
    

(A2, C2), (B2, C4), (A3, B4), (C1, C3), (A4, D2), slopes of the opposite sides of the parallelogram
and (D1, D4). that are not horizontal. The slope of the side with
c0 c
endpoints (0, 0) and (b, c) is  b  0  b , and the
 
EXTENSIONS
slope of the side with endpoints (a, 0) and (d, c)
A. Yes, the converses of the parallelogram conjectures c0 c
d  a  d  a . Because opposite sides of a parallel-
is   
are true:
ogram are parallel (definition of a parallelogram)
Converse of the Parallelogram Opposite Angles
and parallel lines have the same slope, the two
Conjecture: If both pairs of opposite angles of a
quadrilateral are congruent, then the quadrilateral slopes you have found must be equal. Therefore,
c c
is a parallelogram.   , so b  d  a, and d  a  b. By
b da
substituting a  b for d in the midpoint coordi-
Converse of the Parallelogram Consecutive Angles
Conjecture: If consecutive angles of a quadrilateral nates d2, 2c, you can see that both diagonals have
ab c
are supplementary, then the quadrilateral is a the same midpoint,  2 , 2 . Therefore, the diago-
 
parallelogram. nals of the parallelogram bisect each other.
Converse of the Parallelogram Opposite Sides
Conjecture: If both pairs of opposite sides of a USING YOUR ALGEBRA SKILLS 5
quadrilateral are congruent, then the quadrilateral
EXERCISES
is a parallelogram.
Note: Each of the systems in Exercises 1–6 can be solved
Converse of the Parallelogram Diagonals by either the substitution or elimination method.
Conjecture: If the diagonals of a quadrilateral bisect
each other, then the quadrilateral is a parallelogram. 1. 12, 3. The substitution method is a good
choice for solving this system because the first
Investigations will vary.
equation is already solved for y. Substitute 2x  2
B. A coordinate geometry verification of the Parallelo- for y in the second equation and solve for x.
gram Diagonals Conjecture is given here. Other
6x  2(2x  2)  3
parallelogram conjectures could also be verified.
Place a parallelogram on a coordinate system with 6x  4x  4  3
one vertex at the origin and another along the
2x  1
positive x-axis. In order for opposite sides of the
1
parallelogram to be parallel, the side opposite the x  2

86 CHAPTER 5 Discovering Geometry Solutions Manual


©2003 Key Curriculum Press
DG3SM586_056-096.qxd 8/2/06 5:48 PM Page 87

Then substitute x back into the first equation to find y. 4(3)  3y  3


y  2x  2 12  3y  3

 
1
y  2 2  2 3y  9
y  3
y3
The solution of the system is (3, 3).
The solution of the system is 12, 3.
5. Multiply the second equation by 2 and subtract the
2. (7, 2). To solve by the elimination method,
resulting equation from the first equation.
multiply both sides of the second equation by 2 and
add the resulting equation to the first equation. x  6y  10
x  2y  3 (x  6y  10)
 4x  2y  32 00
5x  35 Notice that all the variables cancelled out and you’re
left with a true statement.
x7
To graph the equations in the original system, solve
Now substitute 7 for x in either of the original
equations and solve for y. Here, it is substituted either of the original equations for y. Both of the
into the first original equation. original equations are equivalent to y  16x  53.
The graph is the line with y-intercept 53 and slope
7  2y  3
16.
2y  4 y
y  2 5
1 5
y  – _6x  _3
The solution of the system is (7, 2).
x
5, 3. Solve the first equation for y to get y 
2 –5 5 10
3.
5x  1. Then substitute 5x  1 for y in the
second equation and solve for x. The lines that represent the two equations in the
system are the same. This indicates that all solutions
15x  2(5x  1) of one of the equations are solutions of the other, so
15x  10x  2 the system has an infinite number of solutions,
represented by all of the points on the common line.
5x  2
2 6. Substitute 2x  1 for y in the first equation.
x  5
2x  (2x  1)  30
Now substitute 25 for x in the equation 5x  y 
1 and find the corresponding value of y. 2x  2x  1  30

2
5 5  y  1 1  30
Notice that all of the variables cancel out and you’re
2  y  1 left with a false statement. y

3y To graph the first line, solve 30

 3.
2 the first equation for y:
The solution of the system is ,
5
y  2x  30. The second y = –2x + 30
4. (3, 3). Multiply the first equation by 3 and then equation is already solved
20

add the resulting equation to the second equation. for y. Both lines have the
12x  9y  9 same slope, 2, but the 10

y-intercepts are different.


 7x  9y  6
The lines are parallel. x
5x  15 Because there is no
5 10
y = –2x – 1
x  3 point of intersection, –10
the given system has no
Now substitute 3 for x in either of the original solution.
equations and solve for y. Here, it is substituted into
the first original equation.

Discovering Geometry Solutions Manual CHAPTER 5 87


©2003 Key Curriculum Press
DG3SM586_056-096.qxd 8/2/06 5:48 PM Page 88

2 1
7. a. Plan A. y  4x  20; Plan B: y  7x. To solve 3  3x  3x
the system formed by these two equations by the
x  3
substitution method, substitute 7x for y in the
1
first equation and solve for x. y  3(3)  1
7x  4x  20 The first and second lines intersect at the point
(3, 1).
3x  20
20 y  3  23x and y  43x  3: Substitute 3  23x
x  3
for y in the second equation and solve for x; then
Substitute 230 for x in the second equation and find the corresponding value of y.
solve for y. 2 4
3  3x  3x  3
20 140
y  7 3  3   2x  0
The value of x, 230  623, represents 623 hr, or 6 hr x0
140 2 
40 min. The value of y,  3  46 3  46.6, repre-
  2
y  3  3(0)  3
sents $46.67 (rounded to the nearest cent).
The first and third lines intersect at the point (0, 3).
b. The point of intersection is where both plans are
the same, 6 hr 40 min of rental time for $46.67. y  13x and y  43x  3: Substitute 13x for y in
the third equation and solve for x; then find the
y
50 corresponding value of y.
40 Plan A
6 _23 , 46 _23 1 4
3x  3x  3
Cost ($)

30
Plan B x3
20 (0, 20) 1
y  3(3)  1
10

x The second and third lines intersect at the


2 4
Hours
6
point (3, 1).

c. The best deal for 5 hours is Plan B. Look at the Therefore, the vertices of the triangle are (3, 1),
graph. The line representing Plan B is below the (0, 3), and (3, 1).
line representing Plan A when x  5, which is to
the left of the intersection point, so Plan B is the LESSON 5.6
better deal (less expensive) when x  5.
EXERCISES
The most you can snowboard for $50 is
712 hours, with Plan A. The line y  50 intersects 1. False. The diagonals are congruent only if the
the line representing Plan A farther to the right parallelogram is a rectangle.
than it intersects the line representing Plan B,
indicating that $50 corresponds to more hours
under Plan A than it does under Plan B. To find
the number of hours of snowboarding that you 2. True
can get for $50 with Plan A, substitute 50 for y 3. True
in the equation for Plan A and solve for x: 50 
4x  20, so 4x  30 and x  430  125  712. 4. False. The diagonals of a rectangle
Therefore, the most hours of snowboarding you bisect the angles only if the
can get is 712 hours, with Plan A. rectangle is a square.

8. (3, 1), (0, 3), (3, 1). Find the intersection point 5. True
for each pair of equations by solving that pair 6. False. A rhombus has four congruent
simultaneously. sides, but it does not necessarily have
y = 3 + 23x and y  13x : Substitute 3  23x for y in four congruent (right) angles.
the second equation and solve for x. Then substitute 7. True
the value of x into the second equation to find the
corresponding value of y. 8. True
9. True

88 CHAPTER 5 Discovering Geometry Solutions Manual


©2003 Key Curriculum Press
DG3SM586_056-096.qxd 8/2/06 5:48 PM Page 89

10. False. Consecutive angles of a parallelogram are bisector. Then construct the perpendicular bisector
always supplementary. They are congruent only if . Label the points where this perpendicular
of BK
the parallelogram is a rectangle. intersects the sides of B as E and K. Connect the
points B, A, K, and E to form the rhombus.
11. 20. By the Rectangle Diagonals Conjecture, the diag-
onals of a rectangle bisect each other and are A K
congruent. By the “bisect each other” part, KC 
CR  10, so KR  20, and by the “are congruent”
part, WE  KR, so WE  20. B
E
12. 37°. By the Parallelogram Consecutive Angles
Conjecture, P and PEA are supplementary, so 19. Possible construction: Copy I E

48°  (y  95°)  180°. Then y  95°  132°,  and construct a perpendi-


PS
then y  37°.  through S. Open
cular to PS
your compass to radius PE
13. x  45°, y  90°. A diagonal divides a square into and draw an arc intersecting P S
two isosceles right triangles, so x  45°. The the perpendicular line that
diagonals of a square are perpendicular (Square you have constructed. Label the intersection point
Diagonals Conjecture), so y  90°. of the arc and the perpendicular line as E.
14. DIAM is not a rhombus because it is not equilateral Construct arcs of length PI and EI. Label their
and opposite sides are not parallel. You can use intersection as I. Connect the points P, I, E, and S
slopes to determine that opposite sides of this to form the rectangle.
quadrilateral are not parallel, so it is not even 20. If the diagonals are congruent and bisect each other,
a parallelogram. then the room is rectangular (Rectangle Diagonals
15. BOXY is a rectangle because its sides are Conjecture).
perpendicular. 21. The platform stays parallel to the floor because
16. Yes. TILE is a rhombus, and every rhombus is opposite sides of a rectangle are parallel. (The Paral-
a parallelogram. lelogram Opposite Sides Conjecture applies because
a rectangle is a parallelogram.)
17. Possible construction: You know
by the Square Diagonals E V 22. The crosswalks form a parallelogram: The streets are
Conjecture that the diagonals of different widths, so the crosswalks are of different
of a square are perpendicular lengths. The streets would have to cross at right
and congruent. Copy LV . angles for the crosswalks to form a rectangle. The
Construct its perpendicular L O streets would have to cross at right angles and also
bisector. From the intersection be of the same width for the crosswalk to form a
point of the diagonals, set the radius of your square.
compass at 12(LV ). Draw a circle with this center 23. Place one side of the ruler along one side of the
and radius. Label the two points where the circle angle. Draw a line with the other side of the ruler.
intersects the perpendicular bisector of LV as E Repeat with the other side of the angle. Draw a line
and O. Connect the points L, O, V, and E to form from the vertex of the angle to the point where the
the square. two lines meet.
18. One possible construction: Copy B. Bisect it. Mark
off distance BK along the bisector. At K copy angles
. Label as A
of measure 12 mB on either side of BK
and E the intersections of the rays of B with the
rays of the new angles which are not BK.
A
K

24. Rotate your ruler so that each endpoint of the


segment barely shows on each side of the ruler.
B
E Draw the parallel lines on each side of your ruler.
Now rotate your ruler the other way and repeat the
Another possible construction: Copy B and then
process to get a rhombus. The original segment is
bisect it. Mark off the length BK on the angle

Discovering Geometry Solutions Manual CHAPTER 5 89


©2003 Key Curriculum Press
DG3SM586_056-096.qxd 8/2/06 5:48 PM Page 90

4  (1)
5
one diagonal of the rhombus. The other diagonal that line. First find the slope: m   
3  2 5 

will be the perpendicular bisector of the original 1. Now use the slope and one of the points on the
segment. line to find an equation for the line. Here, (2, 1)
is used.
y  (1)
 x  2  1
y1

x 2  1
y  1  1(x  2)
25. a  54°, b  36°, c  72°, d  108°, e  36°, y  1  x  2
f  144°, g  18°, h  48°, j  48°, and k  84°.
y  x  1
First, a  54° (CA Conjecture). Now look at the
right triangle in which the measure of one of the To find points on this line in addition to the two
acute angles is b. The other acute angle in this that are given, substitute any number for x other
triangle is the vertical angle of the angle of measure than 2 and 3 into the equation y  x  1 to
a, so its measure is 54°, and b  90°  54°  36°. find the corresponding value for y. If x  1, y  0;
Next, 2c  b  180°, so 2c  144°, and c  72°. if x  0, y  1; if x  1, y  2; if x  2,
The angle with measure d forms a linear pair with y  3. Therefore, four additional points on the line
the alternate interior angle of the angle with are (1, 0), (0, 1), (1, 2), and (2, 3). Any three of
measure c, so d  180°  72°  108°. In the these points would be sufficient; there are infinitely
triangle that contains the angle with measure e, many other points on the line.
both of the other angle measures have been found
28. Circumcenter: (1, 3); orthocenter: (2, 1). First,
to be 72°, so this is an isosceles triangle with vertex
sketch the triangle.
angle of measure e. Therefore, e  2  72°  180°,
so e  36°. Now look at the quadrilateral that y

contains the angle with measure f. This is a parallel-


T (–3, 4)
ogram because both pairs of opposite sides are
parallel (from the given pairs of parallel lines). G (5, 2)
Notice that the angle that forms a linear pair with x
the angle of measure e is the angle opposite the R (2, –1)
angle of measure f in the parallelogram. Therefore,
by the Parallelogram Opposite Angles Conjecture
and the Linear Pair Conjecture, f  180°  e 
144°. Now look at the isosceles triangle in which g
It appears that RGT is a right triangle with R as
is the measure of a base angle. In this triangle, the
vertex angle measures 144° (Vertical Angles Conjec- 
the right angle. To verify this, use slopes: slope GR
2  (1)  1  4 5
ture), so 2g  144°  180°; then 2g  36°, and   33  1 and slope TR    
2  (3)
52 5
g  18°. Along 1, there are three marked angles, all 1. Because 1 and 1 are negative reciprocals of
of measure h, and one unmarked angle. The   TR
each other, GR , so R is a right angle.
unmarked angle is the alternate interior angle of the
supplement of the angle of measure f, so its Recall that the circumcenter of a triangle is the
measure is 180°  144°  36°. Then 3h  36°  point of concurrency of the three perpendicular
180°, so 3h  144°, and h  48°. Next, j is an alter- bisectors. In a right triangle, the circumcenter is the
nate interior angle of one of the angles of measure midpoint of the hypotenuse. In RGT, TG  is the
h, so j  48°. Finally, k  j  48°  180° (Triangle 5  (3) 2  4
hypotenuse, and its midpoint is 2, 2 
Sum Conjecture), so k  84°.
(1, 3), so the circumcenter is (1, 3).
26. 2. Given; 4. SSS; 5. CPCTC; 8. Definition of
Recall that the orthocenter of a triangle is the point
rhombus
of concurrency of the three altitudes. In a right
27. Possible answers: (1, 0); (0, 1); (1, 2); (2, 3). triangle, the orthocenter falls at the right-angle
Find an equation for the line passing through the vertex, so the orthocenter of RGT is (2, 1).
points (2, 1) and (3, 4), and then find points on Note: Another method for finding the circumcenter
and the orthocenter is to find equations of two of

90 CHAPTER 5 Discovering Geometry Solutions Manual


©2003 Key Curriculum Press
DG3SM586_056-096.qxd 8/2/06 5:48 PM Page 91

the perpendicular bisectors (for the circumcenter) LESSON 5.7


and two of the altitudes (for the orthocenter). Then
solve the system formed by the equations of the two EXERCISES
lines to find the coordinates of the circumcenter 1. If you start at square 100 and work 1 30
and the orthocenter. This method can be used for backward, the problem becomes 3 6 28
any triangle, not just right triangles, but involves much easier because there is only 8 14 19
more work than the special method shown above one possible path: 100  5; 95  5; 95

19  5; 14  2; 28  2; 30  5;
100
for right triangles.
6  2; 8  5; 3  2; 1.
29. The base angles of an isosceles right triangle
measure 45°; thus they are complementary. 2. Completion of flowchart proof: 3. OA   SK,

definition of parallelogram; 5. AIA; 6. SA  SA ,
45° Same segment; 7. SOA  AKS by ASA
Paragraph Proof: By the definition of a
parallelogram, SO   KA
 and OA  SK. Because 3
and 4 are alternate interior angles formed when
45°
 and KA
the parallel lines SO  are cut by the
, 3  4 by the AIA Conjecture.
transversal SA
30. Velocity  1.8 mi/hr. Angle of path  2 mi/hr
Similarly, 1 and 2 are alternate interior angles
106.1° clockwise from the north. 60°  and SK
 are cut by
formed when parallel lines OA
31. Yes, it’s true for rectangles. the same transversal, so 1  2 by the AIA
Conjecture. SA is a shared side of the two triangles;
A 1 B
every segment is congruent to itself. Therefore,
2 1.5 mi/hr
SOA  AKS by ASA.
3. Completion of flowchart proof: 4. Given; 5. THA,
D C
Conjecture from Exercise 2; 6. HBA  ATH by
Given: BAD  B  C  D CPCTC
Show: ABCD is a rectangle Paragraph Proof: Diagonal BT  divides
Paragraph Proof: By the Quadrilateral Sum parallelogram BATH into two congruent triangles
Conjecture, mBAD  mB  mC  mD  because either diagonal of a parallelogram divides
360°. Because all four angles are congruent, each the parallelogram into two congruent triangles.
360° (This is the conjecture that you proved in
measure is  4  90°. m1  mBAD  180°

because they are a linear pair. We already know Exercise 2.) Therefore, BAT  TBH, and
BAT  THB by CPCTC. Also, HT  divides
mBAD  90°, so m1  90° by subtraction.
By definition of congruence and by substitution, parallelogram BATH into two congruent triangles
1  B. Therefore, AD   BC
 by AIA. m2  for the same reason, so BAH  THA, and
mBAD  180° because they are a linear pair. HBA  ATH by CPCTC.
m2  90° by subtraction. By definition of 4. Completion of flowchart proof: 1. WA  RT,
congruence and by substitution, 2  D.   AT
Given; 2. WR , Given; 3. WT  WT, Same
  DC
Therefore, AB  by AIA. Thus ABCD is a segment; 4. WRT  TAW by SSS; 5. 1  2
parallelogram by definition of parallelogram. by CPCTC; 6. RT   WA
 by Converse of the
Because it is also equiangular, ABCD is a rectangle. Parallel Lines Conjecture; 7. 4  3 by CPCTC;
  TA
8. RW  by Converse of the Parallel Lines
IMPROVING YOUR REASONING SKILLS
Conjecture; 9. Definition of parallelogram
Think about which pairs of items can be together. One
Paragraph Proof: Because WA   RT and
solution: Take the rabbit across and leave it there. Go
  AT
WR  (both given) and also WT  WT,
back. Take the carrots across and return with the rabbit.
Leave the rabbit on the original side. Take the dog WRT  TAW by SSS. Then 1  2 by
CPCTC, so RT   WA
 by the Converse of the Parallel
across, leave it there, and go back. Finally, bring the
Lines Conjecture. Similarly 3  4, so RW   TA.
rabbit across again.
Because both pairs of opposite sides are parallel,
WATR is a parallelogram.

Discovering Geometry Solutions Manual CHAPTER 5 91


©2003 Key Curriculum Press
DG3SM586_056-096.qxd 8/2/06 5:48 PM Page 92

5. Sample flowchart proof: Thus, all four angles of parallelogram BEAR are
1   2  5
congruent, and therefore, BEAR is a rectangle by
SP  OA 
SP  OA SOP  APO
the definition of a rectangle.
Given Given SAS
3
1  2 6 9. 1
PA  TR 3
TZ  RA 4
2  3
3  4
AIA Conjecture CPCTC Given Given CA Conjecture
4  
OP  OP 7   2
ZA  TR
PA  SO
Same segment Z is on PA
Converse of
AIA Conjecture
5
8 ZART is a parallelogram
SOAP is a parallelogram
Definition of parallelogram
Definition of parallelogram
6 7
ZT  AR PT  AR
6. parallelogram
Parallelogram Given
7. Sample flowchart proof: Opposite Sides
Conjecture
1   GO
 2   YO
3 YO 
IY GOY  IYO 8
ZT  PT
Opposite sides of Definition of rectangle Same segment
rectangle are congruent. Substitution

4 9
GOY  IYO 1  2
SAS Isosceles Triangle
Conjecture
5   IO

YG
10
CPCTC 1  3
Substitution
Paragraph Proof: Look at two overlapping
triangles: GOY and IYO. IY   GO because 10. Completion of flowchart proof: 1. Isosceles trape-
opposite sides of a rectangle are congruent.   TH
zoid GTHR; 2. GR ; 3. Same segment;
(Opposite sides of a parallelogram are congruent, 4. RGT  HTG; 5. RGT  HTG by SAS;
and a rectangle is a parallelogram.) GOY  IYO   TR
6. GH  by CPCTC
by the definition of a rectangle. (All angles of a
rectangle are congruent.) Therefore, GOY  Paragraph Proof: Look at the overlapping triangles
IYO by SAS, and YG   OI  by CPCTC. in the figure, RGT and HTG. It is given that
GTHR is an isosceles trapezoid with nonparallel
8. Sample flowchart proof:   TH
sides GR . Also, GT
  GT
 (same segment),
1 BEAR is a parallelogram. and RGT  HTG by the Isosceles Triangle
Conjecture. Then, RGT  HTG by SAS, so
Given
  TR
GH  by CPCTC.
2   AB
   EA
3 BR    EB
4 AR 
RE 11. Opposite sides of a parallelogram are parallel.
Given Opposite sides of Opposite sides of
parallelogram are parallelogram are 12. If the fabric is pulled along the warp or the weft,
congruent. congruent. nothing happens. However, if the fabric is pulled
along the bias, it can be stretched because the
5
EBR  ARB  RAE  BEA rectangles are pulled into parallelograms.
SSS
13. 30° angles in 4-pointed star, 30° angles in 6-pointed
6 star; yes
EBR  ARB  RAE  BEA
CPCTC 4-pointed star: At each vertex of the tiling, two
hexagons, one square, and one vertex of the star
7
BEAR is a rectangle meet. Each interior angle of the hexagon measures
4  180°
Definition of rectangle  6  120°, and each interior angle of the
  AB is given, and square measures 90°. The sum of the angles around
Paragraph Proof: RE
  EA  and AR  EB  because opposite sides of any point is 360°, so the measure of each angle of
BR
a parallelogram are congruent (Parallelogram the 4-pointed star is 360°  2  120°  90°  30°.
Opposite Sides Conjecture). Therefore, 6-pointed star: At each vertex of the tiling, three
EBR  ARB  RAE  BEA by SSS, so squares and three stars meet. Again, the sum of
EBR  ARB  RAE  BEA by CPCTC.

92 CHAPTER 5 Discovering Geometry Solutions Manual


©2003 Key Curriculum Press
DG3SM586_056-096.qxd 8/2/06 5:48 PM Page 93

the angles around any point is 360°, so the measure measure formula equal to the angle measure and
of each acute angle of the 6-pointed star is solve for n. Using the measure of an exterior angle,
(360°  3  90°)  (90°)  30°.
1 1
3 3 divide the angle measure into 360° to find n.
The acute angles of both stars are the same, 30°. 3. Trace both sides of the ruler as shown below.
14. He should measure the alternate interior angles to
see if they’re congruent. If they are, the edges are
parallel.
15. 13. You miss 5 minutes out of 15 minutes, so the
probability of missing the bus is 155  13.
16. The container is 182  23 full, so it will be 23 full no 4. Make a rhombus using the double-edged straight-
matter which of the faces it rests on, which tells you edge, and draw a diagonal connecting the vertex of
that the height of the liquid in the new position will the original angle to the opposite vertex of the
be 23(9 in.)  6 in. rhombus.

9
6

12 3

PROJECT
Project should satisfy the following criteria: 5. Sample answer: Measure the diagonals with string to
see if they are congruent and bisect each other.
● Project questions are answered.
6. Draw a triangle so that the two points are
1. At least two colors are needed. midpoints of the sides. Then measure the side
2. In a quilt of four colors, around any pseudoblock of parallel to the midsegment. The length of the
one color there can be six different arrangements of midsegment is half the length of the third side
pseudoblocks of the other three colors (not (the side to which it is parallel).
counting rotations); three of these arrangements 7. x  10°, y  40°. This figure is a kite, so the
require four different quilt blocks. If four colors are diagonals are perpendicular and form two pairs
used with no touching, then the quilt can be made of congruent right triangles. In the larger right
with two different quilt blocks, each with kites of all triangles (on the right), one of the acute angles of
four colors. If the colors of one block are considered each triangle measures 80°, so the other acute angle
in clockwise order beginning with a kite in the measures 10°, that is, x  10°. In the smaller right
smaller angle of the rhombus, then the other block triangles (on the left), one of the acute angles of
will have the same colors in counterclockwise order each triangle measures 50°, so the other acute angles
beginning with a kite in the larger angle of the measure 40°. Thus, y  40°.
rhombus.
8. x  60 cm. 2x  52  64  266, so 2x  120, and
3. None of the four-color quilts requires more than x  60 cm.
four different quilt blocks.
9. a  116°, c  64°. This figure is an isosceles trape-
● A two- or four-color quilt is correctly assembled. zoid, so a  116° (Isosceles Trapezoid Conjecture)
Extra Credit and c  180°  116°  64° (Trapezoid Consecutive
Angles Conjecture).
● A quilt is designed with five or six colors.
10. Perimeter  100. By the Triangle Midsegment
CHAPTER 5 REVIEW Conjecture, MS  12(OI), so OI  36. Also, because
 and M is the midpoint of
S is the midpoint of IT
EXERCISES 
OT (definition of midsegment), IS  20 and
1. Divide 360° by the number of sides. OM  26. Therefore, the perimeter of MOIS (which
is a trapezoid) is 36  20  18  26  100.
2. Sample answers: Using the measure of an interior
angle, set the expression in the interior angle

Discovering Geometry Solutions Manual CHAPTER 5 93


©2003 Key Curriculum Press
DG3SM586_056-096.qxd 8/2/06 5:48 PM Page 94

11. x  38 cm. Use the Trapezoid Midsegment (6  2)  180° 4  180°


Conjecture. a  6
  6
 120°
1
32  2(x  (x  12)) The angle with measure b forms a linear pair with
1 one of the interior angles of the hexagon, so b 
32  2(2x  12)
180°  120°  60°. The third angle in the triangle
32  x  6 containing angles with measures b and c also forms
a linear pair with an interior angle of the hexagon,
38  x
so this angle also measures 60°. Thus b  c  60°
Thus, x  38 cm.  180°, so c  60°. The angle that forms a linear
pair with the angle of measure c is the alternate
12. y  34 cm, z  51 cm. The key to finding y and z
interior angle of the angle with measure d. There-
is to look for smaller polygons contained within the
fore, d  180°  60°, so d  120°. The measure of
large triangle. First look at the triangle in which y
the angle that forms a linear pair with the angle of
is one of the lengths of the sides. In this triangle,
measure d is 60°, so 2e  60°  180°, and e  60°.
17 cm is the length of the midsegment and y the
Now look at the triangle that contains the angle of
length of the third side, so 17  21y, or y  2  17 
measure f. One of the angles of this triangle forms a
34 cm by the Triangle Midsegment Conjecture. Now
linear pair with an angle that is marked as a right
look at the trapezoid in which one of the bases has
angle, so this angle of the triangle is also a right
length 17 cm and the other has length x. In this
angle and this is a right triangle. The measure of
trapezoid, x is the length of the midsegment, so by
17  z one of the acute angles of this triangle is f and the
the Trapezoid Midsegment Conjecture, y   
2 . measure of the other acute angle is e  60°, so e 
Because y  34 cm, this equation becomes
17  z f  90°, and f  30°. Now look at the equiangular
34   2 , or 68  17 + z. Therefore, z  51 cm.

pentagon on the right. Here, g is the measure of one
13. (See table at bottom of page.) of the interior angles, so
14. a  72°, b  108°. The measure of each (5  2)  180° 3  180°
g      108°
interior angle of a regular decagon is 5 5
(10  2)  180° 8  180° Next, look at the small triangle with three
 10
  10
 144° unmarked angles, one of which forms a linear pair
By looking at a common vertex of two isosceles with the angle of measure d. That angle measures
trapezoids and the decagon along the inner edge of 180°  d  60°, and another angle of the triangle
the frame and by using the fact that the sum of all is an exterior angle of the equiangular pentagon,
360°
so its measure is  5  72° (or 180°  108° 

angles around a point is 360°, you can see that
2b  144°  360°, so 2b  216° and b  108°. 72°). Thus, the measure of the third angle of this
Then, by the Trapezoid Consecutive Angles triangle is 180°  60°  72°  48°. Then, m 
Conjecture, a  b  180°, so a  72°. 108°  48°  180°, so m  24°. Finally, look at the
small triangle that contains an angle of measure m.
15. a  120°, b  60°, c  60°, d  120°, e  60°, Another angle of this triangle measures 72° (exterior
f  30°, g  108°, m  24°, and p  84°. First, look angle of pentagon or supplement of g), and the
at the equiangular hexagon on the left. Because a is third angle measures p (Vertical Angles Conjecture),
the measure of an interior angle of this hexagon, so p  m  72°  180°, and p  84°.

Chapter 5 Review, Exercise 13

Kite Isosceles trapezoid Parallelogram Rhombus Rectangle


Opposite sides are parallel No No Yes Yes Yes
Opposite sides are congruent No No Yes Yes Yes
Opposite angles are congruent No No Yes Yes Yes
Diagonals bisect each other No No Yes Yes Yes
Diagonals are perpendicular Yes No No Yes No
Diagonals are congruent No Yes No No Yes
Exactly one line of symmetry Yes Yes No No No
Exactly two lines of symmetry No No No Yes Yes

94 CHAPTER 5 Discovering Geometry Solutions Manual


©2003 Key Curriculum Press
DG3SM586_056-096.qxd 8/2/06 5:48 PM Page 95

16. 15 stones. Let x represent the measure of an interior Another way to find b is to notice that three angles
angle in the regular polygon. Two obtuse angles with this measure meet at a common vertex, and
measuring 96° (from two adjacent isosceles trape- the sum of the measures of all the angles around a
zoids) and one interior angle of the regular polygon point is 360°, so 3b  360°, and thus b  120°.
meet at a common vertex, so 2  96°  x  360°,
20. Speed  901.4 km/hr. Direction: N
which gives x  168°. Use algebra to find the
slightly west of north. Figure is Resultant
number of sides in a regular polygon in which each vector
approximate.
interior angle measures 96°.
900 km/hr
(n  2)  180°
 n  168°
180°n  360°  168°
50 km/hr
12°n  360°
21. Recall that the diagonals of a rhombus are perpen-
n  30
dicular and bisect each other (Rhombus Diagonals
Thus, the regular polygon is a 30-gon, but because Conjecture); use this conjecture as the basis of your
the arch is semicircular, the original arch contained construction.
15 stones.
First, copy the segment of length y and label the
17. (1, 0). Sketch the kite and observe that A and C copied segment as SR . Construct the perpendicular
are the vertex angles. 
bisector of SR . Also, bisect the given segment of
y length x. Using the midpoint of SR  as the center
1
and 2x as the radius, mark off arcs on both sides of
 along its perpendicular bisector. Label the points
SR
D(0, 2) where these arcs intersect the perpendicular bisector
C(3, 1)
x
as E and Q. To form the rhombus, connect the
vertices S, Q, R, and E.
A(–3, –2) B(2, –2)
E

S R 1x
1x 2

 is the
2
By the Kite Diagonal Bisector Conjecture, AC
, so the diagonals of
perpendicular bisector of BD Q
, which has
the kite intersect at the midpoint of BD
0  2 2 (2) 22. Possible construction: Copy F and draw an arc
coordinates  2
,   (1, 0).
2 centered at F with radius x, intersecting both sides
18. When the swing is motionless, the seat, the bar at of the angle. Label the points where this arc inter-
the top, and the chains form a rectangle. When you sects the angle as R and L. Bisect F. Copy L
swing left to right, the rectangle changes to a paral- with vertex at L and one side along FL. Label the
lelogram. The opposite sides stay equal in length, so point where this angle intersects the angle bisector
they stay parallel. The seat and the bar at the top of F as Y. Connect the vertices F, L, Y, and R to
are also parallel to the ground. form the kite.
19. a  60°, b  120°. Corresponding (and therefore R R

congruent) angles of six of the pentagons meet at Y


Y
a common vertex in the center of the figure, so F F x
L L
6a  360°, and a  60°. By looking at the tiling,
you can see that four of the interior angles of each  of length x and copy L with one
23. Construct LP
pentagon are congruent, with measure b, while the . Placing your compass point at L,
side along LP
fifth angle has measure a. The sum of the measures mark off a distance of x along the other side of
of all the interior angles of any pentagon is 540° L and label the point where the arc intersects the
(Pentagon Sum Conjecture), so 4b  60°  540°,
and thus b  120°.

Discovering Geometry Solutions Manual CHAPTER 5 95


©2003 Key Curriculum Press
DG3SM586_056-096.qxd 8/2/06 5:48 PM Page 96

angle as N. Construct the line parallel to LP 26. 12 cm. First look at the two triangles with solid
through N. Placing your compass point at N, mark sides and ignore the dashed segments. The lower
off a distance of y along this parallel line to form triangle is isosceles, so the shared side of the two
. Connect the vertices E and P to form the
NE triangles has length 48 cm (Converse of the
fourth side of the trapezoid. Isosceles Triangle Conjecture). The middle of the
three dashed segments is the midsegment of the
upper solid triangle, so its length is 12(48)  24 cm
(Triangle Midsegment Conjecture). Now look at the
N y E
smaller triangle in which the dashed segment of
length x is the midsegment and the dashed segment
x of the larger triangle (of length 24 cm) is the third
side. Apply the Triangle Midsegment Conjecture to
L z P this smaller triangle to find x: x  12(24)  12 cm.
24. Compare this situation with Exercise 23. Notice that   DI
27. 1. Given; 2. DE  by definition of rhombus;
Exercise 23 gives the lengths of both bases of the    
3. NE  NI by definition of rhombus; 4. DN
trapezoid, whereas Exercise 24 gives only the length 
DN , Same segment; 5. DEN  DIN by SSS;
of one possible base. Use the same method as 6. 1  2, 3  4 by CPCTC; 7. definition of
outlined in the solution for Exercise 23, but notice angle bisector
that now you can make RY  any length you wish.
TAKE ANOTHER LOOK
The figures below show two possible trapezoids
obtained with different choices for RY. 1. Sample answer: There are (n  2) triangles and for
each triangle, the sum of angle measures is 180°, so
x
R the total is 180°(n  2). The proof, with adaptations
F
Y in particular cases, holds for concave polygons.
2. It’s not possible for a “rectangle” on a sphere to
R z have four right angles, because the sum of the
x
F Y angles of any quadrilateral on a sphere is greater
D than 360°. The proof from Take Another Look
z activity 1, using inequalities, extends to the sphere,
on which the sum of angle measures of each
D triangle is more than 180°. Sample answer: On a
sphere, the sum of the angle measures of an n-sided
polygon is greater than 180°(n  2).
25. 20 sides. Notice that a square, a regular pentagon,
3. Arranging the angles about a point is one way of
and the third regular polygon with an unknown
finding their sum. Because this arrangement forms
number of sides meet at the common vertex, B. The
a complete circle about the point, the sum of the
measure of each interior angle of a regular pentagon
3  180° angle measures is 360°.
is  5  108°, and the measure of each interior
angle of a square is 90°. Because the sum of the 4. In effect, the measure of an exterior angle on a
measures of the angles around any point is 360°, the concave side is 180° minus the measure of the inte-
measure of an interior angle of the third polygon is rior angle and thus is negative. Other definitions of
360°  108°  90°  162°. Now find the number of exterior angle are possible, so results may vary. All
sides that a regular polygon has if the measure of its the kite conjectures also hold for darts (if the diag-
interior angles is 162°. onal joining the vertices is extended to intersect the
(n  2)  180° diagonal lying outside the figure).
 n  162°
5. The sum of the angle measures at a vertex is still
180°n  360°  162° 360°. However, the exterior angles, cut and arranged
18°n  360° around a point, will not quite completely surround
the point, showing that the exterior angle sum is
n  20 less than 360°. For instance, a triangle with three
Therefore, the third polygon has 20 sides. right angles would have an exterior angle sum of
90°  90°  90°  270°.

96 CHAPTER 5 Discovering Geometry Solutions Manual


©2003 Key Curriculum Press
DG3SM586_097-135.qxd 8/2/06 5:56 PM Page 97

10. The longer chord is closer to the center; the longest


CHAPTER 6 chord, which is the diameter, passes through the
center.
LESSON 6.1
EXERCISES
1. x  165° by the definition of the measure of an arc.
The measure of an arc is defined as the measure of
its central angle.
2. z  84° by the Chord Arcs Conjecture. The two 11. The central angle of the smaller circle is larger
marked chords are congruent, so their intercepted because in the smaller circle the chord is closer to
arcs are congruent, and the measure of the the center.
unmarked arc is 128°. The sum of the measures of
all arcs of a circle is 360°; z  128°  20°  128° 
360°, so z  84°.
5 cm
3. w  70° by the Chord Central Angles Conjecture. 5 cm

4. y  96° by the Chord Arcs Conjecture. The three


marked chords are congruent, so their intercepted 12. M(4, 3), N(4, 3), O(4, 3). Possible method:
arcs are congruent. Therefore 3y  72°  360°, so Because the rectangle is centered at the origin, it
3y  288°, and y  96°. must be symmetric to both axes. Reflect P across
5. OQ  8 cm by the Chord Distance to Center the y-axis to get the coordinates of M: (4, 3).
Conjecture. The distance between a point and Then reflect M across the x-axis to get the coordi-
a segment of a line is measured along the nates of N: (4, 3). Finally, either reflect N across
perpendicular. the y-axis or reflect P across the x-axis to get the
coordinates of O: (4, 3).
6. 20 cm; Perpendicular to a Chord Conjecture.
  AB
OP , so OP
 bisects AB
, and therefore 13. The center of the circle is the circumcenter of the
  CD
PB  12(AB)  3 cm. Likewise, OQ , triangle. The circumcenter of a triangle is the point
so QD  12(CD)  4 cm. Thus the perimeter of of concurrency of three perpendicular bisectors.
pentagon OPBDQ is 4  3  6  4  3  20 cm. Construct the perpendicular bisectors of two sides
  68° by the definition of the measure of an of the triangle. Use their intersection point as the
7. mAC center of the circle. The radius is the distance from
arc; mB  34° by the Isosceles Triangle Conjecture the center to any of the three vertices of the
 
and the Triangle Exterior Angle Conjecture: OC triangle. Possible construction:

OB because all radii of a circle are congruent, so
OBC is isosceles, and B  C by the Isosceles
Triangle Conjecture. Also, mB  mC  68°
(Triangle Exterior Angle Conjecture), so 2mB 
68°, and mB  34°.
8. The length of the chord is greater than the length
of the diameter. Because the radius of the circle is
18 cm, the diameter is 36 cm, and a diameter is the
longest chord in any circle. 14. Compass-and-straightedge construction: Draw
any two chords of the circle and construct their
9. The perpendicular bisector of the segment does not perpendicular bisectors. The intersection point of
pass through the center of the circle. This is impos-
sible by the Perpendicular Bisector of a Chord
Conjecture.

Discovering Geometry Solutions Manual CHAPTER 6 97


©2003 Key Curriculum Press
DG3SM586_097-135.qxd 8/2/06 5:56 PM Page 98

the perpendicular bisectors is the center of the reciprocal is 17. Find an equation of the line with
circle. Possible construction: slope 17 and passing through 72, 12:
1
y  2 1

 7 7
x  2

 
O
2 y  12 Multiply fraction on
1
  7 left by 22 to eliminate
 7
2 x  2  fractions.
Patty-paper construction: Fold a piece of patty
2y  1 1
paper so that two semicircles coincide and make a   
2x  7 7
crease. Repeat with a second pair of semicircles. The
2y  1
  
1 Multiply both sides
intersection point of the two creases is the center of 7(2x  7)   
the circle. 2x  7  7(2x  7) 7 of equation by
least common
Fold 2 denominator.
Fold 1 7(2y  1)  1(2x  7)
14y  7  2x  7
14y  2x
1
15.  13.8 cm. Draw two chords that connect pairs of y  7x
points on the arc. Find the center by the compass- If x  0, y  17(0)  0, so (0, 0) is on this line.
and-straightedge construction method used in Therefore the perpendicular bisector of AB  passes
Exercise 14. Measure the radius, and double it to through the center of the circle.
find the diameter. (Note: In order to find the
diameter accurately, trace the arc from the drawing 19. a. Rhombus
of the plate on page 311 of your book, and use this b. Rectangle
arc for your construction.)
c. Kite
d. Parallelogram
20. 78°. From the figure below, you can see that the
measure of the angle to be found is the sum of the
measures of two angles. The portion of the angle
between the two paths that is to the left of the
16. 1. Given; 3. All radii of a circle are congruent; vertical (North) line is the alternate interior angle
4. AOB  COD by the SSS Congruence of the angle marked as 15°, so it also measures 15°
Conjecture; 5. AOB  COD by CPCTC (AIA Conjecture), while the portion of the angle to
the right of the vertical line is the supplement of
17. They can draw two chords and locate the intersec-
the 117° angle, so its measure is 180°  117°  63°.
tion of their perpendicular bisectors. The radius is
Thus, the measure of the angle between the two
just over 5 km.
paths is 15°  63°  78°.
18. y  17x. (0, 0) is a point on this line. The perpen- N N
dicular bisector of AB  is the line that is perpendicu-
117°
lar to AB and passes through its midpoint. The
 is 3  4 4  (3) E
   , . The
7 1
midpoint of AB 2 , 2 2 2
? 9 km
15°
 is  3  4 7
slope of AB     7, whose negative 6 km
43 1
E

98 CHAPTER 6 Discovering Geometry Solutions Manual


©2003 Key Curriculum Press
DG3SM586_097-135.qxd 8/2/06 5:56 PM Page 99

21. 9.7 km. In the figure below, C is the location of the exponent of the previous term, and divide by k  1.
camp, L is the location of the lake, and F is the For example, the sixth term of the first sequence has
place where the family meets their friends. (0.5 cm (15)(2)
coefficient 5  6.
represents 1 km, so 1 cm represents 2 km.)
1. 6xy 5, y 6 2. 7xy 6, y 7 3. 8xy 7, y 8
N N
EXTENSIONS
117°
L
E
A. Results will vary.
9 km
15°
6 km
B. If a segment is drawn from the center of the circle
F to the midpoint of a chord (any chord that is not
E
C a diameter), two triangles will be formed that are
congruent by SSS, so you know by CPCTC that two
The distance to be found is CF. Carefully measure right angles are formed. Therefore the median of
to find that, in this figure, CF  4.85 cm, so, to the the chord must be perpendicular to the chord, so
nearest tenth of a kilometer, the distance from the it coincides with the given perpendicular bisector,
camp to the lake is 4.85 cm 1 cm   9.7 km.
2 km
 forcing the latter to go through the circle’s center.
22. The triangle on the left is isosceles with the angle of
measure x as its vertex angle, so x  55°  55°  LESSON 6.2
180°, and therefore, x  70°. Also, the triangle on
EXERCISES
the right is isosceles with the 40° angle as its vertex
angle, so 40°  y  y  180°, and thus, y  70°. 1. w  50°. Look at the quadrilateral in the figure.
Therefore x  y. Notice that the figure contains two tangents to the
circle and that the two unmarked angles of the
11
23. . Find the number of possible right triangles, and quadrilateral are formed when radii are drawn to
21
divide this by the number of ways of randomly the points of tangency. Therefore both of these
selecting three points from the nine points in the angles are right angles by the Tangent Conjecture.
grid. To find the number of right triangles, consider Now, apply the Quadrilateral Sum Conjecture:
the number of possible triangles with right angles 130°  90°  w  90°  360°, so w  50°.
at the corners of the grid, at the middle of the 2. x  55°. By the Tangent Segments Conjecture, the
sides, and at the middle of the grid. There are four triangle in the figure is isosceles with a vertex angle
distinct triangles that have right angles at each of 70°. Therefore, by the Isosceles Triangle Conjec-
corner (giving 4  4  16 triangles), five right ture, each of the base angles has measure x. Thus,
triangles (including one tilted) have right angles at x  x  70°  180°, so 2x  110°, and x  55°.
the middle point of each side (so 4  5  20), and 3. y  30°. Look at the triangle in the figure. Its angle
eight triangles (four tilted) have right angles measures are y, 90° (Tangent Conjecture), and 60°.
at the middle point (1  8  8). The total, 44, is Therefore y  90°  60°  180° (Triangle Sum
divided by the number of ways of choosing Conjecture), so y  30°.
9(8)(7)
three points from the nine points, which is  
1(2)(3) 4. z  105°. Look at the quadrilateral in the figure.
 84. Therefore the probability of selecting
The two unmarked angles are right angles (Tangent
three points from the grid that form the vertices Conjecture), so z  75°  90°  90°  360°
of a right triangle is 4844  1211 . (Quadrilateral Sum Conjecture). Therefore
IMPROVING YOUR ALGEBRA SKILLS z  105°.
These terms occur in expansions of (x  y)n for n  6, 5. 76. Apply the Tangent Segments Conjecture four
7, and 8, respectively. If you are familiar with Pascal’s times: From point P, PA  PC. From O, OA  OR.
triangle, you can see the pattern in the coefficients. The From S, SR  SD. From T, TD  TC. Also, by
exponents move up or down by 1 in each subsequent markings in the figure, PA  OA, and SR  TC.
term. You might find a missing exponent by symmetry, Therefore PC  PA  OA  OR  13, and
or you might see a different pattern: To calculate the SD  SR  TC  TD  12(ST)  6, so the
perimeter of POST is 4(13)  4(6)  52  24  76.
kth term in a sequence, multiply the coefficient and first

Discovering Geometry Solutions Manual CHAPTER 6 99


©2003 Key Curriculum Press
DG3SM586_097-135.qxd 8/2/06 5:56 PM Page 100

6. When viewed from the side, the ball’s path is actu- 11. Construct a line, and label a point T on it. On the
ally a parabola, but the overhead view shows a line, mark off two points, L and M, each on the
tangent line. Possible answer: same side of T at distances r and t from T, respec-
tively. Construct circle L with radius r. Construct
circle M with radius t. Possible construction:

Target L M T
t
7. Possible answer: The perpendicular to the tangent
line passes through the center of the circle. Use the
T-square to find two diameters of the Frisbee. The
intersection of these two lines is the center. 12. Start with the construction from Exercise 11. On
, mark off length TK so that TK  s and
line LM
8. From the Tangent Conjecture, you know that the
K is on the opposite side of T from L and M.
tangent is perpendicular to the radius at the point
. Construct a line through Construct circle K with radius s. Possible
of tangency. Construct OT
. Possible construction: construction:
point T perpendicular to OT

L M T K
r T
O t s

, OY
9. Construct OX , and OZ. Construct tangents 13. Sample answer: If the three points do not lie on the
through points X, Y, and Z. The three intersection same semicircle, the tangents form a circumscribed
points of pairs of these tangents are the vertices of triangle. If the points lie on the same semicircle,
the circumscribed triangle. Possible construction: they form a triangle outside the circle, with one side
touching (called an exscribed triangle).
14. Sample answer: Internally tangent: wheels on a
Z roller-coaster car in a loop, one bubble inside
another. Externally tangent: touching coins, a
O
snowman, a computer mouse ball and its roller
t
balls. Note that sample answers include tangent
Y
X spheres whose great circles through the point of
tangency are tangent.
15. Draw a diameter. Then bisect each of the radii that
form the diameter to find the centers of the two
10. Construct tangent circles M and N, each with inside circles, which are also the centers of the two
radius s. Construct equilateral triangle MNP. small circles inside of them. Possible construction:
Then construct circle P with radius s. Possible
construction:

s s
M N

100 CHAPTER 6 Discovering Geometry Solutions Manual


©2003 Key Curriculum Press
DG3SM586_097-135.qxd 8/2/06 5:56 PM Page 101

16. 41.6%. Look at the angles in the quadrilateral in the intersection of the two lines. To check, construct the
figure. Two of these angles are formed by radii circle through the three given points. Notice that the
drawn to points of tangency, so each of these is a circumcenter of the triangle (which is the center of
right angle (Tangent Conjecture). Therefore, by the the circle) lies outside the triangle. This is because
Quadrilateral Sum Conjecture, the measure of the the angle with vertex at (3, 11) is obtuse.
central angles, and thus the arc marked with a ques- y
tion mark, is 360°  30°  90°  90°  150°. (3, 11)
150°  
Because 360°  41.6%, 41.6% of the equator is

observable from the satellite.
17. y  185 x  
289
  
15 . Because PA is a radius and AT is a

tangent to the circle, AT is the line perpendicular
 and passing through the point A(8, 15)
to PA (–14, 4)

(Tangent Conjecture). The slope of PA  is 15 , so (–2, –1)


x
8

the slope of AT is the negative reciprocal of 185 ,
(11, –1)

which is 185 . Find the equation of the line through


(8, 15) with slope 185 :
y  15 8
  
x8 15
15(y  15)  8(x  8)
15y  225  8x  64
Note: It may be difficult to find the exact coordi-
15y  8x  289 nates of the center by this geometric method, espe-
8 289
y  1x 
5  15
cially when the coordinates are not integers. Finding
the center of a circle that passes through three given
18. 45°. Look at the angles in quadrilateral PAQC. points is equivalent to finding the circumcenter of
CQA is a central angle that intercepts CA, so
 the triangle with the three given points as vertices.
mCQA  mCA  78° (definition of arc measure). You will learn an algebraic way to do this in Using
  PA
QA  (Tangent Conjecture), so mQAP  90°.
Your Algebra Skills 6: Finding the Circumcenter.
In order to find mPCQ, it is necessary to find The algebraic method requires several steps, but has
some other angle measures. First, BQA is a central the advantage of always giving the exact coordinates
, so mBQA  mBA
angle that intercepts BA 
of the center.
168°. Then, mBQC  168°  78°  360° (sum of
angle measures around a point), so mBQC  21. 1. Given; 2. Definition of perpendicular lines;
114°. Now look at BQC: BQ   CQ  because all 3. Definition of right angle; 4. Substitution
radii of a circle are congruent, so BQC is isosceles property; 5. All radii of a circle are congruent;
with vertex angle BQC. Then, 2mBCQ  114°  6. Definition of isosceles triangle; 7. C  D
180°, so 2mBCQ  66°, and mBCQ  33°. by the Isosceles Triangle Conjecture; 8. OCR 
Therefore mPCQ  180°  33°  147° (Linear ODR by SAA; 9. CR   DR  by CPCTC;
Pair Conjecture). Now apply the Quadrilateral Sum 10. Definition of bisect
Conjecture to PAQC to find mP: mP  147°  22. a. True. If diagonals were perpendicular bisectors of
78°  90°  360°, so mP  45°. each other, it would be a square. Because only
19. Angles A and B must be right angles by the Tangent one diagonal bisects the other, opposite sides
Conjecture, but this would make the sum of the can’t be the same length.
angle measures in the quadrilateral shown greater b. False. It can be an isosceles trapezoid or a kite, so
than 360°. an isosceles trapezoid is a counterexample.
20. (2, 1). Possible method: Plot the three points. c. False. A nonsquare rectangle is a counterexample.
Construct the midpoint and the perpendicular
bisectors of the segments connecting two different 23. The circumcenter of the triangle formed by the
pairs of points. The center is the point of three light switches. Draw the triangle formed by
the three light switches, and find its circumcenter by

Discovering Geometry Solutions Manual CHAPTER 6 101


©2003 Key Curriculum Press
DG3SM586_097-135.qxd 8/2/06 5:56 PM Page 102

constructing the perpendicular bisectors of two of IMPROVING YOUR VISUAL THINKING SKILLS
the sides of the triangle.

s
EXTENSIONS
A. Constructions will vary.
s
B. Conjectures and results will vary.
s
C. See solutions for Take Another Look activities 1 and
2 on page 120.
The circumcenter of the triangle (or center of the
circumscribed circle, if it were constructed) is equi- LESSON 6.3
distant from the three points, so it is the most effi-
cient location for the junction box. EXERCISES
1. a  65°. The arc intercepted by the angle of
24. 7. Make a list of powers of 3, beginning with measure a is also intercepted by the central angle
30  1, and look for a pattern in the units digits: of measure 130°. The measure of this arc is 130°
30  1 34  81 (definition of the measure of an arc), so a 
1
(130°)  65° (Inscribed Angle Conjecture).
31  3 35  343 2
2. b  30°. One of the sides of the triangle is a diam-
32  9 36  1029 eter, so the unmarked angle is inscribed in a semi-
33  27 37  3087 circle. Therefore the unmarked angle in the triangle
is a right angle (Angles Inscribed in a Semicircle
Notice that in all powers of 3, the units digit is Conjecture). Thus, b  60°  90°  180°, so
1, 3, 9, or 7 and that the digits go through this b  30°.
pattern in a repeating cycle. Look at the remainders
when the exponents are divided by 4: When the 3. c  70°. The 95° angle is an inscribed angle whose
remainder is 0, the units digit of the power is 1; intercepted arc measures c  120°. Therefore 95° 
1
when the remainder is 1, the units digit is 3; when (c  120°), or c  120°  2(95°), by the Inscribed
2
the remainder is 2, the units digit is 9; and when Angle Conjecture, so c  70°.
the remainder is 3, the units digit is 7. When 23 is 4. h  50°. Draw the radius to the
divided by 4, the remainder is 3, so the units digit point of tangency. By the Tangent
of 323 is 7. Conjecture, the radius is per-
25. Station Beta is closer. Use a protractor and a pendicular to the tangent at the 40° h
centimeter rule to make a careful drawing. point of tangency, so the triangle
that has been formed is a right
Downed aircraft
N triangle. The figure shows that one of its acute
4.6 mi angles measures 40°, so the other acute angle
7.2 mi 48° measures 50°. This is the central angle that
Beta
intercepts the arc of measure h, so h  50° by
72° 312° the definition of arc measure.
38° 8.2 mi
5. d  140°, e  42°. The diameter divides the circle
Alpha
into two semicircles, and the sum of the measures
26. Yes, as long as the three points are noncollinear. of the arcs that make up a semicircle is 180°. The
Possible answer: Connect the points with segments, arc intercepted by the 20° angle measures 40°
then find the point of concurrency of the (Inscribed Angle Conjecture), so d  40°  180°,
perpendicular bisectors (same as circumcenter and d  140°. Likewise, the measure of the arc
construction). intercepted by the inscribed angle of measure e is
2e, so 2e  96°  180°; thus 2e  84°, and e  42°.

102 CHAPTER 6 Discovering Geometry Solutions Manual


©2003 Key Curriculum Press
DG3SM586_097-135.qxd 8/2/06 5:56 PM Page 103

6. f  90°, g  100°. The quadrilateral in the figure is hexagon intercepts an arc of measure 4s, so the
a cyclic quadrilateral, so its opposite angles are hexagon is equiangular as well as equilateral;
supplementary (Cyclic Quadrilateral Conjecture). that is, it is a regular hexagon. Therefore you
Therefore f  90°, and the angle opposite the could also find r by using the Equiangular
(6  2)  180°
75° angle measures 105°. The 105° angle is an Polygon Conjecture: r   6  120°.)
inscribed angle that intercepts an arc of measure
g  110°, so the measure of this arc must be 12. m  140°, n  111°. Apply the Cyclic Quadrilateral
2(105°)  210° (Inscribed Angle Conjecture). Conjecture and the Inscribed Angle Conjecture.
Therefore g  110°  210°, and g  100°. Because opposite angles of a cyclic quadrilateral are
supplementary, the angle opposite the angle marked
7. w  50°. The angle that intercepts the angle that is as a right angle is also a right angle. This is an
marked as 130° is a central angle, so its measure is inscribed angle that intercepts an arc of measure
130°, and the measure of its vertical angle is also 130° m  40°, so 90°  12(m  40°) (Inscribed Angle
(Vertical Angles Conjecture). This vertical angle, also Conjecture). Therefore m  40°  180°, and
a central angle of the circle, is one of the angles of m  140°. The measure of the fourth (unmarked)
the smallest quadrilateral that contains the angle of arc of the circle is 360°  140°  40°  98°  82°.
measure w. Also, two of the angles of the quadrilat- The angle of measure n is an inscribed angle that
eral are right angles because they are formed by radii intercepts an arc of measure m  82°  140° 
drawn to points of tangency (Tangent Conjecture). 82°  222°, so n  12(222°)  111°.
Therefore, by the Quadrilateral Sum Conjecture,
w  90°  130°  90°  360°, so w  50°. 13. p  71°, q  41°. First apply the Parallel Lines
Intercepted Arcs Conjecture. Because AB   CD
,
8. x  148°. CALM is a rectangle, so its opposite sides   BD
AC , so mBD   p. Next, mAB   120° and
are parallel. (A rectangle is a parallelogram.) mCD  98° because these are the measures of their
Because CM  AL
, CA  ML  by the Parallel Lines
  mCA  central angles. Adding all the arc measures in the
Intercepted Arcs Conjecture, so mML circle, 120°  2p  98°  360°, so 2p  142°, and
  
  ML , CM  AL by the
32°. Likewise, because CA p  71°. Now look at the triangle whose vertices are
Parallel Lines Intercepted Arcs Conjecture, so C, D, and the center of the circle. Because two of its
  mAL
mAL   x. Therefore 2x  2(32°)  360°,
sides are radii and all radii of a circle are congruent,
so 2x  296°, and x  148°. (Note: You could also this is an isosceles triangle with vertex angle of
use the Chord Arcs Conjecture to conclude that measure 98°. Therefore 2q  98°  180°, so
  CA
AL  and CM   AL  because opposite sides
2q  82°, and q  41°.
of a rectangle, or any parallelogram, are congruent.)
14. 180°. Each of the five angles whose vertices are the
  OW
9. y  44°. Because DOWN is a kite, DO , so points of the star is an inscribed angle, so the
 
DO  WO (Chord Arcs Conjecture), and therefore
  mWO   y. Likewise, DN measure of each of them is half the measure of its
mDO   WN, so intercepted arc. Notice that the five intercepted arcs
  
DN  WN , and therefore mWN  mDN   136°.
add up to one complete circle (360°). Therefore
Thus 2y  2(136°)  360°, so 2y  88°, and a  b  c  d  e  12(360°)  180°. (Another way
y  44°. to approach this exercise is to recall your work with
10. k  142°. The triangle is isosceles with vertex angle star polygons in the Sketchpad Exploration in
of measure 38°, so each of its base angles measures Chapter 5. There, you found that the sum of the
1 1 measures of the angles of a 5-pointed star is 180°.)
(180°  38°)  (142°)  71°. Each of the base
2 2
angles of the isosceles triangle is an inscribed angle 15. 75°. Label the figure as shown. 80°
in the circle, and one of these angles intercepts the The angles with measures a
arc of measure k. Therefore, by the Inscribed Angle and b are both inscribed
Conjecture, 71°  12k, or k  2(71°)  142°. angles, so a  12(70°)  35°,
y a
1
11. r  120°, s  60°. Notice that the sides of the and b  2(80°)  40° 70° b

hexagon are chords of the circle. Because the (Inscribed Angle Conjecture).
hexagon is equilateral, the six arcs intercepted by Notice that the angle with measure y
is an exterior angle of the triangle, with a and b the
these chords are all congruent. Therefore 6s  360°,
measures of its remote interior angles. Therefore
so s  60°. Also, the angle with measure r is an
y  a  b (Triangle Exterior Angle Conjecture), so
inscribed angle that intercepts an arc of measure y  35°  40°  75°.
4s  240°, so r  12(240°)  120° (Inscribed Angle
Conjecture). (Notice that each interior angle of the

Discovering Geometry Solutions Manual CHAPTER 6 103


©2003 Key Curriculum Press
DG3SM586_097-135.qxd 8/2/06 5:56 PM Page 104

16. The two inscribed angles intercept the same arc, so (5  2)


a    108°
 180°
they should be congruent (Inscribed Angles Inter- 5
cepting Arcs Conjecture). In the figure, they have by the Equiangular Polygon Conjecture. Then,
different measures, so they are not congruent. b  180°  108°  72° (Linear Pair Conjecture).
17. BFE  DFA (Vertical Angles Conjecture), and (Also, the angle with measure b is an exterior angle
360°
BGD  FHD (all right angles congruent). of the equiangular polygon, so b   5  72°.)


Therefore B  D (Third Angle Conjecture). Look at the triangle that contains the angles with
B and D are inscribed angles, so by the measures b and c. The unmarked angle in this
, and
Inscribed Angle Conjecture, mB  12mAC triangle is also an exterior angle of the equiangular
1  pentagon, so its measure is also b. Therefore the
mD  2mEC . Because mB  mD, it follows
  EC
that AC . triangle is isosceles, and 2b  c  180°, and c  36°.
The angle with measure d is a vertical angle of one
18. Possible answer: Place the corner so that it is an of the interior angles of the equiangular pentagon,
inscribed angle. Trace the inscribed angle. Use the so d  a  108°. Because 1  2, e  d (AIA
side of the paper to construct the hypotenuse of the Conjecture), so e  108°. Notice that e and f are the
right triangle (which is the diameter). Repeat the measures of consecutive angles of a parallelogram,
process. The place where the two diameters intersect so e  f  180° (Parallelogram Consecutive Angles
is the center. Conjecture), and f  72°. Also, e and g are opposite
19. Possible answer: angles of a parallelogram, so g  e  108° (Parallel-
ogram Opposite Angles Conjecture). The angle with
measure h forms a linear pair with a right angle, so
h  90° (Linear Pair Conjecture). To find l, look at
the triangle that contains the vertical angle of the
angle with measure l. The other two angles of this
triangle are exterior angles of the equiangular
pentagon, so this is an isosceles triangle with base
It works on acute and right triangles. To use this angles of measure 72°. So the third angle measures
method on an obtuse triangle, you must construct 180°  2(72°)  36°, and therefore l = 36° (Vertical
the circle with the longest side of the triangle as the Angles Conjecture). Now, look at the small triangle
diameter; the altitudes will intersect the circle at the whose angles measure n, h, and l. n  h  l  180°,
extensions of the sides. so n  90°  36°  180°, and n  54°. Next look at
the small triangle that includes the angle of measure
20. The camera can be placed anywhere on the major m. One of the other angles of this triangle is a
arc (measuring 268°) of a circle such that the row of corresponding angle of the angle with measure h,
students is a chord intersecting the circle to form a and the other is the vertical angle of the angle of
minor arc measuring 92°. This illustrates the measure f. Therefore these angles measure 90° (CA
Inscribed Angles Intercepting Arcs Conjecture, Conjecture) and 72° (VA Conjecture), so m  90° 
which says that inscribed angles that intercept the 72°  180°, and m  18°. Finally, look at the large
same arc are congruent. triangle whose angles measure p, e, and c. Here p 
e  c  180°, so p  108°  36°  180°, and thus
Possible location p  36°.
for the camera
92°
46° Students
  RS
23. It is given that PQ . OP  OQ   OR
  OS

because all radii in a circle are congruent. Therefore
OPQ  ORS by SSS, and 2  1 by CPCTC.
21. You will get two congruent Now look at the smaller right triangles inside
externally tangent circles with OPQ and ORS. It is given that OT   PQ  and
half the diameter of the   RS
OV . Therefore OTQ and OVS are right
original circle. angles by the definition of perpendicular lines,
and OTQ  OVS because all right angles are
congruent. Thus OTQ  OVS by SAA, and OT 
 OV by CPCTC.
22. a  108°, b  72°, c  36°, d  108°, e  108°,
f  72°, g  108°, h  90°, l  36°, m  18°, 24. Start with an equilateral triangle whose vertices are
n  54°, and p  36°. First, a is the measure of the centers of the three congruent circles. Then
an interior angle of an equiangular pentagon, so locate the incenter/circumcenter/orthocenter/centroid

104 CHAPTER 6 Discovering Geometry Solutions Manual


©2003 Key Curriculum Press
DG3SM586_097-135.qxd 8/2/06 5:56 PM Page 105

(all the same point because the triangle is equilat- D. Points A and B will not move, so AB will not
eral) to find the center of the larger circle. To find change when P moves. Therefore the perimeter of
the radius, construct a segment from the incenter of ABP will increase or decrease depending on the
the triangle through the vertex of the triangle to a change in AP  PB. The perimeter decreases until P
point on the circle.  (so that ABP
is directly above the midpoint of AB
is isosceles) and then increases.
Thus there is no length or distance described in
A–D that always increases as P moves from left to
right, so the correct response is E.
IMPROVING YOUR REASONING SKILLS
The second vowel in dinosaur is o, and the letter that
is three letters after o is u. The sixteenth letter of the
alphabet is p. In the alphabet, u comes after p, not
before it. Therefore the word dinosaur should be printed
25. Look at the three smaller triangles inside ABC, vertically. The first vowel is i, and the second letter after
each of which shares a side with one of the equian- i is o, so o should be crossed out.
gular polygons. Notice that in each case, two of the d
angles of the smaller triangle are exterior angles i
of the equiangular polygon; therefore each of the n
small triangles is isosceles, with angles A, B, and C —
o
as their vertex angles. s
The polygon near A is an equiangular hexagon, a
so the measure of each of its exterior angles is u
360°
  60°, and therefore mA  180°  2(60°)  r
6
60°. (Thus the small isosceles triangle containing EXTENSIONS
A is equilateral.) The polygon near B is an
equiangular pentagon, so the measure of each A. Research results will vary.
360°
of its exterior angles is 5  72°, and mB 
 B. See the solutions for Take Another Look activities 3
180°  2(72°)  36°. The polygon near C is an and 4 on page 120.
equiangular octagon, so the measure of each of its
360°
exterior angles is 8  45°, and mC  180° 

LESSON 6.4
2(45°)  90°.
Using the angle measures that you have found, EXERCISES
mA  mB  mC  60°  36°  90°  186°. 1. Paragraph Proof: Let z  mMDR, x  mMDK,
This is impossible because the sum of the measures and w  mKDR.
of the angles in any triangle is 180° (Triangle Sum mKDM  mKDR  mRDM (or x  w  z)
Conjecture). by angle addition.
  mRK
mMR   mMK  by arc addition.
26. E. Consider each of the options listed as choices
 and w  1mRK
z  12mMR  by Case 1.
A–D. 2
  1mMR
x  12mRK  by substitution.
A. The distance PB will decrease until P is directly 2
  mMR
x  12(mRK ) by factoring out the greatest
above B and then increase as P moves farther to
1
the right. common factor, 2.
 by substitution.
x  12mMK
B. The distance from D to line AB is the distance
Therefore mMDK  12mMK  by substitution.
between the parallel lines AB and CD. This will not
change as P moves. 2. Paragraph Proof: By the Inscribed Angle
 is the midsegment of ABC that connects  and also
Conjecture, mACD  12mAD
C. DC
1 
 and PB
the midpoints of PA . By the Triangle mABD  2mAD . Therefore, by substitution,
Midsegment Conjecture, DC  12(AB), so DC will mACD  mABD, so ACD  ABD by the
not change as P moves. definition of congruence.

Discovering Geometry Solutions Manual CHAPTER 6 105


©2003 Key Curriculum Press
DG3SM586_097-135.qxd 8/2/06 5:56 PM Page 106

Flowchart Proof (See flowchart at bottom of page.) Therefore L and C are supplementary. (A similar
3. Paragraph Proof: ACB is an inscribed angle that proof can be used to show that I and Y are
. Because AB
intercepts ADB  is a diameter of the supplementary.)
circle, it divides the triangle into two semicircles, so Flowchart Proof (See flowchart at bottom of page.)
 is a semicircle. So, by the Inscribed Angle
ADB
  1(180°)  90°. 5. Paragraph Proof: Angles 1 and 2 are inscribed
Conjecture, mACB  12mADB 2 angles, so, by the Inscribed Angle Conjecture,
Therefore ACB is a right angle. ) and m2  1(mBC ).
m1  12(mAD 2
Flowchart Proof (See flowchart at bottom of page.) Because AB  DC
, 1  2 by the AIA Conjec-
4. Paragraph Proof: By the Inscribed Angle   1mAD
ture. Therefore 12mBC , so mBC
  mAD .
, and mL  1mYCI
Conjecture, mC  12mYLI   
2
1  1  2 BC  AD by the definition of congruence.
(360°  mYLI )  180°  mYLI  180°  mC.
2 2
Flowchart Proof (See flowchart at bottom of page.)

Lesson 6.4, Exercises 2, 3, 4, 5


2. 1 2 1
⬔ACD is inscribed m⬔ACD  _ AD
 2
in ACD
Inscribed Angle
Given 5 6
Conjecture m⬔ACD  m⬔ABD ⬔ACD  ⬔ABD
3 4 1 Substitution Definition of
⬔ABD is inscribed m⬔ABD  _ AD
 2 congruent angles
in ACD
Inscribed Angle
Given
Conjecture

3.
1  2 
AB is a diameter ADB is a semicircle;

mADB  180°
Given
Definition of semicircle

3 4 1  5
⬔ACB is inscribed m⬔ACB  _ mADB  ⬔ACB is a right angle
in semicircle ACB 2
_1 (180°)  90° Definition of right angle
Given 2
Inscribed Angle Conjecture

4. 1 1 
m⬔C  _ mYLI
2 4 m⬔C  m⬔L 
Inscribed Angle _1  _1 
Conjecture 2 mYLI  2 mYCI 

_1 (mYLI 
2  mYCI )
2 1 
m⬔L  _ mYCI
2 Addition and distributive 5 6
properties m⬔L  m⬔C  ⬔C and ⬔L are
Inscribed Angle _1 (360°)  180° supplementary
Conjecture 2
Definition of
3   Substitution
mYLI  mYCI  360° supplementary

Arc addition

5. 1   2
AB  CD ⬔1  ⬔2
Given AIA Conjecture

3 1  5  _ 1  6   7  
m⬔1  _ mAD _1 mAD  mBC mAD  mBC AD  BC
2 2 2
Multiplication Definition of
Inscribed Angle Substitution
property congruent arcs
Conjecture

4 1 
m⬔2  _ mBC
2
Inscribed Angle
Conjecture

106 CHAPTER 6 Discovering Geometry Solutions Manual


©2003 Key Curriculum Press
DG3SM586_097-135.qxd 8/2/06 5:56 PM Page 107

6. True. measure of 39° (Isosceles Triangle Conjecture).


Paragraph Proof: Opposite angles of a parallelo- Notice that this second base angle (lower left angle
gram are congruent (Parallelogram Opposite Angles of triangle) and the angle marked 39° are corre-
Conjecture), and opposite angles of any quadrilat- sponding angles formed when lines m and n are cut
eral inscribed in a circle are supplementary (Cyclic by one of the transversals. Because the correspond-
Quadrilateral Conjecture). If two angles are both ing angles are congruent, m  n by the Converse of
congruent and supplementary, each of them must the Parallel Lines Conjecture.
be a right angle. Here, there are two pairs of oppo- 2
10. . In Exercise 23 of Lesson 6.1, you worked with
21
site angles that are congruent and supplementary, so the same 3-by-3 grid and determined that the
GOLD has four right angles. The definition of a number of ways to select three points from the nine
rectangle says that a rectangle is a parallelogram 9(8)(7)
points in the grid is 1(2)(3)  84. Now, count the

with four right angles, so GOLD is a rectangle.
number of triples of collinear points: There is a
Flowchart Proof (See flowchart at bottom of page.) collinear triple across each row (3), down each
7. True. column (3), and along each diagonal (2). Therefore
Paragraph Proof: Start with the Parallel Lines there are 8 triples of collinear points out of 84
Intercepted Arcs Conjecture, which guarantees that selections of any three points, so the probability of
  ET
GA . GA  ET by the Converse of the Chord selecting three collinear points is 884  221 .
Arcs Conjecture. Thus, by definition, GATE is an
11. 6. Make an orderly list of the routes, and then count
isosceles trapezoid.  to AL to LG, RA to AN
 to
them. The routes are RA
Flowchart Proof (See flowchart at bottom of page.) , RE
NG  to EC
 to CG, RE  to EN to NG, RT
 to TC

8. a. Sometimes. An equilateral triangle is equiangular, , and RT
to CG  to TL to LG .
but a rhombus is not equiangular. IMPROVING YOUR VISUAL THINKING SKILLS
b. Always The rotating quarter makes two full turns. Visualize
what would happen if the two quarters were enmeshed
c. Never (Only one diagonal is the perpendicular
gears and were turning together. Both would be facing
bisector of the other.)
the same way after a half turn. The same thing happens
d. Always when one of the two quarters is stationary.
e. Sometimes. An equilateral triangle has rota-
tional symmetry and three lines of reflectional USING YOUR ALGEBRA SKILLS 6
symmetry; a parallelogram has rotational
EXERCISES
symmetry, but no line of reflectional symmetry.

1. y  23x  133 . The perpendicular bisector of RE
9. Look at the isosceles triangle that is formed when 
is the line perpendicular to RE and passing
the two slanted lines intersect line n. One of the
through its midpoint. The midpoint of RE  is
base angles of this triangle forms a linear pair with 0  4 0  (6)
the 141° angle, so its measure is 180°  141°  39°. 2, 2  (2, 3), and the slope of RE  is
Therefore the other base angle must also have a

Lesson 6.4, Exercises 6, 7


6. 1
⬔G  ⬔L;
⬔O  ⬔D
3 4
Parallelogram Opposite ⬔G and ⬔L GOLD is a rectangle
Angles Conjecture are right angles;
Definition of rectangle
⬔O and ⬔D
2 are right angles
⬔G and ⬔L are
supplementary; If two angles are congruent
⬔O and ⬔D are and supplementary,
supplementary they are right angles
Cyclic Quadrilateral
Conjecture

7. 1   AT
 2   3   ET
 4
GE GA  ET GA GATE is isosceles
Given Parallel Lines Converse of Definition of
Intercepted Arcs Chord Arcs isosceles trapezoid
Conjecture Conjecture

Discovering Geometry Solutions Manual CHAPTER 6 107


©2003 Key Curriculum Press
DG3SM586_097-135.qxd 8/2/06 5:56 PM Page 108

6  0 6 3 To solve this system by the substitution method,



40  4   2 . The slope of the perpendicular
 
bisector is the negative reciprocal of 32, which is 23. substitute 2x  1 for y in the first equation, and
solve for x.
Find the equation in the form y  mx  b of the
line with slope 23 passing through (2, 3). 2x  1  3x  5
y  (3) 2 x4
  
x2 3
y3 2 Now substitute 4 for x in the second equation, and
   solve for y.
x2 3
3(y  3)  2(x  2) y  2(4)  1  7
3y  9  2x  4 The solution of the system gives the coordinates of
3y  2x  13 the point where the two perpendicular bisectors
intersect, which is the circumcenter of the triangle.
2 13
y  3x  3 Therefore the circumcenter of TRM is (4, 7).
2. (4, 7). Find the perpendicular bisectors of any two 3. 14 , 7. Follow the method given in the solution
69 6
sides of the triangle, and then find the point where for Exercise 2. In this solution, the perpendicular
they intersect. Any two of the perpendicular bisec-  and FH
bisectors of FG  will be found, but you
tors can be chosen; in this solution, the perpendicu- could choose any pair of perpendicular bisectors.
 and RM
lar bisectors of TR  will be used to find the
circumcenter.  
Midpoint of FG 
0  3 6  6
2
,   , 0
2   
3
2
.
First find the perpendicular bisector of TR 6  (6)
    1 2
Slope of FG 30 3 4
Midpoint of TR  
2  4 1  3
2 2 
,   (1, 2)

The slope of the perpendicular bisector of FG
 31 2 1 1
Slope of TR     
4  (2)  6  3 is 4.

The slope of the perpendicular bisector of TR  is the Find the equation in the form y  mx  b for the
negative reciprocal of 13, which is 3. Find the line with slope 14 that passes through the point
2, 0.
3
equation in the form y  mx  b for the line with
slope 3 that passes through (1, 2). y0 1

y2  3  4
  3 x  2
x1
y  2  3(x  1) y 1

 3  4
y  2  3x  3 x  2

y  3x  5  3
4y  1 x  2
Now find the perpendicular bisector of RM. 4y  x  2
3
4  (4) 3  (1)
Midpoint of RM 
  ,   (0, 1)
2 2  1 3
y  4x  8
1  3 4 1

Slope of RM     
4  4  8  2 Midpoint of FH  0  12 6  0
2 
,   (6, 3)
2
The perpendicular bisector of RM  has slope 2 and 0  (6)
    6  1
Slope of FH
y-intercept 1, so its equation is y  2x  1. 12  0 12 2
Now solve the system formed by the equations of  is
The slope of the perpendicular bisector of FH
the perpendicular bisectors. 2. Find the equation in the form y  mx  b for
the line with slope 2 that passes through (6, 3).
 yy  3x 5
2x  1 y  (3)
  2
x6
y  3  2(x  6)
y  3  2x  12
y  2x  9

108 CHAPTER 6 Discovering Geometry Solutions Manual


©2003 Key Curriculum Press
DG3SM586_097-135.qxd 8/2/06 5:56 PM Page 109

Now solve the system formed by the equations of Substitute 54x  241 for y in the first equation.
the perpendicular bisectors. 5 21 14 31
1 3 x    x  
y  4x  8 4 4 3 2
y  2x  9
5
 21
 14
 31
12 4x  4  12 3x  2 
To solve this system by substitution, substitute 15x  63  56x  186
2x  9 for y in the first equation, and solve for x.
41x  123
1 3
2x  9  4x  8
x3

 1
8(2x  9)  8 4x  8
3
 To find the value of y, substitute 3 for x in the first
equation of the system.
16x  72  2x  3
14 31 31 28 31 3
14x  69 y  3(3)  2  14  2  2  2  2
69
x  1
4 The circumcenter is 3, 32.

Now substitute 69
 for x in the second equation, and 5. 9 
2, 0. Find the perpendicular bisectors of CD
14
solve for y. , and then find their point of intersection.
and CE
0  0 6  (6)
 
69
y  2 1
69 63
 
6

4  9  7  7  7
  
Midpoint of CD 
,   (0, 0)
2 2 
 .
69
Therefore the circumcenter of FGH is ,
14 67 Notice that C and D both lie on the line x  0 (the
 y-axis), a vertical line with undefined slope. Any line
4. 3, 32. Find the perpendicular bisectors of MO
perpendicular to a vertical line is a horizontal line.
, and then find their point of intersection.
and NO  will be the hori-
The perpendicular bisector of CD
4  10 0  (3)
Midpoint of MO     2
, 
2  zontal line through its midpoint, which is the
origin, so its equation is y  0; thus the perpen-
 is the x-axis.
 3, 2
3 dicular bisector of CD

  3  0
 
3
 
Midpoint of CE 
0  12 6  0
2
,   (6, 3)
2 
Slope of MO 10  (4)  14 0  6 1
    
Slope of CE
Slope of perpendicular bisector of MO   1 4 120 2
3
y  2
3  is the line
The perpendicular bisector of CE
14
Solve the equation  x  3  3 for y to rewrite

through (6, 3) with slope 2. Solve the equation
this equation in the form y  mx  b: y3
  2 to rewrite this equation as y  2x  9.
y  134 x  321 . x6
To find the circumcenter, solve the system of
0  10 5  (3)
Midpoint of NO 
  
2 2 
,   (5, 1) equations formed by the equations of the two
perpendicular bisectors.
   3  5 8 4
  
Slope of NO 10  0  10  5
Slope of perpendicular bisector of NO   5
y0
y  2x  9
4
y1
You already know that y  0. Substitute 0 for y in
5
x  5  4 for y to obtain
Solve the equation    the second equation, and solve for x.
5 21
y  4x  4.
0  2x  9
Now solve the system formed by the equations of 9
the two perpendicular bisectors. x  2
2, 0.
9


14 31 The circumcenter is
y  3x  2
5 21
y  4x  4

Discovering Geometry Solutions Manual CHAPTER 6 109


©2003 Key Curriculum Press
DG3SM586_097-135.qxd 8/2/06 5:56 PM Page 110

6. The midpoint of the hypotenuse is the circum- y  mx  b of AB , which is the line with slope
3
center. Notice that in Exercise 4, the circumcenter  4 that passes through A(3, 2).
 
of the right triangle, 3, 32, was the same point
. Because y  (2) 3
as the midpoint of one of the sides, MO   
 is 4 and the slope of MN  is 5, x  (3) 4
the slope of NO
  MN
5
, so N is the right angle, and MO  is
4 y  2 3
NO   
x3 4
the hypotenuse.
4(y  2)  3(x  3)
To see why the midpoint of the hypotenuse of any
right triangle is the circumcenter, look at a right 4y  8  3x  9
triangle inscribed in a circle. 4y  3x  17
C 3 17
y  4x  4
B  and PA
QB  lie on parallel lines because both are
perpendicular to AB , so slope of QB
  slope of
  4. Find the equation of QB
PA 3
, which is the line
O with slope 43 passing through Q(11, 0).
y0 4
  
A x  11 3
3y  4(x  11)
Here, C is a right angle because it is inscribed in a 3y  4x  44
semicircle (Angles Inscribed in a Semicircle Conjec- 4 44
y  3x  3
 is both a diameter of the circle and
ture), and AB
the hypotenuse of the right triangle. Because OA, Now solve the system formed by the equations of
 
OB , and OC are all radii of the circle, OA  OB   and QB
AB .


OC, so O is equidistant from the three vertices of 3 17
y  4x  4
the triangle and is the circumcenter of the triangle
4 44
(which is the center of the inscribed circle). Because y  3x  3
O is also the midpoint of the hypotenuse, AB , this
shows that in a right triangle the midpoint of the Solve this system by substitution:
hypotenuse is the circumcenter. 4 44 3 17
x    x  
3 3 4 4
7. The intersection of the altitude from the vertex
angle and the perpendicular bisector of one equal 
4 44
 3 17
12 3x  3  12 4x  4 
side is the circumcenter. In any isosceles triangle, 16x  176  9x  51
the altitude from the vertex angle to the base is
also the median to the base, so it lies on the 25x  125
perpendicular bisector of the base. Therefore the x5
intersection of the altitude from the vertex angle
and the perpendicular bisector of one of the legs Substitute 5 for x in the first equation of the system.
is the circumcenter of the triangle. (It may be 3 17 15 17 32
y  4(5)  4  4  4  4  8
easier to find the equation of an altitude than a
perpendicular bisector because there are fewer Therefore B has coordinates (5, 8).
steps involved.)
 and QB
8. (5, 8). Find equations for AB , and then LESSON 6.5
find the coordinates of their intersection, which is
point B. EXERCISES
2  (6) 1. d  5 cm. Substitute 5 cm for C in the formula
    4
Slope of AP 3  (6) 3 C  d, and solve for d.

Because AB  AP  (Tangent Conjecture), the slope
 3 C  d
of AB is 4. Find the equation in the form
5  d
d  5 cm

110 CHAPTER 6 Discovering Geometry Solutions Manual


©2003 Key Curriculum Press
DG3SM586_097-135.qxd 8/2/06 5:56 PM Page 111

2. C  10 cm. Substitute 5 cm for r in the formula 13. 16 in. Sketch a circle circumscribed about a square.
C  2r, and solve for C.
C  2r
C  2(5)  10 cm
3. r  12 m. Substitute 24 m for C in the formula
C  2r, and solve for r.
C  2r
24 m  2r Notice that the diameter of the circle is a diagonal
24 12 of the square. Find the diameter of the circle:
r  
2   m C  d, so 16  d, and d  16 in. Therefore the
11 length of the diagonal of the square is also 16 in.
4. C  5.5 m, or 
2 m. C  d  (5.5)  5.5 m,
11
or  
2 m. 14. 244 yr. Possible answer: The size of the ring indi-
cates the amount of growth in a particular year,
5. C  12 cm. C  d  (12)  12 cm.
which varies from year to year depending on
6. d  46 m. Use the formula C  d: 46  d, weather patterns.
so d  46 m.
To find the age of “Old Fred,” first find its radius.
7. C  15.7 cm. C  d  (5)  5 cm  15.7 cm.
C  2r
8. C  25.1 cm. C  2r  2(4)  8 cm 
766  2r
25.1 cm.
766
r 
2 122 cm
9. r  7.0 m. Use the formula C  2r: 44  2r,
so r  24
4
 22 m  7.0 m. If each ring has a thickness of 0.5 cm, the tree will
122 cm
have about  0.5 cm  244 rings and is therefore

10. C  84.8 in. C  d  (27)  27 in.  84.8 in.
about 244 years old.
11. A seated person travels about 565 ft. The distance
15. 1399 tiles. Find the perimeter of the figure. Add
from the center to one of the seats is the radius of
the lengths of the two 30-foot sides of the rectangle
the Ferris wheel, and the distance traveled by a
and the circumference of two semicircles each
seated person in one revolution is the circumfer-
with diameter 18 ft, which is the same as the
ence, so find the circumference of a circle with
circumference of one circle with diameter 18 ft:
radius 90 ft. C  2r  2(90)  180 ft  565 ft.
2(30)  (18)  (60  18) ft  12(60  18) in.
12. C  6 cm. Sketch a circle inscribed in a square. 1399 in. Therefore 1399 1-inch tiles will be
needed.
16. Conjecture: The measure of the angle formed by
two intersecting chords is equal to one-half the sum
of the measures of the two intercepted arcs. In the
diagrams, mNEA  12(mAN   mGL ).

You could also say, “The measure of the angle


formed by two intersecting chords is the average of
Notice that the diameter of the circle has the same the measures of the two intercepted arcs.”
length as a side of the square. The perimeter of the 17. mALN  12mAN and mLAG  1mLG  by the
2
square is 24 cm, so the length of each side is 6 cm. Inscribed Angle Conjecture. mNEA  mALN 
Therefore the diameter of the circle is also 6 cm, mLAG by the Triangle Exterior Angle Conjecture.
and the circumference of the circle is d  6. )  1(mGL
By substitution, mNEA  12(mAN ) 
1  
(mAN  mGL ).
2
2
  MT
18. 1. MA ; 2. SA
  ST; 3. Definition of kite;
  by the
4. MS is the perpendicular bisector of AT
Kite Diagonal Bisector Conjecture.

Discovering Geometry Solutions Manual CHAPTER 6 111


©2003 Key Curriculum Press
DG3SM586_097-135.qxd 8/2/06 5:56 PM Page 112

19. b  90°, c  42°, d  70°, e  48°, f  132°, and g  LESSON 6.6


52°. First, b  90° by the Cyclic Quadrilateral Conjec-
ture. Next, c  42° (Inscribed Angles Intercepting EXERCISES
Arcs Conjecture). Now look at the right triangle in 1.  4398 km/hr
which the measures of the acute angles are e and 42°. distance circumference
speed    
time 
Here, e  90°  42°  48°. Then f  180°  48°  12 hr
132°. Next apply the Quadrilateral Sum Conjecture 2(2000 km  6400 km)
 
to the quadrilateral in which the angle measures are 12 hr
d, f, 68°, and 90°: d  132°  68°  90°  360°, so 4398 km/hr
d  70°. Finally, look at the triangle that contains the
76° angle and the angle of measure g. This triangle is 2. 11 m/sec
isosceles with vertex angle of measure 76° (Tangent distance circumference
speed    
time  16 sec
Segments Conjecture). Therefore 2g  76°  180°,
2  28 m
so 2g  104°, and g  52°.  16 sec  11 m/sec

 
150° 5 1
20.  , or . mAB  2 22°  45° (Inscribed 3. 37,000,000 revolutions. First convert 60 cm to
360° 12 2
Angle Conjecture). mA  2212°  105°  180° kilometers: 60 cm  0.6 m  0.0006 km. The diam-
(Triangle Sum Conjecture), so mA  5212°, and eter of the tire is 0.0006 km, so the circumference,
  2 521°  105° (Inscribed Angle Con- which is the length of one revolution, is d 
mCD  2
jecture). Then mAB   mCD   45°  105°  150°. 0.0006 km.
Therefore the probability that a random point on Let x represent the number of revolutions of the tire
 or CD
the circle will lie on either AB  is 
150°

5
 before the warranty is up, and write a proportion;
360° , or 12 .
then solve the proportion.
21. a  b  b  a  180°, so 2a  2b  180°, or
1 revolution x revolutions
2(a  b)  180°, and therefore a  b  90°.   
0.00006 km 70,000 km
22. 10x  2y. Look for a pattern in the lengths of the 70,000  0.00006x
outside edges, whose sum is the perimeter of each 70,000
x 
0.0006 37,000,000
figure. The nth shape has n outside edges of length
x. If n is odd, there is 1 edge of length y and 1 of The tire will make about 37,000,000 revolutions
length w, whereas if n is even, there are 2 edges of before the warranty is up.
length y and none of length w. For the tenth shape,
4. 637 revolutions. 1 revolution  d  0.5 m.
n  10, an even number, so there are 10 edges of
Use a proportion.
length x and 2 of length y. Therefore the perimeter
of the tenth shape is 10x  2y. 1 revolution x revolutions
  
0.5 m 1000 m
PROJECT 1000  0.5x
Project should satisfy the following criteria:
2N
x 637
● For a large number of trials, the ratio  will be close
C
to . The tire will make about 637 revolutions.
● Explanations will take into account that the experi- 5. Mama; C 50 in. First, find the circumference of
ment involves lines and segments, but the result each pizza.
suggests a circle. They might suggest that the different
Baby Bear: C  2r  12 in.
possible orientations of the toothpick lie in a circle.
● Student does experiments with different line widths Mama Bear: C  2r  16 in.
and finds that for a toothpick with length L tossed on Papa Bear: C  d  20 in.
2L
lines a distance D apart, P(crossing a line)   
D . (If L
is longer than D, however, calculating the probability Now use the prices for the three pizzas to find the
is more complicated and requires calculus.) number of inches of edge per dollar for each pizza.
EXTENSIONS 12 in.
Baby Bear:  
$9.75 3.87 in./dollar
A. Research results will vary. 16 in.
Mama Bear:  
$12.00 4.19 in./dollar
B. Ratios should be close to .
20 in.
Papa Bear:  
$16.50 3.81 in./dollar
C. See the solution for Take Another Look activity 5
on page 120.
D. Poems will vary.

112 CHAPTER 6 Discovering Geometry Solutions Manual


©2003 Key Curriculum Press
DG3SM586_097-135.qxd 8/2/06 5:56 PM Page 113

Therefore the Mama Bear pizza has the most pizza 11. Both triangles are isosceles, so the base angles in
edge per dollar. Its circumference is 16 in. each pair are congruent (Isosceles Triangle Conjec-
50 in. ture). Also, the unmarked base angles of the two
triangles are vertical angles, so they are congruent
6.  168 cm. Find the circumference of the tree, and
(Vertical Angles Conjecture). Therefore a  b by
then use the circumference to find the diameter.
transitivity.
The circumference is about 138  136  128  126
C 528
 528 cm. Then, d      168 cm, so the
 12. C. The measure of ABP increases as it changes
diameter of the redwood tree is about 168 cm. from an acute angle to a right angle to an obtuse
angle. The other measures either increase and then
7. d 7.6 ft. The table will fit, but the chairs may be
decrease or do not change as P moves.
a little tight in a 12-by-14 ft room. 12 chairs 
192 in., and 12 spaces  96 in., so the circumfer- 13. 38°. The conjecture discovered in Lesson 6.5,
ence of the circle should be C  288 in. For this Exercise 16 and proved in Exercise 17 says that
C 288
table, d       91.7 in.  7.6 ft.
 the measure of an angle formed by two
intersecting chords is equal to one-half the sum
8.  0.35 ft/sec. Calculate the distance traveled in one
of the intercepted arcs. Applying this conjecture,
revolution, which is the circumference, for each
a  12(32°  44°)  12(76°)  38°.
record.
14. 48°. By the Parallel Lines Intercepted Arcs Conjec-
45 rpm record: d  7 in., so C  7 in.
ture, the unmarked arc has measure b. Therefore
33 rpm record: d  12 in., so C  12 in. 2b  96°  168°  360°, so 2b  96°, and b  48°.
Now, find the speed of a point on the edge of each 15. 30 cm. The figure is a trapezoid, and the segment of
record. length 24 cm is its midsegment. Therefore 24 
1
(d  18) by the Trapezoid Midsegment Conjecture,
45 rpm record: 2
so 48  d  18, and d  30 cm.
distance (7 in./rev)(45 rev)
speed    
time  1 rev  60 sec 544.5 ft/sec
400 rev 26 ft 1 min
1 min 16.   
1 min
 315 in./min 17. 12 cm  third side  60 cm. By the Triangle
Inequality Conjecture, the length of the third side
33 rpm record:
must be between 36  24  12 cm and 24  36 
distance (12 in./rev)(33 rev) 60 cm.
speed    
time  1 min
IMPROVING YOUR VISUAL THINKING SKILLS
 396 in./min
1. Rule: (n  1)n  1  n2  n  1
So the difference in speeds between the two records
is 396  315  81 in./min. Now, convert this
difference from in./min to ft/sec, and use your
calculator to approximate :
81 in. 1 ft 1 min

min 
12 in.  60 sec 0.35 ft/sec
  2. Rule: (n  2)n  2  n2  2n  2
So the difference in speeds between points on the
edges of the 33 rpm record and the 45 rpm record
is about 0.35 ft/sec.
9. mECA  12(mSN   mEA ). Conjecture: The
measure of an angle formed by two intersecting
secants through a circle is equal to one-half the EXTENSION
difference of the larger arc measure and the smaller Problems and solutions will vary.
arc measure.
10. Draw SA  to form inscribed angles ESA and SAN. LESSON 6.7
mESA  12mEA and mSAN  1mSN  by the
2 EXERCISES
Inscribed Angle Conjecture. mSAN  mESA  4   80°, so CD is 8 0 2
360  9 of the circum-
1.  in. mCD 
mECA by the Triangle Exterior Angle Conjecture, 3
  1mEA
so 12mSN   mECA by substitution. So ference. r  3 in., so C  2r  2(3)  6 in.
12    1(mSN
  mEA). Therefore, by the Arc Length Conjecture, the length
mECA  2mSN  12mEA 2  is 2C  2(6)  4 in.
of CD 9 9 3

Discovering Geometry Solutions Manual CHAPTER 6 113


©2003 Key Curriculum Press
DG3SM586_097-135.qxd 8/2/06 5:56 PM Page 114

  120°, so EF is 
120 1
360  3 of the
2. 8 m. mEF   same as the circumference of one circle with a
circumference. r  12 m, so C  2(12)  24 m. diameter of 40 m. Thus the length of one complete
  1C  1(24)  8 m.
Therefore the length of EF lap is 2(100)  40  (200  40) m. The go-cart
3 3
 
3. 14 cm. mGB  150°, so mBIG  360°  150° 
covers four laps in 6 minutes. Find its speed:
 is  210 7 distance 4(200  40) m
360  12 of the circumference. r
  speed      217 m/min
time 
210°, and BIG

12 cm, so C  24 cm, and the length of BIG 6 min
15 min 1 1
7
(24)
12  14 cm. 10.  4200 mi. Because  90 min  6 , Polly has made 6 of
  
  is 1 of the circumference.
4. 9 m. mAB  120°, so AB
a trip around Earth during her lunch. The distance
3 along the equator over which she has passed is 16
Use the Arc Length Conjecture to find the radius.
of the circumference of Earth, or 16(8000) 
1 4200 miles.
6  3C
1 11. The desks are about 17 m from the center. Use the
6  3(2r)
Arc Length Conjecture.
18  2r 1
12  9C
r9m
1
 12  9(2r)
5. 6 ft. By the Inscribed Angle Conjecture, mRT

2(30°)  60°, so RT is  60 1 108  2r
360  6 of the circumference.
 
C  2r  2(18)  36 ft. Therefore the length of
 is 1(36)  6 ft. r  17 m
RT 6
6. 4 m. CO  is
 is a diameter of the circle, so CSO
About four desks will fit because an arc with one-

a semicircle, and mSO  180°  100°  80°.
half the radius and the same central angle will be
C  2r  18 m. Therefore the length of SO  one-half as long as the outer arc.
80 2
360 (18)  9 (18)  4 m.
is    12. The measure of the central angle is 7.2° because of
  2(80°)
7. 27 in. TAV is an inscribed angle, so mTV
the Corresponding Angles Conjecture. Therefore
360  C. So, according to this method, C 
7.2
 500   
 160° by the Inscribed Angle Conjecture, and TV
160 4 25,000, that is, the circumference of Earth is about
360  9 of the circumference. Use the Arc Length
is  
25,000 mi.
Conjecture to find the diameter.
4 13. 18°/sec. No, the angular velocity is the same at every
12  9C point on the carousel.
4 360°
12  9(d) angular velocity   
20 sec  18°/sec.
108  4d 14. Outer horse  2.5 m/sec, inner horse  1.9 m/sec.
One horse has traveled farther in the same amount
d  27 in.
of time (tangential velocity), but both horses have
8. 100 cm. By the Parallel Lines Intercepted Arcs rotated the same number of times (angular
  AR
Conjecture, CE , so 2mAR   146°  70°  velocity).

360°. Therefore 2mAR  144°, and mAR   72°, so
 72 1 Calculation of tangential velocities:
AR is 360  5 of the circumference. Use the Arc
 
Length Conjecture to find the radius. Outer horse:
1
40  5(2r) C  2r  2(8)  16 m
distance along circular path
200  2r tangential velocity   time
r  100 cm 16 m
 
20 sec 2.5 m/sec
9. 217 m/min. Sketch the racetrack. Inner horse:
100 m
C  2r  2(6)  12 m
distance along circular path
40 m tangential velocity   time
12 m
 
20 sec 1.9 m/sec
To make one complete lap around the track, a go-
15. a  70°, b  110°, c  110°, d  70°, e  20°,
cart must cover two straightaways and two semicir-
f  20°, g  90°, h  70°, k  20°, m  20°,
cles. The circumference of the two semicircles is the
n  20°, p  140°, r  80°, s  100°, t  80°,

114 CHAPTER 6 Discovering Geometry Solutions Manual


©2003 Key Curriculum Press
DG3SM586_097-135.qxd 8/2/06 5:56 PM Page 115

and u  120°. First, a  180°  110°  70° (Linear 17. 170°. Sketch the clock at 10:20, and observe the
Pair Conjecture). The segment on the left between position of the hands.
the two parallel lines is the midsegment of a trape-
zoid, so it is parallel to the bases of the trapezoid 12
11 1
(Trapezoid Midsegment Conjecture). Therefore
2
b  110° and c  110° (CA Conjecture). Also, by 10

the CA Conjecture, d  a, so d  70°. Next look 9 3


at the right triangle in the lower left. The acute 8 4
angles of this triangle have measures d and e, so 7 5
6
e  90°  d  20°. f  20° (Vertical Angles Conjec-
ture). Now look at the large isosceles triangle in
which f and k are the measures of the base angles. From 10:00 to 10:20, the minute hand has moved
The figure shows the median to the base of this
from the 12 to the 4, while the hour hand has
triangle. In an isosceles triangle, the median to the
base is also an altitude, so g  90°. f and h are the moved 13 of the way from the 10 to the 11 (because
measures of the acute angles of a right triangle, so 20 min  13 hr), which is 13112   316 of the way
h  90°  f  70°. Also, k  f (Isosceles Triangle around the clock. Because 316 (360°)  10°, this
Conjecture), so k  20°. Next, by the AIA Conjec-
corresponds to an arc or a central angle of 10°.
ture, m  k, so m  20°, and n  f, so n  20°. By
the Triangle Sum Conjecture, m  n  p  180°, so If the hands were at exactly 10 and 4, the angle
p  180°  2(20°)  140°. Now, e  2r  180°, so between them would be 180°, but because the hour
2r  160°, and r  80°. Now look at the trapezoid hand has moved 10° closer to the minute hand, the
in which c and s are opposite angles. (You know
angle between the hands is 180°  10°  170°.
that this is a trapezoid from the Trapezoid Midseg-
ment Conjecture.) By the Trapezoid Consecutive PROJECT
Angles Conjecture, s  180°  r  100°. By the CA Project should satisfy the following criteria:
Conjecture, t  r, so t  80°. Finally, look at the
● The track will have straightaways with lengths of
small triangle at the top of the figure in which all
about 320 meters.
three angles are unmarked. In this triangle, the
lower left angle forms a linear pair with the angle ● The runner in the inner lane starts at the finish line.
of measure t, so its measure is 180°  t  100°,
● Assumptions about the width of the lanes are stated.
and the lower right angle is the vertical angle of the
angle of measure m, so its measure is 20°. The angle ● If the lanes are 1 meter wide, the runner in the outer
with measure u is an exterior angle of this triangle, lane will start about 9.4 meters ahead of the runner in
with its remote interior angles being the two angles the inner lane.
whose measures have just been found. Therefore, by
● Complete explanations of all the factors used in the
the Triangle Exterior Angle Conjecture, u  100° 
calculations are given.
20°  120°.
16. The overlaid figure consists of two pairs of
EXTENSION
congruent equilateral triangles. The length of a side Results and explanations will vary.
in the smaller pair is half the length of a side in the
larger pair. All of the arcs use the lengths of the EXPLORATION • CYCLOIDS
sides of the triangles as radii. Possible construction:
EXTENSION
Possible answer:

Discovering Geometry Solutions Manual CHAPTER 6 115


©2003 Key Curriculum Press
DG3SM586_097-135.qxd 8/2/06 5:56 PM Page 116

CHAPTER 6 REVIEW 9. 91°. The angle marked as a right angle intercepts a


semicircle and is therefore inscribed in the opposite
EXERCISES semicircle, so d  89°  180°, and d  91°. (You
1. Possible answer: The Inscribed Angle Conjecture is could also use the Cyclic Quadrilateral Conjecture
very important because several other conjectures to see that the angle opposite the marked right
build on it, and it can be used in many different angle is the supplement of a right angle, and there-
situations. fore it is also a right angle. Then the intercepted arc
of this right angle must measure 180°, so d  89° 
2. Possible answers:
180°.)
With compass and straightedge: Draw two nonpar-
10. 66°. Look at either of the angles that form a linear
allel chords, and construct their perpendicular
pair with the 88° angle. The supplement of an 88°
bisectors. The intersection of their perpendicular
angle is a 92° angle. Because the measure of an
bisectors is the center of the circle.
angle formed by two intersecting chords is one-half
With patty paper: Fold the paper along a diameter the sum of the measures of their intercepted arcs
so that two semicircles coincide. Repeat with a (see Exercise 8), 92°  12(f  118°), so 184°  f 
different diameter. The center is the intersection of 118°, and f  66°.
the two folds.
11. 125.7 cm. C  2r  2(20)  125.7 cm.
With the right-angled corner of a carpenter’s 132
12. 42.0 cm. C  d, so 132  d, and d   

square: Place the corner in the circle so that it is an
42.0 cm.
inscribed right angle. Trace the sides of the corner.
13. 15 cm. mAB  100° (Chord Arcs Conjecture),
Use the square to construct the hypotenuse of the
right triangle (which is the diameter of the circle). so by the Arc Length Conjecture, the length of
 is 
360° C  18 (2  27)  15 cm.
100° 5
Repeat. The center is the intersection of the two AB  
diameters.
14. 14 ft. In Lesson 6.6, Exercises 9 and 10, you
3. The velocity vector is always perpendicular to the discovered and proved a conjecture that says, “The
radius at the point of tangency to the object’s measure of an angle formed by two intersecting
circular path. secants through a circle is equal to one-half the
difference of the larger arc measure and the smaller
4. Sample answer: An arc measure is between 0° and
arc measure.” First apply this conjecture to find
360°. An arc length is proportional to arc measure : 50°  1(mDL
DL   60°), so 100°  mDL   60°,
and depends on the radius of the circle.  2
and mDL  160°. By the Chord Arcs Conjecture,
mCD  mOL , so 2mCD   160°  60°  360°.
5. 55°. Draw the radius to the point of tangency. By

Then 2mCD  140°, and mCD   70°. Now apply
the Tangent Conjecture, the radius is perpendicular
the Arc Length Conjecture. The length of CD  is
to the tangent, so a right triangle has been formed.
C  (2  36 ft)  14 ft.
70° 7
Therefore the measure of the central angle that 360° 36
intercepts the arc of measure b is 90°  35°  55°,
15. Look at the inscribed angles with measures 57° and
so b  55° by the definition of the measure of
35°. By the Inscribed Angle Conjecture, the sum of
an arc.
the measures of their intercepted arcs is 2(57°) 
6. 65°. The 110° angle is an inscribed angle that inter- 2(35°)  184°. However, the sum of these two arcs
cepts an arc of measure a  155°, so by the is a semicircle, and the measure of a semicircle is
Inscribed Angle Conjecture, 110°  12(a  155°); 180°, so this is impossible.
220°  a  155°, and a  65°.
Another way to look at this is to add the angle
7. 128°. Congruent chords intercept congruent arcs measures in the triangle. The third angle of the
(Chord Arcs Conjecture), so the unmarked arc has triangle must be a right angle because it is inscribed
measure c. Then, 2c  104°  360°, so 2c  256°, in a semicircle (Angles Inscribed in a Semicircle
and c  128°. Conjecture). Therefore the sum of the three angles
of the triangle would be 57°  35°  90°  182°,
8. 118°. First find the measure of either of the two
which is impossible by the Triangle Sum Conjecture.
vertical angles that form a linear pair with the angle
of measure e. The measure of an angle formed by 16. By the Parallel Lines Intercepted Arcs Conjecture,
two intersecting chords is half the sum of the mea- the unmarked arc measures 56°. Then the sum of
sures of the intercepted arcs (see Lesson 6.5, Exer- the arcs would be 84°  56°  56°  158°  354°,
cises 16 and 17), so the measure of either one of but this is impossible because the sum of the mea-
these angles is 12(60°  64°)  62°. Then, e  180° sures of the arcs of a circle must be 360°.
 62° (Linear Pair Conjecture), so e  118°.

116 CHAPTER 6 Discovering Geometry Solutions Manual


©2003 Key Curriculum Press
DG3SM586_097-135.qxd 8/2/06 5:56 PM Page 117

17. mEKL  12mEL  1(180°  108°)  36°  24. Sample answer: Construct acute angle R. Mark off
2
mKLY. Therefore KE   YL  by the Converse of the equal lengths RM and RH. From points M and H,
swing arcs of lengths equal to RM and RH. Label
Parallel Lines Conjecture.
the intersection of the arcs as O. Construct RHOM.
  360°  56°  152°  152°  mMI
18. mJI . Therefore The intersection of the diagonals is the center of the
mJ  mM (Inscribed Angles Intercepting Arcs inscribed circle. Construct a perpendicular to a side
Conjecture), and JIM is isosceles by the Converse to find the radius. It is not possible to construct the
of the Isosceles Triangle Conjecture. circumscribed circle unless the rhombus is a square.
  2mKEM  140°. Then mKI
19. mKIM   140°  H
O

70°  70°  mMI . Therefore mIKM  12mMI 
1 
mKI  mIMK, so IKM  IMK. So KIM is
2
isosceles by the Converse of the Isosceles Triangle
R M
Conjecture.
20. Ertha can trace the incomplete circle on paper. She 25. 4x  3y  32 or y  43x  332 . A tangent is
can lay the corner of the pad on the circle to trace perpendicular to a radius drawn to the point of
an inscribed right angle. Then Ertha should mark tangency (Tangent Conjecture). Here, the slope of
41 3
the radius to the point of tangency is  5  1  4 , so
the endpoints of the intercepted arc and use the pad  
to construct the hypotenuse of the right triangle, the slope of the tangent is the negative reciprocal of
3 3
which is the diameter of the circle. , which is . The tangent is the line with slope
4 3 4
4 passing through (5, 4). Find an equation for
21. Sample answer: Construct
this line.
perpendicular bisectors of two
y4 4 4
sides of the triangle. The point     
at which they intersect (the x5 3 3
circumcenter) is the center of 3(y  4)  4(x  5)
the circle. The distance from 3y  12  4x  20
the circumcenter to each vertex
is the radius. 4x  3y  32

22. Sample answer: Construct the incenter (from the or


angle bisectors) of the triangle. From the incenter, 3y  4x  32
which is the center of the circle, construct a perpen- 4 32
dicular to a side. The distance from the incenter to y  3x  3
the foot of the perpendicular is the radius. 26. (3, 2). This is equivalent to finding the circum-
center of the triangle with the three given points
as vertices. For reference, label the three points as
A(7, 5), B(0, 6), and C(1, 1). Draw the triangle
on a coordinate grid.
y

23. Sample answer: Construct a right angle, and label


B (0, 6)
the vertex R. Mark off RE and RT with any lengths. A (–7, 5)

From point E, swing an arc with radius RT. From


point T, swing an arc with radius RE. Label the
x
intersection of the arcs as C. Construct the diago- C (1, –1)
 and RC
nals ET . Their intersection is the center
of the circumscribed circle. The circle’s radius is
the distance from the center to a vertex. It is not
possible to construct the inscribed circle unless the
rectangle is a square.
It appears from the drawing that ABC is a right
triangle with B as the right angle. Verify this by
E C
finding slopes of the two sides of B: Slope AB
65 1  1  6 7
  , and slope BC     7.
0  (7) 7 10 1
  BC
Therefore AB , so B is a right angle, and AC 
R T
is the hypotenuse of the right triangle. Thus the

Discovering Geometry Solutions Manual CHAPTER 6 117


©2003 Key Curriculum Press
DG3SM586_097-135.qxd 8/2/06 5:56 PM Page 118

circumcenter of the triangle (or the center of the 32. 8 m  25.1 m. Use the formula C  2r. If the
, which has coordinates
circle) is the midpoint of AC radius of the moat is 10 m, the circumference
7  1 5  (1)
2, 2  (3, 2). would be 20 m, whereas if the radius is 6 m, the
circumference would be 12 m. Therefore the larger
Note: You could also use the more general method moat’s circumference should have been 20  12
of finding equations of two perpendicular bisectors  8 m  25.1 m greater.
of the triangle and then finding their intersection by
solving a system of equations. The method shown 33. 12 cm. The circumference of the base of the cone
above is a shortcut that works for right triangles will be the length of the arc of the sector shown in
only. (See Using Your Algebra Skills 6, Exercise 6.) the figure. The circumference of the original circle
is 2(45)  90 cm, so the length of the arc is
27.  0.318 m. Use the formula C  d to find the 48 2
C  (90)  12 cm, and the diameter of the
360 15
diameter of a circle whose circumference (one revo- base is 12 cm.
lution) is 1 m: 1  d, so d  1 m  0.318 m.
34. False. Any obtuse isosceles triangle is a counterex-
28. Melanie: 151 m/min or 9 km/hr; Melody: 94 m/min ample, such as a triangle with angles of measures
or 6 km/hr. For Melanie, the radius is 8 m, so the 20°, 20°, and 140°.
circumference (or length of one revolution) is
16 m, and for Melody, the radius is 5 m, so the 35. True
circumference is 10 m. 36. False
The merry-go-round makes 30 revolutions in D C
10 minutes, or 3 revolutions/minute. Find the
average speeds for Melanie and Melody in
kilometers per hour.
A B
Melanie:
distance 3C 37. True 38. True 39. True 40. True
speed    
time  1 min
3(16 m)
41. False. (7  2)  180°  900°, so it could have seven
60 min 0.001 km
 
1 min  
1 hr  
1m
sides.
42. False. The sum of the measures of any triangle
 9 km/hr
is 180°.
Melody:
43. False. The sum of the measures of one set of exte-
distance 3C rior angles for any polygon is 360°. The sum of the
speed    
time  1 min measures of the interior angles of a triangle is 180°
3(10 m) 60 min 0.001 km
1 min  1 hr 
    and of a quadrilateral is 360°. Neither is greater
1m than 360°, so these are two counterexamples.
 6 km/hr
44. False. The consecutive angles between the bases are
2(6357) 2(6378)
29. 
360  60  1.849  1.852  1.855 
360  60 supplementary.
30. Possible 45. False. 48°  48°  132°  180°.
location
46. False. Inscribed angles that intercept the same arc
7700 ft
are congruent.
5500 ft 5280 ft
D T 47. False. The measure of an inscribed angle is half the
measure of the arc.

Possible 48. True


location
 and BD
49. False. AC  bisect each other, but AC
 is not
200
31. 
 ft  63.7 ft. The circumference of the table is .
perpendicular to BD
100(2)  200 ft. Use the formula C  d to find D C
C 200
the diameter of the table: d  
    63.7 ft.


A B

118 CHAPTER 6 Discovering Geometry Solutions Manual


©2003 Key Curriculum Press
DG3SM586_097-135.qxd 8/2/06 5:56 PM Page 119

50. False. It could be isosceles. (Remember that an equi- Also, q  g (CA Conjecture), so q  116°, and r 
lateral triangle is a special kind of isosceles triangle.) 180°  90° (Linear Pair Conjecture), so r  90°.
Next, s  c (CA Conjecture), so s  58°, and t  d
51. False. 100°  100°  100°  60°  360°.
(CA Conjecture), so t  122°. Now, u  n (CA

CD
52. False. AB Conjecture), so u  105°, and v  75° (CA Conjec-
A ture). Also, w  b (CA Conjecture), so w  61°.
Now look at the right triangle with an acute angle
90° C
90°
of measure w. The other acute angle in this triangle
measures 90°  w  90°  61°  29°, so x  29°
B D
(AIA Conjecture). Finally, y  180°  29° (Linear
Pair Conjecture), so y  151°.
58. TAR  YRA by SAS; TAE  YRE by ASA

53. False. The ratio of the circumference to the diam- 59. FTO  YTO by SAA, SAS, or SSS; FLO 
eter is . YLO by SAA, SAS, or SSS; FTL  YTL by SSS,
SAS, or ASA
54. False. 24  24  48  48  96.
60. PTR  ART by SAS; TPA  RAP by SAS,
24 cm 24 cm
SAA, or ASA
61. ASA
62. mAC  84°, length of AC
  11.2 in. or about 35.2
48 cm 48 cm
in. By the Inscribed Angle Conjecture, mAC 

2mABC  2(42°)  84°. AC is  84 7
360  30 of the
 
circumference, and C  2r  2(24)  48 in.
55. True  is 7C  7(48) 
Therefore the length of AC 30 30
56. This is a paradox. All but seven of the statements in 11.2 in.  35.2 in.
Exercises 34–55 are false. What about the statement 63. x  63°, y  27°, w  126°. Let O be the center of
in Exercise 56? If it is true, then all but eight of the the circle. Look at the angles in quadrilateral ECOD,
statements in Exercises 34–56 will be true, which which is a kite. Angles ECO and EDO are right
would make the statement in Exercise 56 false. And angles (Tangent Conjecture), so 54°  90°  w 
if the statement in Exercise 56 is false, then all but 90°  360° (Quadrilateral Sum Conjecture), and
seven of the statements in Exercises 34–56 are false, thus w  126°. The angle with measure w is the
so the statement in Exercise 56 is true. , so mCD
central angle that intercepts CD   126°
57. a  58°, b  61°, c  58°, d  122°, e  58°, (definition of the measure of an arc). Then, x 
1 
(mCD ) (Inscribed Angle Conjecture), so x  63°.
f  64°, g  116°, h  52°, i  64°, k  64°, 2
l  105°, m  105°, n  105°, p  75°, q  116°, Now look at AOB. AB   CD  (given) and OA
r  90°, s  58°, t  122°, u  105°, v  75°,   OC
OB   OD  (radii of same circle.) Therefore
w  61°, x  29°, and y  151°. First, a  58° AOB  COD by SSS, so AOB  COD by
(Vertical Angles Conjecture). Then, 2b  58°  CPCTC, and mAOB  w  126°. AOB is
180°, so b  61°. Next, c  58° (CA Conjecture), isosceles, so 126°  2y  180°. Therefore 2y  54°,
and d  180°  c (Linear Pair Conjecture), so and y  27°.
d  122°. Also, e  c (Vertical Angles Conjecture), 64. Sample answer:
so e  58°. Next, f  180°  116° (Linear Pair
Conjecture), so f  64°, and g  116° (Vertical
30°
Angles Conjecture).
150°
Next look at the isosceles triangle in which the
vertex angle has measure h. Each of the base angles
65. f(n)  9  4n; 71. Notice that as n increases by 1,
of this triangle measures 64° because one of these
f(n) decreases by 4, so the function rule must
angles is the vertical angle of the angle with
involve subtracting 4 n times to get the nth term. To
measure f. Then h  2(64°)  180°, so h  52°, and
find the number from which 4n is subtracted (call
i  64° (CA Conjecture), using the 64° angle in the
this number a), look at the first term: f(1)  5 
upper left of the isosceles triangle. Next, h  i 
a  4(1), so a  9. Therefore the function rule is
k  180°, so k  64°. Now, l  180°  75° (Linear
f(n)  9  4n. Now use this rule to find the 20th
Pair Conjecture), so l  105°. Then, m  l (AIA
term: f(20)  9  4(20)  9  80  71.
Conjecture), so m  105°, n  l (CA Conjecture), so

Discovering Geometry Solutions Manual CHAPTER 6 119


©2003 Key Curriculum Press
DG3SM586_097-135.qxd 8/2/06 5:56 PM Page 120

66. a. The circle with its contents has 3-fold rotational 4. Rectangles and squares are always cyclic because
symmetry; the entire tile does not. their opposite angles are right angles and thus are
supplementary. An isosceles trapezoid is always
b. No, it does not have reflectional symmetry.
cyclic.
67. Possible construction: Draw a segment for the T R
width. Construct two perpendiculars at the
endpoints of this segment. Mark off the width
P A
twice along each perpendicular to get the lengths.
Connect the last two vertices to form the rectangle. In isosceles trapezoid PART, A and R are
consecutive interior angles and therefore supple-
mentary. R and T are base angles and therefore
congruent. Thus opposite angles A and T are
supplementary. A rhombus has opposite angles
congruent, so any rhombus inscribed in a circle is a
square. A kite is cyclic only if its nonvertex angles
are right angles.
68. 9.375 cm. Sketch the situation described in the 5. Possible answer: y  a  bx; thus C  0  3.1d,
exercise. or C  3.1d. This confirms the Circumference
15 cm Conjecture.
A D E C B y

From the figure, you can see that EC  18(AB) and 30


CB  12(AB). Then EB  EC  CB  18(AB) 

1
6  3.
1 5 5
(AB)  (AB)  (15)  9.375 cm.
_._7
Circumference

m  1_8
2 8 8 20
69. 70. 18.7

10

6
x
12
Diameter

TAKE ANOTHER LOOK


1. EAN and EGN are right angles because the CHAPTER 7
Tangent Conjecture says that a tangent is perpen-
  EG
dicular to the radius. EA  because all radii in
LESSON 7.1
a circle are congruent. Because point E is equidistant
from the sides of ANG, the converse of the Angle EXERCISES
Bisector Conjecture says that ANE  GNE.
  GN 1. Rigid. The size and shape do not change.
Therefore ANE  GNE by SAA, so AN
by CPCTC. Thus the tangent segments are 2. Nonrigid. The shape changes.
congruent.
3. Nonrigid. The size changes.
2. Sample answer: The quadrilateral is a kite because
  EG (all radii in a circle are congruent) and 4. 5.
EA
  GN
AN  (Tangent Segments Conjecture).
P
3. The converse of the conjecture “If a quadrilateral is
inscribed in a circle, then its opposite angles are ᐉ
supplementary” is “If the opposite angles of a
quadrilateral are supplementary, then the quadrilat- 6.
eral can be inscribed in a circle.” To show that this
is true, for quadrilateral ABCD, draw a circle
containing A, B, and C and show D is also on the
circle. You know mABC  mAC   360°, mB 
1 
mD  180°, and mB  2mAC . Combining these
equations, you get the result mABC  2mD. This
is true only if D is on the circle.

120 CHAPTER 7 Discovering Geometry Solutions Manual


©2003 Key Curriculum Press
DG3SM586_097-135.qxd 8/2/06 5:56 PM Page 121

7. Possible answer: a boat moving across the water 15. 7-fold symmetry: Possible answers are F or J. 9-fold
symmetry: Possible answers are E or H. Basket K
8. Possible answer: a Ferris wheel
has 3-fold rotational symmetry but not reflectional
9. a. Sample answer: Fold the paper so that the images symmetry.
coincide, and crease.
16. Number of sides of 3 4 5 6 7 8 n
regular polygon
Number of reflectional 3 4 5 6 7 8 n
symmetries
Number of rotational 3 4 5 6 7 8 n
symmetries (360°)
b. Construct a segment that connects two corre-
sponding points. Construct the perpendicular A regular polygon of n sides has n reflectional
bisector of that segment. symmetries and n rotational symmetries.
17. Notice that each figure is made up of a capital letter
attached to its “mirror image” (reflection across a
vertical line). Because the letters are in alphabetical
order starting with C, the next two figures will use
the letters H and I.

, or

18. The capital letters are in alphabetical order, and


10. a. Extend the three horizontal segments onto the their position moves clockwise through the four
other side of the reflection line. Use your small squares. Also, the placement of the letter is
compass to measure lengths of segments and rotated 90° clockwise each time. Therefore the next
distances from the reflection line. two figures will have F placed like B, and G placed
like C. The circle that is divided into four parts is
always opposite the letter, and the quarter that is
shaded moves clockwise through four positions, so
the circle for F is the same as that for B, and the
circle for G is the same as that for C.

b.

19. P(a, b), Q(a, b), R(a, b). A rectangle has


2-fold reflectional symmetry. In this case, the y-axis
11. and the x-axis are the lines of symmetry. The reflec-
tion of S(a, b) over the y-axis is P(a, b), the
reflection of P(a, b) over the x-axis is Q(a, b),
and the reflection of Q(a, b) over the y-axis is
R(a, b). Notice that you can also find the coordi-
nates of R by considering it as the reflection of S
P over the x-axis.
20. Compass-and-straightedge construction: Place your
compass point at one endpoint of the arc, and draw
12. Reflectional symmetry an arc with a radius that is greater than half the
13. 4-fold rotational and reflectional symmetry length of the original arc. Repeat, using the other
endpoint as center and the same radius. Use your
14. Reflectional symmetry straightedge to draw a segment connecting the two
points of intersection of the arcs you have drawn.

Discovering Geometry Solutions Manual CHAPTER 7 121


©2003 Key Curriculum Press
DG3SM586_097-135.qxd 8/2/06 5:56 PM Page 122

The point where this segment intersects the original Another explanation uses a picture:
arc is P. Possible construction: 1

a
P b
1
a

The quantity a  b represents a(1)  b(1), the sum of


the areas of the rectangles between the perimeters of the
Patty-paper construction: Draw an arc that is more two squares.
than a quarter-circle but less than a semicircle. Fold EXTENSIONS
the patty paper so that the two endpoints of the arc
A. Research and discussions results will vary.
coincide, and crease. The point where the crease
intersects the arc is P, the midpoint of the arc. B. Any number x and the number formed by rotating
x 180° are both in the same 2-by-2 corner square.
96 11 89 68
88 69 91 16
61 86 18 99
P
Fold 19 98 66 81

21. 50th figure: 154 (50 shaded, 104 unshaded);


nth figure: n  2(n  2), or 3n  4 (n shaded, LESSON 7.2
2(n  2) unshaded). Find patterns for the number
of shaded circles and for the number of unshaded EXERCISES
circles, and then add to find the total number of 1. Translation. x  5 means “move right 5 units.” Every
circles. Shaded circles: 1 in the first figure, 2 in the point of the polygon moves right 5 units. Find the
second figure, 3 in the third figure, so there are n image of each vertex of the given triangle, and then
shaded circles in the nth figure. Unshaded circles: 6 connect the image points to form the translated
in the first figure, 8 in the second figure, 10 in the triangle.
third figure, so there is always an even number of y
unshaded circles. Notice that 6  2(3), 8  2(4),
10  2(5), so there are 2(n  2) unshaded circles in
the nth figure. Then the total number of circles in
the nth figure is n  2(n  2), which can also be 5
written as 3n  4. By substituting 50 for n in this
expression, you can find the total number of circles
in the 50th figure: 3n  4  3(50)  4  154. x
5
22. 46. Use the Tangent Segments Conjecture and
subtraction to find the lengths of all the tangent
segments that aren’t given. HI  HM, so HI  4. 2. Reflection. By the Coordinate Transformations
SI  ST, so SI  5. WT  WS  ST  11  5  6. Conjecture, this is a reflection over the x-axis.
WE  WT, so WE  6. OE  WO  WE  14  6 y
 8. OM  OE, so OM  8. Therefore the 8
perimeter of SHOW  SW  WO  OH  HS 
11  14  12  9  46.
IMPROVING YOUR ALGEBRA SKILLS
x
Using the formula for factoring differences of squares, –5

a2  b 2  (a  b)(a  b), in the case where a  b  1


gives a2  b 2  (a  b)(a  b)  a  b, so you can
find a2  b 2 by adding a and b. You can also use the –8
fact that the base numbers are consecutive, so a 
b  1, and substitute into the expression a2  b 2 to
get (b  1)2  b 2  b 2  2b  1  b 2  2b  1 
(b  1)  b  a  b.

122 CHAPTER 7 Discovering Geometry Solutions Manual


©2003 Key Curriculum Press
DG3SM586_097-135.qxd 8/2/06 5:56 PM Page 123

3. Reflection. By the Coordinate Transformations 6. Rules that involve either x or y changing signs, or x
Conjecture, this is a reflection over the line y  x. and y switching places, produce reflections. Rules
y that involve a constant being added to the x and/or
y terms produce translations. If both x and y change
signs, the rule produces a rotation.
5
7. (x, y) → (x, y). Compare the ordered pairs for
V and V, R and R, and Y and Y. In each case, the
x x-coordinates of the original point and its image are
–4 7
the same, whereas the y-coordinates of the original
point and its image are opposites. Use this informa-
–5 tion to write the rule. This is a reflection over the
x-axis. (You can also simply observe from the figure
4. Reflection. Use the given rule to find the image of that the transformation is a reflection over the
each vertex of the given quadrilateral, and then x-axis and use the Coordinate Transformations
connect these image points to form the translated Conjecture to find the rule.)
quadrilateral. The image of (2, 1) is (6, 1), the 8. (x, y) → (x, y). Compare the ordered pairs for
image of (6, 2) is (2, 2), the image of (5, 5) is (3, 5), D and D, H and H, and R and R. In each case,
and the image of (3, 4) is (5, 4). the x-coordinates of the original point and its image
y are opposites, and the y-coordinates of the original
point and its image are opposites. Use this informa-
tion to write the rule. This is a rotation about the
5 origin. (You can also simply observe from the figure
that the transformation is a rotation about the
origin and use the Coordinate Transformations
5
x Conjecture to find the rule.)
9. N

From this drawing, you can see that the transforma-


tion is a reflection over the vertical line x  4. If
you look at this transformation as x  8, you can Cue ball
W 7 E
see that this is a combination of a translation over
the y-axis followed by a translation right 8 units. 8 8 8 ball
For example, the translation over the y-axis moves
(2, 1) to (2, 1), and then translation right 8 units
moves this point to (6, 1).
S
5. Rotation. Use the given rule to find the image of 8
each vertex of the original triangle, and then
connect these image points to form the transformed 10.
N
triangle. The image of (3, 1) is (3, 1), the image
of (4, 3) is (4, 3), and the image of (1, 3) is H
(1, 3). The figure shows that the triangle has been
W E
rotated.
y
T S
5

11. H

x
5
N

W E

T H H
S

Discovering Geometry Solutions Manual CHAPTER 7 123


©2003 Key Curriculum Press
DG3SM586_097-135.qxd 8/2/06 5:56 PM Page 124

12. Possible answer: Use the Minimal Path Conjecture 18. This is the “mirror image” of the message that
 and find intersection point C.
to draw AB appears in the mirror:
Proposed freeway B


C

B
Mason
A
Perry 19. Draw a diameter of the circle. Draw two radii that
form noncongruent acute angles with the diameter,
13. Notice that each figure is made up of a numeral and reflect them across the diameter. Construct the
attached to its “mirror image” (reflection across a lines perpendicular to the radii at the points where
vertical line), with the mirror image directly to the the radii intersect the circle. Each of these perpen-
left of the numeral. Because the numerals are in dicular lines is tangent to the circle (Tangent Conjec-
increasing order, the next two figures will use the ture). Extend the four tangent lines until they meet
numerals 6 and 7. to form a quadrilateral. By the Tangent Segments
Conjecture and addition, this quadrilateral has two
pairs of adjacent congruent sides, so it is a kite.
14. The numbers in the corner square are the odd inte- Sample construction:
gers starting with 1, so the next two figures will
contain 11 and 13. Also, the numerals that form
these odd integers are reflected in the following
order after the first figure: reflection over vertical
line (down center of figure), reflection over hori-
zontal line (through center of figure), reflection over
vertical line, and reflection over horizontal line to
return to the original position. Therefore, in the
fifth figure, 11 will be oriented like 3 in the second
figure, and in the sixth figure, 13 will be oriented
like 5 in the third figure. Also, the small square
opposite the square containing the number is either
shaded (in odd-numbered figures of the sequence)
or contains a dot in the center of the square (in 20. Draw a radius of the circle, extending outside the
even-numbered figures of the sequence.) Therefore circle. Construct the line tangent to the circle at that
the fifth figure will contain a dot in the lower left point. Choose a point on the tangent: this will be
corner (like the second figure), and the sixth figure one vertex of your rhombus. Draw a line through
will have the upper left corner square shaded (like this point and the center of the circle. This is one
the third figure). diagonal of the rhombus. Construct another diam-
eter perpendicular to the first one, also extending
outside the circle. This is the other diagonal. Mark
the point where this line intersects the tangent line:
This is the second vertex of the rhombus. Copy this
length three times to construct the other three sides
15. Possible answer: HIKED. There are many other
of the rhombus. The other vertices are the intersec-
possible answers. Some of these are HEED, HIDE,
tions of this length and the diagonal lines.
COD, DIED, BIKE, and DICE. Any words all of
whose letters are on the following list will work: Sample construction:
B, C, D, E, H, I, K, O, and X. These are all the
letters that have a horizontal line of symmetry.
16. One unless it is equilateral, in which case it has
three.
17. Two unless it is a square, in which case it has four.

124 CHAPTER 7 Discovering Geometry Solutions Manual


©2003 Key Curriculum Press
DG3SM586_097-135.qxd 8/2/06 5:56 PM Page 125

21. False. Possible counterexample: a trapezoid with two Now consider the case in which the centers of
right angles. rotation differ. Example: Draw right triangle ABC
on a coordinate grid. (In this figure, mA  30°,
22. False. Possible counterexample: an isosceles
mB  60°, and mC  60°. All of the rotations
trapezoid.
are assumed to be counterclockwise because the
IMPROVING YOUR REASONING SKILLS direction of rotation is not specified. Rotate the
alphabet v. The third vowel of the alphabet is i, the triangle three times, using a different center of
second consonant after i is k, and the third letter before rotation each time. First rotate 45° counterclockwise
k is h. The twenty-sixth word of the puzzle is the, which through B. Label the new triangle as ABC,
contains the letter h. The fortieth word of the puzzle is where B  B. Now rotate ABC 50° clockwise
alphabet. The twenty-second letter of the alphabet is v. through A. Label the new triangle as A B C ,
Therefore, v should be printed directly after alphabet. where A  A. Finally, rotate A B C 85° coun-
terclockwise through C . Label the new triangle as
EXTENSIONS A
B
C
, where C
 C . Compare the final
A. Sample answer: Rules such as (x, y) → (3x, 3y), triangle, A
B
C
, to the original, ABC. You will
(x, y) → (2x, 2y), (x, y) → 12x, 12y, (x, y) → notice that the original triangle has been rotated
(x, 2y), and (x, y) → (2x, 3y) create nonrigid 180° and also translated from its original position.
In this case, the translation is to the right and
transformations. Either x or y or both x and y must
upward.
be multiplied by constants other than 1 or 1. The
y
same constant or different constants can be used as
multipliers for x and y. C  C B
85°
B. The minimal path on a sphere is an arc of a great A
circle. Minimal paths on the lateral (curved) surface 50° B
A  A
of a cylinder are lines (parallel to the axis of the
C
cylinder), circles (perpendicular to the axis), and A
helixes. The helix can be found by unwrapping the 45° x
lateral surface and laying it flat, and connecting the B  B C

two endpoints with a straight line. Minimal paths


on the ends of the cylinder are lines, and minimal
paths connecting points on the ends of the cylinder 3. a. 20 cm. Use the Reflections over Parallel Lines
with points on the curved surface would vary. Conjecture. The distance between the parallel
lines is 10 cm, so AA  20 cm.
C. H, I, O, and X are the only letters that typically have
both reflectional and rotational symmetry, so a b. 20 cm, but in the opposite direction. See the
word with both types of symmetry would have to figure below. A is the first image (after A is
use only these letters and also be a palindrome. reflected over n), and A is the second image
There aren’t many; an example would be OHO. (after A is reflected over m).

A 20 cm
LESSON 7.3
6 cm
A 10 cm
EXERCISES
16 cm
1. (10, 10). The single translation can be represented by
the rule (x, y) → (x  2  5  13, y  3  7  0), m A
n
or (x, y) → (x  10, y  10). This translation can
be written as (10, 10). 4. a. 80° counterclockwise. Use the Reflections over
2. A 180° rotation. If the centers of rotation differ, Intersecting Lines Conjecture.
rotate 180° and add a translation. If the center is the b. 80° clockwise. Here, B is the reflection of B over
same for all the rotations, simply add the angles of line n, and B is the reflection of B over line m.
rotation: 45°  50°  85°  180°. (You can find the images under these reflections
by measuring, by using the Reflection Line

Discovering Geometry Solutions Manual CHAPTER 7 125


©2003 Key Curriculum Press
DG3SM586_097-135.qxd 8/2/06 5:56 PM Page 126

Conjecture, or by folding patty paper on line m corresponding points on the original figure and the
and then line n.) To find the angle of rotation image after two translations.
(with center P) that rotates the second image,
 and PB
B , back to B, draw PB ; then use a
protractor to find that mB PB  80°. From the R P
figure, you can see that the rotation that rotates
B back to B is 80° clockwise. L A

Fold 2 Fold 1

B B
P
80°

40° 7. Connect a pair of corresponding points. Construct


B
two perpendiculars to the segment connecting these
points with half the distance between the two given
n m
figures between them. There are many possible posi-
 and crease. Draw tions for the lines that will work, but the distance
5. 180°. Fold the patty paper on AP
between them must be half the distance between
the image of the figure. Then fold the patty paper
 and crease again. Draw the image of the first corresponding points on the two figures.
on AL
image. Possible answer:

Fold 1 Fold 2

P L

8. Possible answer: N

Notice that the image under the first reflection


O

shows that the original figure has been rotated 90°


counterclockwise and that the image under the
A

N
second reflection shows that the second figure has
O
H

been rotated an additional 90° counterclockwise. 


A
O A H
This is equivalent to rotating the original figure N
H

180°. Because mPAL  90°, this result is consis- Center of rotation


tent with the Reflections over Intersecting Lines
Conjecture. (Because 180° is half the total angle 9.
measure of a circle, the results of clockwise and
counterclockwise rotations of 180° are the same, so
it is not necessary to specify the direction of rota-
tion when the angle of rotation is 180°.)
6. About 3 cm. By the Reflections over Parallel Lines
Conjecture, the length of the equivalent translation
vector is twice the distance between the parallel
lines. In the figure in your book, the distance 10. Notice that each figure is made up of a capital letter
between PA  and RL
 is about 1.5 cm, so the length and reflections of that letter through a horizontal
of the translation vector will be about 3 cm, which line, a vertical line, and both the horizontal and
you can confirm by measuring the distance between vertical lines. The figures shown use the letters A, B,
C, D, and E in alphabetical order, so the next two
figures will use F and G in the same way.

126 CHAPTER 7 Discovering Geometry Solutions Manual


©2003 Key Curriculum Press
DG3SM586_097-135.qxd 8/2/06 5:56 PM Page 127

11. The letters that appear in the corners are every PROJECT
other capital letter in alphabetical order. In the first Project should satisfy the following criteria:
five figures, the letters are A, C, E, G, I, so the next
● A kaleidoscope is carefully constructed.
two figures will contain K and M. Notice that the
letters move counterclockwise through the four ● There is an understanding of the relationship between
squares and also rotate through four different orien- angle measurement and the number of images created.
tations. Therefore, in the sixth figure, K will be A 60° angle shows six mirrors and five images, plus
placed the same way that C is in the second figure, the original object; a 45° angle shows eight mirrors
and in the seventh figure, M will be placed the same and the seven images, plus the original object.
way as E is in the third figure. The numbers that
appear in the squares are even numbers in order, LESSON 7.4
starting with 2, so the sixth figure will contain 12
(positioned like 4 in the second figure) and the EXERCISES
seventh figure will contain 14 (positioned like 6 in 1. Answers will vary. Possible answers include floor,
the third figure). Also, the shaded triangles and ceiling, and wall tilings, and a checkerboard.
shaded sectors of circles in the sixth figure will
match those in the second figure, while those in the 2. Answers will vary.
seventh figure will match those in the third figure. 3. 33.42. Each vertex is surrounded by three equilateral
triangles and then two squares.
4. 34.6. Each vertex is surrounded by four equilateral
triangles and then one regular hexagon.
12. Sample answer: Draw a figure on an overhead trans- 5. 32.4.3.4. Each vertex is surrounded by two equilat-
parency, and then project the image onto a screen. eral triangles, then a square, then another equilateral
triangle, and then another square.
13. Possible answers: rotational: playing card, ceiling
fan, propeller blade; reflectional: human body, 6. 3.4.6.4 3.42.6. One type of vertex is surrounded by
backpack an equilateral triangle, then a square, then a regular
hexagon, and then another square. The other type
14. Two reflections about intersecting lines yield a rota- of vertex is surrounded by an equilateral triangle,
tion. The measure of the angle of rotation is twice two squares, and a regular hexagon.
the measure of the angle between the lines of
reflection, or twice 90°, which is 180°. 7. 33.42 32.4.3.4. One type of vertex is surrounded by
three equilateral triangles and then two squares. The
15. One: yes; two: no; three: yes. An isosceles triangle other type is surrounded by two equilateral trian-
that is not equilateral (that is, a triangle with exactly gles, then a square, then another equilateral triangle,
two congruent sides) has exactly one line of and then another square.
symmetry, while an equilateral triangle has three
lines of symmetry. 8. 36 32.4.12. One type of vertex is surrounded by six
equilateral triangles. The other kind is surrounded
by two equilateral triangles, then a square, then a
regular dodecagon.
9. a. The dual of a square tessellation is a square
tessellation.

16. Label the two points A and B. Draw AB. Find the

midpoint of AB , and label it O. Connect O to one
of the endpoints (either A or B) with a curve. Copy
the curve onto patty paper and rotate it about O to
form the other half of the curve. Possible answer:

A O B

Discovering Geometry Solutions Manual CHAPTER 7 127


©2003 Key Curriculum Press
DG3SM586_097-135.qxd 8/2/06 5:56 PM Page 128

b. The dual of a hexagon tessellation is a triangle 16. y  12x  4. From the graph, you can see that the
tessellation. reflected line has y-intercept 4 and slope 12, so its
equation is y  12x  4.
y

x
–3 5
c. If tessellation A is the dual of tessellation B, then
tessellation B is the dual of tessellation A.
–6
10. The dual is a 34 38 tessellation of isosceles right
triangles.
17. Possible answers: TOT, AHA. To fit the criteria, all
the letters in the word need to have a vertical line
of symmetry themselves or be the mirror image of
another letter. The following list will work: A, H, I,
M, O, T, U, V, W, X, and Y. These are all the letters
that have a vertical line of symmetry.
11. 12. 18. Work backward. Reflect a point of the 8-ball over
the S cushion. Then reflect this image over the N
cushion. Aim at this second image.
8

13. A ring of ten pentagons fits around a decagon,


and another decagon can fit into any two of the
pentagons. However, if two decagons fit into pairs
of pentagons that share a common pentagon, then
the decagons overlap.
N

8-ball
8 10
7
W 15 E
4 Cue ball
6

14. S

IMPROVING YOUR REASONING SKILLS


1. Possible answers: 65  76  141; 66  75  141

15. Answers will vary. 2. Because it is not possible to multiply a two-digit


number by a two-digit number and get a two-digit
number, the answers must be a two-digit number
times a one-digit number, giving a three-digit
number. Possible answers: 17  9  153;
9  17  153.
851 851
3. Possible answers: 3 
7  23; 23  37

128 CHAPTER 7 Discovering Geometry Solutions Manual


©2003 Key Curriculum Press
DG3SM586_097-135.qxd 8/2/06 5:56 PM Page 129

LESSON 7.5 5. By the Triangle Sum Conjecture, a  b  c  180°.


Around each point, the sum of the angle measures
EXERCISES is 2(a  b  c)  2  180°  360°. Therefore a
1. Answers will vary. triangle will fill the plane edge-to-edge without gaps
or overlaps. Thus any triangle can be used to create
2. To construct the dual tessellation, connect centers
monohedral tiling.
across the common side. The dual is a 53 54
tessellation. c a c a
b b b
a c b a c b a c
c a c a c a
b b
b a c b a c b

6. 3 ways

7.

3.

8. y
8

4. Yes. The tessellation shown below uses the concave


quadrilateral shown in your book, but you can
x
create your own quadrilateral and follow the same 5
process to create a tessellation. Start with one –2

quadrilateral. Rotate it 180° around the midpoint of


one of its sides to form an adjacent figure. The side
y  2x  3. From the graph above, you can see
around whose midpoint you rotated the figure will
that the image line has y-intercept 3 and slope 2,
be the common side of the two adjacent figures.
so its equation is y  2x  3.
Continue this process to create the tessellation. The
four angles of the quadrilateral will be around each IMPROVING YOUR VISUAL THINKING SKILLS
point of intersection in the tessellation. There are several possible answers. Here is one answer
You may find it helpful to draw one quadrilateral for each of the two patterns.
and then cut it out so that you can move it around 1. 2.
and copy it in various positions. You may also find
it helpful to label the sides, as in the figure below,
to keep track of the location of each side of the
original figure.
c d PROJECT
b a
b Project should satisfy the following criteria:
a ● The tessellation has no gaps or overlaps.
c d
c d
d
b a ● Dots are correctly placed so that the tiling is
b a
a
nonperiodic.
c d
c d Extra credit
d
b a
b a ● Exceptional creativity and care are shown in the
coloring.
EXTENSION
Research results will vary.

Discovering Geometry Solutions Manual CHAPTER 7 129


©2003 Key Curriculum Press
DG3SM586_097-135.qxd 8/2/06 5:56 PM Page 130

LESSON 7.6 12. y  23x  3. The lines have the same y-intercept
and slopes of the opposite sign. Therefore the image
EXERCISES line has slope 23, and its equation is y  23x  3.
1. Answers will vary. You can add features to make this y
look like a bat.
5
2. Answers will vary. You can add an eye and feathers
to make this look like a profile of a bird’s head.
x
–10 10
3. Answers will vary. You can add an eye and feathers
to make this look like a profile of a bird.
4. Regular hexagons
5. Squares or parallelograms 13. 3.4.6.4 4.6.12. One type of vertex is surrounded by
6. Squares or parallelograms an equilateral triangle, then a square, then a regular
hexagon, and then another square. The other type
7. Repeat the shape over and over so that the shapes of vertex is surrounded by a square, then a regular
fit together and fill the grid. hexagon, and then a regular dodecagon.
400 rev 2  26 ft 1 min
14. 1 min   
 1 rev 60 sec  1089 ft/sec
15. a. True. Possible explanation: The kite diagonal
between vertex angles is the perpendicular
bisector of the other diagonal; in a square,
diagonals bisect each other.
b. False. It could be an isosceles trapezoid.
c. False. It could be a rectangle.
d. True. Parallel lines cut off congruent arcs of a
circle, so inscribed angles (the base angles of the
8. Repeat the shape over and over so that the shapes
trapezoid) are congruent.
fit together and fill the grid.
IMPROVING YOUR VISUAL THINKING SKILLS

EXTENSION
Tessellations will vary.

LESSON 7.7
EXERCISES
9. Answers will vary.
1. Equilateral triangles
10. Answers will vary.
2. Regular hexagons
11. B 3.
E

A
S

130 CHAPTER 7 Discovering Geometry Solutions Manual


©2003 Key Curriculum Press
DG3SM586_097-135.qxd 8/2/06 5:56 PM Page 131

4. LESSON 7.8
EXERCISES
1. Parallelograms
2. Parallelograms
3.

5. Answers will vary.


6. Answers will vary.
7. Sample design: 4.

8. False. If the diagonals of a quadrilateral bisect each


other, the quadrilateral is a parallelogram. The diag-
onals of a parallelogram are congruent only if the
quadrilateral is a rectangle. 5. Answers will vary. Use a grid of kites like the one
9. True below. Notice that in this grid, rows of kites alter-
nate between those with “long sides down” and
10. True those with “long sides up.”
11. False. Possible counterexamples: isosceles trapezoid,
kite
12. Approximately 6280 miles, which will take
126 hours, or around 514 days.
13. a. Yes. It has reflectional symmetry, so legs and base
angles are congruent.
b. Greatest: near each of the acute vertices; least: at
the intersection of the diagonals (where both A,
C, and B become collinear and A, C, and B
become collinear). 6. Answers will vary. Use a grid of parallelograms like
IMPROVING YOUR REASONING SKILLS the one in the example on page 399 in your book.
Here’s a sample of the kind of reasoning required: Both 7. Circumcenter is (3, 4); orthocenter is (10, 8).
Denise and Charles claim to be second, so at least one
8. 9.
of them is lying; therefore Kai must be either first or
third. Kai claims to have been last and that Leyton was
best; because Kai can’t be last, Leyton was best.
Continue with this kind of reasoning. Frances claims
that Kai was second; because Kai must be first or third,
Frances was fourth. Leyton said that he was third, but
you already know that he was best, so Charles was last. 10.
Charles claims that he came in second, but you know
that he was last, so Kai was third. There is only one
student, Denise, and one position, second, left, so Denise
must be second. You can check this by checking Denise’s
statements: She claimed that Kai won, but you know
that Kai was third, so Denise was second. From first to
last, the order of finish was Leyton, Denise, Kai, Frances,
and Charles.

Discovering Geometry Solutions Manual CHAPTER 7 131


©2003 Key Curriculum Press
DG3SM586_097-135.qxd 8/2/06 5:56 PM Page 132

IMPROVING YOUR ALGEBRA SKILLS Any line through N(0, 5) has y-intercept 5, so an
x  15. Apply the definitions of the two new equation of the line containing the median from R
 is y  13 x  5.
to RO
functions: a  b  a2  b 2 and a ° b  ab. 8  x  6
0  10 5  (3)
82  x 2 and 17 ° 2  172. Therefore the equation
8  x  17 ° 2 means 82  x 2  172. Then 64  x 2 
  
Midpoint of NO  ,   (5, 1)
2 2 
289, so x 2  225, and x  15.
Find the equation of the line through M(4, 0) and
10 1
5  (4)  9 . Find
(5, 1). The slope of this line is   
USING YOUR ALGEBRA SKILLS 7 the equation of the line passing through (4, 0)
with slope 19.
EXERCISES
y0 1
1. y  16x. The median from R to ES  is the segment   
x  (4) 9
whose endpoints are R and the midpoint of ES . y 1
 4  8 6  4   
The midpoint of ES is 2, 2  (6, 1). x4 9
To find the equation of the line through R(0, 0) 9y  x  4
10 1
and (6, 1), first find its slope: 60   6 . The
  1 4
 goes y  9x  9
line containing the median from R to ES
through the origin, so its y-intercept is 0. Therefore Solve the system formed by the equations of the line
the equation of this line is y  16x. containing the two medians.
1 4


2. y  2x  2. First find the slope of RS , and then y  9 x  9
use this slope to find the slope of the altitude from 13
40 y  6 x  5
. The slope of RS  is  4 1
8  0  8  2 , so the
E to RS   
slope of the altitude to RS is 2, the negative To eliminate fractions, multiply both sides of the
1 first equation by 9 and both sides of the second
reciprocal of 2. Find the equation in the form
y  mx  b of the line through E(4, 6) with equation by 6.
slope 2.
y  (6)
9y6y  x13x
4
 30
 x  4  2 To solve this system by substitution, solve the first
y  6  2(x  4) equation for x; then substitute the resulting expres-
y  6  2x  8 sion in the second equation and solve for y.

y  2x  2 x  9y 4

3. Centroid is 2, 23; orthocenter is (0, 5). There are 6y  13(9y  4)  30


two methods for finding the centroid. 6y  117y  52  30
Method 1: Find the equations of the lines containing 123y  82
two of the medians, and solve a system of equations 82 2
to find their point of intersection, which will be the y  
123  3
centroid.
Now substitute 23 for y in the equation x  9y 4
4  10 0  (3)
Midpoint of MO    2
, 
2  and solve for x.

 3, 2
3 
2
x  9 3  4  2

Therefore the coordinates of the centroid are 2, 23.


Find the equation of the line through N(0, 5) and
3, 2. The slope of this line is
3
Method 2: Find the mean of the x-coordinates and
the mean of the y-coordinates of the three vertices
 
5  32 1
2
3
13
of the triangle.
    
03 3 6

132 CHAPTER 7 Discovering Geometry Solutions Manual


©2003 Key Curriculum Press
DG3SM586_097-135.qxd 8/2/06 5:56 PM Page 133

4  0  10
The mean of the x-coordinates is   2. From the graph, you can see that the vertices of
3
0  5  (3) the triangle are (0, 0), (0, 4), and (3, 0). The
The mean of the y-coordinates is 3  23. 030
mean of the x-coordinates is    1,
3

Therefore the coordinates of the centroid are 2, 23 . and the mean of the y-coordinates is  400
  .
4
3 3
Now find the orthocenter. Because the legs of a Therefore the centroid of the triangle is 1, 34.
right triangle are perpendicular to each other (to
form the right angle), they are two of the altitudes 6. (1, 1). Find the vertices of the triangle by
of the triangle. This means that the orthocenter of solving three systems of equations, each made up
a right triangle is the right-angle vertex. From the of two of the three equations. First solve the system
figure or by computing slopes, you can see that N formed by the first and second equations:
is the right angle in MON. Therefore the ortho-
center is N(0, 5).  8x  3y  12
6y  7x  24
4. Centroid is (4, 0); orthocenter is (3, 0). First use the To solve this system by elimination, rewrite the
means of the coordinates of the vertices of CDE second equation as 7x  6y  24, multiply the
to find the centroid. first equation by 2, and then subtract the second
resulting equation from the first.
0  12  0
Mean of x-coordinates  3  4
16x  6y  24
6  0  (6)
Mean of y-coordinates  3  0 7x  6y  24
The centroid is (4, 0). 23x  0
To find the orthocenter, find the equations of two of x0
the altitudes of CDE, and then solve a system of Substitute 0 for x in the first original equation and
equations to find their intersection. solve for y.
In an isosceles triangle, the median to the base is 16(0)  6y  24
also an altitude. The figure shows that the median
y4
from E to CD is the x-axis, which is the line y  0.
One vertex is (0, 4).
Now find an equation for the altitude from D to
. The slope of CE  is  06 1 Now solve the system formed by the first and third
12  0   2 , so the slope of
CE  
equations.
 is 2, the negative reciprocal of
the altitude to CE
12. The y-intercept of any line through C(0, 6) is 8xx 9y3y3312 0
6, so the equation of the altitude from D to CE 
Multiply the first equation by 3, and rewrite the
is y  2x  6. To solve the system
y0
y  2x  6
, second equation by subtracting 33 from both sides.
substitute 0 for y in the first equation to get Then subtract the second resulting equation from
the first.
0  2x  6, or x  3. Therefore the orthocenter
is (3, 0). 24x  9y  36

5. 1, 43. Sketch the triangle and find the coordinates x  9y  33
of its vertices. Graph the line 12x  9y  36 23x  69
by finding its intercepts: If x  0, y  4, so the
y-intercept is 4. If y  0, x  3, so the x-intercept x3
is 3. Substitute 3 for x in the equation x  9y  33
y and solve for y.
3  9y  33
(0, 4)
12x  9y  36 9y  36
x y  4
(0, 0) (3, 0)
The second vertex is (3, 4).

Discovering Geometry Solutions Manual CHAPTER 7 133


©2003 Key Curriculum Press
DG3SM586_097-135.qxd 8/2/06 5:56 PM Page 134

Now solve the system formed by the second and would be completely unpainted. So the only possibilities
third equations, with both of them rewritten in the are that the large cube contain 43  64 or 53  125
same ways as before. small cubes. In the latter case, there will be 27 small
cubes, necessarily unpainted, in the interior, so the
7x  6y  24
x  9y  33
painting must avoid exactly 5 cubes in the outer layer. If
even one face of the large cube is left unpainted, 9 cubes
Multiply the second equation by 7, and add the in the middle of that face will remain clear, so more
result to the first equation. than 32 small cubes altogether will be clear. Therefore
the large cube must be 4 by 4 by 4. If one face of this
7x  6y  24
large cube is painted, then 16 small cubes are painted on
7x  63y  231 one face. If two opposite faces are painted, then 32 small
cubes are painted, so 32 remain unpainted. Alternatively,
69y  207
you could consider the 8 interior cubes that are neces-
y  3 sarily unpainted and see how to arrive at 24 unpainted
small cubes in the outer layer.
Substitute 3 for y in the equation x  9y  33
and solve for x.
CHAPTER 7 REVIEW
x  9(3)  33
EXERCISES
x  27  33
1. True 2. True
x  6
3. True 4. True
The third vertex is (6, 3).
5. True 6. True
Finally, use the coordinates of the three vertices,
(0, 4), (3, 4), and (6, 3), to find the 7. False. A regular pentagon does not create a mono-
coordinates of the centroid. hedral tessellation, whereas a regular hexagon does.
0  3  (6) 8. True 9. True
Mean of x-coordinates  3
3 10. False. Two counterexamples are given in Lesson 7.5.
 3 1
11. False. Any hexagon with all opposite sides parallel
4  (4)  (3) and congruent will create a monohedral tessellation.
Mean of y-coordinates  3
3 12. This statement can be both true and false. Among
 3  1
Exercises 1–11, 8 of the statements are true and 3
The centroid is (1, 1). are false. If the statement in Exercise 12 is true, then
there are 9 true and 3 false statements, so there are
IMPROVING YOUR VISUAL THINKING SKILLS
at least three times as many true statements as false
Two opposite ones. So the statement in Exercise 12 is true. If the
faces painted
statement in Exercise 11 is false, then there are 8
true and 4 false statements, so there are not at least
three times as many true statements as false ones.
So the statement in Exercise 11 is false. Therefore
if the statement is true, it’s true, and if it’s false,
it’s false.
13. 6-fold rotational symmetry
14. Translational symmetry
Inside every cube is a smaller cube with 2 fewer small
cubes on each side; these will have no painted faces. The 15. Reflectional. Color arrangements will vary, but the
large cube has more than 3 cubes on each edge because white candle must be in the middle. Notice that
32 is more than 33  27, and it doesn’t have 6 cubes on there is only one white candle, while there are two
each edge because then 43  64 cubes in the middle candles of each of the other colors. One way to

134 CHAPTER 7 Discovering Geometry Solutions Manual


©2003 Key Curriculum Press
DG3SM586_097-135.qxd 8/2/06 5:56 PM Page 135

make the colors symmetrical is to move the white The minimal path from A (the person’s eyes) to B
candle to the middle, leave the four candles to the (the top of her head), which are on the same side of
right of the middle where they are, and arrange the line  (the mirror), is found by reflecting point B
candles on the left in the following order (left to , and then drawing AD
over line , drawing AB  and
right): blue, pink, green, yellow. , where D is the point of intersection of AB
DB  and
line . Likewise, reflect C, find intersection E, and
16. The two towers are not the reflection (or even the  and EC to find the minimal path from her
draw AE
translation) of each other. Each tower individually   BB
  CC
, with F on
eyes to B (her feet). Draw AF
has bilateral symmetry. The center portion of the . You can easily show that BCCB, ABBF, and
BC
façade has bilateral symmetry.  
ACCF are rectangles. In rectangle BCCB, BC
. In rectangle ABBF, diagonal AB
BC  is bisected by
 (Rectangle Diagonals Conjecture), so
diagonal BF
  DB
AD . Similarly, in rectangle ACCF, AE   EC
.
Therefore DE is the midsegment of ABC, so
DE  12BC  12BC, by the Triangle Midsegment
Conjecture.
22. Use the Minimal Path Conjecture. The path will
involve three bounces. Start at the hole, and
work backward from the hole. This problem is
similar to Example B on pages 369–370 of your
book, but here there are three bounces rather
than two.
T H

Left tower Center Right tower 23. Use a grid of squares. Tessellate by translation.

17. Answers will vary. 24. Use a grid of equilateral triangles. Tessellate by
rotation.
18. Answers will vary.
25. Use a grid of parallelograms. Tessellate by glide
19. 36 32.4.3.4; 2-uniform. One kind of vertex is reflection.
surrounded by six equilateral triangles. The second
kind is surrounded by two equilateral triangles, then 26. Yes. It is a glide reflection for one pair of sides and
a square, then an equilateral triangle, and then a midpoint rotation for the other two sides.
square. 27. No. Because the shape is suitable for glide
20. 4.82;semiregular. All vertices have the same arrange- reflection, the rows of parallelograms should
ment. Each is surrounded by a square and two alternate the direction in which they lean (row 1
regular octagons. leans right, row 2 leans left, row 3 leans right, and
so on).
21. y  12 x. Make and label a diagram as shown.
28.
B B
F A
D
y
x

C C

Discovering Geometry Solutions Manual CHAPTER 7 135


©2003 Key Curriculum Press
DG3SM586_136-181.qxd 8/2/06 6:03 PM Page 136

11. 6 square units. The figure is a parallelogram with


CHAPTER 8 base length 3 units and height 2 units, so its area is
(3)(2)  6 square units.
LESSON 8.1
12. 712 square units. The trapezoid
EXERCISES can be subdivided into two right
1. 228 m2. Use the Rectangle Area Conjecture. triangles and a rectangle. Each
A  bh  (19)(12)  228 m2. of the triangles is half of a
rectangle.
2. 41.85 cm2. A  bh  (9.3)(4.5)  41.85 cm2.
Area of triangle on left  12(2)(3)  3 square units
3. 8 yd. A  bh, so 96 yd2  b  12. 96
  8,
so b  8 yd.
12 Area of rectangle  (1)(3)  3 square units
273
4. 21 cm. A  bh, so 273  13h.  Area of triangle on right  12(1)(3)  32 
13  21,

so h  21 cm. 112 square units

5. 91 ft2. First use the given perimeter and height Area of trapezoid  3  3  112  712 square units
to find the length of the base. P  2b  2h, 13. Look for ways to factor 64. Use one factor for the
so 40  2b  2(7)  2b  14. Therefore 2b  26, length of the base and the other for the height.
and b  13 ft. Now use the length of the base and Sample answer:
the height to find the area of the rectangle: 16 cm
A  bh  (13)(7)  91 ft2. 8 cm
64 cm2 4 cm
64 cm2
6. 182 m2. The shaded area is the difference between
the area of the large rectangle, which has base 8 cm
length 21 m and height 12 m, and the area of the
small (unshaded) rectangle, which has base length 14. There are many possible solutions. Sample answer:
21  11  10 m and height 12  5  7 m. Then 16 cm
16 cm
the shaded area is (21)(12)  (10)(7)  252  70  4 cm
16 cm
182 m2. 16 cm

7. 96 in.2. Use the Parallelogram Area Conjecture. 15. 23.1 m2. The area of each panel is (1)(0.7)  0.7 m2,
A  bh  (12)(8)  96 in.2. and there are (11)(3)  33 panels, so the total area
8. 210 cm. First use the given area and height to find of the arch is 33(0.7)  23.1 m2.
the length of the base, and then use the length of 16. 57 m2. Two of the walls will be rectangles with
the base and the height to find the perimeter. dimensions 4 m by 3 m, and the other two walls
A  bh will be rectangles with dimensions 5.5 m by 3 m.
Therefore the total area of the four walls is
2508  b  44 2(4)(3)  2(5.5)(3)  57 m2.
b  57 17. 625 m2. For a constant perimeter, the rectangle of
P  2b  2h maximum area is a square. Therefore, the length of
each side of the pen will be 100  4  25 m, and
P  2(57)  2(48)  210 cm the area will be (25)(25)  625 m2.
9. A = 42 ft2. The shaded region is one of two 1
18. .
530 First find the area of the football field: A 
congruent triangles that make up a parallelogram. (53)(100)  5300 yd2. The area of each square is
Area of shaded region  12(area of parallelogram) 
1 1 1 yd2, so the probability that the math club wins is
bh  (12)(7)  42 ft2. 10 yd2 1
2 2 2   
5300 yd 530 .
10. Factor 48 in two different ways. For example,
48  4  12 and 48  8  6. (Other factorizations 19. 112 tiles. First convert the dimensions of the wall
are possible.) Sample answer: from feet to inches: 4 ft  4(12 in.)  48 in.,
7 ft  7(12 in.)  84 in. The area of the wall is
12 cm 6 cm
(48)(84)  4032 in.2, and the area of each tile is
4 cm 48 cm2
(6)(6)  36 in.2, so the number of tiles required
8 cm 4032 in.2
is 36 in.2  112.
48 cm2 

136 CHAPTER 8 Discovering Geometry Solutions Manual


©2003 Key Curriculum Press
DG3SM586_136-181.qxd 8/2/06 6:03 PM Page 137

20. x 3 21. border is 2(15)(114)  2(72)(15)  3420 


2 4x 10
2160  5580 in.2.
x x2 3x
15 in.
3x 6x2 15x
84 in.

5 5x 15
2x 5
15 in.
72 in.
22. 100 cm2; (36  64) cm2. The area of the large
square equals the sum of the areas of the small
squares.
23. 96 square units. A  (6)(16)  96 square units.
24. 500 cm2. The dashed segments show that the trape- 26. a  76°, b  52°, c  104°, d  52°, e  76°,
zoid can be subdivided into two right triangles and f  47°, g  90°, h  43°, k  104°, and m  86°.
a rectangle, where each of the right triangles is half For reference, let P be the center of the circle. First,
of a rectangle. mDBC  90° (Angles Inscribed in a Semicircle
Conjecture), and also mPBA  90° (Tangent
Area of trapezoid  12(5)(20)  (12)(20)  Conjecture). Therefore, both DBP and ABC are
1
(21)(20)  50  240  210  500 cm2. complements of PBC, so b  52°. Next, AB  AC
2
25. a. In one Ohio Star block, the sum of the areas of (Tangent Segments Conjecture), so ABC is
the red patches is 36 in.2, the sum of the areas of isosceles with BAC as the vertex angle. Then, a 
the blue patches is 72 in.2, and the area of the 2b  a  2(52°)  180° (Triangle Sum Conjecture),
yellow patch is 36 in.2. so a  76°. Next, DPB is isosceles with vertex
angle DPB because two of its sides are radii, so
b. The complete quilt requires 42 blocks. The area d  52° (Isosceles Triangle Conjecture). Then,
of the quilt is (72)(84)  6048 in.2, and the area e  2(52°)  180° (Triangle Sum Conjecture), so
of each square is (12)(12)  144 in.2, so the e  76°, and c  d  52°  52°  52°  104°
6048 in.2
number of squares is  144 in.2  42. Notice that (Triangle Exterior Angle Conjecture). Also, k  104°

each dimension of the quilt is a multiple of because this arc is intercepted by the central angle
12 in., so 42 squares will cover the area exactly. of measure c (definition of the measure of an arc).
12  6; 12  7; 6  7  42.
72 84 Next, f  12(94°)  47° (Inscribed Angle Conjec-
ture), and g  90° (Angles Inscribed in a Semicircle
c. About 1814 in.2 of red fabric, about 3629 in.2 of Conjecture). Then, h  f  g  180° (Triangle Sum
blue fabric, and about 1814 in.2 of yellow fabric. Conjecture), so h  43°, and m  2(43°)  86°
The border requires 5580 in.2 (if it does not need (Inscribed Angle Conjecture). (Also, m  180° 
the extra 20%). 94°  86° because DEC is a semicircle.)
Red fabric: There are 42 quilt squares, each . Construct a line perpendicular to AM at
27. Copy AM
with 36 in.2 of red fabric; also multiply by 1.2
M. Construct a 60° angle (an angle of any equilat-
(120%) to allow for the extra 20%: 42(36)(1.2) 
eral triangle), and bisect it to form two 30° angles
1814 in.2.
 as one of their sides. Extend the other
that have AM
Blue fabric: There are 42 quilt squares, each sides of these angles to intersect the perpendicular
with 72 in.2 of blue fabric; also multiply by 1.2 through M. The two intersection points will be the
(120%) to allow for the extra 20%: 42(72)(1.2)  other two vertices of the required equilateral
3629 in.2. triangle. Sample construction:
Yellow fabric: The same amount of yellow fabric
is needed as red, so the amount of yellow fabric
is also about 1814 in.2.
Border: From the diagram, you can see that the
area of the border can be calculated by adding
30° M
the areas of two rectangles with dimensions A
30°
114 in. by 15 in. and two rectangles with dimen-
sions 72 in. by 15 in. Therefore the area of the

Discovering Geometry Solutions Manual CHAPTER 8 137


©2003 Key Curriculum Press
DG3SM586_136-181.qxd 8/2/06 6:03 PM Page 138

28. a.

b.

c.

PROJECT
Project should satisfy the following criteria:
● Student provides clear descriptions, constraints,
explanations, and predictions.
● Student includes graphs similar to these:

Extra credit
● Other graphs and relationships are presented.
EXTENSIONS
A. Results will vary.
B. There are two answers: 4 by 4 and 3 by 6. Possible
explanation: For the perimeter to be equal to the
area, there must be the same number of unit
squares in the rectangle as there are unit lengths on
the sides. If you count one square of area for each
unit of length, you will count each square on the
corner twice, for a total of four extra squares. The
perimeter will be equal to the area only if the area
of these four extra squares equals the area of the
internal squares—the squares not counted in the
perimeter. This happens only in rectangles that are
4 by 4 or 3 by 6.
C. Approximate areas: Tennessee: 42,000 mi2;
Utah: 85,000 mi2; Wyoming: 98,000 mi2.

LESSON 8.2
EXERCISES
1. 20 cm2. Use the Triangle Area Conjecture.
A  12bh  12(8)(5)  20 cm2.
2. 49.5 m2. A  12bh  12(11)(9)  49.5 m2.
3. 300 square units. Use the Kite Area Conjecture
with d1  9  16  25 and d2  12  12  24.
A  12d1d2  12(25)(24)  300 square units.

138 CHAPTER 8 Discovering Geometry Solutions Manual


©2003 Key Curriculum Press
DG3SM586_136-181.qxd 8/2/06 6:03 PM Page 139

You can also find the area of the kite by adding the 10. 168 cm. Let b represent the unmarked side of the
areas of the four right triangles that are formed by triangle. The height to this side is 24 cm. Find b,
the diagonals. If you use this method, the area of and then use this side length to find the perimeter
the kite is 212(12)(9)  212(12)(16)  108  of the triangle.
192  300 square units. 1
A  2bh
4. 60 cm2. Use the Trapezoid Area Conjecture with
1
b1  14 cm and b2  6 cm. A  12h(b1  b2)  924  2b(24)
1
(6)(14  6)  60 cm2.
2 924  b(12)
5. 6 cm. Use the Triangle Area Conjecture.
b  77 cm
1
A  2bh P  51  40  77  168 cm
1
39  2(13)h 11. 12 cm. Let b1  10 cm and let b2 represent the
length of the unmarked base of the trapezoid. First
78  (13)h
use the perimeter and the known side lengths to
78
h 13  6 cm find b2.

6. 7 ft. Use the Triangle Area Conjecture. P  15  10  13  b2


1 62  38  b2
A  2bh
1 b 2  24 cm
31.5  2(b)(9)
Now use the Trapezoid Area Conjecture to find h.
63  b(9)
1
A  2h(b1  b2)
b  7 ft
1
7. 30 ft. Use the Kite Area Conjecture with d1  BU  204  2h(10  24)
20  8  28 ft and d2  LE. 204  h(17)
1
A  2d1d2 h  12 cm
1 12. x  3.6 ft, y  10.8 ft. The area of the triangle can
420  2(28)d2
be calculated in three different ways (using each
420  (14)d2 base and its corresponding altitude), but all of them
d2  LE  30 ft must give the same area. Notice that two of the alti-
tudes lie outside the triangle.
8. 5 cm. Use the Trapezoid Area Conjecture with
1 1 1
b1  13 cm and b2  7 cm. A  2(15)(x)  2(5)(y)  2(6)(9)  27
1 1 1
A  2h(b1  b2) (15)(x)  27
2
(5)(y)  27
2
1 (15)(x)  54 (5)(y)  54
50  2h(13  7)
54 54
50  10h x  1
5 y  5
h  5 cm  3.6 ft  10.8 ft
9. 16 m. Use the Trapezoid Area Conjecture with 13. Find two positive integers whose product is 2 times
b1  24 m and b2  b. 54, or 108. Two such integers are 9 and 12. In the
1 pair of triangles shown below, one uses 9 cm as the
A  2h(b1  b2) length of the base and 12 cm as the height, while
1 the other uses 12 cm as the length of the base and
180  2(9)(24  b)
9 cm as the height. Sample answer:
360  (9)(24  b)
40  24  b
b  16 m
12 cm 9 cm

9 cm 12 cm

Discovering Geometry Solutions Manual CHAPTER 8 139


©2003 Key Curriculum Press
DG3SM586_136-181.qxd 8/2/06 6:03 PM Page 140

14. Choose a factor of 56 for the height. Divide 56 by find the amount of balsa wood, and the total area
that number to get the average of the lengths of the of the kite and flaps to find the amount of Mylar.
two bases. For example, if you choose 8 cm for the The sum of the lengths of the diagonals is
height, the average of the lengths of the bases will 2(15)  20  36  86 in. To find the area of
be 14 cm, and one possibility is to make the bases the kite, substitute 30 for d1 and 20  36  56
have lengths 10 cm and 4 cm. Sample answer: for d2 in the kite area formula, A  12d1d2. A 
1
4 cm (30)(56)  840, so the area of the kite is
2
7 cm
840 in.2. Each of the four flaps is a trapezoid
with height 1 in.; two have bases of lengths 21 in.
8 cm 7 cm and 25 in., while the other two have bases of
lengths 35 in. and 39 in. Find the total area of all
the flaps: 212  1(21  25)  212  1(35  39)
10 cm 9 cm

15. Choose a factor of 1092 as the length of one diag-  46  74  120, so the area is 120 in.2. The
onal. One possible choice is 24. You can use 24 cm total amount of Mylar needed is 840  120 
as the length of the diagonal that is bisected by the 960 in.2.
other diagonal. Because one-half the product of the b. 56 in. (or less, if he tilts the kite). Because the
lengths of the diagonals is 1092, the product is length of the shorter diagonal is 30 in., the easiest
2184. Then the length of the other diagonal will be way to cut the kite out of a 36-inch wide piece of
2184
  91. You can split the length of 91 cm in Mylar is to place the shorter diagonal along the
24
many ways, corresponding to different positions width of the unrolled Mylar. Then the length of
where the diagonals intersect. The figures below Mylar needed would simply be the length of the
show two ways to do this. Sample answer: longer diagonal of the kite, which is 56 in.
20. 3600 shingles (to cover an area of 900 ft2). To find
the total area to be covered with shingles, add the
46 cm 35 cm
areas of the two congruent trapezoids and the two
congruent triangles.
12 cm 12 cm
Area of two trapezoids: 212  15(20  30)  750,
so the sum of the areas of the two trapezoids is
750 ft2.
45 cm 56 cm
Area of two triangles: 212  10  15  150, so the
sum of the areas of the two triangles is 150 ft2.
Therefore the total area of the roof is 750  150 
16. The length of the base of the triangle equals the 900 ft2. Because each shingle covers 0.25 ft2, Crystal
900 ft2
should buy  0.25 ft2  3600 shingles.

sum of the lengths of both bases of the trapezoid.
2 cm 21. The isosceles triangle is a right triangle because the
angles on either side of the right angle are comple-
3 cm 3 cm
mentary. If you use the trapezoid area formula, the
7 cm 9 cm area of the trapezoid is 12(a  b)(a  b). If you add
 of TPR. Then the area of the areas of the three triangles, the area of the
17. 12. Draw altitude PQ
trapezoid is 12ab  12c 2  12ba  12c 2  ab.
TPR  12(TR)(PQ)  12(TR)(AR)  12(area of
rectangle ARTY). Notice that this result does not 22. Given: trapezoid ABCD with A b1 B
depend on the position of P, which can be any height h. Area of ABD  12hb1. h
point on AY . Area of BCD  12hb2. Area of
D b2 C
trapezoid  sum of areas of two
18. More than half, because the top card completely 1 1 1
triangles  2hb1  2hb2  2h(b1  b2).
covers one corner of the bottom card. Notice that
this is like the figure in Exercise 17, but the covered 23. 1114 square units. Count the complete and fractional
area of the card here is a larger fraction of the squares inside the figure. There are 7 whole squares,
rectangle (or card) than in Exercise 17, so the top 7 half-squares, and 1 three-quarter square. Add
card covers more than half the bottom card. these to get the area of the figure: A  7(1) 
712  134  7  312  34  1114, so the area of
19. a. 86 in. of balsa wood and 960 in.2 of Mylar.
the figure is 1114 square units.
Find the sum of the lengths of the diagonals to

140 CHAPTER 8 Discovering Geometry Solutions Manual


©2003 Key Curriculum Press
DG3SM586_136-181.qxd 8/2/06 6:03 PM Page 141

24. 7 square units. Enclose the triangle 29. Draw the prism unfolded. The path from C to M to
in a square. Find the area of the I
II T lies on a straight line, and therefore must be
original triangle by subtracting shorter than the path from C to A to T.
the areas of the three surrounding III P A T
right triangles from the area of the
square. M

Area of square  42  16 C U B

Area of triangle I  12(4)(2)  4 30. 32.623.6.3.6. There are two types of vertices. One
type of vertex is surrounded by two equilateral
Area of triangle II  12(2)(3)  3 triangles, then two regular hexagons. The other type
Area of triangle III  12(4)(1)  2 of vertex is surrounded by an equilateral triangle,
then a regular hexagon, then another equilateral
Therefore the area of the original triangle is triangle, and then another regular hexagon.
16  (4  3  2)  16  9  7 square units.
PROJECT
25. 70 m. First find the length of the base of the
rectangle. Let b represent the length of the base. You might gather some data points and use quadratic
A  bh, so 264  b(24), and b  11 m. Now find regression rather than finding and graphing a function
the perimeter of the rectangle: P  2b  2h  2(11) expression. Projects should satisfy the following criteria:
 2(24)  70 m. ● The expression and equation are equivalent to
A  x(10  2x).
26. 144 cm2. The parallelogram has two sides of length
10 cm. Let b represent the length of each of the ● The graph shows the maximum area at x  2.5.
other two sides. Using the given perimeter, 2b  Extra credit
2(10)  52 cm. Then, b  16 cm, and A  bh 
● The project includes an explanation of why the
(16)(9)  144 cm2.
maximum area is 12.5 m2. One reason, not using the
27. a  34°, b  68°, c  68°, d  56°, e  56°, f  90°, graph, is that if there were twice as much fencing
g  34°, h  56°, m  56°, n  90°, and p  34°. available and no barn, then the region of maximum
For reference, let O be the center of the circle. First, area would be a square (by Exercise 17 in Lesson 8.1);
  2(56°)  112° (Inscribed Angle Conjecture).
mBC the barn wall can be considered a line of symmetry,
Because DBC is a semicircle, b  180°  112°  68°.
cutting the amount of fencing in half but making the
Next, a  12b (Inscribed Angle Conjecture), so a  shape half a square, with width 14 the total amount of
34°. By the definition of the measure of an arc, c  b, fencing.
so c  68°. Now look at BAO and CAO. Notice
  OC
that OB  (all radii of a circle are congruent), EXTENSIONS
  AC
AB  (Tangent Segments Conjecture), and AO  A. In a triangle, each midsegment is one-half the
 (same segment), so BOA  COA by SSS.
 AO length of a side. If you consider one side as the
Therefore d  e by CPCTC. Observe that c  d  e base and label its length as b, then length of
 180°, so 68°  2d  180°. Then, 2d  112°, so midsegment  12b. Then, area of triangle  12bh 
d  56°, and also e  56°. By the Tangent Conjec- (midsegment)(height).
ture, OC  AC, so f  90°. Next, g  e  f  180° In a trapezoid, there is one midsegment, and its
(Triangle Sum Conjecture), so g  56°  90°  180°, length is half the sum of the lengths of the bases,
and g  34°. Next, h  56° (Inscribed Angles Inter- that is, length of midsegment  12(b1  b2). Then,
cepting Arcs Conjecture) and m  12(112°)  56° area of trapezoid  12h(b1  b2)  12(b1  b2)h 
(Inscribed Angle Conjecture). Now, by the conjec- (midsegment)(height).
ture about the measure of an angle formed by two
intersecting chords (see Lesson 6.5, Exercise 16), B. Possible answers (all use trapezoid ABCD with long
n  12(b  112°)  12(68°  112°)  12(180°)  90°. , short base CD
base AB , b1  AB, b2  CD, and h
Finally, p  n  56°  180° (Triangle Sum Conjec- as the perpendicular height between the bases):
ture), so p  180°  90°  56°  34°. Area of trapezoid  area(ABC)  area(CDA)
28. T1 1 1 1
 2b1h  2b2h  2h(b1  b2)
T2
D b2 C

h h

A b1 B

Discovering Geometry Solutions Manual CHAPTER 8 141


©2003 Key Curriculum Press
DG3SM586_136-181.qxd 8/2/06 6:03 PM Page 142

 so that ED
Construct point E on AB  is parallel to 2. He should buy at least four rolls of wallpaper. Two
. Therefore, area of trapezoid  area(AED) 
BC walls are rectangles with dimensions 11 ft by 10 ft.
area(parallelogram EBCD)  12h(b1  b2)  hb2  The other two walls are rectangles with dimensions
1
h(b1  b2). 13 ft by 10 ft. Find the sum of the areas of these
2
rectangles: 2(11)(10)  2(13)(10)  480 ft2, so the
D b2 C
total surface area to be papered is 480 ft2. Now find
h h the area of each roll: (2.5)(50)  125, so the area of
each roll is 125 ft2. Finally, find the number of rolls
A E b1 B of wallpaper that is needed: 480 ft2   1 roll
125 ft2 

3.84 rolls; round up to 4 rolls. If paper cut off at
  AB
Construct DE , with points E and F on the corners is wasted, he’ll need 5 rolls.
. Let c  AE. Then, FB  b1  b2  c.
AB
Therefore, area of trapezoid  area(AED)  3. 1,552 ft2; 776 ft2 more surface area. First, find the
area(rectangle EFCD)  area(FBC)  12ch  b2h  area of 65,000 rectangular cells: 65,000(1.25)(2.75)
1 1
(b1  b2  c)h  h(b1  b2).  223,437.5, so the total area of the cells is
2 2
223,437.5 in.2. Convert this area to square feet.
b2
D C
There are 12 in. in a foot, so there are 12(12) 
h h
144 square inches in a square foot. 223,437.5 in.2 
1 ft2
2  1,552 ft2. If the cells are only 12%
c 144 in.
A E b1 F B efficient, 50% more area will be needed than if they
are 18% efficient, so the additional surface area that
Start as you did in the previous method, then dupli- would need to be covered would be 0.5(1,552) 
cate AED as DEA and FBC as CFB, 776 ft2.
forming rectangle ABFE. Therefore, area of trape-
zoid  area(rectangle ABFE)  area(DEA)  4. $760. The easiest way to find the total area to
area(CFB)  b1h  12ch  12(b1  b2  c)h  be carpeted is to find the area of the complete
1
h(b1  b2). rectangle at the top of the figure and then subtract
2
the areas of the bathrooms. This way, you don’t
E' D b2 C F' have to figure out the dimensions of the hallway.
h h Length of rectangle  10  10  7  27 ft
A b1 B Width of rectangle  9  8  17 ft
Total area to be carpeted
C. Approximate areas: California: 164,000 mi2; Texas:
 total area of three bedrooms and hallway
269,000 mi2; Nevada: 111,000 mi2.
 (27  17)  (6  10  7  9)  336 ft2

LESSON 8.3 Convert to square yards. There are 3 ft in a yard,


so there are 3(3)  9 square feet in a square yard.
1 yd2
EXERCISES 336 ft  9 ft2  3713 yd2; round up to 38 yd2
2  
1. a. 121,952 ft2. Each room has two walls with because carpeting, padding, and installation are
dimensions 14 ft by 10 ft and two walls with priced per square yard.
dimensions 16 ft by 10 ft, while the ceiling is
14 ft by 16 ft. Find the sum of the areas of these The cost for carpeting, padding, and installation is
rectangles: 2(14)(10)  2(16)(10)  (14)(16)  $14  $3  $3  $20 per square yard, so the cost
824, so the total area to be painted for one room for 38 yd2 will be 38($20)  $760.
is 824 ft2, and the total for 148 rooms is 148(824) 5. 220 terra cotta tiles, 1107 blue tiles; $1,598.15. First
 121,952 ft2. find the total area of the entryway and kitchen.
b. 244 gallons of base paint and 488 gallons of Look at the rectangle that contains the entryway,
finishing paint. Find the number of gallons of kitchen, living room, and dining room, and at the
each kind of paint separately. two smaller rectangles containing the entryway and
1 gal
Base paint: 121,952 ft2   
500 ft2  243.9 gal;
kitchen (on the left) and living room and dining
round up to 244 gal. room (on the right). From the dimensions of these
1 gal rooms, you can see the total area of the entryway
Finishing paint: 121,952 ft2   
250 ft2  487.8 gal; and kitchen is (10)(22)  220 ft2. Because 1-foot-
round up to 488 gal. square tiles are to be used for the entryway and
kitchen, 220 tiles will be needed.

142 CHAPTER 8 Discovering Geometry Solutions Manual


©2003 Key Curriculum Press
DG3SM586_136-181.qxd 8/2/06 6:03 PM Page 143

The dimensions of the bathrooms are 6 ft by 10 ft Then substitute into Hero’s formula:
and 7 ft by 9 ft, so the total area of the two bath- A   s(s  a )(s  b
)(s  c) 
rooms is 60  63  123 ft2. The tiles for the bath-  8)(2 0  15 )(20  17) 
 20(20
room floors are 4-inch-square tiles, so there are 9 of
20 • 12 • 5 3  3600
•  60, so the area of the
these tiles per square foot: 142  3; 3  3  9. Find
the number of blue tiles needed: 123 ft2   9 tiles
1 ft2 
 triangle is 60 cm2.
1107 tiles. Using the standard triangle area formula,
Finally, find the cost of the tiles. A  12bh  12(15)(8)  60, so by this formula,
the area is 60 cm2.
Terra cotta tiles: 220($5)  $1,100
  BO
10. AO  because all radii of a circle are
Blue tiles: 1107($0.45)  $498.15 congruent, so AOB is isosceles. Therefore,
Total cost: $1,100  $498.15  $1,598.15 mA  20° and mAOB  140°. mAB   140°

and mCD  82° (definition of arc measure).
6. 21 gallon containers. First find the area of the kite.
  mBD
mAC  because parallel lines
The lengths of the diagonals are 40 ft and 70 ft,
so the area of the kite is 12(40)(70)  1400 ft2. Find intercept congruent arcs on a circle.
360°  140°  82°
the amount of sealant needed for one application: x    69°.
2
1400 ft2  
1 gal
400 ft2  3.5 gal. Harold will make 6

11. E. None of the values changes.
applications of sealant (twice a year for three years),
A. Moving P doesn’t change the height of ABP, so
so he will need 6(3.5)  21 gal of sealant. Therefore the area of ABP won’t change.
he should buy 21 one-gallon containers.
B. Moving P doesn’t change the height of PDC,
7. 336 ft2; $1,780. First find the area of one flowerbed. so the area of PDC won’t change.
Each flowerbed is a trapezoid with height 7 ft and C. P is not part of trapezoid ABCD, so its position
bases of lengths 12 ft and 20 ft, so its area is can’t affect the area.
1
(7)(12 + 20)  112 ft2. Then the total area of the
2 D. mPCD  mB (CA Conjecture), so mA 
three flowerbeds is 3(112)  336 ft2, and the cost
mPCD  mCPD  180°, regardless of the
will be $100  336($5)  $1,780.
position of P.
8. $384 (16 gal). Add the areas of all the walls to be
painted. IMPROVING YOUR VISUAL THINKING SKILLS
This problem can be done in hundreds of ways. One
Front rectangle: 10(40)  400 ft2 unusual way is to rearrange pieces of the triangle to
2 rectangles of front portion: 2(28)(10)  560 ft2 make a parallelogram or rectangle of the same area,
divide the resulting rectangle into four pieces, and then
2 rectangles of back portion: 2(18)(10)  360 ft2 rearrange those pieces back into a triangle.
Far-back rectangle: 24(10)  240 ft2
Rectangle at back of front portion: 16(10)  160 ft2
Front triangle: 12(40)(16)  320 ft2
Back triangle: 12(24)(10)  120 ft2
Partially covered rectangle at back of front portion:
1 1
(40)(16)  (24)(10)  320  120  200 ft2
2 2
Total surface area of walls  2360 ft2
Find the number of gallons of paint required.
2360 ft2  
1 gal
150 ft2  15.7 gal; round up to 16 gal.


The cost of 16 gal of paint is 16($24)  $384. EXTENSIONS


A. Results will vary. Divide the figure into more and
9. 60 cm2 by either method. The first step in using
more pieces and add the resulting areas. The areas
Hero’s formula is to find the semiperimeter: will approach some number (the actual area) as the
abc 8  15  17
s  2
    20.
2 number of pieces increases.
B. Results will vary.

Discovering Geometry Solutions Manual CHAPTER 8 143


©2003 Key Curriculum Press
DG3SM586_136-181.qxd 8/2/06 6:03 PM Page 144

LESSON 8.4 Use the formula A  12asn to approximate the area


of the hexagon. By construction, s  4 cm.
EXERCISES Measure the apothem with a ruler: a  3.5 cm.
1. 2092 cm2. The figure is a heptagon, so n  7. Therefore, A  12asn  12(3.5)(4)(6)  42 cm2.
1 1 10.  58 cm2. Draw a circle with radius 4 cm, and use
A  2asn  2(24.9)(24)(7)  2092 cm2
your protractor to form five congruent central
360°
2. 74 cm. The figure is a pentagon, so n  5. angles, each of measure  5  72°. Use the sides

1 of these angles to form five radii of the circle. Use
A  2asn a protractor and the Tangent Conjecture to draw
1 tangents to the circle at each of the points where
19,887.5  2a(107.5)(5)
the five radii touch the circle. The tangent segments
39,775  a(537.5) form a regular pentagon circumscribed about the
39,775 circle.
a 
537.5  74 cm
3. 256 cm. Use the formula A  12aP.
1
4940.8  2(38.6)P
9881.6  38.6P
9881.6
P 
38.6  256 cm
4. 33 cm2. A  12asn  12(3)(4.4)(5)  33 cm2
5. 63 cm. Use the formula A  12aP.
302.4  12(9.6)P
Because each radius is perpendicular to a tangent,
604.8  9.6P
the radii form apothems of the pentagon, so
P  63 cm a  4 cm, as required. Measure one of the
tangent segments to find the length of a side of
6. 490 cm2. A  12aP  12(12)(81.6)  490 cm2
the pentagon: s  5.8 cm. Then, A  12asn 
1
7. 57.6 m. Use the formula A  12aP. Solve the (4)(5.8)(5)  58 cm2.
2
equation 259.2  12(9)P to obtain P  57.6.
11. It is impossible to increase its area, because a
The perimeter is 57.6 m.
regular pentagon maximizes the area. Any dragging
8. 25 ft. Use the formula A  12asn to find the value of the vertices decreases the area. (Subsequent drag-
of s. ging to space them out more evenly can increase the
1 area again, but never beyond that of the regular
20,000  2(80)s(20) pentagon.)
20,000  800s 12.  996 cm2. First find the area of the complete
s  25 octagon.
1 1
The length of each side is 25 ft. A  2asn  2(20)(16.6)(8)  1328 cm2

9.  42 cm2. Construct a circle with radius 4 cm. The shaded area is 68, or 34, of the octagon, so
Mark six 4-centimeter chords around the circle. its area is 34 of the area of the octagon: 34(1328) 
996 cm2. Because the given measurements for the
s octagon are approximate, the shaded area is approx-
imately 996 cm2.
a 13.  497 cm2. The area of the hexagonal donut is the
difference between the area of the large hexagon
and the area of the smaller hexagon. In a regular
hexagon, the distance from the center to each vertex
is equal to the length of each side, so in the small
hexagon, s  r  8 cm, and in the large hexagon,
s  2r  16 cm. The given measurements are

144 CHAPTER 8 Discovering Geometry Solutions Manual


©2003 Key Curriculum Press
DG3SM586_136-181.qxd 8/2/06 6:03 PM Page 145

approximate, so the areas calculated from these To find the area of triangle A, use the side along the
measurements will also be approximate. y-axis as the base. Because the y-intercept of line
y  12x  5 is 5, the length of this base is 5 units.
Area of large hexagon  12asn  12(2  6.9)(16)(6) The two lines intersect at (2, 6), so the length of the
 662.4 cm2
altitude to this base is 2 units. (This altitude will fall
Area of small hexagon  12asn  12(6.9)(8)(6)  outside the triangle.) Therefore, area(triangle A) 
1 1
165.6 cm2 bh  (5)(2)  5 square units.
2 2
Area of donut  (area of large hexagon)  To find the area of triangle B, use the side along
(area of small hexagon)  497 cm2 the x-axis as the base. Because the x-intercept of
the line y  2x  10 is 5, the length of this base
14. Total surface area  13,680 in.2  95 ft2; cost 
is 5 units. The two lines intersect at (2, 6), so the
$8,075. To find the total surface area, divide the
length of the altitude to this base is 6 units.
lower portion of the kitchen floor plan into three
Therefore, area(triangle B)  12bh  12(5)(6) 
rectangles and the upper portion into two rectangles
15 square units. Thus, the total area of the
and one regular octagon.
quadrilateral is 5  15  20 square units.
144 in.
120 in. 16. 36 square units. As in Exercise 15, divide the
24 in. 60 in. quadrilateral into two triangles, A and B, by
60 in. connecting the origin to the point where the
36 in.
two lines intersect, which is (6, 4) in this case.
24 in.
y
48 in.
24 in.
24 in.
48 in.
36 in.
72 in. y  – _4 x  12
60 in. 3
24 in. A
138 in. (6, 4) y  – _1 x  6
3
186 in. B
x

In the octagon, s  24 in., a  12(60)  30 in., and


n  8.
Let A be the upper triangle. The y-intercept of the
Lower portion: (24)(60)  (24)(138)  (24)(72)  line y  13x  6 is 6, so the vertices of this triangle
6480 in.2 are (0, 0), (0, 6), and (6, 4). Use the side along the
y-axis as the base. The length of the altitude to this
Upper portion: (24)(60)  (120)(24) 
base is 6, so area(triangle A)  12(6)(6)  18 square
2(30)(24)(8)  7200 in.2
1
units. Let B be the lower triangle. The x-intercept of
The total surface area of the countertops is 6,480  the line y  43x  12 is 9, so the vertices of this
7,200  13,680 in.2. triangle are (0, 0), (9, 0), and (6, 4). The length of
the altitude to this base is 4, so area(triangle B) 
Convert to square feet: 13,680 in.2  1 ft 2
144 in.2  95 ft .
 2 1
(9)(4)  18 square units. Thus, the total area of
2
The cost of the countertop will be 95($85)  the quadrilateral is 18  18  36 square units.
$8,075.
17. Conjecture: The three medians of a triangle divide
15. 20 square units. Divide the quadrilateral into two the triangle into six triangles of equal area.
triangles, A and B, as shown in this figure. Find the Argument: Triangles 1 and 2 have equal area
areas of each of these triangles, and add to find the because they have equal bases and the same height.
area of the quadrilateral. Because the centroid divides each median into
y thirds, the height of triangles 1 and 2 is 13 the height
y  _1x  5
2 of the whole triangle. Each has an area 16 the area of
the whole triangle. By the same argument, the other
small triangles also have areas of 16 the area of the
(2, 6)

A y  –2x  10
whole triangle.
B
x

h 2
1

Discovering Geometry Solutions Manual CHAPTER 8 145


©2003 Key Curriculum Press
DG3SM586_136-181.qxd 8/2/06 6:03 PM Page 146

18. nw  ny  2x. Write the expressions for the LESSON 8.5


perimeter of the first few figures and look for a
pattern. Each figure has two sides of length x, and EXERCISES
the number of sides of lengths w and y increases 1. 9 in.2. A  r 2  (3)2  9 in.2.
by 1 each time, so the perimeter of the nth figure
2. 49 cm2. A  r 2  (7)2  49 cm2.
is nw  ny  2x.
3.  0.79 m2. A  r 2  (0.5)2  0.25 m2 
19. a. Incenter. The construction marks show that this
0.79 m2.
is the point of concurrency of the angle bisectors.
4. 3 cm. A  r 2  9 cm2, so r 2  9, and r  3 cm.
b. Orthocenter. The construction marks show that
(Note that r must be positive because it represents
this is the point of concurrency of the altitudes.
the length of a segment; the negative square root of
c. Centroid. The construction marks show that this 9, which is 3, is not relevant in this situation.)
is the point of concurrency of the medians.
5. 3 in. A  r 2  3 in.2, so r 2  3, and
IMPROVING YOUR VISUAL THINKING SKILLS r  3 in.
A: 292  841; B: 332  1089; C: 372  1369; D: 422  6.  0.5 m. A  r 2  0.785 m2, so r 

1764; E: 252  625; F: 162  256; G: 182  324;

0.785
 

0.5 m.
H: 242  576; I: 92  81; J: 72  49; K: 152  225;
L: 22  4; M: 172  289; N: 62  36; O: 112  121; 7. 36 in.2. C  2r  12 in., so r  6 in. Then,
P: 192  361; Q: 352  1225; R: 82  64; S: 272  729. A  r 2  (6)2  36 in.2.
314
EXTENSIONS 8.  7846 m2. C  2r  314 m, so r   
2 m. (Keep
this number stored in your calculator.) Then,
A. Possible answer: Consider a regular polygon with a 314 2
A  r 2  2   7846 m .
 2
point at the center and divided into triangles with
each side of the polygon as a base, where a is the 9. (25  48)  30.5 square units. The area of the
apothem, s is the length of each side, and n is the shaded region is the difference between the area
number of sides in the regular polygon. If the of the circle and the area of the rectangle. Circle:
polygon has an odd number of sides, the triangles r  5, so A  25. Rectangle: The dimensions are
can be arranged into a trapezoid with long base 2(3)  6 by 2(4)  8, so A  6  8  48. The area
n1 n1
s, short base s, height a, and area
2 2 of the shaded region is (25  48) square units.
n1 n1 10. (100  128)  186 square units. The area of the

 
s  s
2 2 1 shaded region is the difference between the area of
 a  nas
2 2 the circle and the area of the triangle. Circle: r  10,
so A  100. Triangle: Use the horizontal segment
at the top of the triangle as the base. Its length is
n1 s
_____ 2(8)  16, and the corresponding height is 6  10
2
 16. Therefore, A  12(16)(16)  128. The area of
a a the shaded region is (100  128) square units.
s s
n1 s
_____
2
11. A  r 2  324 cm2, so r  324
 18 cm.
Odd number of sides
Regular pentagon
(n  5)
r  18 cm
If the polygon has an even number of sides, the
triangles can be arranged into a parallelogram with
base n2s, height a, and area 12nas.

n
_s
2
12. 804 m2. A  r 2  (16)2  256  804 m2.
a a 13. 11,310 km2. A  r 2  (60)2  3,600 
s s
11,310 km2.
n
_s
2 14. 154 m2. A  r 2  (7)2  49  154 m2.
Even number of sides
Square 15. 4 times. Find the areas of the two circular regions
(n  4) and then compare the areas.
B. Results will vary.

146 CHAPTER 8 Discovering Geometry Solutions Manual


©2003 Key Curriculum Press
DG3SM586_136-181.qxd 8/2/06 6:03 PM Page 147

Muscle with radius of 3 cm: A  9 cm2 IMPROVING YOUR VISUAL THINKING SKILLS
8
Muscle with radius of 6 cm: A  36 cm2 .There are 2 isosceles triangles with vertex angle at
21
strength of second muscle 36 cm2 each corner, 4 with vertex angle at each side midpoint,
strength of first muscle  9 cm2  4
Ratio:  
and 8 with vertex angle in the center: 2  4  4  4 
Therefore the second (larger) muscle is 4 times as 8  32. There are 84 possible ways to select three
strong as the first (smaller) muscle. 987
points from the grid:  2  3  84. 32 out of 84 gives
16. A  r 2 because the 100-gon almost completely a probability of 3824  281 .
fills the circle. Observe how, as the number of sides
of a regular polygon increases, more and more of LESSON 8.6
the circle is filled.
EXERCISES
a
r a r 1. 6 cm2. The shaded region is a sector of the circle
with a  60°. Asector  
360° r   360° (6) 
a
 2 60°
 2

6(36)  6. The area of the shaded region is


1

Hexagon Dodecagon 6 cm2.


64
17. 456 cm2. The figure is a rhombus, so use the 2. 
3 cm2. The shaded region is a sector of the
Parallelogram Area Conjecture: A  bh  circle with a  360°  240°  120°. Asector 
64

360°  r   360° (8)   3 (64)  3 . The area
a 120° 1
(24)(19)  456 cm2.  2  2  
64 2
18. 36 ft2. Use the Triangle Area Conjecture. The side of of the shaded region is  
3 cm .
length 9 ft is the altitude to the side of the triangle 3. 192 cm2. The shaded region is a sector of the
with length 8 ft. A  12bh  12(8)(9)  36 ft2. circle with a  360°  90°  270°. Asector 

360°  r   360°  (16)   4 (256)  192.
a 2 270° 2 3
19. The triangles have equal area when the point is at   
the intersection of the two diagonals. There is no The area of the shaded region is 192 cm2.
other location at which all four triangles have equal
area. 4. (  2) cm2. The shaded region is a segment of
  2 24°  48° (Inscribed Angle Conjecture). the circle with a  90°. The area of the segment
20. mDE 
  48° (Parallel Lines Intercepted Arcs
x  mDE is the area of the sector minus the area of the
Conjecture). triangle. Asegment  
360° r  2 bh   360° (2) 
a
 2 1
 
90° 2

21. 90°  38°  28°  28°  180°. The measure of the


1
(2)(2)
2  14(4)  2    2. The area of the
unmarked base angle of the isosceles triangle is 28° shaded region is (  2) cm2.
(Isosceles Triangle Conjecture). Add the angle mea-
sures of the large right triangle: 90°  (38°  28°)  5. (48  32) cm2. The area of the shaded region is
28°  184°, but this is impossible, because the sum the sum of the areas of the sector, which is 34 of the
of the angles of any triangle must be 180° (Triangle circle, and the triangle, so A  34(64)  12(8)(8) 
Sum Conjecture). 48  32. The area of the shaded region is (48 
32) cm2.
22.
6. 33 cm2. The shaded region is an annulus of the
circle. Aannulus  R2  r 2  (7)2  (4)2 
49  16  33. The area of the shaded region
is 33 cm2.
24 cm
7. 21 cm2. The shaded region is an annulus of the
12 cm circle. The radius of the larger circle is 12(10) 
5 cm. Aannulus  R2  r 2  (5)2  (2)2 
25  4  21. The area of the shaded region is
18 cm 21 cm2.
6 cm

Discovering Geometry Solutions Manual CHAPTER 8 147


©2003 Key Curriculum Press
DG3SM586_136-181.qxd 8/2/06 6:03 PM Page 148

105 300° 5
360°  6 of an
8. 
2 cm2. The shaded region is    14. $448. The path around the circular fountain can be
annulus of the circle. A  56(12)2  (9)2  considered an annulus of a larger circle. The radius
5 5 105 of the smaller circle is 12(8)  4 m, and the radius
(144  81)  (63)  . The area of the
6 6 2 of the larger circle is 4  1.5  5.5 m. Area of path
105
shaded region is 2 cm2.  (5.5)2  (4)2  30.25  16  14.25. The
9. r  6 cm. The shaded region is a sector of the circle area of the path is 14.25 m2. Now find the cost to
with a  120°. pave this path: 14.25 m2   $10
1 m2  $142.5  $448.


Asector   
a

360° r
2
15. a.

1
12  3r 2
36  r 2 b.
r6
The radius is 6 cm.
10. r  7 cm. The shaded region is an annulus of the c.
circle. The radius of the larger circle is 12(18)  9 cm.
Aannulus  R2  r 2
32  81  r 2
r 2  49
r  7 cm d.
11. x  75°. The shaded region is a sector of the circle
with a  x and r  24 cm.

 
Asector  
a

360° r
2

120  
360° ()(24)
x
 2

x 120 5
    
360° 576 24
24x  5(360°)
24x  1800° 16.
x  75°
x
12. x  100°. The shaded region is 
360° of an annulus 17. The area of each square is (12)(12)  144 cm2.
of the circle. Subtract the area of the circle or circles from the
A   
x

360° (10  8 )
2 2 area of the square to find the area of the shaded
region.
10    
x

360° 36
a. (144  36) cm2; 78.54%. The radius of the
circle is 6 cm, so its area is 36 cm, and the area
10 x of the shaded region is (144  36) cm2. Now
  
36 360° compare the area of the circle to the area of the
10 x square:
  
36 360°
area of circle 36 cm2
  
36x  3600° area of square 144 cm2  78.54%
x  100°
b. (144  36) cm2; 78.54%. The radius of each
13. 42 cans. The pizza slice is a sector of the circle. circle is 14(12)  3 cm, so the area of the four
Area of slice   360° (20 )  10 (400)  40.
36°
 2 1
 circles is 4()(3)2  4(9)  36 cm2. Therefore,
The area of the pizza slice is 40 ft2. Now find the as in 17a, the area of the shaded region is
number of cans of tomato sauce needed to cover (144  36) cm2, and the area of the circles
this slice: 40 ft2  
1 can 40
3 ft2  3  41.9 cans. Round
  is about 78.54% of the area of the square.
up to 42 cans.

148 CHAPTER 8 Discovering Geometry Solutions Manual


©2003 Key Curriculum Press
DG3SM586_136-181.qxd 8/2/06 6:03 PM Page 149

c. (144  36) cm2; 78.54%. The radius of each EXPLORATION • GEOMETRIC PROBABILITY II
circle is 16(12)  2 cm, so the area of the nine
circles is 9()(2)2  9(4)  36 cm2. PROJECT
Therefore, as in 17a, the area of the shaded Project should satisfy the following criteria:
region is (144  36) cm2, and the area of ● Predictions mention possible outcomes and most likely
the circles is about 78.54% of the area of the outcomes.
square.
● Irregularities in weight and shape of dice are minimized.
d. (144  36) cm2; 78.54%. The radius of each
circle is 18(12)  182  1.5 cm, so the area of ● The histogram’s range reflects the minimum and
the sixteen circles is 16()(1.5)2  16(2.25)  maximum possible outcomes and has the approximate
36 cm2. Therefore, as in 17a, the area of the correct shape.
shaded region is (144  36) cm2, and the area ● Simulation results roughly match the theoretical
of the circles is about 78.54% of the area of the probability.
square.
● Report makes a connection between the number of
18. 480 m2. Combine two conjectures about trials and how closely the outcomes match the
trapezoids to find an alternate formula for the expected values.
area of a trapezoid. By the Trapezoid Area
Conjecture, A  12 h(b1  b2). By the Trapezoid Extra credit
Midsegment Conjecture, the length of the ● Unusual dice are explored.
midsegment is 12(b1  b2). Therefore, by substitu-
● Report compares theoretical outcomes for several
tion, A  (midsegment)(height). Thus the area of a
kinds of dice.
trapezoid with height 14 m and midsegment with
length 32 cm is (32)(15)  480 cm2.
LESSON 8.7
90
19. True. If 24  
360  2r, then r  48 cm.
360 EXERCISES
20. True. If 
n  24, then n  15.
1. 150 cm2. The solid is a cube. Each of the six faces is
21. False. It could be a rhombus. a square with area (5)(5)  25 cm2, so the surface
22. True. Triangle Inequality Conjecture. area of the cube is 6(25)  150 cm2.

IMPROVING YOUR REASONING SKILLS 2. 4070 cm2. The solid is a rectangular prism so it has
six faces: two squares with sides of length 37 cm,
1. 45 Degrees in an Acute Angle of an Isosceles Right
and four rectangles, each with dimensions 37 cm by
Triangle
9 cm. To find the surface area of the prism, add the
2. 7 Sides on a Heptagon areas of the six faces: 2(37  37)  4(37  9) 
2738  1332  4070, so the surface area of the
3. 90 Degrees in each Angle of a Rectangle
rectangular prism is 4070 cm2.
4. 5 Diagonals in a Pentagon
3. 216 cm2. The solid is a triangular prism so it has
EXTENSIONS five faces: one rectangle with base of length 6 cm
A. Research results will vary. and height 7 cm, one rectangle with base of length
2R  2r
8 cm and height 7 cm, one rectangle with base of
B. This method works because  (R  r)  length 10 cm and height 7 cm, and two triangles
2
(R  r); (R  r)  (R  r 2)  R 2  r 2.
2
each with base of length 8 cm and height 6 cm. To
find the surface area of the prism, add the areas
R of the five faces: (6)(7)  (8)(7)  (10)(7) 
212  8  6  42  56  70  48  216, so the
r surface area of the triangular prism is 216 cm2.
4. 340 cm2. The solid is a square pyramid. Use one of
the formulas from Investigation 1 on page 448 of
Rr
your book. Here, a  5 (the apothem connects the
2R  2r center of the square of the midpoint of its sides),
C.  is the average of the two circumferences.
2 l  12, and P  nb  4(10)  40. Substitute these
values in the formula SA  12P(l  a) to obtain
SA  12  4  10(12  5) 340. Thus the surface
area of the square pyramid is 340 cm2.

Discovering Geometry Solutions Manual CHAPTER 8 149


©2003 Key Curriculum Press
DG3SM586_136-181.qxd 8/2/06 6:03 PM Page 150

5. 103.7 cm2. The solid is a cone. To find its surface the areas of the top and bottom surfaces is 24.
area, use the formula from Investigation 2 on The area of the outer lateral surface is (8)(9) 
page 449 of your book. Substitute 3 for r and 8 for 72, and the area of the inner lateral surface is
l in the formula SA  rl  r 2 to obtain SA  (4)(9)  36. Therefore the surface area of the
(3)(8)  (3)2  24  9  33  103.7. solid is (24  72  36)  132  414.7 cm2.
Thus the surface area of the cone is approximately
10. 329.1 cm2. The solid is a rectangular prism with a
103.7 cm2.
hole through its center. First find the areas of the
6. 1187.5 cm2. The solid is a cylinder. To find its rectangular faces. The top and bottom faces each
surface area, use the formula from Example B on have area lw, so the sum of the areas of these two
pages 446 and 447 of your book. Substitute 7 for faces is 2(8)(4)  64 cm2. The side faces each have
r and 20 for h in the formula SA  2(r 2)  area hw, so the sum of the areas of these two faces
(2r)h  2(  72)  (2    7)  20  98  is 2(10)(4)  80 cm2. Next find the areas of the
280  378  1187.5. Thus the surface area of front and back faces by subtracting the area of a
the cylinder is approximately 1187.5 cm2. circle with radius 2 cm from that of a rectangle
with area lh. The sum of the areas of these two
7. 1604.4 cm2. The solid is a hexagonal prism so it has
faces is 2(lh  r 2)  2(8  10  (2)2) 
eight faces: two hexagonal faces (the bases) and six
160  8. Now find the surface area of the hole.
congruent rectangular lateral faces with sides of
Notice that the hole is a cylinder with radius 2 cm
length 14 cm and height 7 cm. To find the sum of
and height 4 cm. The lateral surface area of this
the areas of the two hexagonal bases, substitute 12.1
cylinder is 2rh  2(2)(4)  16. Add all the
for a, 14 for s, and 6 for n in the formula A  12asn,
areas that you have found to obtain the total surface
and multiply by 2 to obtain (2)12(12.1)(14)(6) 
area of the solid: 64  80  (160  8)  16 
1016.4. Thus the total area of the hexagons is
304  8  329.1. The surface area of the solid is
1016.4 cm2. Now find the sum of the areas of the
approximately 329.1 cm2.
six rectangular lateral faces: 6(14)(7)  588, so the
total area of the lateral faces is 588 cm2. (Another 11. Area of square  4(area of trapezoid) 
way to think of this is to imagine unwrapping the 4(area of triangle)
six rectangles into one rectangle. The lateral area of
12. $1,570. Find the cost of the paint and then the cost
this “unwrapped” rectangle is the height times the
of the shingles. Paint: Use the measurements given
perimeter, which gives the same result.) Therefore
the surface area of the hexagonal prism is 1016.4  in the end view to find the area of each end wall.
588  1604.4 cm2. The area of each end wall is the sum of the areas of
a rectangle, a trapezoid, and a triangle: (30)(24) 
8. 1040 cm2. The solid is a pyramid whose base is 1 1
(12)(12  30)  (2.5)(12)  720  252  15 
a regular pentagon so it has six faces: the pentag- 2 2
2
987 ft . There are two end walls, so the sum of the
onal base and the five triangular lateral faces. To
areas is 2(987)  1974 ft2. Each of the side walls is
find the area of the base, substitute 11 for a, 16
for s, and 5 for n in the formula for the area of a a rectangle with b  40 ft and h  24 ft, so each
regular polygon: A  12asn  12(11)(16)(5)  440. side wall has an area of (40)(24)  960 ft2, and
Therefore the area of the base is 440 cm2. Find the sum of the areas of these two walls is 2(960) 
the area of each triangular lateral face: 12sl  1920 ft2. Therefore the total surface area of all the
1
(16)(15)  120, so the area of each lateral face is
2 walls is 1974  1920  3894 ft2. Find the number
120 cm2, and the total area of the five lateral faces of gallons of paint needed to paint these walls:
3894 ft2  
is 5(120)  600 cm2. Thus the surface area of the 1 gal
250 ft2  15.6 gal. Because the paint

pyramid is 440  600  1040 cm2. must be purchased in gallons, round up to 16 gal.
9. 414.7 cm2. Draw and label all the faces of this solid. The cost of 16 gal of paint at $25 per gallon is
4
16($25)  $400.
C  8␲ C  4␲
8 9 9 Shingles: Find the total surface area of the roof,
which is made up of two rectangles with dimen-
Top and Outer surface Inner surface sions 40 ft by 15 ft and two rectangles with dimen-
bottom
sions 40 ft by 6.5 ft. 2(40)(15)  2(40)(6.5) 
To find the areas of the bottom surfaces, find the 1200  520  1720, so the total surface area of
area of an annulus with outer radius 4 and inner the roof is 1720 ft2. Find the number of bundles of
radius 2: (4)2  (2)2  16  4  12. The shingles needed for the roof: 1720 ft2  1 bundle
100 ft2 


area of the top surface is the same, so the sum of 17.2 bundles. Because the shingles must be
purchased in complete bundles, round up to

150 CHAPTER 8 Discovering Geometry Solutions Manual


©2003 Key Curriculum Press
DG3SM586_136-181.qxd 8/2/06 6:03 PM Page 151

18 bundles. The cost of 18 bundles of shingles at and k  g  80° (Isosceles Triangle Conjecture).
$65 per bundle is 18($65)  $1,170. Next, m  g  k  180° (Triangle Sum Conjec-
ture), so m  180°  2(80°)  20°. Finally, n  k
The total cost for painting the walls and putting
(AIA Conjecture), so n  80°.
new shingles on the roof is $400  $1,170  $1,570.
18. 398 square units. Find the surface area of each of
13. Sample answer:
the first four solids and look for a pattern.

Solid number (n) 1 2 3 4


Surface area 6 14 22 30

The surface area of the first solid in this pattern


is 6, and each number after the first is found by
adding 8 to the previous one, so the surface area of
the nth solid will be 6  (n  1)(8)  6  8n 
14. Sample tiling: 2  8n  2. Therefore, the surface area of the 50th
solid will be 8(50)  2  400  2  398 units2.
Another way to find the expression 6  (n  1)(8)
is to observe that the first solid, which is a single
cube, has a surface area of 6, and that each solid
after that adds 2 more cubes, each of which
increases the surface area by 4. Each additional cube
adds 5 units to the surface area, but subtracts 1
33.42/32.4.3.4/44
because one face of a cube in the previous solid is
covered. Therefore, the number of cubes added in
15. a  75°, b  75°, c  30°, d  60°, e  150°, and each solid after the first is 2(4)  8.
f  30°. First, a  12(150°)  75° (Inscribed Angle
IMPROVING YOUR VISUAL THINKING SKILLS
Conjecture). Then, b  a  75° (Isosceles Triangle
Conjecture). Next, c  180°  2(75°)  30° Move g and j to the second row, and move a to make a
(Triangle Sum Conjecture), and d  2(30°)  60° new bottom row:
(Inscribed Angle Conjecture). Also, e  2b  150° g b c j
(Inscribed Angle Conjecture). Finally, f  12((150°  d e f
d)  e)  12(210°  150°)  12(60°)  30° because
h i
the measure of an angle formed by two tangents to
a
a circle is one-half the difference between the larger
arc measure and the smaller arc measure. (This is
EXTENSION
similar to the conjecture about the measure of any
angle formed by two secants through a circle. See Prism: 2lw  2lh  2wh or 2(lw  lh  wh). Square-
Lesson 6.6, Exercise 9.) based pyramid: b 2  2bl or b(b  2l). Cylinder: 2r 2 
2rh or 2r(r  h). Cone: r 2  rl or r(r  l).
16. About 23 days. Each sector is about 1.767 km2.
The number of days that the cattle will graze on
365 EXPLORATION • ALTERNATIVE AREA FORMULAS
each sector is 16  23.
 The area of each sector is
1 1 9
r  (3)2    1.767 km2.
2
IMPROVING YOUR VISUAL THINKING SKILLS
16 16 16
17. a  50°, b  50°, c  80°, d  100°, e  80°,
f  100°, g  80°, h  80°, k  80°, m  20°,
and n  80°. First, a  50° (AIA Conjecture),
and b  a, so b  50°. a  b  c  180°, so
c  180°  2(50°)  80°. The angle with measure d
forms a linear pair with the corresponding angle of
the angle with measure c, so d  180°  c  100°.
Likewise, the angle with measure e forms a linear z x
pair with the corresponding angle of the angle of y
measure d, so e  180°  d  80°. By the same y x
x
reasoning, f  180°  e  100°, and g  180°  z

f  80°. Also, h  g (AIA Conjecture), so h  80°,

Discovering Geometry Solutions Manual CHAPTER 8 151


©2003 Key Curriculum Press
DG3SM586_136-181.qxd 8/2/06 6:03 PM Page 152

CHAPTER 8 REVIEW and the height is r, so its area is r 2. Thus, the area
of a circle is given by the formula A  r 2.
EXERCISES
1. Parallelogram (B) 2. Triangle (A)
r
3. Trapezoid (C) 4. Kite (E)
␲r
5. Regular polygon (F) 6. Circle (D)
7. Sector (J) 8. Annulus (I) 17. 800 cm2. Use the midsegment formula for the area
9. Cylinder (G) 10. Cone (H) of a trapezoid: A  (midsegment)(height) 
(40)(20)  800 cm2.
11. 12.
Apothem 18. 5990.4 cm2. The figure is a regular octagon, so use
the formula for the area of a regular polygon.
A  12asn  12(36)(41.6)(8)  5990.4 cm2.
19. 60  188.5 cm2. The shaded region is an annulus.
Aannulus  R2  r 2  (8)2  (2)2  64 
4  60  188.5 cm2.
13.
20. 32 cm. Use the formula A  12bh. Here,
576  12  36  h, so 576  18h, and h  32 cm.
21. 32 cm. The figure is a kite, so use the formula
A  12d1d2. Here, 576  12  d1  36, so 576  18d1,
and d1  32 cm.

14. Sample answer: Construct an altitude from the 22. 15 cm. The figure is a trapezoid, so use the formula
vertex of an obtuse angle to the base. Cut off the A = 12h(b1  b2).
right triangle and move it to the opposite side, 1
126  2(9)(13  b)
forming a rectangle. Because the parallelogram’s
area hasn’t changed, its area equals the area of the 252  9(13  b)
rectangle. Because the area of the rectangle is given
28  13  b
by the formula A  bh, the area of the parallelo-
gram is also given by A  bh. b  15 cm
b b 23. 81 cm2. Find the radius of the circle and then use
h h
the radius to find the area. C  2r, so 18  2r,
and r  9 cm. Then, A  r 2  (9)2  81 cm2.
24. 48 cm. Find the radius of the circle and then use
15. Sample answer: Make a copy of the trapezoid and the radius to find the circumference. A  r 2, so
put the two copies together to form a parallelogram 576  r 2, r 2  576, and r  24 cm. Then,
with base (b1  b2) and height h. Thus, the area of C  2r  2(24)  48 cm.
one trapezoid is given by the formula
A  12(b1 b2)h, or A  12h(b1  b2). 25. 40°. The shaded region is a sector of a circle with
radius 12 cm. The area of the sector is 16 cm2 and
b1 b2
the area of the complete circle is 144 cm2, so
Asector 16 cm2 1
 
Acircle    
144 cm2  9
h h

b2 b1
Therefore the sector is 19 of the circle, so
16. Sample answer: Cut a circular region into 16 wedges mFAN  19(360°)  40°.
and arrange them into a shape that resembles a 26. 153.9 cm2. To find the area of the shaded region,
rectangle. The base length of this “rectangle” is r subtract the areas of the two small semicircles from
the area of the large semicircle.

152 CHAPTER 8 Discovering Geometry Solutions Manual


©2003 Key Curriculum Press
DG3SM586_136-181.qxd 8/2/06 6:03 PM Page 153

Large semicircle: r  14 cm; A  12r 2  32. A  112 square units. The figure is a parallelogram
1
(14)2  98 cm2 with base length 14 units and height 8 units, so its
2
area is 14  8  112 square units.
Two small semicircles: r  7 cm; A  212r 2 
y
(7)2  49 cm2
Therefore the area of the shaded region is
98  49  49  153.9 cm2. D (6, 8) C (20, 8)

27. 72 cm2. By rearranging this figure into a rectangle,


you can show that its area is one-half the area of A (0, 0) B (14, 0)
x
the square. (See Lesson 8.6, Exercise 15a. The figure
here is the same, except that it has been rotated 90°,
which does not affect its area.) Because the arcs in  to divide quadrilat-
33. A  81 square units. Draw RO
this figure are arcs of a circle with radius 6 cm, the
eral FOUR into RFO and RUO. Find the areas
square has sides of length 12 cm. Therefore the area  as the base for both
of these two triangles, using RF
of the square is 144 cm2, and the area of the shaded
triangles. Notice that 15  (3)  18.
region is 12(144)  72 cm2.
y
28. 30.9 cm2. To find the area of the shaded region,
R (4, 15)
subtract the combined area of the four quarter-
circles from the area of the square. As in Exer-
cise 27, the area of the square is 144 cm2. The area U (9, 5)
of the four quarter-circles is 414(6)2  36 cm2. F (0, 0)
x
Therefore, the area of the shaded region is
(144  36)  30.9 cm2. O (4, –3)

29. 300 cm2. The solid is a triangular prism so it has


Area of RFO  12(18)(4)  36 square units
five faces: one rectangle with dimensions 8 cm by
12 cm, one rectangle with dimensions 8 cm by Area of RUO  12(18)(5)  45 square units
5 cm, one rectangle (the slanted face) with dimen-
Area of quadrilateral FOUR  36  45 
sions 8 cm by 13 cm, and two right triangles each
81 square units
with base of length 12 cm and height 5 cm. To
find the surface area of the prism, add the areas 34. 6 cm. Use the formula A  12h(b1  b2). Notice that
of the five faces: (8  12)  (8  5)  (8  13)  although the values of b1 and b2 are not given indi-
212  12  5  96  40  104  60  300, so the vidually, the value of b1  b2 is provided.
surface area of the triangular prism is 300 cm2. 1
A  2h(b1  b2)
30. 940 cm2. To find the surface area of the prism, add 1
the areas of its six faces. 66  2h(22)

Area of two trapezoids: 212  12(35  10)  66  11h


12(45)  540 cm2 h6
Area of four rectangles: 15  5  10  5  20  5  The height of the trapezoid is 6 cm.
35  5  75  50  100  175  400 cm2
35. 172.5 cm2. Use the formula A  12asn. The measure-
Surface area of prism  540  400  940 cm2 ments given in this exercise are approximate, so
31. 1356 cm2. The solid is a pyramid with a rectangular A  12(6.9)(10)(5)  172.5, and the area of the
base. To find the surface area of the pyramid, add pentagon is approximately 172.5 cm2.
the areas of its five faces. 36. Sample answers:
Area of rectangular base  30  14  420 cm2
Area of two triangles with base length 30 cm and
height 20 cm: 212  30  20  600 cm2
Area of two triangles with base length 14 cm and
height 24 cm: 212  14  24  336 cm2
Surface area of pyramid  420  600  336  37. 1250 m2. This problem was investigated in the
1356 cm2 Maximizing Area project at the end of Lesson 8.2
(see page 421 in your book). The only difference is

Discovering Geometry Solutions Manual CHAPTER 8 153


©2003 Key Curriculum Press
DG3SM586_136-181.qxd 8/2/06 6:03 PM Page 154

that here there are 100 m of fencing rather than Giant slice: 4($2.85)  $11.40
10 m. If you completed that project, apply your
Large slice: 6($2.25)  $13.50
result to this problem. If not, work through the
project with your graphing calculator, but use 100 Regular slice: 8($1.75)  $14.00
rather than 10. The rectangle with maximum area
Because the cost of a whole pizza is smallest for the
has the following shape.
giant slice, the giant slice is the best deal.
2x
An equivalent approach is to find the amount of
x Pen x pizza per dollar that you get with each kind of slice.
14
Castle wall Giant slice: 
$2.85  0.088 pizza/dollar

38. The circle gives the maximum area. For the square, 16
Large slice:   0.074 pizza/dollar
100  4s, s  25 ft, A  252  625 ft2. For the $2.25
circle, 100  2r and r  15.9, so A  (15.9)2  18
795 ft2. Regular slice: 
$1.75  0.071 pizza/dollar

39. A round peg in a square hole. Draw two circles with Because the amount of pizza per dollar is greatest
the same radius, r, one with the circle inscribed in a for the giant slice, the giant slice is the best deal.
square and the other with a square inscribed in the 41. About 14 oz. Compare the areas of the two pizzas.
circle. For the 12-inch diameter pizza, r  6 in., so A 
36 in.2. For the 16-inch diameter pizza, r  8 in.,
r r
so A  64 in.2. The amount of dough that is
r r
r needed for the larger pizza can be found by solving
a proportion. Let x represent the number of ounces
For each of these figures, find the fraction of the of dough needed for the 16-inch diameter pizza.
8 oz 36 in.2 9
hole that is covered by a cross section of the peg.   2  
x 64 in. 16
Round peg in square hole: Area of circle  r 2. The 9x  8(16)
length of a side of this square is 2r, so area of
square  (2r)2  4r 2. Fraction of square hole filled 9x  128
128
by round peg: x  9  14.2
area of circle r 2 
    About 14 oz of dough will be needed to make the
area of square 4r 2 = 4
16-inch diameter pizza.
Square peg in round hole: Area of circle  r 2. The
diagonals of the square are diameters of the circle, 42. Biggest slice: one-eighth of a 12-inch diameter pie.
so they each have length 2r. Look at the four trian- Most crust: one-fourth of a 6-inch pie and one-
gles formed by the diagonals. Each has area eighth of a 12-inch pie both have the same length
of crust, and more than one-sixth of an 8-inch pie.
1 1
bh  r 2 To find the biggest slice, compare the areas of the
2 2
so the area of the whole square is 412r 2  2r 2. slices, which are sectors of a circle.
Fraction of round hole filled by square peg:
One-fourth of 6-inch diameter pizza: r  3 in.;
area of square
  2  
2r 2 2 A  14(9)  94in.2
area of hole r 
Now compare 4 and 2 . 4  0.785 and 2  0.637, One-sixth of 8-inch diameter pizza: r  4 in.;
so 4 2. Therefore the round peg fills a larger A  16(16)  83in.2
fraction of the square hole (78.5%) than the square One-eighth of 12-inch diameter pizza: r  6 in.;
peg fills in a round hole (63.7%). This means that A  18(36)  92in.2
the round peg in the square hole is the better
(tighter) fit. Because 92 83 94, one-eighth of a 12-inch diam-
eter pizza is the largest slice. To find out which slice
40. Giant. From the drawing, the giant slice is 14 of has the most crust around its edge, compare the
the pizza, the large slice is 16 of the pizza, and the outer circumferences of the pizzas (sectors) by
regular slice is 18 of the pizza. An easy way to figure finding the appropriate fraction of the circumfer-
out the best deal is to find the cost for the whole ence of each pizza.
pizza based on the cost per slice.

154 CHAPTER 8 Discovering Geometry Solutions Manual


©2003 Key Curriculum Press
DG3SM586_136-181.qxd 8/2/06 6:03 PM Page 155

One-fourth of 6-inch diameter pizza: C  14d  Find the cost for the silver to electroplate these
1 3
()(6)   in. wedges.
4 2
$1
One-sixth of 8-inch diameter pizza: C  16d  600,000 cm2   
200 cm2  $3,000
1 4
()(8)   in.
6 3 45. $4,160. First find the surface area of the top and
One-eighth of 12-inch diameter pizza: C  18d  lateral faces of the cylinder, which has radius 3.5 m
1 3
()(12)   in. and height 10 m. Because the bottom faces will not
8 2
be painted, there is only one circular face, so use
Because 32 43, the slice of the 6-inch diameter pizza
r 2 rather than 2r 2 in computing the surface area.
and the slice of the 12-inch diameter pizza have the
same length of crust around the edge, and this is SA  r 2  2rh  (3.5)2  2(3.5)(10) 
more than the amount of crust around the edge of 12.25  70  82.25 m2
the 8-inch diameter pizza.
There are nine containers to be painted, so the
43. a. Perimeter at widest part  96 ft. Perimeter at total surface area to be painted is 9(82.25) 
bottom  40 ft. The widest part of the balloon is 740.25 m2. Next find the number of gallons of
at the bottom of the top row of trapezoids. Here, sealant that is needed.
the perimeter is 8(12)  96 ft. The perimeter at 1 gal
740.25 m2   
18 m2  129.2 gal
the bottom of the balloon is 8(5)  40 ft.
Because the sealant is sold in gallon containers,
b. 3290 ft2. This figure may 3.2
round up to 130 gal. The cost for 130 gal of sealant
help you see the panels that
is 130($32)  $4,160.
make up the balloon more
clearly. 46. $2,002. First find the lateral surface area of one
Find the surface area of the cone, which has radius 24 in. and slant height 51 in.
balloon by adding the areas Lateral surface area of one cone  rl 
of all the panels. (24)(51)  1,224 in.2
Octagon: A  12asn  12
1 Lateral surface area of 100 cones  100(1,224) 
(3.8)(3.2)(8)  48.64 ft2
2 122,400 in.2
8 small trapezoids: 5
Now find the number of pints of oxidizer that is
A  812h(b1  b2) 
needed to cover 100 cones.
812  12(3.2  12)  729.6 ft2 1 pt
122,400 in.2   
5,000 in.2  76.9 pt
8 squares: A  8(s 2)  8(122)  1152 ft2
8 large trapezoids: A  812h(b1  b2)  The oxidizer is sold in pints, so round up to 77 pt.
The cost for 77 pt of oxidizer is 77($26)  $2,002.
812  20(12 + 5) = 1360 ft2
Total surface area of balloon  48.64  729.6  47. It’s a bad deal. 2r1  44 cm. 2r2  22 cm, which
1152  1360  3290.24. To the nearest square implies 4r2  44 cm. Therefore r1  2r2. The area
foot, the total surface area of the balloon is of the large bundle is 4(r2)2 cm2. Thus he is
3290 ft2. getting half as much for the same price.

44. $3,000. First find the surface area of one wedge, TAKE ANOTHER LOOK
which is a triangular prism so it has 5 faces: two 1. A possible approach for the triangle: To make a base
triangular bases that are right triangles with base of constant length, construct line j between points A
length 6 cm and height 8 cm, one rectangle with and B. Mark vector AB. Construct point C on line j.
dimensions 6 cm by 0.5 cm, one rectangle with Translate point C to C by the marked vector. Then
dimensions 8 cm by 0.5 cm, and one rectangle CC will be constant. To create a constant altitude,
with dimensions 10 cm by 0.5 cm. construct point D not on line j. Construct line k
through point D parallel to line j. Construct point E
Area of two triangles  212  6  8  48 cm2
on line k. Construct segments to form CCE of
Sum of areas of three rectangles  6(0.5)  constant base and height. Then hide points A, B,
8(0.5)  10(0.5)  12 cm2 and D and lines j and k.
Surface area of wedge  48  12  60 cm2 2. True. Given ABC with inscribed circle O touching
 at point D, CA
BC  at point E, and AB
 at point F,
The total surface area of 10,000 metal wedges is
let a  AF  AE, b  BF  BD, c  CD  CE,
10,000(60)  600,000 cm2.

Discovering Geometry Solutions Manual CHAPTER 8 155


©2003 Key Curriculum Press
DG3SM586_136-181.qxd 8/2/06 6:03 PM Page 156

and r  OD  OE  OF. The triangle is composed length of the midsegment is half the length of
of three pairs of congruent right triangles. Its area is the third side, or m  12b. By substitution A  mh.
therefore 12r(a  b  b  c  c  a)  12Pr.
C m
c
c
D h
E
b b
a

A F B
a b
CHAPTER 9
3. Yes. Area of dart  12  d1  d2  12  x  2y  xy;
area of two triangles  2  12  x  y  xy.
LESSON 9.1
x
EXERCISES
y y
1. 12 cm. Use the Pythagorean Theorem to find a, the
4. See Lesson 8.5, Exercise 16. As n →
, a → r. length of one of the legs of the right triangle.
Therefore, A  12aP → A  12r  2r  r 2. a2  b 2  c 2
r r r r a2  (5)2  (13)2

a a a a
a2  25  169
5. Area of two bases  2  12  a  P  aP; area of n a2  144
rectangular sides  n  side length of polygon  h 
P  h; total surface area  Ph  aP  P(h  a). a  
144

6. As the number of sides of the base gets larger, the a  12 cm


prism approaches a cylinder and a becomes r. Or 2.  19.2 cm. Use the Pythagorean Theorem to find c,
use the method used in activity 5, except with C the length of the hypotenuse of the right triangle.
and r:
a2  b 2  c 2
Area of two bases  2  r 2
(12)2  (15)2  c 2
Area of side  C  h
144  225  c 2
Total surface area  C  h  2r 2
369  c 2
 C  h  2r  r
c  
369
 Ch  C  r
c  19.2 cm
 C (h  r)
3.  5.3 cm. Use the Pythagorean Theorem to find a,
7. The Trapezoid Area Conjecture tells you that the length of one of the legs of the right triangle.
A  12(b1  b2)h. But the Trapezoid Midsegment
Conjecture tells you that m  12(b1  b2). Thus, a2  b 2  c 2
if A  12(b1  b2)h and m  12(b1  b2), then a2  (6)2  (8)2
A  mh.
a2  36  64
b2
a2  28
m
a  
28
h

b1
a  5.3 cm
4. 10 cm. The diagonal of the rectangle is the
Similarly, the Triangle Area Conjecture says that the hypotenuse of two congruent right triangles.
formula for the area of a triangle is A  12bh. But (6)2  (8)2  d 2, so 36  64  d 2. Then
the Triangle Midsegment Conjecture tells us that the d 2  100, so d  100  10 cm.

156 CHAPTER 9 Discovering Geometry Solutions Manual


©2003 Key Curriculum Press
DG3SM586_136-181.qxd 8/2/06 6:03 PM Page 157

5. 26 cm. The solid is a prism whose bases are right legs of the right triangle are 5 cm and 12 cm. Then
triangles, so the Pythagorean Theorem can be (5)2  (12)2  r 2, so 25  144  169  r 2, or
applied to either of the congruent triangular bases r  169
 13 cm.
to find s: (10)2  (24)2  s 2, so 100  576  s 2.
12. 127 ft. Let s represent the length of the side of the
Then s 2  676, so s   676  26 cm.
square and d represent the distance from home
6.  8.5 cm. This is an isosceles triangle, so each leg plate to second base.
has length 6 cm. (6)2  (6)2  c 2, so 36  36  c 2, Second base
or 72  c 2. Then c  72
 8.5 cm.
7. 24 cm. The solid is a rectangular prism. Apply the s s
Pythagorean Theorem to one of the right triangles
formed by the diagonal of the front face of the
prism. d

(7)2  b2  (25)2
49  b2  625 s s

b2  576
Home plate
b  576
 24 cm
8. 3.6 cm. The solid is a pyramid. Apply the Notice that d is the length of a diagonal of the
Pythagorean Theorem to one of the triangular square and is also the hypotenuse of the two
lateral faces to find x. Notice that the edge of the isosceles right triangles into which the diagonal
pyramid with length 3.9 cm is the hypotenuse of divides the square.
the right triangle. s2  s2  d2
x 2  (1.5)2  (3.9)2 (90)2  (90)2  d 2
x 2  2.25  15.21 8,100  8,100  d 2
x 2  12.96 16,200  d 2
x  12.96
 3.6 cm d  16,200
 127
9. 40 cm. The solid is a cone. Apply the Pythagorean The distance from home plate to second base is
Theorem to one of the right triangles; the lengths about 127 ft.
of the legs are the radius and height of the cone,
and the length of the hypotenuse is the slant height. 13. 512 m2. Let s represent the length of the side of the
square. Then s 2  s 2  (32)2, so 2s 2  1024, and
(9)2  x 2  (41)2 s 2  512. The area of the square is s 2, so the area is
81  x 2  1681 512 m2.

x 2  1600 14.  11.3 cm. Let s represent the length of the side of
the square and d represent the length of the diag-
x  1600
 40 cm onal. s 2  s 2  d 2, so 2s 2  d 2. The area of the
10.  3.5 cm. Apply the Pythagorean Theorem to either square is s 2, which is 64 cm2, so substitute 64 for s 2
of the isosceles triangles formed by the diagonal of in the equation 2s 2  d 2: 2(64)  d 2, so d 2  128,
the square. and d  128  11.3. The length of the diagonal
of the square is about 11.3 cm.
s 2  s 2  (5)2
15. 3, 4, 5. Because these numbers are small integers,
2s 2  25 you may be able to find them by trial and error:
s 2  12.5 (3)2  (4)2  9  16  25  (5)2.

s  12.5 To solve this problem algebraically, let n, n  1,


 3.5 cm
and n  2 represent the three consecutive integers.
11. 13 cm. The radius of the circle is the hypotenuse The three integers represent the lengths of the
of the right triangle. The dashed segment, which is three sides of a right triangle, so the Pythagorean
perpendicular to the x-axis, has endpoints with Theorem applies. n  2 is the largest of the three
coordinates (5, 0) and (5, 12), so the lengths of the integers, so it must represent the length of the

Discovering Geometry Solutions Manual CHAPTER 9 157


©2003 Key Curriculum Press
DG3SM586_136-181.qxd 8/2/06 6:03 PM Page 158

hypotenuse. Substitute n for a, n  1 for b, and 19. x  21 cm. x is the sum of the lengths of the sides
n  2 for c; then use algebra to solve for n. of the two squares. The side of the large square 
 225  15 cm. The side of the small square 
a2  b 2  c2 The Pythagorean
Theorem.  36  6 cm. So x  15  6  21 cm.
20. 3632.4.3.4. There are two types of vertices. One
n2  (n  1)2  (n  2)2 Substitute the
known values. type is surrounded by six equilateral triangles. The
other type is surrounded by two equilateral trian-
n2  n2  2n  1  n2  4n  4 gles, then a square, then another equilateral triangle,
Expand the and then another square.
binomials.
21. Mark the unnamed angles as shown in the figure
n2  2n  3  0 Rewrite equation below. By the Linear Pair Conjecture, p  120° 
with 0 on one side.
180°. Therefore, p  60°. By the AIA Conjecture,
(n  3)(n  1)  0 Factor the equation. m  q. By the Triangle Sum Conjecture, q  p 
n  180°. Substitute m  q and p  60° to get
n  3  0 or n  1  0 m  60°  n  180°. Therefore m  n  120°.
Zero Product
Property.

n  3 or n  1 m
Solve the linear 120°
equations. p
n
Reject 1 as a possible solution because n repre- q

sents the length of a side of a triangle, which


cannot be negative. Therefore, n  3, n  1  4,
and n  2  5, so the integers are 3, 4, 5. 22. a  122°, b  74°, c  106°, d  16°, e  90°, f 
74°, g  74°, h  74°, n  74°, r  32°, s  74°,
16. 28 m. A sketch of this situation will be the same
t  74°, u  32°, and v  74. First, by the Linear
as the figure for Exercise 4 in your book, except
Pair Conjecture, a  180°  58°  122° and b 
that now d  10 m is given and the length of
180°  106°  74°. By the Vertical Angles Conjec-
the rectangle is not given. From Exercise 4 or by
ture, c  106°. Now look at the large triangle that
applying the Pythagorean Theorem, you can see that
has angle measures c, d, and 58° (by the Vertical
the length of the garden is 8 m. Then P  2(8) 
Angles Conjecture). c  d  58°  180°, so d 
2(6)  16  12  28 cm.
16°. By the Tangent Conjecture, e  90°. Now look
17. The area of the large square is 4  area of triangle  at the triangle containing the angle measures d
area of small square. and f. The third angle of this triangle is a right
1 angle (Linear Pair Conjecture), so d  f  90°, and
c 2  4  2ab  (b  a)2
f  74°. Next look at the quadrilateral whose angle
c 2  2ab  b 2  2ab  a2 measures include c, e, and g. The fourth angle of
this quadrilateral is a right angle (Tangent Conjec-
c 2  a2  b 2
ture), so e  g  90°  c  360° (Quadrilateral
Sum Conjecture); thus g  74°. Because h is the
b measure of the arc intercepted by the central angle
a of measure g, by the definition of the measure of an
b a arc, h  g, so h  74°. Now look at the triangle
that contains the angle between the angles with
c measures g and n, a right angle (where the radius
intersects 1), and a 58° angle (by the Vertical
Angles Conjecture). The measure of the unknown
angle, which is a central angle of the circle, is
180°  58°  90°  32°. Then, n  32°  g 
18. Sample answer: Yes, ABC  XYZ by SSS. Both 180°, or n  32°  74°  180°, so n  74°. The
triangles are right triangles, so you can use the arc with measure r is intercepted by the central
Pythagorean Theorem to find that CB  ZY  angle of measure 32°, so r  32° by the definition
3 cm. of the measure of an arc. Next look at the arcs
with measures s and t. The arc with measure t is
intercepted by the central angle with measure n, and
the arc with measure s is intercepted by the central

158 CHAPTER 9 Discovering Geometry Solutions Manual


©2003 Key Curriculum Press
DG3SM586_136-181.qxd 8/2/06 6:03 PM Page 159

angle that is the vertical angle of the angle with 7. No. (9)2  (12)2  81  144  225, while (18)2 
measure n. So, by the definition of the measure of 324, so this is not a right triangle. Another way to
an arc, s  74°, and also t  n  74°. f  e  look at this is to observe that because 9-12-15 is
(measure of the angle that intercepts the arc with a Pythagorean triple (because it is a multiple of
measure u)  180°, so the unknown angle measure 3-4-5), it is not possible for 9-12-18 to be a
is 16°; therefore u  2(16°)  32° (Inscribed Angle Pythagorean triple.
Conjecture). Finally, look at the arc with measure v.
8. No, the window frame is not rectangular. (408)2 
Because the arc with measure (v  u  t) is a
(306)2  166,464  93,636  260,100, while
semicircle, v  u  t  180°, so v  180° 
(525)2  275,625, so the diagonal does not form
32°  74°  74°.
right triangles. Therefore, the window frame is not
PROJECT rectangular.
Project should satisfy the following criteria: 9. y  25 cm. Look at the large right triangle in which
● Movement in the flip book is smooth. y is the length of the hypotenuse. The smaller
square has area 25 cm2, so each side has length
● The student can explain the dissection used.
5 cm. Therefore, the length of the longer leg of the
Extra credit large right triangle is 15  5  20 cm. The shorter
● The student adds other animation, for example, leg is a side of the larger square, so its length is
drawing a hand flipping the pages of a flip book, thus 15 cm. Multiplying 3-4-5 by 5 gives the Pythagorean
creating a flip book of a flip book! triple 15-20-25, so y  25 cm.

● An animation is created using geometry software. 10. y  24 units. Drop a perpendicular from the point
(7, y) to the x-axis to form a right triangle. The
EXTENSIONS point where the perpendicular touches the axis is
A. Research results will vary. (7, 0), so the triangle has one leg of length 7 units
and hypotenuse of length 25 units. Because 7-24-25
B. See the solutions to Take Another Look activities 1,
is a Pythagorean triple, y  24 units.
2, and 3 on page 181.
11. y  301
m, or y  17.3 m. y 2  (18)2  (25)2,
LESSON 9.2 so y  (25)2  (18)2  625  324  301. Thus
2

y  301
 17.3 m.
EXERCISES
12. 6, 8, 10. Check the list of Pythagorean triples in this
1. Yes. (8)2  (15)2  64  225  289  (17)2, so lesson to find a triple made up of three consecutive
this is a right triangle. Also, notice that 8-15-17 is even integers. Another way to find this triple is to
on the list of Pythagorean triples in the Investiga- start with the triple 3-4-5, which is made up of
tion Is the Converse True? on page 468 of your consecutive integers (which differ by 1) and then
book. multiply by 2 to get 6-8-10, three consecutive even
2. Yes. 50-120-130 is a multiple of the Pythagorean integers (which differ by 2).
triple 5-12-13, so this is a right triangle. 13. 60 cm2. Because 8-15-17 is a Pythagorean triple, the
3. No. (12)2  (35)2  144  1225  1369, while length of the other leg is 8 cm. The legs of a right
(36)2  1296. Because (12)2  (35)2  (36)2, this triangle are perpendicular, so the area of any right
is not a right triangle. triangle is half the product of the lengths of the
legs. Thus, A  12(8)(15)  60 cm2.
4. No. (12)2  (18)2  144  324  468, while
(22)2  484. Because (12)2  (18)2  (22)2, this 14. About 14.1 ft. Let h represent the
is not a right triangle. height that the ladder reaches on the
building and make a sketch.
5. No. (10)2  (20)2  100  400  500, while
(24)2  576. Because (10)2  (20)2  (24)2, this In this right triangle, (5)2  h2 
is not a right triangle. (15)2, so h2  (15)2  (5)2  225  15 ft h
25  200. Then h  200  14.1,
6. No. (1.73)2  (1.41)2  2.9929  1.9881  4.981, so the ladder will touch the wall about
while (2.23)2  4.9729, so this is not a right 14.1 ft up from the ground.
triangle. Notice that for this triangle, a 2  b 2 is
close to c 2, so a triangle drawn accurately with these
measurements may appear to be a right triangle. 5 ft

However, in order to be a right triangle, a 2  b 2


must be exactly equal to c 2.

Discovering Geometry Solutions Manual CHAPTER 9 159


©2003 Key Curriculum Press
DG3SM586_136-181.qxd 8/2/06 6:03 PM Page 160

15. About 17.9 cm2. Recall that the altitude from the 20. Sample answer: (BD)2  62  32  27; (BC)2 
vertex angle to the base of an isosceles triangle is (BD)2  92  108; then 36  108  144, so
also a median. Make a sketch. (AB)2  (BC)2  (AC)2, and ABC is a right
triangle by the Converse of the Pythagorean
Theorem.
6 cm 6 cm
h 21. Centroid. The construction marks show that this is
the point of concurrency of the medians of the
triangle.
4 cm
8 cm 22. Because mDCF  90° (Tangent Conjecture),
mDCE  90°  x. Because DCE is isosceles
Use the Pythagorean Theorem to find h: h2 
(two sides are radii), mDEC  mDCE (Isosceles
(4)2  (6)2, so h2  (6)2  (4)2  36  16  20.
Triangle Conjecture), so mDEC  90°  x.
Thus h  20 , so the height of the triangle is
Therefore, mD  180°  2(90°  x)  180° 
 20 cm. Then A  12bh  12(8)20
 420 
180°  2x  2x. Because D is a central angle,
17.9 cm2.
2x  a by the definition of the measure of an arc.
16. 102 m. Let w represent the width of the rectangle. Therefore x  12a.
Make a sketch. 3
23. .The total number of ways to randomly choose
10
3 points from the 10 points on the grid is
39 m
 
103  
w 10 9 8
3  2  1  720
36 m Use systematic counting or make an organized list
to find the number of isosceles triangles that can be
36  3(12) and 39  3(13); the sides of this formed with 3 of these points as vertices. Remember
triangle are a multiple of the Pythagorean triple to include the equilateral triangles because an equi-
5-12-13, so w  3(5)  15 m. Then, P  2(36)  lateral triangle is a special kind of isosceles triangle.
2(15)  72  30  102, so the perimeter of the In order to make a list of all the isosceles triangles,
rectangular lot is 102 m. This means that 102 m of label the points on the grid.
fencing will be needed.
A
17. a. About 1442.2 cm2. First find the length of the
J D
diagonal of the rectangular base of the box. Let l
represent this length. Apply the Pythagorean F O G
Theorem to one of the right triangles that make C H E B
up the rectangular base: l 2  (40)2  (60)2 
1600  3600  5200, so l  5200 cm. The This solution shows one of many ways to categorize
cm
cardboard rectangle has base length 5200 the triangles. It is important to do this in an orga-
and height 20 cm, so A  bh  5200  20  nized way so that you don’t miss any triangles and
1442.2 cm2. don’t list any of them more than once. In each
triangle listed below, the vertices will be written in
b. About 74.8 cm. Let d represent the diagonal of alphabetical order to avoid listing the same triangle
the cardboard rectangle. This diagonal divides the more than once. In the system used below, all
rectangle into two right triangles, each with base triangles within a subcategory (such as small
length  5200 cm and height 20 cm, so d 2  equilateral triangles) are congruent.
( 5200 )  (20)2  5200  400  5600. There-
2

fore d   5600  74.8 cm. Equilateral triangles:

18. Because DEF is a right triangle, a 2  b 2  x 2. By Large: ABC (1)


substitution, c 2  x 2 and c  x. Therefore, EFD  Medium-sized vertical: AFG, CEJ, BDH (3)
BCA by SSS and C  F by CPCTC. Hence,
C is a right angle and BCA is a right triangle. Medium-sized on a “slant”: DEF, GHJ (2)

19. Sample answer: The numbers given satisfy the Small: ADJ, FJO, DOJ, DGO, CFH,
Pythagorean Theorem, so the triangle is a right FHO, EHO, EGO, BEG (9)
triangle; but the right angle should be inscribed in Total number of equilateral triangles: 15
an arc of 180°. Thus the triangle is not a right
triangle.

160 CHAPTER 9 Discovering Geometry Solutions Manual


©2003 Key Curriculum Press
DG3SM586_136-181.qxd 8/2/06 6:03 PM Page 161

Now count the isosceles triangles that are not USING YOUR ALGEBRA SKILLS 8
equilateral.
EXERCISES
“Tall and skinny”: AEH, BFJ, CDG (3)
1. 6 . 3  2  3 • 2  6
.
Triangles formed by the longer diagonal of a
2. 5. (5 )2  5  5  25  5, so (5
)2  5 by
rhombus: AJO, ADO, BGO, BEO, CFO,
the definition of square root.
CHO, FHJ, HJO, DFJ, DFO, DEO,
DEG, EGH, EFH, EFO, GHO, GJO, 3. 182 . (36 )(23 )  3  2  6
• 3  618

DGJ (18) 69 2  69  2  6  3  2  182 .

Total number of isosceles triangles that are not 4. 147. (73 )2  73  73  7  7  3  3 
equilateral: 21 49  3  147.
Total number of isosceles triangles: 15  21  36  22  4  2  8.
5. 8. (22 )2  22
Total number of ways to use 3 points from  9
6. 32 . 18 • 2  9  2
 32 .
grid: 120
. The largest perfect square factor of 40 is 4.
7. 210
 10
Probability that 3 points selected at random form 40  4 • 10  4
36 3   210 .
120  10
an isosceles triangle:  

24. 790 square units. The first solid is made up of a


 25
8. 53 . 75 • 3  25
 3  53 .
single cube and has a surface area of 6 because a . 85 has no perfect square factors other than 1,
9. 85
cube has 6 faces. Each solid after the first is formed is already in its simplest form.
so 85
by adding 4 more cubes, each one of the “arms” 10. 46 . The largest perfect square factor of 96 is 16.
of the solid. Whenever a new cube is added, the
96  16
  6
• 6  16  46 .
surface area is increased by 5 (all but the face that is
covered by a face of a cube in the previous solid), 11. 24. 242  576, so 576
 24.
but decreased by 1 (because one face of the
12. 125 . The largest perfect square factor of 720 is
previous solid is covered). Therefore, each new cube
adds 4 to the surface area, and the 4 new cubes  144
144. 720  5
• 5  144  125 .
added at each stage add 4(4)  16 to the surface 13. 192 . The largest perfect square factor of 722 is
 2
area. Thus a formula for the surface area of the
 361
361. 722 • 2  361  192 .
nth solid is 6  16(n  1)  6  16n  16 
16n  10. Therefore the surface area of the 50th 14. 28. 282  784, so 784
 28.
solid is 16(50)  10  800  10  790.
. The largest perfect square factor of 828 is 36.
15. 623
25. 828  36
 • 23  36  23  623 .
16. 682 . The largest perfect square factor of 2952 is
 32
36. 2952 • 82  36  82  682 .
17. 882 . The largest perfect square factor of 5428 is
 64
64. 5428 • 82  64  82  882 .

IMPROVING YOUR ALGEBRA SKILLS 18. 1082 . The largest perfect square factor of 8200 is
You might think of number combinations, Pascal’s  100
100. 8200 2  82
• 8  100  1082 .
triangle, or symmetry. Or you might see this pattern: 19. 1282 . The largest perfect square factor of 11,808
After the first term, each coefficient can be determined  144
is 144. 11,808 2  82
• 8  144 
by multiplying the previous coefficient by the exponent 12 82 .
on x in the previous term and then dividing by the
number of the term (counting from zero). For example, 20. 1482 . The largest perfect square factor of 16,072
the term that includes x 6 is the third term, so its  196
is 196. 16,072 2  82
• 8  196 
7(36)
coefficient is 3  84. The next three terms in 14 82 .
the sequence are 36x 2y 7, 9xy 8, and y 9.  1682
21. 20,992 . From Exercises 16–20, the
EXTENSIONS square roots in this pattern can be written in
simplest form as 6 , 1082
82 , 882 , 1282 , and
A. If the sum is less, the angle is obtuse. If the sum is
14 82 , so the next term in the pattern will be
greater, the angle is acute.
16 82  16 22 • 8 
 20,992 .
B. Results will vary.

Discovering Geometry Solutions Manual CHAPTER 9 161


©2003 Key Curriculum Press
DG3SM586_136-181.qxd 8/2/06 6:03 PM Page 162

IMPROVING YOUR VISUAL THINKING SKILLS length of a side of the square. By the Isosceles Right
A good problem-solving technique is to eliminate as Triangle Conjecture, s  18 cm. The perimeter of
many choices as possible. Look at each net and see the square is 4s  4(18)  72 cm.
whether adjacent faces are related as they are in the 7. 123 cm. Draw DB  to form two isosceles right
cube. For example, 1A can be eliminated because the triangles, DAB and CBD, on the base of the
base of the green face is touching the red face, unlike its cube, and right triangle HDB in the interior of the
position in the original cube. cube. In DAB, AB (the length of a leg)  12 cm,
1. D so DB  12 2 cm. Now look at HDB, in which
HDB is a right angle and d is the length of the
2. C  is an edge of the cube, HD 
diagonal. Because HD
12 cm. Apply the Pythagorean Theorem to HDB
LESSON 9.3 to find d:
2
EXERCISES (12)2  12
2   d2
1. 722 cm. The length of each of the legs of this 144  12  12  2  2  d 2
isosceles right triangle is 72 cm, so by the Isosceles
144  144(2)  d 2
Right Triangle Conjecture, the length of the
hypotenuse is 72 2 cm, so a  722 cm. 144  288  d 2
2. 13 cm. The length of the hypotenuse of this 432  d 2
isosceles right triangle is 132 cm, so by the
d  432

Isosceles Right Triangle Conjecture, the length of
each of the legs is 13 cm, so b  13 cm. The length  
144  3
of the legs of an isosceles right triangle can be
 12
3 cm
found by dividing the length of the hypotenuse
by  2. 8. g  50 cm, h  100 cm. First look at the right
triangle with sides of length g, 120 cm, and 130 cm.
3. a  10 cm, b  53 cm. This is a 30°-60°-90°
Because 120  10(12) and 130  10(13), the side
triangle in which the given length, 5 cm, is the
lengths are a multiple of the 5-12-13 Pythagorean
length of the shorter leg (the side opposite the
triple, so g  10(5)  50 cm. Now look at the small
30° angle). By the 30°-60°-90° Triangle Conjecture,
isosceles triangle in the lower left part of the figure.
the length of the hypotenuse is twice the length of
The angle between the congruent sides of this
the shorter leg, so a  2(5)  10 cm, and the
triangle is a right angle (Vertical Angles Conjecture),
length of the longer leg is the length of the shorter
so this is an isosceles right triangle. Because the
leg times  3 , so b  5 3 cm.
quadrilateral with sides of length 120 cm and g 
4. c  103 cm, d  10 cm. This is a 30°-60°-90° 50 cm is a rectangle, the side of this rectangle oppo-
triangle in which the given length, 20 cm, is the site the side of length g also has length 50 cm.
length of the hypotenuse. By the 30°-60°-90° (Opposite sides of a rectangle, or any parallelogram,
Triangle Conjecture, the length of the shorter leg is are congruent.) Therefore both legs of the isosceles
half the length of the hypotenuse, so d  12(20)  right triangle have length 50 cm. Finally, look at the
10 cm, and the length of the longer leg is the length triangle in the upper left in which h is the length of
of the shorter leg times  3 , so c  10
3 cm. one side and 30° is the length of one angle. This is a
30°-60°-90° triangle with a shorter leg of length
5. e  34 cm, f  17 cm. This is a 30°-60°-90° triangle
50 cm. Because h is the length of the hypotenuse,
in which the given length, 173 cm, is the length
h  2(50)  100 cm.
of the longer leg. By the 30°-60°-90° Triangle
Conjecture, the length of the shorter leg is the 9. 16 cm2. Let s represent the length of a leg of the
length of the longer leg divided by 3 , so f  triangle. By the Isosceles Right Triangle Conjecture,
8
17 cm, and the length of the hypotenuse is twice the s
cm. Because this is an isosceles right
2 8
length of the shorter leg, so e  34 cm. triangle, use 
2
as both the length of the base
and the height to find the area of the triangle:
 divides the square into two
6. 72 cm. Diagonal EQ
 is the
congruent isosceles right triangles, and EQ
hypotenuse of both triangles. Let s represent the
1 1 8

A  2bh  2   
2 2
8

1 64 1
 2  2  2(32)  16 cm2

162 CHAPTER 9 Discovering Geometry Solutions Manual


©2003 Key Curriculum Press
DG3SM586_136-181.qxd 8/2/06 6:03 PM Page 163

14. a. CDA, AEC, AEB, BFA, BFC


 
1 1
10. ,
2
. Draw a perpendicular segment from P to
2
the x-axis to form an isosceles right triangle. b. MDB, MEB, MEC, MFC, MFA
y
15. Set up and solve the Pythagorean Theorem for the
P (?, ?)
triangle.
1
y c2  x2  x2 The Pythagorean Theorem.
45° x
x (1, 0) c 2  2x 2 Combine like terms.

c  x 2 Take the square root of both sides.


(0, –1)
16. 1693 m2. Draw an altitude of the equilateral
Let (x, y) be the coordinates of P. From the figure, triangle to form two congruent 30°-60°-90° trian-
you can see that x and y are also the lengths of the gles. (Recall that every altitude of an equilateral
legs of the triangle; because it is isosceles, x  y. triangle is also a median and an angle bisector.)
The radius of the circle is 1. From the figure, notice
that the hypotenuse of the right triangle is a radius
30°
of the circle, so its length is 1. By the Isosceles Right
1 1
Triangle Conjecture, x   2
and y   2
.
  26 m 26 m
11. A 30°-60°-90° triangle must have sides whose
lengths are multiples of 1, 2, and 3 . The triangle
shown does not reflect this rule.
60°
12. Possible answer: 13 m

Apply the 30°-60°-90° Triangle Conjecture to the


3 2 30°-60°-90° triangle on the left. The hypotenuse of
6 this triangle is one of the sides of the equilateral
1 triangle, so its length is 26 m. So the shorter leg has
2
3 3
3
length 13 m and the longer leg has length 13 3 m.
1
The longer leg is the altitude of the equilateral
2 triangle. Therefore A  12bh  12(26)(13 3) 
3 (13)(13 3 )  169 3 , so the area of the equilateral
1 triangle is 169 3 cm2.
3
17. 390 m2. Make a sketch.
By the Pythagorean Theorem, the length of the
longer leg of the large right triangle is  62  32 
 36  9  27 . By the 30°-60°-90° Triangle 30°
Conjecture and the way in which the figure was
drawn, the length of this segment is 33 . This 26 m
illustrates that 
27 is equivalent to 3 3.
13. Possible answer:
60°

4 2
4 Look at the 30°-60°-90° triangle on the left. The
altitude of the equilateral triangle is the longer
leg of this triangle, so the length of the shorter leg
4 26
is 
m, and the length of the base (or any side of
3
By the Pythagorean Theorem, the length of the the equilateral triangle) is 2  
26
 52

m. Now
3 3
hypotenuse of the right triangle is  42  42  find the area:
. By the Isosceles Right Triangle Conjecture and
32
the way in which the figure was drawn, the length
1 1 52
3
  676
A  2bh  2  (26)    390
3
of this segment is 4
2 . This illustrates that 
32 is To the nearest square meter, the area of the triangle
equivalent to 4 2. is 390 m2.

Discovering Geometry Solutions Manual CHAPTER 9 163


©2003 Key Curriculum Press
DG3SM586_136-181.qxd 8/2/06 6:03 PM Page 164

18. along the perpendicular. Connect the endpoint


where your arc intersects the perpendicular to the
30° other endpoint of the original segment to form an
isosceles right triangle. The length of its hypotenuse
will be a 2 . Now repeat the process to construct a
x 3 right triangle in which the segment of length a 2
2x
45°
is used as one leg and length a is copied along a
x 2 perpendicular to form the second leg. The length of
the hypotenuse of the second triangle will be a 3.
x
15°
Finally, construct a third right triangle by using the
segment of length a 3 as one leg and copying the
segment of length a 2 along a perpendicular.
45° The length of the hypotenuse of the third triangle
x
will be a 5.
3

1 .Find the area of each triangle in terms of x and
a 2
then find the ratio of the two areas.
 x  x 3 
1 a
Area of 30°-60°-90° triangle  
2
x 2  3
a 3 a 5
 2 a a 2
 x  x  x2 1 2
Area of 45°-45°-90° triangle  
2 a
x 2  3
area of 30°-60°-90° triangle  2
 20. Areas: 4.5 cm2, 8 cm2, 12.5 cm2. Notice that
area of 45°-45°-90° triangle  
Ratio: 
x2 4.5  8  12.5, that is, the sum of the areas of

2 the semicircles on the two legs is equal to the area
x 2  3 of the semicircle on the hypotenuse.
 2  x22
Compass-and-straightedge construction (with
3
 centimeter ruler): Use the ruler to draw a segment
1 of length 8 cm and construct a segment perpendi-
19. Compass-and-straightedge construction: Construct cular to the segment you have drawn at one of its
an isosceles right triangle with legs of length a; endpoints. Use the ruler to draw the second
construct a 30°-60°-90° triangle with legs of lengths segment with length 6 cm. Connect endpoints to
a and a 3 ; and construct a right triangle with legs form the hypotenuse, which will have length 10 cm
of lengths a 2 and a 3 . If d is the length of the by the Pythagorean Theorem. Use your compass
hypotenuse of the third right triangle, d 2  and straightedge to bisect two of the legs of the
(a 2 )2  (a 3 )2  2a 2  3a 2  5a2, so triangle. These perpendicular bisectors will intersect
d  5a 2  a 5.
 at the midpoint of the hypotenuse. Use your
compass to construct a semicircle on each side,
using the midpoint as the center and half the length
a 3
a 5 of the side as the radius.
a 2 a 2a
a

a a 3 a 2 10 cm
6 cm
Another way to construct the segment of length
a 3 is to construct a triangle with legs of length 8 cm
a and a 2 (copying the length a 2 from the
isosceles right triangle). If c is the length of the
hypotenuse of this new triangle (which replaces the
middle triangle in the figure above), c 2  a2 
(a 2 )2  a2  2a2  3a2, so c   3a2  a 3.
Patty-paper construction: Copy the given segment
of length a onto a piece of patty paper. Construct
a perpendicular at one of the endpoints of the
segment by folding. Use a compass to copy length a

164 CHAPTER 9 Discovering Geometry Solutions Manual


©2003 Key Curriculum Press
DG3SM586_136-181.qxd 8/2/06 6:03 PM Page 165

Patty-paper construction: Follow the same proce- cone is unwrapped to form the sector, the lateral
dure, but use folding to construct the right angle area of the cone is equal to the area of the sector.
and then to find the midpoints of the sides. Notice also that the slant height of the cone
becomes the radius of the sector. Use these two
Fold facts to find a.

 
rconel  
a

360° (rsector)
2

(6)(27)  
360° (27)
a
Fold
 2

162  360° (729)


a

Using either method of construction, the radii of
the three semicircles are 3 cm, 4 cm, and 5 cm, a 162 2
    
1 1
so their areas are 2(3)2  4.5 cm2, 2(4)2  360° 729 9
1
8 cm2, and 2(5)2  12.5 cm2. 9a  2(360°)  720°
73
21. 6 chih  12.2 chih. Draw a sketch; form a triangle a  80°
to show the situation where the rope is tightly The measure of the angle at the vertex of the sector
stretched. Let x represent the length of the rope. is 80°.
IMPROVING YOUR VISUAL THINKING SKILLS
Each Monster visitor must go across alone. It makes no
x3 x3 x sense for one to return, so before each Monster crosses,
the two Smallville players must cross and leave one
on the other side to bring the boat back on the next
3 8 trip. This will require 23 round trips for the 11 players
and the coach. The team has enough money for 20 one-
Apply the Pythagorean Theorem to this right way trips. Even without seeing how to arrange the
triangle. Smallville players to return the boat at least 11 times,
(x  3)2  82  x 2 the Monsters don’t have enough money.

x 2  6x  9  64  x 2 EXTENSION
See the solutions to Take Another Look activities 4 and
73  6x 5 on page 181.
73
x  6  12.2
The length of the rope is 763 chih  12.2 chih. LESSON 9.4

22. Extend the rays that form the right angle. m4  EXERCISES
m5  180° by the Linear Pair Conjecture, and it’s 1. 10 m. The tree was 36 m tall before it cracked, so
given that m5  90°. Therefore m4  90°. m2 the length of the hypotenuse is (36  x) m. Use the
 m3  m4  m2  m3  90°  180°. Pythagorean Theorem with a  x, b  24, and
Therefore m2  m3  90°. m3  m1 by c  36  x to find x.
the AIA Conjecture. Therefore m1  m2  90°.
a2  b2  c2
x 2  (24)2  (36  x)2
1
5 x 2  576  (36)2  2(36)x  x 2
4
3 2 x 2  576  1296  72x  x 2
72x  576  1296

23. 80°. The lateral surface area of a cone is rl, where 72x  720
r is the radius and l is the slant height, while the x  10
area of the sector of a circle is 
360°  r , where a is
a
 2

the number of degrees in the central angle and r is He would have to climb 10 m from the base of
the radius. Because the lateral surface area of the the tree.

Discovering Geometry Solutions Manual CHAPTER 9 165


©2003 Key Curriculum Press
DG3SM586_136-181.qxd 8/2/06 6:03 PM Page 166

2. No. The space diagonal of the box is 30.1 in. Refer 5. $7,200. Make a sketch of the cabin and of its trian-
to the example on page 482 of your book. The situ- gular front wall.
ation here is the same, but with different measure-
ments. As in the figure for the example, let d
represent the length of the diagonal of the base 13 m 13 m 13 13
h
(bottom of the box) and x represent the length of
the space diagonal (the diagonal inside the box). 10 m 5 5

First apply the Pythagorean Theorem to the base,


To find the area of the front wall, first find h, the
which is a rectangle that measures 24 in. by 18 in.
height of the triangle. Notice that this altitude
Because 24  6(4) and 18  6(3), the side lengths
divides the isosceles triangle representing the wall
of either right triangle on the base are a multiple of
into two congruent right triangles in which the
the Pythagorean triple 3-4-5, so the length of the
length of the shorter leg is 5 m and the length of
hypotenuse is 6(5)  30. Thus d  30 in. Next
the hypotenuse is 13 m. Because 5-12-13 is a
apply the Pythagorean Theorem to the right triangle
Pythagorean triple, h  12 cm. Now find the area
legs of lengths d  30 in. and 2 in. (the height of
of the isosceles triangle: A  12bh  12(10)(12) 
the box). Here, (30)2  (2)2  x 2, so x 2  904, and
60 m2. The cost of the glass is 60 m2   $120
1 m2 

x   904  30.1 in. The space diagonal is the
$7,200.
longest segment inside the box. Because the bat is
34 in. long, it will not fit in the box. 6. Surface area of prism  (273 + 180) cm2 
226.8 cm2; surface area of cylinder  78 cm2 
3. 50 km/hr. Draw a sketch that shows the distances
245.0 cm2. First find the surface area of the prism,
involved.
which has two congruent hexagonal faces (the
bases) and six congruent rectangular faces (the
lateral faces). To find the area of a hexagonal base,
160 km d you need to find the length of its apothem. When a
regular hexagon is inscribed in a circle, the length of
Lost Pecos each of its sides is equal to the radius of the circle.
Wages 30 km/hr Gulch In this case, the diameter of the cylinder, and thus
for 4 hrs
of the circular bases, is 6 cm, so the radius of the
Paul traveled 4 hours at 30 km/hr, a distance of circular bases of the cylinder is 3 cm.
4(30)  120 km, so the lengths of the legs of the _3 cm
2
right triangle are 120 km and 160 km. Notice that
120  40(3) and 160  40(4), so these lengths are a 60°
3 cm
multiple of 3-4-5. Therefore, d  40(5)  200 km.
3 cm 30° a
Rhaina must travel 200 km in 4 hr, so her average
200 km
speed must be  4 hr  50 km/hr.


4. 8 ft. Let h represent the height of the ladder on


the building before it slips, and let x represent the
number of feet that the bottom of the ladder
slips out.
Notice that the apothem forms the larger leg of a
30°-60°-90° triangle in which the length of the
h 25
h4 25 hypotenuse is 3 cm and the length of the shorter
leg is 32 in., so a  32  3 . To find the area of one
7 x hexagonal base, use the formula for the area of
7x
a regular hexagon: A  12asn  12  323  3  6 
7
27
3 2
In the first right triangle, h  24 ft because 7-24-25 2 cm . Now find the area of one rectangular
is a Pythagorean triple. Then the height of the (lateral) face. Each rectangle has a base of length 3 cm
second right triangle is h  4  20 ft. In this (a side of the hexagonal base) and height 10 cm
triangle, one leg has length 20 ft and the hypotenuse (the height of the cylinder and prism), so each has
has length 25 ft. Because 20  5(4) and 25  5(5), area 30 cm2. To find the total surface area of the
these lengths are a multiple of 3-4-5, so the length prism, add the areas of the two hexagons and the six
of the other leg is 3(5)  15 ft. Then 7  x  rectangles: 2227 3   6(30)  273  180, so the
15 ft, so x  8 ft. surface area of the prism is (273  180) 
226.8 cm2.

166 CHAPTER 9 Discovering Geometry Solutions Manual


©2003 Key Curriculum Press
DG3SM586_136-181.qxd 8/2/06 6:03 PM Page 167

Now find the surface area of the cylinder by using 11. 2 2 2 2


a formula from Lesson 8.7: SA  2r 2  2rh  2 2
2
2(3)2  2(3)(10)  18  60  78  2 2
2
245.0 cm2. 4 2
2 2
36 2 2
7.  cm. Recall that any median of an equilateral 4

3 2
triangle is also an altitude. This segment divides the 4 2
equilateral triangle into two congruent 30°-60°-90°
triangles. Let s represent the length of a side of the 12.
equilateral triangle.

30°
Cat Rabbit Swan Horse with Rider
s 6 cm s
13. 12 units. Draw a perpendicular from the point in
Quadrant II to the x-axis to form a 30°-60°-90°
triangle.
60°
_s y
2

Look at the 30°-60°-90° triangle on the left. The (?, 6)


6r 150°
median (and altitude) of the equilateral triangle is 30° x
the longer leg of this triangle, so the length of the Q
6
shorter leg is 
cm. This is half the length of a
3
side of the equilateral triangle, so s  2 6


3
12
 cm, and the perimeter of the equilateral triangle
3
is 3 3 
12
36
3
cm. From the y-coordinate of the point, you know that
 
the shorter leg of the triangle has length 6, so the
8.  48.2 ft;  16.6 lb. If the slope of the ramp is 112 , length of the hypotenuse is 2(6)  12. Because the
the ramp must cover a linear distance of 48 ft to hypotenuse is a radius of the circle, the radius of
rise 1 4
gain a height of 4 ft: Slope  run  12  48 . Let l
   circle Q is 12.
represent the length of the ramp.
14. 182 cm. Draw the altitude from C to AB  to form
l
4 ft a 45°-45°-90° triangle on the left and a 30°-60°-90°
48 ft
triangle on the right. Let D be the point where the
.
altitude meets AB
The ramp forms the hypotenuse of a right triangle
and the lengths of the legs are known, so you can C

use the Pythagorean Theorem to find l: l 2  (4)2  36 cm


(48)2  16  2304  2320, so l  2320 ft  45° 30°
A B
48.2 ft. D

The work required to lift 200 lb by 4 ft is In CDB, BC  is the hypotenuse and CD is the
(200 lb)(4 ft)  800 ft-lb. Because the person and shorter leg (opposite the 30° angle), so CD  12(BC)
wheelchair go up a 48.2 ft ramp, the force required  12(36)  18 cm. In CDA, CD is one of the legs
800 ft-lb
to go up the ramp is  48.2 ft  16.6 lb.
 and AC  is the hypotenuse. This is an isosceles right
9. a. 160 ft-lb. Work  force  distance  triangle, so AC  CD  2  182 cm.
(80 lb)(20 ft)  160 ft-lb.  and CD
15. Draw radii CB . By the Tangent Conjecture,
160 ft-lb   AB
CB  and CD   AD
, so ABC and ADC are
b. 40 lb. 
4 ft  40 lb.
right angles. Now apply the Quadrilateral Sum
160 ft-lb
c. 20 lb. 
8 ft  20 lb. Conjecture to quadrilateral ABCD: 54°  90° 
10.  4.6 ft. First find the work required: (160 lb)(2 ft) mBCD  90°  360°, so mBCD  126°. But
 320 ft-lb. The equation work  force  distance then the sum of the arc measures of the circle
can be written as W  fd, or d  Wf. In this case, would be 226°  126°  352°. This is impossible
d 320 ft-lb because the sum of the arc measures in any circle
70 lb  4.6 ft.

must be 360°.

Discovering Geometry Solutions Manual CHAPTER 9 167


©2003 Key Curriculum Press
DG3SM586_136-181.qxd 8/2/06 6:03 PM Page 168

 and also AP


16. (4, 43 ). Draw AA , where AP
 is the . In PRO, P is opposite RO
20. PO , R is opposite
.
altitude from A to OA  
PO , and O is opposite PR . By the Triangle Sum
y Conjecture, mP  mR  mO  180°, so
A mO  180°  70°  45°  65°. Therefore R is
the smallest angle in the triangle, so by the Side-
Q
T
Angle Inequality Conjecture, PO is the shortest side.

30° A
x
IMPROVING YOUR VISUAL THINKING SKILLS
O P (8, 0)

Because A is the image of A after reflection over


, you know that OT
OT  is the perpendicular bisector
 
of AA. Because OT  AA , OQA is a 30°-60°-90°
triangle with hypotenuse OA  with length 8, so
AQ  4 and OQ  4  is the
3 . Also, because OT

perpendicular bisector of AA, AQ  AQ  4, so EXTENSION
AA  8. This makes OAA isosceles, but because See the solution for Exercise 15 in Lesson 9.1 on
the vertex angle QAO is a 60° angle, it must be pages 157 and 158.
equilateral. Now look at OAP. Because OAA is
equilateral, OA  8, and because every altitude of
an equilateral triangle is a median, OP  4. By the LESSON 9.5
Pythagorean Theorem or by noticing that OAP is EXERCISES
a 30°-60°-90° triangle with shorter leg of length 4
(and hypotenuse of length 8), you can tell that 1. 5 units. Label the points A(10, 20) and B(13, 16),
AP  4 3 . Because OP  4 and AP  4 3, and substitute their coordinates in the distance
the coordinates of A are (4, 4 3 ). formula.

17. SAA. Use the pair of vertical angles and the marked (AB)2  (x 2  x1)2  (y2  y1)2 The distance
formula.
pairs of congruent angles and congruent segments.
 (13  10)2  (16  20)2 Substitute the
18. Orthocenter. The construction marks show that this
given values.
is the point of concurrency of the two altitudes.
 (3)2  (4)2 Subtract.
19. 115°. When a polygon is named, the vertices are
listed in consecutive order, so Q and U are  9  16 Square numbers.
consecutive angles of parallelogram QUID, and Q
 25 Add.
and I are opposite angles. Therefore, Q I
(Parallelogram Opposite Angles Conjecture), so AB  25
5 Take the positive
2x  5°  4x  55°. Solve this equation. square root of
both sides.
2x  5°  4x  55°
2. 45 units. Label the points A(15, 37) and B(42, 73),
60°  2x and substitute their coordinates in the distance
x  30° formula.

Then mQ  2x  5°  2(30°)  5°  65°. (AB)2  (x 2  x1)2  (y2  y1)2

(As a check, you can find that mI  4x  55°   (42  15)2  (73  37)2
4(30)  55°  65°.)  (27)2  (36)2
By the Parallelogram Consecutive Angles Conjec-  729  1296
ture, Q and U are supplementary, so mU 
180°  65°  115°.  2025
AB  2025
 45

168 CHAPTER 9 Discovering Geometry Solutions Manual


©2003 Key Curriculum Press
DG3SM586_136-181.qxd 8/2/06 6:03 PM Page 169

3. 34 units. Label the points A(19, 16) and 9. Center is (2, 5), r  6. y  5 can be written as
B(3, 14), and substitute their coordinates in y  (5), so the given equation can be rewritten
the distance formula. as (x  2)2  (y  (5))2  62. Then, h  2,
k  5, and r  6.
(AB)2  (x 2  x1)2  (y2  y1)2
10. Center is (0, 1), r  9. The given equation can be
 ((3)  (19))2  (14  (16))2
rewritten as (x  0)2  (y  1)2  92, so h  0,
 (16)2  (30)2 k  1, and r  9.
 256  900 11. (x  3)2  (y  1)2  18. Use the distance formula
to find the radius of the circle. The radius of a
 1156
circle is the distance between the center and any
AB  1156
 34 point on the circle, so in this case, it is the distance
between the center (3, 1) and the point (6, 2).
4. 354 m. Viki’s and Scott’s locations vary by 5 blocks
Because the equation of a circle involves r 2, it is
vertically and 5 blocks horizontally. By the distance
more convenient to find r 2 : r 2  (6  3)2 
formula, the shortest distance between them is
(2  (1))2  (3)2  (3)2  18. Now find the
 52  52   50 blocks. Because each block is
equation of the circle.
approximately 50 m long, this distance is about
50 50  354 m. (x  h)2  (y  k)2  r 2 Equation of a circle.

You could also consider the distance between Viki (x  3)2  (y  (1))2  18 Substitute 3 for h, 1
and Scott as the hypotenuse of an isosceles right for k, and 18 for r 2.
triangle with legs of length 5 blocks. The length (x  3)2  (y  1)2  18 Simplify.
of the hypotenuse is 52 blocks, or 50(52 ) 
354 m. 12. a. 14 units. This is the same as finding the space
diagonal of a rectangular prism. Look at the
5. 52.4 units. Use the distance formula to find each
“imaginary box” drawn on the figure. First
side length in the triangle, and then add the lengths
of the three sides to find the perimeter. find the diagonal of the base of the box in the
x-y plane. In this plane, this segment has
(AB)2  (8  2)2  (12  4)2  (6)2  (8)2  endpoints (0, 0) and (2, 1), so its length is
36  64  100, so AB  100
 10.
(2  0 )2  (1  0) 2  4  1  5 .
(BC)2  (24  8)2  (0  12)2  (16)2  Find the length of the space diagonal with
(12)2  256  144  400, so AB  400  20. endpoints (0, 0, 0) and (2, 1, 3). Let d represent
(AC)2  (24  2)2  (0  4)2  (22)2  (4)2  the length of this diagonal. This segment is the
484  16  500, so AB  500
. hypotenuse of a right triangle with legs of lengths
 5 and 3, so d 2  ( 5 )2  (3)2  5  9  14,
Perimeter of ABC  AB  BC  AC  10  and d   14 .
20  500
 52.4
Another way to find the distance from the
6. Isosceles. Use the distance formula to find each side origin (0, 0, 0) to (2, 1, 3) is to extend the
length in the triangle, and then see whether any of distance formula to three dimensions:
the lengths are equal. d   (2  0 )2  (1  0)2  (3  0)2 
(DE)2  (39  6)2  (12  (6))2  (33)2   41  9   14 .
(6)2  1089  36  1125, so DE  1125
.  411
b. 176 units.
(EF)2  (24   (18 
39)2 
(12))2 (15)2  PQ  
(5  1
)2  (6
 2)2
 (15
 3)2
(30)2  225  900  1125, so EF  1125
.
 
42  (
4)2 
(12)2
(DF)2  (24   (18 
6)2  
(6))2 (18)2
 
16  16
 144
(24)2  324  576  900, so DF  900
 30.
  176
Thus, DE  EF  DF, so the triangle has two
congruent sides, but not three congruent sides. This radical can be simplified:
Therefore, DEF is isosceles, but not equilateral.  16
176  11  411 .

7. x 2  y 2  16. If a circle has center (0, 0), its equa- c. AB  (x 1  x


2)  (
2 y1  y
2)  (z
2
1  z 2
)2 or
tion is (x  0)2  (y  0)2  r 2, or x 2  y 2  r 2.
(x 2  x
1)  (y
2  y
1)  (z 2  z1)
2 .
2 2

8. (x  2)2  y 2  25. Substitute 2 for h, 0 for k, and


5 for r in the equation of a circle.

Discovering Geometry Solutions Manual CHAPTER 9 169


©2003 Key Curriculum Press
DG3SM586_136-181.qxd 8/2/06 6:03 PM Page 170

3 1 Apply the Pythagorean Theorem and solve for x.


13.   2 , 2 . Draw a perpendicular segment from

A to the x-axis to form a 30°-60°-90° triangle. x 2  (40)2  (x  8)2
(The measure of the angle that forms a linear x 2  1600  x 2  16x  64
pair with the 150° angle is 30°.) The hypotenuse of
this triangle is a radius of the circle, so its length 0  16x  1536
is 1. Therefore the length of the shorter leg (oppo- 16x  1536
site the 30° angle) is 12(1)  12, and the length
3 x  96
of the longer leg is 12( 3)   2 . Because A is in
Quadrant II, its x-coordinate is negative and its The depth of the pond is 96 cm.
y-coordinate is positive, so the coordinates of A 17. The angle of rotation is approximately 77°. Connect
3 1
are   2 , 2 .
 two pairs of corresponding points. Construct the
14. k  2 , m  6 . This figure includes both a perpendicular bisector of each segment. The point
45°-45°-90° triangle and a 30°-60°-90° triangle. where the perpendicular bisectors meet is the center
In the isosceles right triangle, the legs have length of rotation. This point is labeled as P in the
 3 and the hypotenuse has length m, so m  diagram below. Measure the angle of rotation with a
 3  
2   6 . In the 30°-60°-90° triangle, k is protractor. The rotation is counterclockwise, and the
the length of the shorter leg and m is the length of angle is approximately 77°.
the longer leg, so m  k   3 , or k  m


6


3 3

3 
6
 
2 .
6 12
15. x  
 23 , y  
 43 . Because the
3 3 R R
large triangle is equilateral, the medians that are
shown in the figure are also altitudes and angle C
T
bisectors. This means that the small triangle in the T U

lower-left corner of the figure is a 30°-60°-90°


triangle. (The 30° angle is half of a 60° angle of the
C
equilateral triangle.) U

P
60° Fold 2
y Fold 1
x

30° IMPROVING YOUR VISUAL THINKING SKILLS


6 Here are two routes that are shorter than 42 ft. The
In this triangle, x is the length of the shorter leg, shortest path measures 40 ft.
y is the length of the hypotenuse, and the length of Side
17
40.7
wall Ceiling
the longer leg is 6 because this segment is half of 6 ft
6 37
a side of the equilateral triangle. Therefore x  
3 Side
and y  2x  2  6
3 
12
3
. 11 ft
Back wall wall
 
1 ft 30 ft
16. 96 cm. Let x represent the depth of the water. Then, 6 ft
x  8 represents the length of the lily stem. Make a
sketch. 30 ft
Side 40
wall A Ceiling 24
8 cm 40 cm
Surface 6 ft
32
Back 40 ft
12 ft wall
6 ft Floor B
x x8 Side
wall
1 ft 30 ft 1 ft

170 CHAPTER 9 Discovering Geometry Solutions Manual


©2003 Key Curriculum Press
DG3SM586_136-181.qxd 8/2/06 6:03 PM Page 171

64
LESSON 9.6 4. 3  163  cm2. The area of the shaded region,
which is a segment of the circle, is the difference
EXERCISES between the area of the sector and the area of the
1. 456 cm2. Look at the angles in quadrilateral triangle. Let P be the center of the circle. In the
BODY. By the Tangent Conjecture, OBY and triangle, draw the altitude from the center to OH.
ODY are right angles, so by the Quadrilateral Sum Let R be the point where the altitude meets the
Conjecture, 90°  105°  90°  mDOB  360°, . Because the original triangle is isosceles
base, OH
and mDOB  75°. Because DOB is the central (two sides are radii), the altitude to the base is also
 (the minor arc), mBD
angle that intercepts BD 
a median and an angle bisector, so this segment
75°, and the measure of major arc BD is 360°  75° divides the original triangle into two congruent
 285°. Now find the area of the shaded region, 30°-60°-90° triangles.
which is a sector of the circle. P

A   a
 2   
285°
360° r  360° (24)
2 60° 60°

30° 30°
19
 2(24)
4 
2  (19 24)  456 cm2 O R H

2. (32  32 2
3 ) cm . The area of the shaded region Because OH  8  is a median, as
3 cm and PR
is the difference between the area of the semicircle well as an altitude, OR  43 cm. Because PR  is
and the area of the triangle. T is a right angle also the bisector of the 120° angle, mOPR  60°.
(Angles Inscribed in a Semicircle Conjecture), so  is the longer leg (opposite the 60°
Therefore OR
RTH is a 30°-60°-90° triangle in which HT  is angle) in 30°-60°-90° triangle POR, PR is the
 is the
the longer leg (opposite the 60° angle), RT 
shorter leg, and PO is the hypotenuse. Because
 is the
shorter leg (opposite the 30° angle), and RH OR  4 3 cm, PR  4 cm, and PO  2(4) 
hypotenuse. Because HT  8 3 , RT  8, and 8 cm. Notice that PO is a radius of the circle, so
RH  2(8)  16. RH  is a diameter of the circle, r  8 cm.
so the radius is 8 cm. 64π

360° r  3 (8)  3 cm
120° 1
Area of sector   2  2  2
1 1 1
Area of semicircle  r 2  (8)2  64 
2 2 2 Area of triangle  12bh  12(83 )(4)  163 cm2
32 cm2
64
Area of triangle  12bh  12(8)(83 )  323 cm2
Area of shaded region  3  163  cm2
, OB
5. 18 m. Draw OA , and OT
.
Area of shaded region  (32  323 ) cm2
64
3. 643   3  cm . The area of the shaded region
 2

is the difference between the area of quadrilateral O


A
HART and the area of the sector of the circle with a T
central angle that measures 120°. Draw RH, which
divides quadrilateral HART (a kite) into two B
congruent 30°-60°-90° triangles. (Using the Tangent  and OB are radii of
OAB is isosceles because OA
Segments Conjecture, the Tangent Conjecture, and  is tangent to the inner
the outer circle. Because AB
the congruent radii of the circle, HAR HTR   AB by the Tangent Conjecture, so
circle, OT
by SAS, so ARH TRH by CPCTC. Then
 is the OTA is a right angle, and OTA is a right triangle
mARH  mTRH  60°.) In HAR, HA . OT
 is a radius of the inner
 is the shorter leg, and HR
 is the with hypotenuse OA
longer leg, AR  is a radius of the outer circle, so
circle and OA
hypotenuse. Because HA  8 3 cm, AR  8 cm.
OA  12 m and OT  12  3  15 m. Now apply
Therefore the area of HAR is 12(8)(8 3) 
the Pythagorean Theorem to OTA. Because 12 
32 3 cm2, and the area of HART is 2(32 3) 
3(4) and 15  3(5), the side lengths in this triangle
64 3 cm2.
are a multiple of the Pythagorean triple 3-4-5.
 is a
To find the area of the sector, notice that AR Therefore AT  3(3)  9 m. Because OAB is
radius of the circle, and you found that AR  8 cm , OT
isosceles with base AB  is a median as well as
from HAR, so r  8 cm. The area of the sector is an altitude. Therefore AT  BT, so AB  2(AT) 
2(9)  18 m.

360° (8)  3 64  3  cm . Therefore the area
120° 2 1 64 2
  
64
of the shaded region is 643   3  cm .
 2

Discovering Geometry Solutions Manual CHAPTER 9 171


©2003 Key Curriculum Press
DG3SM586_136-181.qxd 8/2/06 6:03 PM Page 172

6. 324 cm2. Let x represent the radius of the inner 8. (40 + 603 )  230 cm. Refer to the figure below.
circle. Radii of the circles are 18 cm and 12 cm. Notice
that the triangles formed are 30°-60°-90° triangles
because in each of them, the length of the shorter
A leg (a radius) is half the length of the hypotenuse.
x 2  182
18 cm Use the 30°-60°-90° Triangle Conjecture to find
O
x T
that the lengths of the longer legs are 18 3 cm
in the larger circle and 12 3 cm in the smaller
circle.
B
18 3
18 12 3
12
As in Exercise 5, the radius of the inner circle is 60° 60°
60° 60°
perpendicular to the chord of the outer circle
and also bisects that chord. Here, AT  12(AB)  18 3 12 3
1
(36)  18 cm. Because OT  AT , OAT is 36 24
2
. So by
a right triangle with hypotenuse OA Length of belt  circumference of 240°-sector of
the Pythagorean Theorem, OA   x 2  1
82  large circle (360°  120°  240°)  sum of lengths
 x  3
2 24 cm. Recall that the area of an annulus of two tangent segments to larger circle  sum of
is the difference between the area of the outer circle the lengths of the two tangent segments to the
and the area of the inner circle. Here, the outer small circle plus 23 of the circumference of the
circle has radius OA   x 2  3
24 and the inner small circle:
circle has radius x. 2
(2  18)  218 3  + 212 3   23(2  12)
Aannulus  Aouter circle  Ainner circle 3
 24  36 3  24 3  16
     x 2
x 2  324 2

 40  60
3   230 cm
 (x 2  324)  x 2
5
9. .
When a regular hexagon is inscribed in a circle,
6
 x 2  324  x 2 the length of each side of the hexagon (which is a
 324 cm2 chord of the circle) will be the radius of the circle.
Make a sketch.
7. 102 cm. Let x be the distance from the center of the
A B
circle (the original pipe) to the chord. Then the 6
radius of the pipe is (x  6) cm. 6
T Q

S R

x6 x From this figure, you can see that ABC will be
an acute triangle if C is on SR and will be a right
24
6
triangle if C is exactly at S or exactly at R.
Therefore, if C is anywhere on major arc SAR
Notice that the segment of length x is the altitude (except exactly at A or exactly at B), ABC will be
of an isosceles triangle whose legs are radii of the an obtuse triangle. The measure of each of the six
circle, so it forms two congruent right triangles,   300°.
minor arcs of the circle is 60°, so mSAR
each with legs of lengths 24 cm and x cm, and Therefore the probability that ABC will be an
hypotenuse of length (x  6) cm. Apply the obtuse triangle if point C is selected randomly on
Pythagorean Theorem to one of these right triangles 300° 5
360°  6 . (Note: If C coincides with A
the circle is   
and solve for x. or B, there will be no triangle. Because there are
x 2  (24)2  (x  6)2 infinitely many points on the circle, this can be
ignored in calculating the probability.)
x 2  576  x 2  12x  36
540  12x
x  45
r  x  6  51 cm, so d  2r  2(51)  102 cm.
The pipe’s original diameter was 102 cm.

172 CHAPTER 9 Discovering Geometry Solutions Manual


©2003 Key Curriculum Press
DG3SM586_136-181.qxd 8/2/06 6:03 PM Page 173

10. Inscribed circle: 3 cm2; circumscribed circle: Inscribed circle: r  OG  1 cm


12 cm2. The area of the circumscribed circle is
A  r 2  (1)2   cm2
four times as great as the area of the inscribed
circle. Draw a perpendicular segment from the Circumscribed circle: r  OD  2 cm
. Let O be
center of the inscribed circle to AB
A  r 2  (2)2  4 cm2
the center of both circles and D be the point where
the perpendicular segment meets AB . Also draw As in Exercise 10, the area of the circumscribed
, which is a radius of the circumscribed circle.
OA circle is four times the area of the inscribed circle.
C 12.  3931 cm2. First find the area of the equilateral
triangle. Because it is equilateral, the altitude to the
O horizontal base will form two congruent 30°-60°-90°
triangles. In each of these triangles, the length
A D B of the shorter leg is 40 cm, so the length of the
altitude, which is the longer leg, will be 40 3 cm.
 bisects CAB and mCAB  60° because
OA 1 1
Atriangle  2bh  2  80  40
3  16003 cm.
CAB is equilateral, so AOD is a 30°-60°-90°
triangle. AB  6 cm, so AD  3 cm. In the Now look at the outer edges of the arch. The line
 is the longer leg (opposite segment marked with length 80 cm in the center of
30°-60°-90° triangle, AD
the figure shows that the edge on the left is an arc
  is the
the 60° angle), OD is the shorter leg, and OA of a circle with center in the lower right corner of
3
hypotenuse. Because AD  3 cm, OD  
cm the arch and radius 80 cm. By symmetry, the edge
3
and OA  2  3

 6

cm. Now that you have on the right of the arch is an arc of a circle with
3 3
found the radii of the two circles, you can find center in the lower left corner of the arch and
their areas. radius 80 cm. (Notice that these are different circles
because they have different centers, although they
3
Inscribed circle: r  OD  
cm have the same radius.) This means that the parts of
3

  9
A  r 2       3  3 cm2
3 the interior of the arch that are outside the triangle
2

3 are segments of the two circles described above.


6 Each segment is formed with a 60° angle (from the
Circumscribed circle: r  OA  
cm
3
equilateral triangle) and an 80 cm radius. Find the
  36
A  r 2       3  12 cm2
6 2
area of one of these segments, using the area of the
3
12  4(3), so the area of the circumscribed equilateral triangle.
circle is four times the area of the inscribed circle. Asegment  Asector  Atriangle
11. Inscribed circle:  cm2; circumscribed circle:
4 cm2. The area of the circumscribed circle is four
   
60°

360° (80)  16003 
2

1
times as great as the area of the inscribed circle. As  6(6400)  16003
in Exercise 10, draw segments to form a 30°-60°-90°
triangle. Let O be the center of both circles, and let
G be the point where the perpendicular segment

3200
 3  16003 cm2 
.
meets DE Aarch  2Asegment  A triangle


3200

F
 2 3  16003  16003 

O
 6400

 3  16003 cm2  3931 cm2
D G E 13. (12  6 3 ) cm. Each side of the triangle in the
center of the figure is made up of two radii, so this
DE  23 cm, so DG  3 cm. In DOG, DG  is is an equilateral triangle with side length 12 cm.
 is the shorter leg, and OD
the longer leg, OG  is the The altitude of this triangle divides the triangle
hypotenuse. Because DG  3 cm, OG  1 cm into two congruent 30°-60°-90° triangles, each with
and OD  2 cm. Use the radii of the two circles to shorter leg of length 6 cm. The altitude is the longer
find their areas. leg of this triangle, so its length is 6
3 cm. If you

Discovering Geometry Solutions Manual CHAPTER 9 173


©2003 Key Curriculum Press
DG3SM586_136-181.qxd 8/2/06 6:03 PM Page 174

transfer the lengths that you know to the left side Triangle Conjecture, the length of the shorter
of the rectangle, you will see that the height of (vertical) leg is 12, and the length of the longer
the rectangle is 6  63  6  (12  63 ) cm. (horizontal) leg is  3
2 . Because K is in Quadrant III,
6 6 both of its coordinates are negative. Therefore the
3
coordinates of K are   2 ,  2 .
1


6 3 18. (x  3)2  (y  3)2  36. Substitute 3 for h, 3 for k,


and 6 for r in the equation (x  h)2  (y  k)2  r 2.
6 6
19. Center  (1, 0), r  10. Rewrite the equation in the
80° form (x  h)2  (y  k)2  r 2 to find the center
14. 77 cm. The arc length of sector ABC is  
360° (2r) and radius.
(2    9)    18  4 cm. Therefore the
80° 2
 360° 9
circumference of the base of the cone is 4 cm. Use x 2  y 2  2x  1  100 Given equation.
this to find the radius of the circular base of the (x 2  2x  1)  y2  100 Rearrange and group
cone: C  2r  4, so r  2 cm. The radius of terms.
the sector, which is 9 cm, becomes the slant height
of the cone. Because the altitude of the cone is (x  1)2  y 2  100 Rewrite perfect-square
perpendicular to the base, the height of the cone, trinomial as square of
the radius of the base, and the slant height form binomial.
a right triangle in which the slant height is the (x  1)2  (y  0)2  (10)2 Write equation in
hypotenuse. Thus (2)2  h 2  (9)2, so 4  h 2  81. the form (x  h)2 
Then h 2  77 and h   77 . (y  k)2  r 2.

15. 76 cm. Use the circumference of the circular cross From the last equation, h  1, k  0, and r  10,
C
section to find its diameter: C  d, so d    = so the circle has center (1, 0) and radius 10.
336
 cm. The diagonals of the square are diameters
 20. The diameter is the transversal, and the chords
of the circle. A diagonal of a square divides the
are parallel by the Converse of the Parallel Lines
square into two isosceles right triangles. By the
Conjecture. The chords are congruent because they
Isosceles Right Triangle Conjecture, the length of a
are the same distance from the center. Sample
leg of an isosceles triangle can be found by dividing
construction:
the hypotenuse by 2 . Therefore the side length of
the largest square that can be cut from the cross
section is
336

d 
    76 cm
2 2
16.   1
,1
2 
. Draw a perpendicular segment from M 21. Any long diagonal of a regular hexagon divides it
2
to the x-axis to form a right triangle. Because into two congruent quadrilaterals. Each angle of
180°  135°  45°, this is an isosceles right triangle. (6  2)  180°
a regular hexagon is  6  120°, and the
The hypotenuse of this triangle is a radius, so the diagonal divides two of the 120° angles into 60°
length of the hypotenuse is 1. By the Isosceles Right angles. Look at the diagonal as a transversal. The
Triangle Conjecture, the length of each leg can be alternate interior angles are congruent; thus, the
found by dividing the length of the hypotenuse by opposite sides of a regular hexagon are parallel.
 2 . Because M is in Quadrant II, its x-coordinate is
120° 60°
negative and its y-coordinate is positive. Therefore 60°
the coordinates of M are   1
2
1
,2 
.
  120° 120°

3
17.  2 ,
 . Draw a perpendicular segment from K
12
60°
to the x-axis to form a right triangle. Because 60° 120°
210°  180°  30°, this is a 30°-60°-90° triangle.
The hypotenuse of this triangle is a radius, so the 22. Write a separate rule for each column of numbers
and combine them: n  (n  2)  3  (n  3) 
length of the hypotenuse is 1. By the 30°-60°-90°
(n  1).

174 CHAPTER 9 Discovering Geometry Solutions Manual


©2003 Key Curriculum Press
DG3SM586_136-181.qxd 8/2/06 6:03 PM Page 175

23. a. Because a carpenter’s square has a right angle congruent right triangles. Because 5-12-13 is a
and both radii are perpendicular to the tangents, Pythagorean triple, the shorter leg of one of these
a square is formed. The radius is 10 in.; so the right triangles has length 5 cm, so AB  2(5) 
diameter is 20 in. 10 cm.
10 in. 3. Obtuse. (70)2  (240)2  4,900  57,600  62,500,
and (260)2  67,600. Because the sum of the
10 in. 10
in. squares of the two shorter sides is less than the
square of the longest side, C is obtuse, and there-
10 in.
d  20 in. fore the triangle is obtuse.
4. 26 cm. Find the diagonal of the base and then AB,
the length of the space diagonal. The diagonal of
b. Possible answer: Measure the circumference with the base (bottom rectangular face) is  82  2 42 
string and divide by .  64  5 76   640 . Then, (AB)  (
2 640 ) 
2

IMPROVING YOUR REASONING SKILLS (6)2  640  36  676, so AB   676  26 cm.


1. One approach: A  0, and C  1, so B  5. You could also find AB by extending the
2. One approach: D  2; 7F  4 ends in F, so F is 1 Pythagorean Theorem to three dimensions, which
or 6. Trying each possibility leads to J  6. is similar to using the distance formula in three
dimensions: AB   (24)2  (8)2 
(6)2 
EXTENSION  576  64  36   676  26 cm.
Let c be the length of the hypotenuse of the right  
2 , 2 . Draw the perpendicular segment from U
3 1
5.
triangle.
down to the x-axis to form a 30°-60°-90° triangle.
c2
Area of semicircle with diameter c  122c2  8
 The hypotenuse of this triangle is a radius of the
Area of right triangle  12ab circle. The circle has radius 1, so the length of the
2 a 2 hypotenuse is also 1. Then, by the 30°-60°-90°
Area of semicircle with diameter a  122a  8
 Triangle Conjecture, the shorter (vertical) leg has
2 b 2
length 12(1)  12, and the length of the longer (hori-
Area of semicircle with diameter b  122b  8
3
zontal) leg is 12( 3)  2 . Because U is in Quad-
Total area of two segments of circle with diameter c 
rant I, both of its coordinates are positive. Therefore
area of semicircle with diameter c  area of triangle  3 1
c 2 1
  ab
the coordinates of U are   2 , 2 .

8 2
Total area of shaded regions  area of semicircle with 6.  1

,  1

. Draw the perpendicular from V to
2 2
the x-axis. Because 225°  180°  45°, the triangle
diameter a  area of semicircle with diameter b 
that is formed is a 45°-45°-90° triangle. The
total area of two segments of circle with diameter c: hypotenuse of this triangle is a radius of the circle,

8a  8b  8 2 


c
2 2 1 2 and the circle has a radius of 1, so the length of the
ab
hypotenuse is also 1. Then, by the Isosceles Right
a2 b 2 c 2 1 Triangle Conjecture, the length of each leg is  1
.
 8  8  8  2ab 2
Because V is in Quadrant III, both of its coordinates
 1
 8(a 2  b 2  c 2)  2ab are negative. Therefore the coordinates of V are
 2 ,  2 .
1 1
By the Pythagorean Theorem, a2  b 2  c 2, so a 2   
b 2  c 2  0. Therefore the area of the shaded region is 7. 2003 cm2.
This is a 30°-60°-90° triangle with
 1 1
(0)  ab  ab, which is the area of the triangle. hypotenuse of length 40 cm. By the 30°-60°-90°
8 2 2
Triangle Conjecture, the length of the shorter leg
CHAPTER 9 REVIEW (opposite the 30° angle) is 20 cm, and the length
of the longer leg is 20 3 cm. Then the area of
EXERCISES the triangle is 12(20)(20
3 )  200 3 cm2.
1. 20 cm. 15  5(3) and 25  5(5). This is a multiple 8. d  122 cm. The diagonal divides the square into
of the Pythagorean triple 3-4-5, so x  5(4)  two congruent isosceles right triangles. From the
20 cm. given area, each side of the square has length 12 cm.
2. 10 cm. The altitude from C to AB  is also a median Because the length of each leg of the isosceles right
because C is the vertex angle of an isosceles triangle. triangles is 12 cm, by the Isosceles Right Triangle
This altitude divides the isosceles triangle into two Conjecture, d  12 2 cm.

Discovering Geometry Solutions Manual CHAPTER 9 175


©2003 Key Curriculum Press
DG3SM586_136-181.qxd 8/2/06 6:03 PM Page 176

9. 246 cm2. Notice that the trapezoid is made up of E 2 R


two right triangles, so the Pythagorean Theorem can
be used to find the lengths of its bases. In ADC, 2
2 2
2
 and DC
DA  are the legs and AC is the hypotenuse.
Because 12  4(3) and 20  4(5), the side lengths Q
in this triangle are a multiple of the Pythagorean S 2

triple 3-4-5, so DC  4(4)  16 cm. In ABC,


 and BC
AC  are the legs and AB is the hypotenuse.
Because 15  5(3) and 20  5(4), the side lengths
in this triangle are also a multiple of 3-4-5, so
AB  5(5)  25 cm. You can now find the area of From the figure, you can see that each half of the
trapezoid ABCD either by finding the areas of the shaded region is equal to the area of a segment of
two triangles separately and adding the results or by the circle in which the central angle is 90°. (The
applying the Trapezoid Area Conjecture to find the  and SQ
sector is a quarter-circle.) Notice that SE 
area of the trapezoid directly. Both of these methods are radii of the circle, so r  2 cm. Find the area of
are shown here. the segment.
Method 1: Use the Triangle Area Conjecture: Asegment  Asector  Atriangle
A  12bh 1 1
 4(2)2  2(2)(2)  (  2) cm2
Area of trapezoid ABCD  area of ADC  area
Area of shaded region  2Asegment
of ABC  12(16)(12)  12(20)(15)  96  150 
246 cm2  2(  2)  (2  4) cm2
Method 2: Use the Trapezoid Area Conjecture: 13. 222.8 cm2. From the figure, you can see that BD  is
A  12h(b1  b2) both a diagonal of the square and a diameter of the
Area of trapezoid ABCD  12(12)(25  16)  circle. Use r and d to represent the radius and diam-
246 cm2 eter of the circle, respectively. First use the given
area of the circle to find the radius: A  r 2 
10. 72 in.2. The diameter of the semicircle is the
 and r 

350 350
350 cm2, so r 2     cm. Then, d 

longer leg of the right triangle. Because 7-24-25 is
2r  2

350

 . The diameter is the hypotenuse of
a Pythagorean triple, the length of this segment is
isosceles right triangle BAD, so by the Isosceles
24 cm, so the radius is 12 cm. Then the area of
the semicircle is 12r 2  12(12)2  12144  Right Triangle Conjecture,
72 in.2. 2 350


11. 24 cm2. The shaded region is a sector of the AB  AB  
circle. In order to find its area, you need to know 2
the radius of the circle. Draw OT to form two  
AB and AD are sides of the square. Therefore, the
congruent 30°-60°-90° triangles. mOBT  90° by area of square ABCD is
the Tangent Conjecture, and mBOT  60° because

 
 
350
2  2 350
 bisects the 120° angle, BOA. Therefore, in
OT  4  
BOT, OB  is the shorter leg, BT
 is the longer leg,  
 is the hypotenuse. To find the radius of the 2 2
and OT
circle, apply the 30°-60°-90° Triangle Conjecture to
BOT: BT  6 3 cm, so OB  6 cm. Now find
 
350 700
 2     222.8 cm2
Note: In order to get an accurate result, it is impor-
the area of the shaded sector. The angle for this
tant that you do not round until the final step.
sector is 360°  120°  240° and the radius is 6 cm.
14. Isosceles right triangle. First use the distance formula
Asector    
a
 2
 
240°

360° r  360° (6)
2
to find the lengths of the three sides of the triangle.
2
 3(36)  24 cm2 AB  (11
 3)
2  (3  5)2 
8 2  (2)2

12. (2  4) cm2. Because SQRE is a square, ESQ is  
64  4  
68
an isosceles triangle with hypotenuse QE. Because BC  (8
 11)
2  (8  3)2 
(3) 2  52

QE  2 2 cm, the length of each side of the
square is 2 cm. Use a compass to draw a circle with  
9  25  
34
center S and radius SE. AC  (8
 3)
2  (8  5)2 
5 2  32

 
25  9  
34

176 CHAPTER 9 Discovering Geometry Solutions Manual


©2003 Key Curriculum Press
DG3SM586_136-181.qxd 8/2/06 6:03 PM Page 177

Because BC  AC, ABC is isosceles with legs AB  17. 1.4 km; 8.5 min. First find the distances that Peter

and BC . To determine whether this is an isosceles and Paul walk before they stop. Peter walks 2 km/hr
right triangle, see whether the side lengths satisfy for 30 min, or 12 hr, so he walks 1 km. Paul walks
the Pythagorean Theorem. 3 km/hr for 20 min, or 13 hr, so he also walks 1 km.
Because their paths are at right angles to each other,
(BC)2  (AC)2  34
2  34
2 the distance between them after they have both
 34  34  68; stopped walking is the length of the hypotenuse of
an isosceles right triangle in which the length of
(AB)2  68
2  68 each leg is 1 km. This distance is  2 km  1.4 km.
Therefore ABC is an isosceles right triangle with If Peter and Paul start running straight toward each
.
hypotenuse BC other with both of them running at 5 km/hr, each
2
of them will travel half this distance, or 2 km.
15. No. The closest she can come to camp is 10 km. Let The time required to travel   2
2 km at 5 km/hr is
d represent the (direct) distance that Sally has trav-
eled from camp. Make a sketch of the situation  2
 km
described in the exercise. 2
 2 hr
5 km/hr 10

“Pay dirt”
2 60 min
1 hr 
Convert this time to minutes:  hr 
45 km/hr 10
d
for 2 hrs 2 min  8.5 min.
6
Camp “No luck” Note: To get an accurate result for the time it will
60 km/hr
for 2 hrs take Peter and Paul to reach each other, work with
radicals as shown above, rather than rounding inter-
She has traveled 2(60)  120 km east and 2(45)  mediate results. If you use the distance of 1.4 mi to
90 km north. The distances 120 km and 90 km calculate this time, you will get 8.4 min, rather than
form the legs of a right triangle in which d is the 8.5 min.
length of the hypotenuse. Thus, (120)2  (90)2 
d 2. Notice that 120  30(4) and 90  30(3), so the 18. Yes. Find the length of the space diagonal of the
side lengths in this triangle are a multiple of 3-4-5, box. Let d represent the length of the space diagonal.
and therefore, d  30(5)  150 km. The distance Use the second method shown in the solution for
back to camp is 150 km. Sally’s complete trip covers Exercise 4.
a distance of 120  90  150  360 km, but she d  
(12)2   (14)2
(16)2
has enough gas to travel only 350 km, so she will
fall 10 km short of making it back to camp.  
144 
256 
196  
596  24.4
16. No. The 15 cm diagonal is the longer diagonal. A Thus the length of the space diagonal of the box is
diagonal of a rectangle divides the rectangle into about 24.4 in., so the 24 in. long flute will fit in
two right triangles, each with the diagonal as a the box.
hypotenuse, so you can determine whether the 19. 29 ft. Make a sketch.
parallelogram is a rectangle by seeing whether the
given lengths 8.5 cm, 12 cm, and 15 cm satisfy the
Pythagorean Theorem.
(8.5)2  (12)2  72.25  144  216.25, while (15)2
 225, so the parallelogram is not a rectangle.
Because (8.5)2  (12)2 (15)2, the triangle formed 45°
by the two sides of the parallelogram and the 12 ft

diagonal of length 15 cm is an obtuse triangle,


Because the flagpole makes a 45° angle with the
with the angle between the two sides of the parallel-
ground, the triangle is an isosceles right triangle, so
ogram an obtuse angle. Every parallelogram that is
the vertical leg must also be 12 ft long, and the
not a rectangle has two congruent obtuse angles
length of the diagonal will be 122 ft  17 ft. The
(a pair of opposite angles) and two acute angles
original height of the flagpole is the sum of the
(the other pair of opposite angles). The diagonal
lengths of the vertical leg and the hypotenuse, or, to
opposite an obtuse angle of a parallelogram is
the nearest foot, 12  17  29 ft.
longer than the diagonal opposite an acute angle, so
the 15 cm diagonal must be the longer diagonal of 20.  45 ft. By the Tangent Conjecture, the angle of the
the parallelogram. triangle whose vertex is the point of tangency is a
right angle. Therefore the triangle shown in the

Discovering Geometry Solutions Manual CHAPTER 9 177


©2003 Key Curriculum Press
DG3SM586_136-181.qxd 8/2/06 6:03 PM Page 178

figure is a right triangle with legs of lengths r and


35 ft, and hypotenuse of length (r  12)2. Apply the
Pythagorean Theorem to this triangle and solve for r. 30°
s s
r 2  (35)2  (r  12)2 s 3
2
r 2  1225  r 2  24r  144
1081  24r 60°
_s
1081 2
r  24  45
In each right triangle, s is the length of the
The radius of the tank is approximately 45 ft.
hypotenuse, so by the 30°-60°-90° Triangle Conjec-
21. 50 mi. This is similar to Exercise 20, but you need ture, the length of the shorter leg is 2s and the
to find the length of the tangent segment rather length of the longer leg is 2s  
3 s 3
2 . The
than the radius of the circle. Make a sketch. longer leg is the altitude of the equilateral triangle.
Use the Triangle Area Conjecture and the given
area to find the value of s.
3960 mi x
1
A  2bh
3960 mi
1 s 3
0.34 mi
36 3  2  s  
2
s 23
36 3 
4
Apply the Pythagorean Theorem to the right s2
36  4
triangle in the figure and solve for x.
s 2  144
(3960)2  x 2  (3960  0.34)2
s  12 m
(3960)2  x 2  (3960.34)2
25. 42. As in the solution for Exercise 24, let s represent
x 2  (3960.34)2  (3960)2
the side length of the equilateral triangle. Refer to
x  (3960.
34)2  )2  52
(3960 the sketch in the solution for Exercise 24. In this
case, the height of the equilateral triangle, which
To the nearest 10 mi, the maximum broadcasting
is the longer leg of each 30°-60°-90° triangle, is
radius is 50 mi. s 3
7 3 , so  2  7 3 . Solve this equation to find s.
22. 225 m2  707 m2. Draw the radius of the circle to
s 3

the feet of the diver to form a right triangle. The 2  73
lengths of the legs of this triangle are 20 m and the
s 3  143
radius, while the length of the hypotenuse is 25 m.
Because 20  5(4) and 25  5(5), the side lengths s  14
in this triangle are a multiple of 3-4-5, so the length
P  3s  3(14)  42
of the third side is 5(3)  15. Therefore the radius
of the circle is 15 m. Find the area of the circular The perimeter is 42.
region: A  r 2  (15)2  225  707 m2.
26. No. If you reflect one of the right triangles into the
23. 63 and 18. By the 30°-60°-90° Triangle Conjec- center piece, you’ll see that the area of the kite is
ture, the length of the shorter leg is half the length almost half again as large as the area of each of the
of the hypotenuse and the length of the longer leg triangles.
is the product of the length of the shorter leg and
3 . In this triangle, the length of the shorter leg is Extra
1
(123
2 )  63 , and the length of the longer leg
is (63 )(3 )  (6)(3)  18.
24. 12 m. Let s represent the side length of the equilat-
30°
eral triangle. Draw an altitude (which is also a 30°
median and angle bisector) to form two congruent
30°-60°-90° triangles. 30°

178 CHAPTER 9 Discovering Geometry Solutions Manual


©2003 Key Curriculum Press
DG3SM586_136-181.qxd 8/2/06 6:03 PM Page 179

2 2
Or you might compare areas by assuming the short For any value of s, s s4 because 1 14. Thus the
leg of the 30°-60°-90° triangle is 1. The area of quarter-circle gives the maximum area.
3
each triangle is then 2 and the area of the kite 29. 1.6 m. For reference, redraw the figure in your
is 3   3.
book, label it as shown below, and draw BD.
27. 79. Draw a circle with center O and points A and B 1.4 m B
on the circle so that mAOB  80°. Extend radii
3.9 m
OA and OB  to form diameters AY  and BX
. 1.5 m
A
A
C D ?

First look at right triangle BCA, which represents


the situation before the wire is pulled over the
X
O 80°
B pulley. Because BC  1.5  0.3(5) and AB  3.9 
80°
0.3(13), the side lengths of this triangle are a
multiple of the Pythagorean triple 5-12-13, so AC 
80° 0.3(12)  3.6 m. After the wire is pulled 1.4 m in
the direction of the arrow, the length of wire in the
Y
triangle will be shortened from 3.9 m to 3.9  1.4 
From the figure, you can see that ABC will be an 2.5 m. The new situation is represented by BCD,
 (between X and Y ) and
acute triangle if C is on XY which is also a right triangle. Here, BC  1.5 
will be a right triangle if C is placed exactly at X or 0.5(3) and BD  2.5  0.5(5), so the side lengths
exactly at Y. Therefore, if C is placed anywhere on are a multiple of the 3-4-5 Pythagorean triple, and
major arc XAY (except exactly at X or exactly at Y ), CD  0.5(4)  2.0. The distance that the block
ABC will be an obtuse triangle. mXOY  80° moves, labeled with a question mark in the figure,
by the Vertical Angles Conjecture, so mXY   80° by is 3.6  2.0  1.6 m.
the definition of the measure of an arc. Therefore 30. Draw a segment of any length for one of the legs.
the measure of major arc XAY  360°  80°  Its endpoints will be two of the vertices of the
280°. Thus the probability that ABC will be an isosceles triangle. Bisect this leg to get the base
obtuse triangle if C is selected randomly on the length. Using one endpoint as center, draw an arc
280° 7
360°  9 .
circle is   
whose radius is the base length. Using the other
28. The quarter-circle gives the maximum area. endpoint as center, draw an arc whose radius is the
length of the leg. The point where the two arcs
Triangle: intersect will be the third vertex of the triangle.
Connect the endpoints of your segment to the third
45° vertex to complete the triangle.
s_
___ s
2

45°
1 s s 2
    s4
s_
___
2 A  2
2 2
Square:
_1 s 31. 4; 0; 10. The rule is n2 if n is even, but 0 if n is odd.
2
Draw a regular octagon and a regular nonagon.
_1 s
2
1 1 s2
A  s
2  2s  4
Quarter-circle:

s
1
s  4  2r
2s
r  
First look at the octagon. Any diagonal that passes
 
2
1 2s s2 through the center of the circle is a diameter that
2s_
__
␲ A  4 
  
connects two opposite vertices of the octagon.

Discovering Geometry Solutions Manual CHAPTER 9 179


©2003 Key Curriculum Press
DG3SM586_136-181.qxd 8/2/06 6:03 PM Page 180

Because there are 8 vertices equally spaced around 48. a., b. Use the Minimal Path Conjecture.
the circle, there are 4 pairs of opposite vertices that N
are connected by diameters, so there are 4 diagonals
8 8 Eight ball
that pass through the center.
B
Now look at the nonagon, which has 9 vertices W E
equally spaced around the circle. Here, there are no Cue ball

pairs of opposite vertices, so none of the diagonals A


of the nonagon are diameters of the circle. There-
S
fore there are no diagonals that pass through the
center of the circle.
For a 20-gon, the situation will be similar to the
octagon. In the 20-gon, there are 220  10 pairs of 8

opposite vertices, so there are 10 diagonals that pass


49. A  34 cm2. P  (22  42 ) cm  27.7 cm. First
through the center of the circle.
use the Trapezoid Area Conjecture to find the area.
For a general n-gon inscribed in a circle, if n is 1 1
even, there are n2 pairs of opposite vertices, so A  2h(b1  b2)  2(4)(12  5)  2(17)  34 cm2
n
 diagonals pass through the center. However, if
2 In order to find the perimeter, you must find the
n is odd, there are no pairs of opposite vertices, so
length of the unmarked side of the trapezoid. Draw
0 diagonals pass through the center of the circle.
another perpendicular segment between the bases so
32. 4, or approximately 12.6 in./sec. The fan makes that the trapezoid will be divided into two right
one revolution per second, so the bug goes around a triangles and a rectangle.
circle with radius 2 inches each second. The circum- 12 cm
ference of this circle is 2r  2(2)  4 in., so 4 cm 5 cm 3 cm
the bug travels at a velocity of 4 in./sec, or
approximately 12.6 in./sec. 4 cm 4 cm 5 cm
4 2 cm
33. True 34. True 5 cm

35. False. The hypotenuse has length x 2 .


Because opposite sides of a rectangle are congruent,
36. True the length of the new segment will also be 4 cm.
Now look at the top base of the trapezoid, which is
37. False. AB  (x
2  x
1)  (
2 y2  y 2
1) . divided into three parts. The triangle on the right is
38. False. A glide reflection is a combination of a trans- a right triangle with hypotenuse of length 5 cm and
lation and a reflection. one leg of length 4 cm, so this is a 3-4-5 triangle,
and the length of the other leg is 3 cm. This is the
39. False. Equilateral triangles, squares, and regular
length of the right-hand section of the upper base.
hexagons can be used to create monohedral
The middle section of the upper base is 5 cm
tessellations.
because opposite sides of a rectangle are congruent.
40. True 41. D 42. B 43. A Because the total length of the upper base is 12 cm,
the length of the left-hand section is 12  5  3 
44. C 45. C 46. D
4 cm. This section is one leg of the right triangle
47. (See figure at bottom of page.) on the left, and the other leg also has length 4 cm,
so this is an isosceles triangle. Therefore, by the

Chapter 9 Review, Exercise 47

1
ABCD is 4
⬔D ⬔B
a rectangle
Definition of
Given rectangle

2 3   5 7
ABCD is DA 储 CB ⬔DAC ⬔BCA 䉭ABC 䉭CDA
a parallelogram
Definition of AIA Conjecture SAA Congruence
Definition of parallelogram Conjecture
rectangle
6  
AC AC
Same segment

180 CHAPTER 9 Discovering Geometry Solutions Manual


©2003 Key Curriculum Press
DG3SM586_136-181.qxd 8/2/06 6:03 PM Page 181

Isosceles Right Triangle Conjecture, the hypotenuse 54. 322 ft2. To find the surface area of a cylindrical
of this triangle has length 42 cm. This hypotenuse container with an open top, add the area of one
is the unmarked side of the trapezoid. Add the side circular base and the lateral surface area, and then
lengths in the trapezoid to find its perimeter: multiply by 2 for the other side of the container.
You can use the formula for surface area of a
P  5  5  12  42  22  4
2
cylinder given in Lesson 8.7 of your book if you
 27.7 cm modify it to reflect the fact that there is only one
40

circular face in this case. The radius is 12(5)  2.5 ft,
50. 3 cm2. The shaded region can be called “a sector
and the height is 9 ft.
of an annulus.” Combine what you have learned
about finding the area of an annulus of a circle with SA  r 2  2rh  (2.5)2  2(2.5)(9)
what you have learned about finding the area of a
 6.25  45  51.25  161.0
sector of a circle to find the area of the shaded
region. Notice that the radius of the larger circle is To the nearest square foot, the surface area of one
3  4  7 cm. side of the cylinder is 161 ft2, so the surface area of
both sides of the container is 2(161)  322 ft2.
Aannulus  Aouter circle  Ainner circle
 (7)2  (3)2
TAKE ANOTHER LOOK
1. Demonstrations should include shapes other than a
 49  9  40 cm2 square. (Any regular polygon can be used. In fact,
Asector of annulus    
120°

1

360° (40)  3 (40)  any three similar figures will work.)
2. Demonstrations will vary.
40
 3 cm2
3. The small square in the center has sides with length
51. 25 5 m  55.9 m. Let d represent the diagonal b  a, the slanted square has area c 2, and the trian-
distance across the pool. The pool is rectangular, so gles each have area a2b . The equation c 2  (b  a)2
this diagonal divides the pool into two congruent  4a2b  simplifies to c 2  a 2  b 2.
right triangles. Apply the Pythagorean Theorem to b
one of these triangles to find d, the length of the
hypotenuse of a right triangle whose legs have a
lengths 50 m and 25 m. c

a2  b2  c 2
(50)2  (25)2  d 2
b a
2500  625  d 2
d 2  3125
d  3125
 
625  5  25 5 b a

The diagonal distance across the pool is 255 m  4. Possible proof: Given ABC with BC  x, AC 
55.9 m. x 3 , and AB  2x, construct 30°-60°-90° right
triangle DEF with right angle F, 30° angle D,
52. About 61.5 cm2. Use the Regular Polygon Area and EF  x. DF  x 3 and DE  2x, by the
Conjecture: 30°-60°-90° Triangle Conjecture. ABC  DEF
1 1 by SSS. mC  mF  90°, mA  mD  30°,
A  2asn  2(4.1)(6)(5)  61.5
and mB  mE  60°.
Because the length of the apothem is about 4.1 cm,
the area of the pentagon is about 61.5 cm2. 5. 1
1
1
53. 48 cm. Let d represent the length of the diagonal of 1
the base. Look at the triangle inside the box. The 8 7
space diagonal of the box is the hypotenuse of this 9
6 1
triangle; its legs have lengths 25 cm and d. Because
5
65  5(13) and 25  5(5), d  5(12)  60 cm.
Now look at one of the right triangles on the 4 1
bottom base of the box. In this triangle, the length
of the hypotenuse is d  60 cm, and the lengths of 1 3
the legs are 36 cm and x. Because 60  12(5) and 2 1
36  12(3), x  12(4)  48 cm. 1

Discovering Geometry Solutions Manual CHAPTER 9 181


©2003 Key Curriculum Press
DG3SM586_182-228.qxd 8/2/06 6:07 PM Page 182

27. True
CHAPTER 10
28. False. This statement is true only for a right prism.
LESSON 10.1 29. True 30. True
EXERCISES 31. False. It is a sector of a circle.
1. polyhedron; polygonal; triangles 32. True
2. PQR, TUS 3. PQUT, QRSU, RPTS 33. False. Counterexample:
, PT
4. QU , RS
 5. 6 cm
6. GYPTAN 7. Point E
, YE
8. GE , PE
, TE
, AE
, NE

9. 13 cm 10. C (Square prism) 34. True 35. True
11. D (Square pyramid) 12. B (Cone) 36. Sample answer: The painting “disappears” into the
view out the window. Also, the cone-shaped tower
13. H (Triangular prism) 14. J (Rectangular prism) appears similar to the road disappearing into the
15. M (Hemisphere) 16. I (Trapezoidal prism) distance.
17. L (Hexagonal prism) 18. G (Octagonal prism) 37. (See tables at bottom of page.)
19. E (Sphere) 20. A (Cylinder) Possible answer: The number of lateral faces of an
antiprism is always twice the number for the related
21. J (Rectangular prism) 22. H (Triangular prism) prism. The total number of faces is n more for the
23. 24. antiprism than for the related prism. The number
of edges is n more for the antiprism than for the
related prism. The number of vertices is the same
for each related prism and antiprism.

25. x 26. 38. 8. You can fit two layers of blocks in the box with
x 4 blocks (2  2) in each layer, so the total number
of blocks is 2(4)  8.
2x
2x 39. 60. You can fit five layers of blocks in the box with
12 blocks (3  4) in each layer, so the total number
of blocks is 5(12)  60.

Lesson 10.1, Exercise 37


Triangular Rectangular Pentagonal Hexagonal n-gonal
prism prism prism prism prism
Lateral 3 4 5 6 n
faces
Total 5 6 7 8 n2
faces
Edges 9 12 15 18 3n
Vertices 6 8 10 12 2n

Triangular Rectangular Pentagonal Hexagonal n-gonal


antiprism antiprism antiprism antiprism antiprism
Lateral 6 8 10 12 2n
faces
Total 8 10 12 14 2n  2
faces
Edges 12 16 20 24 4n
Vertices 6 8 10 12 2n

182 CHAPTER 10 Discovering Geometry Solutions Manual


©2003 Key Curriculum Press
DG3SM586_182-228.qxd 8/2/06 6:07 PM Page 183

40. 30. Use one of the 3-by-5 rectangular faces of the 4. 160 cm3  502.65 cm3. Use B  r 2 to find the
large box as the base. You can fit 15 (3  5) small area of the circular base.
boxes on that base. Then the height of the large
box will be 4 cm, so you can fit two layers of V  BH  (r 2)H  (4)2  10  160  502.65
small blocks inside the large box. Therefore the The volume of the cylinder is 160 cm3 
maximum number of small boxes that will fit 502.65 cm3.
inside the large box is 2(15)  30.
5. 36 cm3  113.10 cm3. The solid is half a cylinder,
41. a. Yes b. Yes so each base is a semicircle with radius 3 cm. You
can find its volume either by finding the volume of
␣ ␣ the complete cylinder and dividing by 2 or by using
B  12r 2 to find the area of the base and then
multiplying by the height. These two methods are
c. No d. Yes equivalent.
First method:
 
Vcylinder  (3)2(8)  72
1 1
Vhalf-cylinder  2Vcylinder  2(72)  36
IMPROVING YOUR VISUAL THINKING SKILLS
Second method:

 1
 1

Vhalf-cylinder  BH  2r 2 H  2(3)2 8
9
 2(8)  36

The volume of the half-cylinder is 36 cm3 


113.10 cm3.
90 1
6. 324 cm3  1017.88 cm3.  360  4 of the cylinder
 
3
has been removed, so 4 of the cylinder remains. As
in Exercise 5, you can find the required volume in
either of two equivalent ways.
LESSON 10.2
First method: Find the volume of the complete
EXERCISES cylinder and then multiply by 34.
1. 72 cm3. Use B  bh to find the area of the rectan- Vcylinder  (6)2(12)  432;
gular base.
3
Vthree-quarter cylinder  4(432)  324
V  BH  (bh)H  (4  3)  6  12  6  72
The volume of the rectangular prism is 72 cm3. Second method: Use B  34r 2 to find the area of
the base and then multiply by the height.
2. 24 cm3. Each base of the prism is a right triangle
with hypotenuse of length 5 cm and shorter leg of  3

Vthree-quarter cylinder  BH  4r 2 H
length 3 cm, so each is a 3-4-5 triangle. Therefore
the length of the longer leg is 4 cm. Use B  12bh to  3

 4(6)2  12  27(12)
find the area of the triangular base.  324
1
  1
V  BH  2bh H  2   3  4  4  24 The volume of the three-quarter cylinder is
324 cm3  1017.88 cm3.
The volume of the triangular prism is 24 cm3.
7. a. 480 cm3; V  BH  12bhH 
3. 108 cm3. Notice that the top and bottom faces of
2  6  8  20  480 cm3
1
the solid are not congruent, so these can’t be the
bases of the prisms. However, the front and back b. 1080 cm3; V  BH  12bhH 
2  9  12  20  1080 cm3
faces are congruent trapezoids, so these are the 1
bases, and the height of the prism is 9 cm. Use B 
1
h(b1  b2) to find the area of the trapezoidal base. c. 1656 cm3; V  BH  12bhH 
2
2  8  18  23  1656 cm3
1
1 1
 
V  BH  2h(b1  b2)H  2  3(3  5)  9  108
d. 960 cm3; V  BH  (bh)H 
The volume of the trapezoidal prism is 108 cm3. (6  8)  20  960 cm3

Discovering Geometry Solutions Manual CHAPTER 10 183


©2003 Key Curriculum Press
DG3SM586_182-228.qxd 8/2/06 6:07 PM Page 184

e. 2160 cm3; V  BH  (bh)H  11. 2x 3. V  BH  (bh)H  (x  x)  2x  2x 3.


(9  12)  20  2160 cm3
12. 3r 3. V  BH  (r 2)  H  (r)2  3r  3r 3.
f. 3312 cm3; V  BH  (bh)H 
(8  18)  23  3312 cm3 13. 13x 3. Subtract the volume of the hole from the
volume of the prism.
g. 1040 cm3; V  BH  12h(b1  b2) H
Vprism  3x  5x  x  15x 3
2  8(6  7)  20  1040 cm3
1

h. 2520 cm3; V  BH  12h(b1  b2) H Vhole  x  2x  x  2x 3


2  12(9  12)  20  2520 cm3 Vsolid  15x 3  2x 3  13x 3
1

i. 5589 cm3; V  BH  12h(b1  b2) H 14. Margaretta has room for 0.5625 cord. She should
2  18(8  19)  23  5589 cm3
1 order a half cord.

j. 180 cm3; V  BH  (r 2)  H  (3)2  20  Volume of each box: 2  3  4  24 ft3


180 cm3 Volume of 3 boxes: 3(24)  72 ft3
k. 720 cm3; V  BH  (r 2)  H  (6)2  20  1 cord
72ft3  
720 cm3 128 ft3  0.5625 cord
15. 170 yd3. This is a triangular prism whose bases are
l. 1472 cm3; V  BH  (r 2)  H  (8)2  23  right triangles with base length 17 yd and height
1472 cm3 2 yd.
8. 960 in.3
12 in.
 
1

1

V  BH  2bh H  2  17  2  10  170 yd3

4 in. 16. 5100 lb. First find the volume of one block, which is
a rectangular prism.
24 in. V  (3)(4)(2.5)  30 ft3
Now find the weight of one block.
170 lb
30 ft3 1 ft3  5100 lb
8 in.
17. 11,140 swimming pools. First find the volume of
1

V  BH  2h(b1  b2)  H  one swimming pool, which is a rectangular prism.
1

 2  4(8  12)  24  960 V  (20)(30)(5)  3,000 ft3

The volume of the prism is 960 in.3. Convert 250 million gal to cubic feet.
0.13368 ft 3
9. QT units3 250,000,000 gal    33,420,000 ft3
1 gal
Q Now find the number of swimming pools that
T could be filled with 250 million gal of crude oil.
33,420,000 ft3

3,000 ft3  11,140
V  BH  (r 2)  H  Q
  T  QT
2 Therefore, 250 million gal of crude oil could fill
11,140 swimming pools with the given dimensions.
The volume of the cylinder is QT units3.
18. The volume of the quilt in 1996 was 4000 ft3.
10. Possible answer: The quilt panels were stacked 2 ft 8 in. high.
12
12
Area of 3-by-6-foot panel  18 ft2
12
8 Area of 12-by-12-foot panel  144 ft2
2 3
144 ft2

18 ft2 8, so eight 3-by-6-foot panels can fit
Prism on left: V  BH  (bh)H  (12  12)  2  in 1 ft3 of storage space.
288 cm3 32,000
  4,000, so the quilt’s volume was 4,000 ft3.
8
Prism on right: V  BH  (bh)H  (12  8)  3  4,000 ft2 2
2  2 ft, or 2 ft 8 in.
288 cm3 1,500 ft 3

184 CHAPTER 10 Discovering Geometry Solutions Manual


©2003 Key Curriculum Press
DG3SM586_182-228.qxd 8/2/06 6:07 PM Page 185

19. 20.
r OM  SM  6, and SMO is an isosceles right
triangle. By the Isosceles Right Triangle Conjecture,
r OS  6 2 . Then, OP  62 because OS and OP
r
are both radii of the circle. Because the diagonals of
the square bisect each other, OA  2(6)  12, so
PA  OA  OP  12  6 2.
21. True
PROJECT
22. False. If you cut off a corner, then the section is a
triangle. If you cut through the midpoints of two Project should satisfy the following criteria:
adjacent edges of the six faces, then the section is ● The seven unique pieces are built.
a regular hexagon. The section can also be a
pentagon. ● The project shows at least one of the 240 distinct ways
to arrange the seven pieces into a cube. (The number
exceeds 1,000,000 if rotations are included.)

23. 24. Possible answer: prism

Salt crystal
Cube
25. Approximately 1.89 m. The
six points on the circular ● Solutions are given for the four shapes pictured in
20 m the book.
track form the vertices of a
regular hexagon inscribed in 20 m
the circle.
The length of Ben’s path is 26  13
of the circumference of the circle, so this is an arc
length on the circle: The length of Ben’s path 
C  (2r)  (2  20)   m.
1 1 1 40
3 3 3 3
The length of Al’s path is the sum of the lengths of
two sides of the hexagon. The side length of an
inscribed hexagon is equal to the radius of the Tunnel
Sofa
circle. The length of Al’s path  2r  2(20)  40 m.
Difference between length of Ben’s path and length
40
of Al’s path  3  40  1.89 m.


, OT
26. 12  62. Draw OS , S A
.
and MT 6 P

6
OSA and OTA are right M
O T
angles by the Tangent Conjec-
ture, so SOTA is a rectangle
because it is a quadrilateral
with four right angles. Also, Castle Winner's Podium
OS  OT because OS  and OT  are radii of the same
● An interesting shape is built and drawn.
circle. Because opposite sides of a rectangle (or any
parallelogram) are congruent, this means that SOTA Extra credit
is a square because it has four congruent sides and ● The student completes research on the number and
four congruent (right) angles. Because SOTA is a strategies of solutions.
 and OA
square, its diagonals, ST , are congruent and
are perpendicular bisectors of each other. Thus,

Discovering Geometry Solutions Manual CHAPTER 10 185


©2003 Key Curriculum Press
DG3SM586_182-228.qxd 8/2/06 6:07 PM Page 186

EXTENSIONS m3
cm3. V  13BH  13  m2  m  m3. The volume
3
7. 
3 3
A. Research results will vary. of the square pyramid is m3 cm3.
B. Results will vary. 8. 23b 3 cm3. V  13BH  13(r 2)H  13(b 2)(2b) 
2 2
b 3. The volume of the cone is b3 cm3.
3 3
LESSON 10.3
9. 324x 3 cm3; 29.6%. First find the volume of the
EXERCISES cone.
1 1 1
1. 192 cm3. Use the Pyramid-Cone Volume Conjecture. V  3BH  3(r 2)H  3(9x)2(12x)
1 1
V  3BH  3(8  8)  9  192 1
 3(81x 2)(12x)  324x3

The volume of the square pyramid is 192 cm3. The volume of the cone is 324x 3 cm3.

2. 84 cm3. Use the Pyramid-Cone Volume Now find the volume of the portion of the cone
Conjecture. that is filled with the liquid. This portion forms a
smaller cone with radius 6 cm and height 8x.
1 1 1
V  3BH  3(r 2)H  3((6)2  7)  84 1 1 1
V  3BH  3(r 2)H  3(6x)2(8x)
The volume of the cone is 84 cm3. 1
 3(36x 2)(8x)  96x 3
3. 150 cm3. Use the Pyramid-Cone Volume Conjecture.
The volume of the portion of the cone filled with
Use the formula B  12h(b1  b2) to find the area of
the liquid is 96x 3 cm3.
the trapezoidal base.
96x 3
 
1 1 1 96 8
V  3BH  3 2h(b1  b2)  H 3      0.296 or 29.6%
324x 324 27


1 1

 3 2  5(8  4)  15  150
Therefore about 29.6% of the volume of the cone is
filled with liquid.
The volume of the trapezoidal pyramid is 150 cm3.
10. a. 120 cm3; V  13BH  1312bhH 
  6  6  20  120 cm3
4. 60 cm3. The base of the pyramid is a right triangle 1 1
3 2
with hypotenuse of length 13 cm and shorter leg of
length 5 cm. Because 5-12-13 is a Pythagorean b. 240 cm3; V  13BH  1312bhH 
  9  8  20  240 cm3
1 1
triple, the length of the longer leg is 12 cm. 3 2
c. 884 cm3; V  13BH  1312bhH 
1
 
1 1 1 1

V  3BH  3 2bh H  3 2  5  12  6  60   13  17  24  884 cm3
1 1
3 2
The volume of the triangular pyramid is 60 cm3.
d. 240 cm3; V  13BH  13(bh)H 
(6  6)  20  240 cm3
5. 84 cm3. The volume of a semicircular cone is half 1
3
the volume of a complete cone with the same radius
and height. e. 480 cm3; V  13BH  13(bh)H 
(9  8)  20  480 cm3
1

   
1 1 1 1 3
V  2 3BH  2 3(r 2)H
f. 1768 cm3; V  13BH  13(bh)H 
(13  17)  24  1768 cm3
 
1 1 1
 2 3  (6)2  14  84 3
g. 260 cm3; V  13BH  1312h(b1  b2)  H 
The volume of the semicircular cone is 84 cm3.
  6(6  7)  20  260 cm3
1 1
3 2
6. 384 cm3. The volume of a cylinder with a cone
3 cm ; V  3 BH  3  2 h(b1  b2)  H 
2480 1 1 1
removed is the difference between the volume of the h.   3   

  8(9  22)  20  3 cm3


1 1 2480
cylinder and the volume of the cone. Notice that the  
3 2
cylinder and the cone have the same radius and the
i. 2856 cm3; V  13BH  1312h(b1  b2)  H 
same height.
  17(13  29)  24  2856 cm3
1 1
1 2 3 2
Vsolid  Vcylinder  Vcone  BH  3BH  3BH
j. 60 cm3; V  13BH  13(r 2)H 
2 2 2
 3(r 2)H  3((6)2)  16  3(36)(16) ((3)2)  20  (9)(20)  60 cm3
1 1
3 3
 384
The volume of the solid is 384 cm3.

186 CHAPTER 10 Discovering Geometry Solutions Manual


©2003 Key Curriculum Press
DG3SM586_182-228.qxd 8/2/06 6:07 PM Page 187

k. 240 cm3; V  13BH  13(r 2)H  14. 78,375 grams. First use the formula for the area of a
((6)2)  20  (36)(20)  240 cm3 pentagon to find the area of the base.
1 1
3 3
1 1
l. 512 cm3; V  13BH  13(r 2)H  B  2asn  2(27.5)(40)(5)  2,750 cm2
((8)2)  24  (64)(24)  512 cm3
1 1 Now find the volume of the pentagonal prism.
3 3
1 1
11. V  13M 2H ft3 V  3BH  3(2,750)(30)  27,500 cm3
Finally, find the weight of the sculpture.
H
2.85 g
27,500 cm3   
1 cm2  78,375 g
M
M The weight of the sculpture is 78,375 grams.

12. Possible answer: 15. 48 in.3. Draw the solid for this net.

Tall cone: V  13BH  13(r 2)H  13((16)2)  27 


2304 cm3
8
Short cone: V  13BH  13(r 2)H  13((48)2)  3  6

2304 cm3
6

The solid is an oblique triangular prism.


1 1 1
  1 1

V  3BH  3 2bh H  3 2  6  6   8  48 in.
27
3
3

16 48 The volume is 48 in.3, so the container will not hold


50 in.3.
13. Mount Etna is larger. The volume of Mount Etna is
approximately 2193 km3, and the volume of Mount 16. 4 units3. If the triangle is rotated about the x-axis,
Fuji is approximately 169 km3. First convert the the side of the triangle that lies along the y-axis will
heights of both volcanoes from meters to kilometers sweep out a circle with radius 2 units, and the solid
by dividing by 1000. formed will be a cone with radius 2 units and
height 3 units.
Height of Mount Fuji  3776 m  3.776 km 1 1
V  3r 2H  3(2)2  3  4 units3
Height of Mount Etna  3350 m  3.35 km
17. 144x 3 cm3. The volume of the liquid is half the
Because Mount Fuji has a slope of 30°, you can volume of the rectangular prism.
find its radius by using the 30°-60°-90° Triangle 1 1
Conjecture. V  2BH  2((12x)(4x)(6x))  144x 3 cm3
18. 40,200 gal; 44 hr 40 min. The two congruent
3.776 km parallel faces are the pentagons at the front and
30° back of the solid, so these are the faces and the
r solid is a pentagonal prism. Notice that the
pentagons are not regular, so you can’t use the
The height of the volcano is the shorter leg of the
formula for the area of a regular polygon. Instead,
triangle (opposite the 30° angle), so the radius of
subdivide one of these pentagons into a trapezoid
the base of the volcano, which is the length of the
and a rectangle.
longer leg, can be found by multiplying the length
of the shorter leg by 3 . Thus, for Mount Fuji, 17 ft 13 ft
4 ft
r  3.776 3 km. For Mount Etna, the diameter is
given as 50 km, so r  25 km. 14 ft 14 ft

Find the volumes of the volcanoes, each of which is 13 ft


a cone. 1
1 1 Area of trapezoid  2h(b1  b2)
VMount Fuji  3BH  3r 2H
1
1 2  2  17(4  14)  153 ft2
 33.7763 (3.776)  169 km3
Area of rectangle  bh  13  14  182 ft2
1 1
VMount Etna  3BH  3r 2H
1
 3(25)2(3.35)  2193 km3

Discovering Geometry Solutions Manual CHAPTER 10 187


©2003 Key Curriculum Press
DG3SM586_182-228.qxd 8/2/06 6:07 PM Page 188

B  area of trapezoid  area of rectangle b. 96 cm2. A regular hexahedron is a cube. Its six
faces are congruent squares. The edges of the
 153  182  335 ft2
hexahedron are the sides of the squares, so the
V  BH  335  16  5360 ft3 area of each face is 16 cm2, and the surface area
of the hexahedron is 6(16)  96 cm2.
Convert this volume from cubic feet to gallons.
7.5 gal c. 803 cm2. A regular icosahedron has 20 faces
5,360 ft3   
1 ft3  40,200 gal that are congruent equilateral triangles. From
Now find how long it will take to fill the pool. 21a, the area of an equilateral triangle with side
1 min length 4 is 43 cm2, so the surface area of the
40,200 gal   
15 gal  2,680 min icosahedron is 204 3   80 3 cm2.
Finally, convert this time to hours and minutes.
1 hr 2
d. (2413  120) cm2. Add the areas of the four
2,680 min    
60 min  443 hr, or 44 hr 40 min congruent rectangular faces and the areas of the
eight congruent triangular faces. The area of each
19. About 71 ft3. The retaining wall is a trapezoidal
rectangle is 5  6 = 30 cm2. The triangular faces
prism.
of this polyhedron are the lateral faces of two
1 1
V  BH  2h(b1  b2)H  2  31(18  48)(120) pyramids. Sketch a right triangle inside the solid
whose right-angle vertex is at the center of the
 122,760 in.3
square, with one leg that is an altitude of the
Convert this volume from cubic inches to cubic feet. pyramid and with the other leg that is an
apothem of the square.
1 ft3  (12 in.)(12 in.)(12 in.)  1,728 in.3
1 ft3
122,760 in.3  
1,728 in.3  71 ft
3

20. 403 barrels. First find the volume of one barrel in 13 cm 3 cm


cubic feet.
V  r 2H  (0.8)2(2.8)  1.792 ft3 2 cm

Convert this volume to gallons. By the Pythagorean Theorem, the length of the
7.5 gal 22  32 
1.792 ft3   
1 ft3  13.44 gal
hypotenuse of this right triangle is 
 13 cm. This length is the slant height of the
Therefore one barrel holds 13.44 gal of oil sludge. pyramid and also an altitude of the triangular
Now find the number of barrels needed to hold faces. Now find the area of each triangular face.
17,000 gal of sludge. The bases of these triangles are the edges that
1 barrel are sides of a square with side length 6 cm.
17,000 gal   
13.44 gal  402.6 Therefore the area of each triangle is 12bh 
1
The number of barrels must be an integer, so round (6)
2 13   3 13 cm2.
up to 403 barrels. The total surface area of the polyhedron is
4(30)  83 13   120  24 13  cm2.
21. If you are not familiar with the polyhedrons
described in 21a–c of this exercise, see the pictures 22. Possible answer: From the D
A
D
on page 528 of your book. properties of reflection,
31
1  3 and 2  4.
a. 163 cm2. A regular tetrahedron has four faces 2
m1  m2  90°, so C 4 B
that are congruent equilateral triangles. The edges
m3  m4  90°, and
of the tetrahedron are the sides of the triangles.
m1  m2  m3  D
Sketch one of the faces.
m4  180°. Therefore D,
C, and D are collinear.

4 cm
30°
4 cm 23. a. Y(a  c, d)
2 3 cm b. Y(a  c, b  d)
60°
2 cm c. Y(a  c  e, b  d  f )
1 1
The area of one face is 
bh
2 2 
(4)23

4 3 cm2, so the surface area of the tetrahedron
is 443   163 cm2.

188 CHAPTER 10 Discovering Geometry Solutions Manual


©2003 Key Curriculum Press
DG3SM586_182-228.qxd 8/2/06 6:07 PM Page 189

PROJECT 4. 11 cm. Let h represent the height of the trapezoidal


Project should satisfy the following criteria: base.

● The volume of the pyramid at Cholula:


volume of bottom section  162,374,66613 ft3,
1 1

V  BH  3 2h(b1  b2) H 
volume of top section  14,511,100 ft3, total
volume  176,885,66613 ft3  6.6 million yd3.
1 1

3168  3 2h(20  28)   36
● The volume of the Great Pyramid at Giza: V  3168  288h
1
(756)2(481)  91,636,272 ft3  3.4 million yd3. 11  h
3
● Accurate models are built using the same scale for The height of the trapezoidal base is 11 cm.
both pyramids.
5. 5.0 cm. Substitute the given values in the formula
EXTENSION for the volume of a cylinder.
If the dimensions of the base are doubled, the volume V  BH  r 2H
will be multiplied by 4. If all the dimensions are
doubled, the volume will be multiplied by 8. 628  r 2(8)
r 2  78.5
LESSON 10.4
78.5
r 2  
EXERCISES
1. 58.5 in.3. Make a sketch
of the piece of paper 1 in.
1 in.
r 

78.5

  5.0
with the corners cut out. The radius of the base is about 5.0 cm.
When the paper is folded,
6. Find the volumes of the two cylinders:
the dimensions of the
base will be 9 in. by 6.5 in.
9 in. 11 in.
  82.5 2  11  63.24 in.3;   2112  8.5 
because 1 in. has been 81.85 in.3. Thus, the short, fat cylinder has
taken off both ends of 6.5 in. greater volume.
both the length and the
8.5 in. 7. 257 ft3. For the circular base, C  2r  44 ft, so
width. Because the paper
r  24
4
 22 ft.
is folded up along the dashed lines shown in the

   5  257 ft
figure above, the height of the box is 1 in. 1 1 22 2
V  3r 2H  3  3
V  BH  (9)(6.5)(1)  58.5 in.3
8. 4 cm. The smallest face measures 6 cm by 12 cm, so
2. 323 cm3. First find the area of the triangular base.
if the container sits on this face, its height will be
The altitude divides the triangle into two congruent
15 cm. If it is filled with water up to 5 cm from the
30°-60°-90° right triangles.
top, then it is filled with water to a depth of 10 cm.
Find the volume of the water.
4 cm
30°
4 cm V  BH  6  12  10  720 cm3
2 3 cm
The largest face of the container measures 12 cm by
60°
2 cm
15 cm. Use the volume that you have calculated to
find the height of the water if the container sits on
1 1

Area of base: B  2bh  2 (4)23  43 cm2 the base.
Now find the volume of the pyramid. V  BH
V  BH  4
3 (8)  32
3 cm3 720  12  15  H  180H
3. 15 cm. Let b represent the side length of the triangle H4
to be found.
If the container sits on its largest face, the height of
1 1

V  3 2b  6   12 the water will be 4 cm.

180  12b
b  15, so the length of the side is 15 cm.

Discovering Geometry Solutions Manual CHAPTER 10 189


©2003 Key Curriculum Press
DG3SM586_182-228.qxd 8/2/06 6:07 PM Page 190

9. He must refute the statement. To find the area of By the 30°-60°-90° Triangle Conjecture, the length
the base of each prism, use the formula B  12aP of the apothem is 323 ft. The perimeter is 6(3) 
for the area of a regular polygon. 18 ft.

 
1

1 3
V  BH  2aP H  2  23  18  3 
81
 23 ft3

3 3 cm Convert this volume to gallons.


30° 2 3 cm
81 7.5 gal
3 ft3  
6 cm
1 ft3  526 gal
30° 4 cm 2
60°
60° Because 526 is approximately halfway between 500
3 cm 2 cm and 550, it will take about 13 min to raise the water
6 cm 4 cm
temperature from 93°F to 103°F.
Outer hexagon Missing hexagon
13. Any values of r and H such that r 2H  14.4 in.3.
Vlarger prism
1
  1
 BH  2aP H  2(3
3)  36  2 The volume of the juice box is

 1083 cm3 1.8 in.3


(8 oz)   
1 oz  14.4 in.
3

Vmissing prism
1
  1
 BH  2aP H  2(2
3)  24  2 so the volume of the can should be the same. Using
the volume formula for a cylinder, this means that
 48
3 cm3 r 2H  14.4 in.3. Any values of r and H satisfying
this equation will work. Choose a reasonable value
Vring  Vlarger prism  Vmissing prism
for the can’s height, such as 4 in., and solve the
 1083  48
3  60
3 cm3 equation to find r  1.07 in.
Therefore the volume of the ring is not equal to the 14. Approximately 38 in.3. If you ignore the thickness of
volume of the hole, so the claim is false. the plastic, the ice tray is two halves of a cylinder
with radius 1 in. and height 12 in.
10. 1502 lb. First convert 8 in. to 23 ft and find the
volume of the mattress in cubic feet. V  r 2H    12  12  12 in.3  38 in.3
V  BH  (5.5  6.5)23  
71.5 3

3 ft
15. 8.2 cm. Connect the center of the
square base to one of the vertices 10 cm
71.5
Now find the weight of 
3 ft3 of water. of the base. Let h represent the h
71.5 63 lb height of the pyramid and x
 ft3  
3 1 ft3  1501.5  1502 represent the distance from the x
8 cm 8 cm
center to the vertex. The segment
The water in the waterbed weighs about 1502 lb.
of length x is half of a diagonal of
11. 192.4 gal. First convert 8 in. to 23 ft and find the the square. By the Isosceles Right Triangle
volume of the water in the wading pool in cubic Conjecture, the length of the diagonal is 82 cm,
feet. The pool is a cylinder with radius 3.5 ft and so x  4 2 cm. The segments of lengths x and h
height 23 ft. are the legs of a right triangle with hypotenuse

 23  
24.5 3 10 cm. Apply the Pythagorean Theorem to this
V  r 2H  (3.5)2 
3 ft triangle and solve for h.
Convert this volume to gallons. h 2  x 2  (10)2
24.5 7.5 gal
 ft3  
3 1 ft3  192.4 gal 2 2  100
h 2  4
To the nearest 0.1 gallon, the wading pool will hold h 2  32  100
192.4 gal of water.
h 2  68
12. 13 min. First look at the base of the hot tub. The
h  
68  8.2
base is a regular hexagon with side length 3 ft.
The height of the pyramid is about 8.2 cm.
16. x  140°. By the Inscribed Angle Conjecture,
_3
  2mBAD  2(80°)  160°. Then,
mBCD
3
a 2 3
  mBCD
mCD   mBC   160°  120°  40°.
_3
2
Now look at DEC. By the Vertical Angles
Conjecture, mDEC  x. Draw radii OC  and OD
.

190 CHAPTER 10 Discovering Geometry Solutions Manual


©2003 Key Curriculum Press
DG3SM586_182-228.qxd 8/2/06 6:07 PM Page 191

Look at quadrilateral B 120° c. Never. ABCD can’t be a kite because two pairs of
OCED, which is a kite opposite sides of ABCD are congruent, while in a
because OC  OD kite, two pairs of consecutive sides are congruent.
(radii of the same circle) O C Also, you know from 19a that ABCD is a parallel-
A 80°
  ED
and EC  (Tangent ogram, and a kite is not a parallelogram.
E
Segments Conjecture).
 and OD
 D

d. Sometimes. AMD  AMB only if AMD 
Because OC
AMB. Because these angles form a linear pair,
are radii drawn to points
  DE and OC
  CE  (Tangent they can be congruent only if they are right
of tangency, OD
angles, which means that the diagonals would
Conjecture), so ODE and OCE are right angles.
COD is a central angle of the circle and CD  is its have to be perpendicular. This happens only

intercepted arc. You found earlier that mCD  40°,
when the parallelogram is a rhombus (which
could be a square). From 19b, you know that
so mCOD  40° (definition of the measure of an
ABCD might be a rhombus but is not necessarily
arc). Now apply the Quadrilateral Sum Conjecture
a rhombus.
to kite OCED: 90°  40°  90°  mDEC  360°,
so mDEC  140°. mDEC  x, so x  140°.  BC
e. Always. DA  because opposite sides of a
parallelogram are congruent. CA is a transversal
Another way to approach this exercise is to notice
to these parallel lines, so DAM  BCM by
that DEC is formed by two tangents intersecting
the AIA Conjecture.
outside the circle. The measure of an angle formed
in this way is half the difference between the mea- IMPROVING YOUR REASONING SKILLS
sures of the intercepted arcs. (This is the same rule From the fact that A  B  3  B  12  10, you can
as for an angle formed by two secants intersecting find A  19, so A  7  1  11 gives the magic sum 38.
outside a circle in Lesson 6.6, Exercise 9.) Use the fact that each letter appears at least once in a sum
1  ) with only numbers and itself to find the other values:
x  mDBC  2(mDBC  mCD
B  16, C  13, D  15, E  14, F  18, and G  17.
1 1
 2((360°  40°)  40°)  2(320°  40°)
3
1 19 17
 2(280°)  140° 16 7 18
2 1
  EC
17. AB  because the opposite sides of a parallelo- 12
4
5
6
11

gram are congruent. EC   BD  because the diago- 10 8 9


13 14
nals of a rectangle are congruent. So AB   BD 15
because both are congruent to EC . Therefore
ABD is isosceles. EXTENSION
18. Possible construction: Results will vary.

D x C LESSON 10.5
45° 45°
x x EXERCISES
45° 45° 1. 675 cm3. The new “slice” of water has a volume of
A x x x B 15  15  3  675 cm3, so the volume of the rock is
3x
675 cm3.
2. 36 cm3. The volume of the displaced water is
equal to the volume of a cylinder with radius 6 cm
19. a. Always. If the diagonals of a quadrilateral and height 1 cm.
bisect each other, then the quadrilateral is a V  BH  r 2H  (6)2(1)  36 cm3
parallelogram.
3.  47 in.3. This situation is the reverse of displace-
b. Sometimes. From 19a, ABCD is a parallelogram, ment because the goldfish is removed from the
and a rhombus is a parallelogram. There is not tank. The decrease in volume when the fish is
enough information to determine that all four removed is equal to the volume of a rectangular
sides of ABCD are congruent and also no infor- prism with base 10 in. by 14 in. and height 13 in.
mation that would mean that all four sides are
not congruent. Therefore ABCD might be a
1

V  BH (10  14) 3  47
rhombus but is not necessarily a rhombus.
To the nearest cubic inch, Columbia’s volume is
47 in.3.

Discovering Geometry Solutions Manual CHAPTER 10 191


©2003 Key Curriculum Press
DG3SM586_182-228.qxd 8/2/06 6:07 PM Page 192

4. 1798.4 g. First find the volume of the aluminum 9. The volume of the medallion is 160 cm3. Yes, it is
block, which is a rectangular prism. gold, and the Colonel is who he says he is.
V  BH  (4  8)  20  640 cm3 Vdisplacement  (10  10)(54.6  53)  100(1.6)
From the table on page 535 in your book, the  160 cm3
density of aluminum is 2.81 g/cm3. mass 3088 g
density    
volume  160 cm3  19.3 g/cm
3
mass
density   
volume The table shows that the density of gold is
mass  volume  density 19.3 g/cm3.
60 1
2.81 g 10. 679 cm3. The slice removed is     of the
 640 cm3   
1 cm3  1798.4 g 1 360 6
cylinder, so its volume will be 6 of the volume of
5. The gold weighs 2728.4 g, and the platinum weighs a cylinder with radius 6 cm and height 36 cm.
7529.8 g. The solid cone of platinum weighs more. 1 1
V  6(r 2H)  6    (6)2  36  679
Find the volume and then the mass for the cylinder
and the cone. To the nearest cubic centimeter, the volume of the
slice is 679 cm3.
Gold cylinder:
11. Approximately 194 lb; 22 fish. 8 in.
V  r 2H  (3)2(5)  45 cm3
19.30 g First find the volume of the
mass  volume  density  45 cm3   
1 cm3  aquarium. Look at one of its
2728.4 g regular hexagonal bases.
Platinum cone:
1 1
 
1
V  BH  2asn H
8 in.
30° 4 3 in.

V  3r 2H  3(4)2(21)  112 cm3


   24
60°
1
21.40 g  2  4 3 86 4 in.
mass  volume  density  112 cm3   
1 cm3  8 in.
7529.8 g  2304
3

6. 1.5 cm. First find the volume of the displacement. The volume of the aquarium is 23043 in.3.
From the table on page 535, the density of sodium Convert this volume to cubic feet.
is 0.97 g/cm3. 1 ft3  12 in.  12 in.  12 in.  1728 in.3
mass
density    23043 in.3  
1 ft3 4

1728 in.3  3 3 ft
3
volume
mass 145.5 g
volume    
density  0.97 g/cm3  150 cm
3
Now find the weight of the water.
4 63 lb
Also, V  BH  (10  10)  H  100H. Therefore 3 ft3  
3 1 ft3  145 lb
100H  150, or H  1.5 cm. The aquarium itself weighs 48 lb, so its total weight
7. 10.5 g/cm3; silver. First find the volume of the when filled with water is about 145  48  194 lb.
displaced water. Finally, find the number of fish that this aquarium
V  BH  (5  5)  2  50 cm3 can house.
1 fish
Now divide the mass by the volume to find the 2304 3 in.3  
180 in.3  22
density. The aquarium can house 22 fish.
mass 525 g
density    
volume  50 cm3  10.5 g/cm
3
12. 923. Recall that the centroid is the point of
From the table, the metal is silver. concurrency of the three medians of a triangle.
Because AE  passes through M, AE  is the median to
8. 8000 cm3. First find the volume of the displaced BC. In an equilateral triangle, every median is also
water. an altitude. Look at CEA. CB  AB because
V  BH  (35  50)  4  7000 cm3 ABC is equilateral, and CE  12(CB) because AE  is
a median. Therefore CE  6. Because CEA is a
Because 78 of the volume of the ice floats below the 30°-60°-90° triangle, AE (length of longer leg) 
water level, 3 3 . Because AE  CE, area of CEA  1bh 
2
7 1
(3)3
9
3   2
Vdisplacement  8Vblock of ice 2 3.
8 8
Vblock of ice  7Vdisplacement  7(7000)  8000 cm3

192 CHAPTER 10 Discovering Geometry Solutions Manual


©2003 Key Curriculum Press
DG3SM586_182-228.qxd 8/2/06 6:07 PM Page 193

13. Paragraph Proof: S  R because SP  RQ .


First find the perpendicular bisector of AB
 
(AIA Conjecture). SM  MR because it is given. x1  x2 y1  y1
SMP  RMQ because they are vertical angles. Midpoint of AB   2 , 2
SMP  RMQ by ASA. MP   MQ because
 2, 2  (5, 5)
46 73
they are corresponding parts of congruent triangles.
Thus M is the midpoint of PQ because MP  MQ .
y2  y1 3  7 4

14. 15 sides. Look at a vertex that is common to all Slope of AB x2  x1    
6  4  2  2
three regular polygons. One of these polygons is The slope of the perpendicular bisector of AB  is
a regular decagon and another is an equilateral the negative reciprocal of 2, which is 12, and it
triangle. Recall that the measure of each interior passes through the midpoint, (5, 5). Find the
(n  2)  180°
angle of a regular n-gon is  n , so the equation in the form y  mx  b for this line.
measure of each interior angle of the decagon is y5 1
8  180°   
  144°. The measure of each interior x5 2
10
angle of an equilateral triangle is 60°. Let x 2(y  5)  1(x  5)
represent the angle measure of an interior angle 2y  10  x  5
in the red polygon. The sum of the angle
2y  x  5
measures around a vertex is always 360°, so
1 5
x  360°  144°  60°  156°. Then find the y  2x  2
number of sides in a regular polygon in which Now follow the same steps to find the perpen-
each interior angle measures 156°. dicular bisector of BC .
(n  2)  180° 6  1 3  (2)
 n  156° Midpoint of BC    2
,   , 
2    7 1
2 2
(n  2)  180°  156°  n  2  3  5
Slope of BC      1
16 5
180°  n  360°  156°  n The slope of the perpendicular bisector of BC  is
24°  n  360° the negative reciprocal of 1, which is 1, and it
n  15 passes through the midpoint, 72, 12. Find the
equation in the form y  mx  b for this line.
Therefore the red polygon has 15 sides. 1
y  2
15. a. (1, 3). Plot and label the three given points and   1
7
sketch a circle passing through them. (You will x  2
not yet know exactly where the center is.)
Connect the three points to form a triangle  1
2 y  2 
inscribed in the circle.   1
y
A (4, 7)
 7
2 x  2 
2y  1
  1
2x  7
B (6, 3)
2y  1  1(2x  7)
x
2y  1  2x  7
C (1, –2)
2y  2x  8
y  x  4
The center of the circumscribed circle is the To find the intersection point of the two perpen-
circumcenter of the triangle. Recall that the dicular bisectors, solve the system formed by the
circumcenter of a triangle is the point of concur- two equations that you have found.
rency of the three perpendicular bisectors of the


1 5
sides of the triangle. Therefore you can find the y  2x  2
coordinates of the circumcenter by finding equa-
y  x  4
tions for any two of the three perpendicular
bisectors and then solving a system of equations
to find the intersection point of the two perpen-
dicular bisectors.

Discovering Geometry Solutions Manual CHAPTER 10 193


©2003 Key Curriculum Press
DG3SM586_182-228.qxd 8/2/06 6:07 PM Page 194

To solve this system by substitution, substitute LESSON 10.6


x  4 for y in the first equation.
EXERCISES
1 5
x  4  2x  2 1. 36 cm3. Use the Sphere Volume Conjecture with
r  3 cm.
1
 5
2(x  4)  2 2x  2  4 4 4
V  3r 3  3(3)3  3  27  36
2x  8  x  5
The volume of the sphere is 36 cm3.
3x  3

2. cm3. Use the Sphere Volume Conjecture with
x1 6
r  12 cm.
Substitute 1 for x in the second equation of the
   43  18  16  6
4 4 1 3
system and solve for y. V  3r 3  3 2
y  x  4  1  4  3 The volume of the sphere is 
 cm3.
6
Therefore the center of the circle through A, B, 9
and C is (1, 3). 3. 
32 cm3. The solid is a hemisphere with r  34 cm.

   23  2674  39


2 2 3 3 9
b. (x  1)2  (y  3)2  25. To find the equation of V  3r 3  3 4 2 

32
a circle, you need to know its center and radius.
You found that the center is (1, 3) in 15a. The 9
The volume of the hemisphere is 
32 cm3.
radius is the distance between the center and any
point on the circle. Let P represent the center of 4. 720 cm3. This solid can be called a capsule. It is
the circle. Use the distance formula to find PA. made up of a cylinder with a hemisphere attached
to each base of the cylinder. The radius for both the
PA  (x
 1 x 
2)  (y
2
2  y
1)
2
cylinder and the hemispheres is 6 cm, and the
 
(4  1
)2  (7
 3)2 height of the cylinder is 12 cm.

 3
2  42  25  5 Vcapsule  2Vhemisphere  Vcylinder
 
 is a radius of the circle, so r  PA  5.
PA 2

 2 3    (6)3  (  (6)2  12)
Finally, substitute 1 for h, 3 for k, and 5 for r in
4
the equation of a circle.  3  216    36  12
(x  h)2  (y  k)2  r 2  288  432  720
(x  1)2  (y  3)2  (5)2 The volume of the capsule is 720 cm3.
(x 1)2  (y  3)2  25 5. 30 cm3. The solid is made up of a hemisphere
16. 58; 3n  2. In each case, the function value is 2 less sitting on top of a cone. To find the volume of this
than 3 times the number, so 20 → 3(20)  2  58, solid, add the volume of the cone to the volume of
and n → 3n  2. the hemisphere. To find the height of the cone, look
at the right triangle in which the length of the
PROJECT shorter leg is 3 cm (the radius of the cone) and the
Project should satisfy the following criteria: length of the hypotenuse is 5 cm (the slant height
of the cone). This is a 3-4-5 triangle, so the length
● The equation y  x(10  2x)(10  2x) is graphed (on of the longer leg, which is the height of the cone,
a calculator, in a window such as [0, 6, 1, 0, 80, 10]). is 4 cm.
1 1 1
● An interpretation is given of the x-intercepts when the Vcone  3BH  3(r 2)H  3    (3)2  4
volume is zero.
 12 cm3
● The maximum point at (1.67, 74.07) is explained as
2 2 2
representing a maximum volume of 74.07 in.3 when Vhemisphere  3r 3  3(3)2  3  27  18 cm3
the cut is 1.67 in.
The volume of the solid is 12  18  30 cm3.
Extra credit
● In general, the maximum volume occurs when the cut
equals 16 of the side length. For a 12-inch-square sheet,
the cut should be 2 inches to maximize the volume.

194 CHAPTER 10 Discovering Geometry Solutions Manual


©2003 Key Curriculum Press
DG3SM586_182-228.qxd 8/2/06 6:07 PM Page 195

40° 1
360°  9 of the hemisphere is missing,
6. 3456 cm3.    11. They have the same volume.
8
so 9 of the hemisphere remains. 2 2 16
Vhemisphere  3r 3  3(2)3  3 cm3

8 2
 16
V  9 3r 3  2
7    (18)3  1267    5832 1
  2 16
Vcones  2 3r 2H = 3  (2)2  2 = 3 cm3
 3456
12. 9 in. Substitute 972 for V in the formula for the
The volume of the solid is 3456 cm3. volume of a sphere and solve for r.
7. 18 m3. The radius of the hemisphere can’t exceed 4
V  3r 3
the smallest dimension of the block, which is 3 m,
4
so find the volume of a hemisphere with radius 972  3r 3
3 m. 4
2 2 2 972  3r 3
V  3r 3  3(3)2  3    27  18
The volume of the largest hemisphere that you
3 3 4
(972)   r 3
4 4 3  
could carve out of the block is 18 m3. 729  r 3
3
8. No. The volume of the ice cream is 85.3 cm3, and r 9
729
the volume of the cone is 64 cm3. The radius of
The radius of the sphere is 9 in.
both the cone and the sphere is 4 cm.
4 4 4 256 13. 3 cm. Substitute 18 for V in the formula for the
Vsphere  3r 3  3(4)3  3    64  3
volume of a hemisphere and solve for r.
 85.3 cm3 2
V  3r 3
1 1
Vcone  3(r 2)H  3(4)2  12  64 cm3 2
18  3r 3
9. Each container will hold only 20 scoops. Find the
2
volume of the cylinder and the sphere. For the 18  3r 3
cylinder, r  3 in. and h  10 in. For the sphere,
r  32 in.
3 3 2
 
(18)   r 3
2 2 3
Vcylinder  r 2H    (3)2  10  90 in.3 27  r 3
3
r 3
    287  92
27
4 4 3 3 4
Vsphere  3r 3  3 2  3
The radius of the hemisphere is 3 cm.
 4.5 in.3 8192
14.  cm3. Use the area of its circular base to find
3
To find out how many scoops of ice cream will fit the radius of the hemisphere.
in the container, divide the volume of the cylinder
by the volume of the sphere. B  r 2  256

Vcylinder r 2  256
90 in.2
  
Vsphere 4.5 in.2  20 r  16
20 scoops of ice cream will fit into the container. Use the radius of 16 cm to find the volume of the
148 hemisphere.
10. 
3 m3  155 m3. To find the volume of the shell,
2 2 2
find the difference between the volume of a sphere Vhemisphere  3r 3  3(16)3  3    4096
with diameter 8 m and a sphere with diameter 8192
6 cm.  3
8192
Sphere with d  8 m (r  4 m): The volume of the hemisphere is  3
 cm3.
4 4 4 256 15. 18 in.3  57 in.3. Find the volume of a hemi-
V  3r 3  3(4)3  3    64  3 m3
sphere with r  3 in.
Sphere with d  6 m (r  3 m): 2 2
4 4 4 Vhemisphere  3r 3  3(3)3  18
V  3r 3  3(3)3  3    27  36 m3
The volume of the student’s brain is 18 in.3  57 in.3.
256 256 108 148
Vshell  3  36  3  3  3
148
The volume of the shell is  3 m  155 m .
 3 3

Discovering Geometry Solutions Manual CHAPTER 10 195


©2003 Key Curriculum Press
DG3SM586_182-228.qxd 8/2/06 6:07 PM Page 196

16. No. The unused volume is 16 cm3, and the volume Parking lot: Find the area of the annulus of a
of the golf ball is 10.6 cm3. circle with R  (75  x) ft and r  75 ft, where
R is the outer radius and r is the inner radius.
First find the volume of a golf ball, which is a
sphere with radius 2 cm. Aannulus  Aouter circle  Ainner circle
4 4 32  R2  r 2
Vsphere  3r 3  3(2)3  3 cm3
32   (75  x)2  (75)2
3 cm  10.6 cm
The golf ball will displace   3 3

of water. Now find the unused volume of the  (75  x)2  5625
cylinder, which is equal to the volume of a cylinder
with radius 4 cm and height 1 cm. The parking lot will be equal in area to the floor of
the building if (75  x)2  5625  5625. Solve
Vunused  r 2H    (4)2  1  16 cm3 this equation.
The golf ball will displace a smaller volume of water (75  x 2)  5625  5625
than the unused volume of the glass, so the water
will not overflow. (75  x)2  11,250
17. Approximately 15,704 gallons; approximately (75  x)2  11,250
53 days. The drawing shows two measurements. 75  x  
11,250
9 ft is the diameter of both the hemispheres and the
cylinder, so r  92 ft, while 36 ft is the sum of the x  
11,250  75  31
height of the cylinder and one radius of each of the To the nearest foot, the parking lot should extend
hemispheres. Therefore H  36  292  36  9  31 ft from the building.
27 ft. Find the volume of the tank.

20. ABCD is a parallelogram because the slopes of CD
Vtank  Vcylinder  Vhemispheres  are both 0 and the slopes of BC
and AB  and AD
are both 53.
2
  4
 r 2H  2 3r 3  r 2H  3r 3
y


9 2 4
   2  27  3  2
9 3

C (3, 5) D (9, 5)
81 4 729 2187 243
   4  27  3  8  4  2
x
 546.75  121.5  668.25 ft3
Convert this volume from cubic feet to gallons. B (–3, –5) A (3, –5)
1 gal
668.25 ft3   
0.13368 ft3  15,704 gal
The storage tank will hold approximately 15,704 gal 21. First construct an equilateral triangle. To find its
of gasoline. circumcenter, construct the perpendicular bisectors
of two sides of the triangle. The circumcenter is the
Each day, the service station fills twenty 15-gallon
point of intersection. Using this center and the
tanks from the storage tank, so the storage tank is
distance from the center to any vertex as the radius,
emptied at a rate of 20(15 gal)  300 gal per day.
construct the circumscribed circle. The incenter
1 day
15,704 gal    
300 gal  52.346 gal/day
(center of the inscribed circle) is the same point
as the circumcenter. Using this center and the
It will take approximately 53 days to completely
distance from the center to the point where the
empty the storage tank.
perpendicular bisector meets the circle as the
18. Lithium. First find the volume of displacement. radius, construct the inscribed circle.
Vdisplacement  r 2H    (4)2  2  32 cm3
mass 54.3 g
density    
volume  32 cm3  0.54 g/cm
3

From the table on page 535 of your book, the metal


that has a density of 0.54 g/cm3 is lithium.
19. 31 ft. Let x represent the width of the parking lot.
The floor area of the building is the area of a circle
with radius 75 ft.
A  r 2    (75)2  5625 ft2
196 CHAPTER 10 Discovering Geometry Solutions Manual
©2003 Key Curriculum Press
DG3SM586_182-228.qxd 8/2/06 6:07 PM Page 197

22. w  110°, x  115°, y  80°. The polygon is a non- S  4r 2


regular pentagon. The two interior angles of the
64  4r 2
pentagon at the bottom of the figure both have
measure x. The one in the lower right forms a linear 64  4r 2
pair with the exterior angle of measure 65°, so x 
r 2  16
180°  65°  115°. Similarly, the exterior angle of
measure y forms a linear pair with a 100° angle, so r4
y  180°  100°  80°. The five interior angles of 4 4 4 256
V  3r 3  3(4)3  3  64  3
this pentagon include two angles each with measure
256
x  115°, two angles each with measure 100°, and The volume of the sphere is   3
3 cm .
one angle with measure w. The sum of the measures
6. 144 cm2. Use the given volume to find the radius
of the interior angles of any n-gon is (n  2)  180°,
of the sphere; then find the surface area.
so the sum of the interior angles of the pentagon is
4
(5  2)  180°  3  180°  540°. Therefore V  3r 3
2(115°)  2(100°)  w  540°, so w  110°. 4
288  3r 3
IMPROVING YOUR VISUAL THINKING SKILLS
The large cube has an odd number of small cubes.
Because of the placement of pink cubes at the corners,
3
4
3 4
4 3 
(288)   r 3
r 3  216
there must be one more pink cube than blue cubes, so
3
the single cube must be pink. You can imagine laying r 6
216
out the pairs so that the empty space occurs at a corner,
S  4r 2  4(6)2  144
at the middle of a face, or at the middle of a cube, so
any of the pink cubes might be the single cube. The surface area of the sphere is 144 cm2.
7. Area of great circle  r 2. Total surface area of
LESSON 10.7 hemisphere  3r 2. Total surface area of hemi-
sphere is three times that of area of great circle.
EXERCISES
As in Exercise 3, the total area of the hemisphere
1. V  972 cm3, S  324 cm2. Substitute 9 for r in includes the surface area of half a sphere (2r 2) and
the formulas for the volume and surface area of a the surface area of the circular bottom (r 2).
sphere.
4 4 4 8. 2 gal. The hemisphere and the circular floor will
V  3r 3  3(9)3  3  729  972 cm3 have the same radius. If r represents this radius, the
S  4r 2  4  (9)2  4(81)  324 cm2 surface area of the ceiling is 12(4r 2)  2r 2, and
the area of the floor is r 2, so the surface area of
2. V  0.972 cm3, S  3.24 cm2. For this sphere, the ceiling is twice the area of the floor. Therefore,
d  1.8 cm, so r  0.9 cm. because it takes 4 gal of sealant to cover the ceiling,
4 4 4
V  3r 3  3(0.9)3  3  0.729  0.972 cm3 it will take 2 gal to cover the floor.

S  4r 2  4  (0.9)2  4(0.81)  3.24 cm2 9. V  10.368 m3; S  13.68 m3. Find the volume
of the wigwam by adding the volume of the
3. V  1152 cm3, S  432 cm2. For this hemi- semicylinder and two half-hemispheres.
sphere, r  12 cm.
2 2 Vwigwam  Vsemicylinder  Vhemisphere
V  3r 3  3  (12)3  1152 cm2
Find the surface area using the same shape.
1
S  2(4r 2)  r 2  3r 2  3(12)2  432 cm2
4. 160 cm2. Because the surface area formula
  
1 4 1

V  2 3 (1.8)3  2 (1.8)2(4.0)

contains r 2, which is the area, it is most efficient  10.368 m3


to substitute the given area directly into the surface
area formula. 
1 1

S  2 (4)(1.8)2  2 2(1.8)(4.0)  13.68 m2

S  4r 2  4(40)  160 10. a. Approximately 3082 ft2. The total surface area
that needs to be treated is the lateral surface area
The surface area of the sphere is 160 cm2.
of the cylinder (walls) plus the surface area of the
256
5.  cm3. First use the given surface area to find the hemisphere (dome) plus the area of the circular
3
radius of the sphere. Then use the radius to find the floor. The height to the top of the dome is 50 ft,
volume. and the radius of both the hemisphere and the

Discovering Geometry Solutions Manual CHAPTER 10 197


©2003 Key Curriculum Press
DG3SM586_182-228.qxd 8/2/06 6:07 PM Page 198

cylinder is 9 ft, so the height of the walls, which Compare the two volume expressions:
is the height of the cylinder, is 50  9  41 ft.
volume of noncrust 43(6,351)3
S  2rh  2r 2  r 2  2rh  3r 2     0.9887
volume of Earth 4
(6,375)3
 2(9)(41)  3(9)2  738  243 3
The volume of the noncrust is about 0.9887 
 981  3082 98.87% of the volume of Earth, so the volume of
The surface area that needs to be treated is the crust is about 1.13% of the volume of Earth.
approximately 3082 ft2.
14. 150 cm3. Find the volume of the displaced water.
b. 13 gal. 3082 
ft2
1 gal
2  12.3 gal. The farmer
250 ft Vdisplacement  r 2H    (5)2  3  75 cm3
will need to purchase a whole number of gallons
of resurfacing compound, so round up to 13 gal. Because this wood floats half out of the water and
half under the surface, the displaced water repre-
c. Approximately 9568 bushels. Find the volume of
sents only half its volume. The volume of the piece
the silo, which is the sum of the volume of the
of wood is 2(75)  150 cm3.
cylinder and the volume of the hemisphere.
2 2 15. 14. Make a sketch. In the
Vsilo  r 2h  3r 3    (9)2  41  3  (9)3 30°-60°-90° triangle shown in
 3321  486  3807 this figure, the shorter leg is a
radius of the inscribed circle r
The volume of the silo is 3807 ft3. and the hypotenuse is a radius 60° _1 r
2
30°
0.8 bushel
3807 ft3     9568 bushels
1 ft3
of the circumscribed circle.
Let r represent the radius of
The silo will hold about 9568 bushels of grain. the circumscribed circle. Then 12r
11. 153,200,000 km2. The area not covered by water is represents the radius of the inscribed circle because
30%, or 0.3 times the surface area of a sphere with in a 30°-60°-90° triangle, the length of the shorter
r  12(12,750)  6,375 km. leg is half the length of the hypotenuse (30°-60°-90°
Triangle Conjecture). Compare the areas of the two
S  0.3(4r 2)  0.3(4)(6,375)2  153,211,547 circles.
 
2
To the nearest 100,000 km2, the area of Earth’s  12r
area of inscribed circle 1
   
surface not covered by water is 153,200,000 km2. area of circumscribed circle r2  4
12. The total cost is $131.95. He will stay under budget. 16. 12. Make a sketch. In the
Surface area of one hemisphere (r  3 cm) without 45°-45°-90° triangle shown in
a base: this figure, one of the legs is a
S  2r 2  2(3)2  18 cm2 radius of the inscribed circle
r r
and the hypotenuse is a radius 45° _____
2
Surface area of one sphere (r  4 cm): of the circumscribed circle. Let
45°
S  4r 2  4(4)2  64 cm2 r represent the radius of the
r
circumscribed circle. Then  represents the radius
Total surface area to be electroplated  6(18)  2
of the inscribed circle because in a 45°-45°-90°
3(64)  108  192  300 cm2  942.5 cm2.
triangle, the length of each leg is the length of the
$0.14
942.5 cm2   
1 cm3  $131.95 hypotenuse divided by  2 (Isosceles Right Triangle
It will cost $131.95 for the electroplating. Conjecture). Compare the areas of the two circles.

 
2
13. Approximately 1.13%. First write an expression  r r 2

for the volume of Earth. For this sphere, area of inscribed circle 2 2
   
d  12,750 km, so r  6,375 km. area of circumscribed circle r 2  r 2
r 2 1 1
4 4
V  3r 3  3(6,375)3  2    
r 2  2
Now write an expression for the volume of the
“noncrust.” For this sphere, r  6,375 km  24 km
 6,351 km.
4 4
V  3r 3  3(6,351)3

198 CHAPTER 10 Discovering Geometry Solutions Manual


©2003 Key Curriculum Press
DG3SM586_182-228.qxd 8/2/06 6:07 PM Page 199

17. 34. Make a sketch. In the IMPROVING YOUR REASONING SKILLS


30°-60°-90° triangle shown in In Problem 3, note that each letter represents a different
this figure, the longer leg is a digit. In Problem 4, remember that 5 can be multiplied
radius of the inscribed circle 30° by numbers other than 0 to get 0 as the second digit.
and the hypotenuse is a radius r r 3
_____
2
1. C  4 2. D  4 3. K  8 4. N  8
of the circumscribed circle. 60°

Let r represent the radius of _r


2 CHAPTER 10 REVIEW
the circumscribed circle. Then,
by the 30°-60°-90° Triangle EXERCISES
Conjecture, the length of the shorter leg will be 2r 1. They have the same formula for volume: V  BH.
r3
and the length of the longer leg will be  2 .
Compare the areas of the two circles. 2. They have the same formula for volume: V  13BH.

area of inscribed circle


  
 
r 

2
3 2

3r 2
 4     3. 6240 cm3. The solid is a rectangular prism.
V  BH  (bh)H  (12  20)  26  6240 cm3
area of circumscribed circle r 2 r 2
3r 2 1 3 4. 1029 cm3. The solid is a cylinder with radius
 4    
r 2  4 7 cm.
18. The ratio gets closer to 1. From the figures shown V  BH  (r 2)H    (7)2  21  1029 cm3
for Exercises 15–17, you can see that as the number
of sides increases, the areas of the inscribed and 5. 1200 cm3. The solid is a rectangular prism with a
circumscribed circles get closer together because the piece missing. First find the area of the base of the
polygon gets closer and closer to both circles. solid by subtracting the area of the base of the
missing piece from the area of the complete prism.
19. a. n 1 2 3 4 5 6 n 200 The base of the missing piece is a rectangle with
f(n) 2 1 4 7 10 13 3n  5 595 dimensions (12  6) cm by (12  8) cm, or 6 cm
by 4 cm. The base of the complete prism is a square
The entries for f(n) increase by 3 each time n with side length 12 cm. Use these measurements to
increases by 1, so the entries for n  5 and n  6 find the area of the base of the solid.
are 10 and 13, respectively. This pattern tells you
that 3n will be part of the general expression for B  (12 12)  (4  6)  144  24  120 cm2
f(n). For each value of n, the corresponding value V  BH  120  10  1200 cm3
of f(n) is 5 less than 3n, so the general rule is
f(n)  3n  5. Apply this rule when n  200: 6. 32 cm3. The solid is a square pyramid.
1 1
f(200)  3(200)  5  595. V  3BH  3(4  4)  6  32 cm3
b. n 1 2 3 4 5 6 n 200 7. 100 cm3. The solid is a cone. Its diameter is
1 1 3 2 5 n1 199
10 cm, so its radius is 5 cm.
f(n) 0       
n1 1 1 1
V  3BH  3(r 2)H  3  (5)2  12  100 cm3
3 2 5 3 7 201

Here you find the value of f(n) by taking each n 8. 2250 cm3. The solid is a hemisphere.
and forming a fraction in which the numerator is 2 2
one less than n and the denominator is one more V  3r 3  3  (15)3  2250 cm3
n1
than n, that is, f(n)   
n  1 , and you write this 9. H  12.8 cm. The solid is a triangular prism. Each
fraction in lowest terms. This pattern can be seen base is a right triangle with shorter leg of length
most clearly in the given entries for n  2 and 8 cm and hypotenuse of length 17 cm. Because
n  4. It is less obvious for n  3 because the 8-15-17 is a Pythagorean triple, the length of the
original fraction obtained by the rule, 24, has been longer leg is 15 cm.
changed to 12.
20. They trace two similar shapes, except that the one
1
 
V  BH  2bh H
traced by C is smaller by a scale factor of 1:2.
768  2  8  15H
1
21. The line traces an infinite hourglass shape. Or it
traces the region between the two branches of a 768  60H
hyperbola.
H  12.8 cm

Discovering Geometry Solutions Manual CHAPTER 10 199


©2003 Key Curriculum Press
DG3SM586_182-228.qxd 8/2/06 6:07 PM Page 200

10. h  7 cm. The solid is a trapezoidal pyramid. 15. 851 cm3. Find B, the area

 
1 1 1 of the base of the solid, 6 cm
V  3BH  3 2h(b1  b2) H by subtracting the area of

 
1 1 the base of the cylinder 30°
896  3 2h(20  12)  24 (a circle) from the area of 8 cm 60°
4 3 cm

1 the base of the hexagonal


896  6h  32  24 prism (a regular hexagon). 4 cm
896  128h Make a sketch.
1 1
h  7 cm Area of hexagon  2asn  2  4 3 86

11. r  12 cm. The solid is a cone.  96 3 cm 2

1 1
V  3BH  3(r 2)H Area of circle  r 2    (6)2  36 cm2
1 B  96
3  36 cm2
1728  3    r 2  36
1728  12r 2
V  BH  96
3  36  16  851
To the nearest cubic centimeter, the volume of the
r 2  144
solid is 851 cm3.
r  12 cm
16. Four times as great. First compare the areas of the
12. r  8 cm. One-fourth of the hemisphere is missing, rectangular bases of the two prisms. If the length
so the solid is three-fourths of a hemisphere. and width of a rectangle are both doubled, the area
will be multiplied by 2  2  4, so the area of the
3 2
 1
V  4 3r 3  2r 3 larger prism is four times that of the smaller one.
1 Because the heights are the same, the volume of
256  2r 3
the larger prism is also four times that of the
r 3  512 smaller one.
3
r   8 cm
512 17. a. Vextra large  3 3  6  54 in.3
1
13. 960 cm3. Let a, b, and c represent the three dimen- Vjumbo  3  (4)2  12  64 in.3  201.1 in.3
sions of a rectangular prism. Then the volume of
the prism is abc. If each dimension is doubled, the Vcolossal    (5)2  10  250  785.4 in.3
result will be a larger rectangular prism with dimen- V 250 in. 3
b. 14.5 times as great. Vextra large  54 in.3  14.5
colossal

sions 2a, 2b, and 2c, so the volume of the larger
prism will be (2a)(2b)(2c)  8abc, that is, eight 18. Cylinder B weighs 83 times as much as cylinder A.
times the volume of the original prism. In this case, Let x represent the radius of cylinder A and y
the volume of the original rectangular prism is represent the height of cylinder A. Then the
120 cm3, so the volume of the prism obtained by radius of cylinder B is 4x and the height of
doubling all three dimensions is 8(120)  960 cm3. cylinder B is 16y.
14. 9 m. Substitute the given volume and base area in VA  r 2H    (x)2 y  r 2y
the formula for the volume of a cone from the
VB  r 2H    (4x)2  6y  (16x 2)6y
1 1
Pyramid-Cone Volume Conjecture and solve for H.
Because the base area is given, it is not necessary to 16 8
 6x 2y  3x 2y
find the radius.
1 8
V  3BH 3x 2y
VB 8
 
1 VA x 2y  3
138  3(46)H
19. 2,129 kg; 9 loads. First find the volume of one pipe.
1

3(138)  3 3(46) H  Vone pipe  Vouter cylinder  Vinner cylinder
414  46H
  (3)2(160)    (2.5)2  160
414  46H
 1,440  1,000
H9
 440
The height of the cone is 9 m.

200 CHAPTER 10 Discovering Geometry Solutions Manual


©2003 Key Curriculum Press
DG3SM586_182-228.qxd 8/2/06 6:07 PM Page 201

The volume of one pipe is 440 cm3, so the total 23. The statue is composed of 5 cm

volume of 200 pipes is 200(440 cm3)  platinum. Find the volume of


88,000 cm3. Density   mass the displaced water. Sketch the
volume , so mass 
 _5
hexagonal base of the prism.
volume  density  88,000 cm3  
7.7 g 3 cm
1 cm3 
2

By the 30°-60°-90° Triangle 5 cm
677,600 g. Conjecture, the length of the 30°

Convert the weight of 200 pipes from grams to apothem is 52


3 cm. Use the 60°

formula for the area of a _5 cm


kilograms. 1 kg  1,000 g, so 677,600 g  2
677.6 kg  2,129 kg. regular polygon to find the 5 cm

area of the hexagonal base.


Rosa’s truck can handle a total weight of a quarter-
tonne, which is 14(1,000 kg)  250 kg. Find the
number of loads needed to transport the 200 pipes.
1
1 5
 150
B  2asn  2 23 (5)(6)  43
75
2,129 kg  23 cm2
  8.5
250 kg
Round up to 9 loads.
 75

Vdisplacement  BH  23 (4)  1503 cm3
Now find the density of the statue.
20. 52.4% of the box is filled by the ball. H  2r mass 5560 g
density    
volume  1503 cm3  21.4 g/cm
3
because the height of the box is the diameter of
the ball. From the table, the metal that has density 21.40 is
Vsphere 43r 3 43r 3 platinum, so the statue is composed of platinum.
 
Vbox (2r)3  8r 3  0.524  52.4% 24. No. The ball weighs 253 lb.
21. Approximately 358 yd3.
First find B, the area of Volume of steel ball with radius 6 in.:
the base of the slab floor. This is the area of a 4 4
V  3r 3  3(6)3  288 in.3
70-by-50 ft rectangle minus the area of a 40-by-
0.28 lb
15 ft rectangle. (70  30  40; 50  35  15) weight  volume  density = 288 in.3  
in.3
B  70  50  40  15  3500  600  2900 ft2  253 lb
1
Convert 4 in. to  ft and find the volume of cement To the nearest pound, the solid steel ball weighs
3
for one floor. 253 lb, so it is too heavy to lift.
V  BH  2900  13  
2900 3

3 ft 25. 256 lb. Find the difference in volume between a
sphere with radius 7 in. and a ball with radius 5 in.
The volume of ten identical floors is 10 2,900

3 
ft3
4 4
29,000
 ft3.
3 Vhollow ball  3  (7)3  3  (5)3
4 4
Convert this volume from cubic feet to cubic yards.  3  343  3  125
1 yd  3 ft, so 1 yd3  (3  3  3) ft3  27 ft3 4
1 yd3  3(343  125)
29,000
 ft3  3  358 yd3 4
3 27 ft  3(218)
Approximately 358 yd3 of cement will be needed.
872
 3 in.3
22. No. The unused volume is 98 in.3, and the volume
872 0.28 lb
of the meatballs is 32 in.3. First find the volume of weight  volume  density = 3 in.3   
in.3
one meatball. Each meatball has a 1-inch radius.
4 4  256 lb
Vmeatball  3  12  3 in.3
The weight of the hollow steel ball is about 256 lb.
The volume of 2 dozen meatballs is 2443 
32 in.3. The unused volume is equal to the volume 26. Approximately 3 in. First find the volume of the
of a cylinder with radius 7 in. and height 2 in., so hollow steel ball.
Vunused    (7)2  2  98 in.3. density  
weight

volume
The unused volume in the pot is a lot more than the weight 327.36 lb
volume of the meatballs, which is the volume of the volume    
density  0.28 lb/in.3  1169 in.
3

displaced sauce, so the sauce will not spill over. Let x represent the thickness of the ball. Then
Vhollow ball  (volume of sphere with radius 7 in.) 
(volume of sphere with radius (7  x)).

Discovering Geometry Solutions Manual CHAPTER 10 201


©2003 Key Curriculum Press
DG3SM586_182-228.qxd 8/2/06 6:07 PM Page 202

4
1169  3  (7)3  43  (7  x)3   AB
First, OM  because M is the midpoint of AB 
4 4 and the median from the vertex angle to the base
1169  3  343  3(7  x)3 in an isosceles triangle is also an altitude. Thus,
4 1372 OAM is a right triangle in which the length of
(7  x)3    1169
3 3 the shorter leg (OM) is half the length of the
3 4
4 3  3
4
1372
 (7  x)3     1169
3    hypotenuse (OA), so this is a 30°-60°-90° triangle.
The shorter leg is opposite the 30° angle, so
3 1372 3 1169
4  3  4 
(7  x)3       mOAM  30° and mAOM  60°. By the
1
30°-60°-90° Triangle Conjecture, the length of
1372 3507
(7  x)3  4   
4 the longer leg can be found by multiplying
3507 the length of the shorter leg by  3 , so AM 
(7  x)3  343   
4  64 1 3

4 3  
4 m.
3
7x 
64 3 
Because M is the midpoint of AB , AB  2  
3
4
7x4  cm. Also, because the median from the vertex
2
x3 angle of an isosceles triangle is also an angle
bisector, mAOB  120°.
The thickness of the ball is approximately 3 in.
 33 Recall that the area of a segment of a circle is found
27. 8  
32  m  0.23 m . Note: To make the given
3 3
by subtracting the area of a triangle from the area
measurements easier to work with, change them
from decimals to fractions before solving the of a sector.
problem: 0.25 m  14 m and 1.5 m  32 m. Asegment  Asector  Atriangle
To find the volume of the water in the barrel, you 120° 2 1
 r  bh
360° 2
must find the volume of a “solid” that is a portion
of a cylinder. To find this volume, you need to find
   
1 3 1
1 1 2
 3 2  2 
2

4
the area of its base. The base is a portion of one of
the circular ends of the barrel, and the height is the 1  
3
same as the height of the cylinder, which is 32 m.  3  14  
16
 3
The base of this portion of the cylinder is a  1 
2  16
segment of one of the circles of radius 12 m that are
 3
bases for the complete cylinder. Sketch this segment The area of the segment is 12  
16  cm . This
2
and draw a triangle by connecting the center of the is the area of the base of the portion of the cylinder.
circle to the endpoints of the chord that cuts off the Let B represent this quantity.
segment as shown. Also draw a radius that passes
through the midpoint of the chord. The height of both the barrel and the portion filled
with water is 32 cm. Now use the Prism-Cylinder
Volume Conjecture to find the volume of this
portion of the barrel.
O

 
3 3 3 33
 
_1 m
2 V  BH  1    
2  16 2  24  32
M
A B
_1 m  33
4  8  32
C

, OB
OA , and OC are all radii of the circle, so The volume of the water in the barrel is
 33
because the diameter of the water barrel is 1 m, 8  32  m  0.23 m .
3 3

each radius has length 12 m. MC  14 m, so 28. 160 units3. The volume of the solid will be the
OM  OC  MC  12  14  14 m. Now look at difference between the volumes of two cylinders,
AOB and AOM. with R  6 units and r  2 units, and each with
O
_1 m height 5 units.
_1 m 4
V  R2H  r 2H    (6)2  5    (2)2  5
2 60° 60°

30° 30°
A B
3 m
____ M  180  20  160
4
3 m
____
2 The volume of the solid is 160 units3.

202 CHAPTER 10 Discovering Geometry Solutions Manual


©2003 Key Curriculum Press
DG3SM586_182-228.qxd 8/2/06 6:07 PM Page 203

TAKE ANOTHER LOOK answer, use three congruent pyramids each with
1. The models should have eight pieces. Volumes: one vertex of the base where three right triangular
a3, three a 2b, three ab 2, and one b 3; (a  b)3  faces meet.
a 3  3a2b  3ab 2  b3. 5. Answers will vary depending on the solid.
2. If you double all three dimensions of a rectangular Sample: Almost four hemispheres 12  fill a
prism, you get V  (2l)(2w)(2h)  8(lwh), but cube of s  2r.
S  2(2l)(2w)  2(2l)(2h)  2(2w)(2h)  12
8r 3  
4(2lw  2lh  2wh). Thus the volume increases 2
eightfold, but the surface area is increased only four Vhemisphere  3r 3
times. Similarly, for a cylinder, V  (2r)2(2h)  6. No. The spheres wouldn’t have the same surface
8(r 2h), but S  2(2r)2  2(2r)(2H)  4(2r 2  area. The surface area of the bumpy sphere would
2rH). Similarly, for a pyramid or a cone, V  be greater because the pyramids add surface.
(4B)(2H)  8BH , but S  4(rl  4r 2) for
1 1
3 3
a cone and S  4B  12bhn for a pyramid.
CHAPTER 11
3. Let r represent the radius of the sector and x repre-
sent the central angle. When the sector is rolled into USING YOUR ALGEBRA SKILLS 9
a cone, the outside edge becomes the circumference
x
of the base of the cone: C   
360° (2r). Let R EXERCISES
represent the radius of the base of the cone, and
x 1. 38; 35. There are 8  4  32 squares in the whole
find the radius in terms of r: 2R   
360° (2r), or figure, of which 12 are shaded and 32  12 are
  r. When the sector is rolled into a cone,
x
R 360° unshaded. Therefore the ratio of the shaded area
the radius of the sector becomes the slant height of 43
the cone. Use the Pythagorean Theorem to find the to the whole figure is 1322   3
4  8  8. The ratio
perpendicular height, H, of the cone: of the shaded area to the unshaded area is
43
  
12 3
20 4  5  5.
R2  H 2  r 2
AC 3 CD 5 BD 8
2.   ,   ,   
 
x 2 CD 5 BD 8 BC 13
  r  H 2  r2
360° 3. a. 31. RSH is a right triangle
with RH  15 
3  5 and SH  39  3  13, so the side lengths

r  

(360°)

  r
x 2
H 2
2
2
are a multiple of the Pythagorean triple 5-12-13,
and RS  3  12  36. Therefore the perimeter
of RSH is 15  36  39  90. MFL is a


r 1 
(360

   r 1

 60° 
x 2 x 2
2
2  5-12-13 triangle, so MF  12, and the
°)
perimeter of MFL is 5  12  13  30.
Now use the formula to find the volume of a cone:
1 perimeter of RSH 90 3
V  3R2H   3 
01
perimeter of MFL

  

 
1 x 2 x 2
360°  r r 1  360°
 3    b. 91. Area of RSH  12bh  12(36)(15)  270;
area of MFL  12bh  12(12)(5)  30.
area of RSH
  1
3

270 9
 60° 
    
1 x 2 x 2
 3r 3  
360°
 area of MFL 30 1
4. Sample answer: 4. a  6. Solve the proportion:
7 a
   Original proportion.
21 18
7 a
21  21  21  18
 Multiply both sides by 21.
21a
7  18
21a
The first two of these pyramids have the same base 18  7  18  18 Multiply both sides by 18.
and height as the prism, so the two have equal
126  21a
volumes. If you consider halves of the front face
126 21a
of the prism as the bases of the second and third    Divide both sides by 21.
21 21
pyramids, they also have equivalent bases and
heights and hence equal volumes. For another a6

Discovering Geometry Solutions Manual CHAPTER 11 203


©2003 Key Curriculum Press
DG3SM586_182-228.qxd 8/2/06 6:07 PM Page 204

Another way to solve the given proportion is to 10. z  6. Multiply both sides of the given
observe that 271  13, so 1a8  13. Because 18  6  3, proportion by 56(10  z), the product of the
a  6  1  6. two denominators.
5. b  16. Solve the proportion: 10 35
  
10  z 56
10 15
   56(10  z)  
10 35
10  z  56(10  z)  56
Original proportion.  
b 24
10 15
24  b  24  2
4 Multiply both sides by 24.
560  (10  z)(35)
240 560  350  35z
  15
b
240 210  35z
b  b  b  15 Multiply both sides by b. 210
z  3
5 6
240  15b
240 15b 11. d  1. Multiply both sides of the given proportion
   Divide both sides by 15. by the least common denominator, which is 20.
15 15
b  16 d d3
  
5 20
6. c  39. Rather than multiplying both sides by 13 d d3
20  5  20  2
0
and then by c, you can save steps by multiplying by
their product, 13c. 4d  d  3
20
  
60 3d  3
13 c
20 60 d1
13c  1 3  13c  c

12. y  5. Multiply both sides of the given proportion
20c  780 by 21(y  2), the product of the two denominators.
780 y 15
c  2 0  39 
y  2  21
7. x  5.6. Multiply both sides of the given y 15
21(y  2)  
y  2  21(y  2)  2 1
proportion by 35, the product of 5 and 7.
4 x 21y  (y  2)(15)
  
5 7
4 x 21y  15y  30
35  5  35  7
6y  30
28  5x
28 y5
x  5  5.6 bc
13. x  .
z Multiply both sides of the given proportion
8. y  8. Multiply both sides of the given by c.
proportion by 32y, the product of y and 32. x b
2 y    
  32 c z
y x b
2 y c  c  c  z
32y  y  32y  32
bc
x  z
64  y 2
14. 318 mi. Let x represent the distance the car can go
y  
64  8 on 12 gallons of gas.
9. x  12. Multiply both sides by the least common 106 mi
  
x mi
multiple of the denominators, which is 30. 4 gal 12 gal
14 x+9 106 x
     
10 15 4 12
14
30  1
x+9 Because 12  3(4), x  3(106)  318. The car can
0  30  15

go 318 mi on a full tank of 12 gal.
3  14  2(x  9)
15. 2.01. Let x represent Ernie’s earned run average.
42  2x  18 34 runs x runs
  
24  2x 152 innings 9 innings
34 x
x  12   
152 9

204 CHAPTER 11 Discovering Geometry Solutions Manual


©2003 Key Curriculum Press
DG3SM586_182-228.qxd 8/2/06 6:07 PM Page 205

Multiply both sides by 152  9, or 1368. IMPROVING YOUR ALGEBRA SKILLS


34 x Write equations from two rows or columns in which the
1368  152  1368  9
  coefficients of x are different. In the second and third
9  34  152x rows, for example, the coefficients of x are both 2, so the
equations would be 2x  20  2x  20, not yielding a
306  152x way to solve for x. Many other pairs of equations lead to
x
306
 the solution, x  7.
152  2.01
Ernie’s earned run average is 2.01. EXTENSION
Results will vary.
16. 12 ft by 15 ft. Let w represent the width of the
room and l represent the length of the room.
1 1 3 LESSON 11.1
 in.  in. 34 in.
4 3 in. 4
    EXERCISES
1 ft w ft 1 ft l ft
1 1 3 1. A 2. B
  34
4 3 4
    3. Possible answer: 4. Possible answer:
1 w 1 l
1 3 1 5
   
4 w 1 4
1 3   
4w  4  4w  w 4 l
1 15
4l  4  4l  4
w  12 ft
l  15 ft
5. Possible answer:
Thus the actual size of the room is 12 ft by 15 ft.
17. Almost 80 years old. Find the sum of the lengths of
the antennae for both Altor and Zenor.
Altor: 8  10  13  16  14  12  73 cm
Zenor: 7(17)  119 cm
6. Figure A is similar to Figure C. Possible answer:
Let A represent Altor’s age.
73 cm 119 cm
   If and C, then C.
A yr 130 yr A B B A
73 119
   7. AL  6 cm, RA  10 cm, RG  4 cm, KN  6 cm.
A 130
73 119 Corresponding sides of similar polygons are
130A  A  130A    TK KN NI TH HT
proportional, so LE  EG  RG  RA  AL .
130     
9490  119A Substitute the given side lengths.
9490 4 KN 2 5 3
A 
119  79.7         
8 12 RG RA AL
Altor is almost 80 years old. Because TLK 4 1
E  8  2 , each side of the smaller
  
18. AB  3 cm, BC  7.5 cm. Let p  AB; then pentagon is half as long as the corresponding side
BC  10.5  p. of the larger pentagon, so KN  6 cm, RG  4 cm,
AB BC RA  10 cm, and AL  6 cm.
  
XY YZ 8. No; the corresponding angles are congruent, but the
p 10.5  p
2  5 corresponding sides are not proportional. Three
pairs of corresponding angles are marked as
10.5  p

p

10 2  10 5  congruent, so the fourth pair of angles must also be
congruent by the Quadrilateral Sum Conjecture.
5p  2(10.5  p) Compare ratios of corresponding sides.
150 10 120 15
5p  21  2p    and   
165 11 128 16
7p  21
p  3 and 10.5  p  7.5
Therefore AB  3 cm and BC  7.5 cm.

Discovering Geometry Solutions Manual CHAPTER 11 205


©2003 Key Curriculum Press
DG3SM586_182-228.qxd 8/2/06 6:07 PM Page 206

Because these two pairs of corresponding sides are angles are congruent. Now check the ratios of
not proportional, the pentagons are not similar. (It corresponding sides:
is not necessary to check the ratios of the other two AE 2  2  2  3  3
   
pairs of corresponding sides because for polygons AB 2 14 1 14 7
to be similar, all pairs of corresponding sides must 43 
3
be proportional.) AD 3
  
AC 7
9. NY  21 cm, YC  42 cm, CM  27 cm,
ED 4  4  4  3  3
MB  30 cm   
BC 1 28 1 28 7
SP ER DE ID PI 93 
         3
HN MB CM YC NY
Therefore all pairs of corresponding sides are
88 40 36 56 28
         proportional. Because corresponding angles are
66 MB CM YC NY
congruent and corresponding sides are proportional,
88 22  4 4 4 40 36 56 28 AED ABC.
    3, so 3      
66 22  3 MB  CM  YC  NY
28  4(7), so NY  3(7)  21 cm 14. m  92 cm, n  94 cm. ABC DBA, so  AB
DB 

BC AC 3 4 6
  . Then,     . To find m and n, solve
56  4(14), so YC  3(14)  42 cm BA DA n 3 m
two proportions:
36  4(9), so CM  3(9)  27 cm 3 4 4 6
     
n 3 3 m
40  4(10), so MB  3(10)  30 cm 3 4 4 6
3n  n  3n  3 3m  3  3m  m
10. Yes; the corresponding angles are congruent, and
the corresponding sides are proportional. Because 9  4n 4m  18
four of the five pairs of corresponding angles 9 18 9
n  4 m  4  2
are marked as congruent, the angles in the fifth
pair must also be congruent because the sum of 15. 13. Let R, O, and Y represent the coordinates of
the measures of the angles of any pentagon is 540° the dilation of ROY.
by the Pentagon Sum Conjecture. Corresponding y
sides are proportional: Y (6, 15)
15
30 27 39 78 21 3
           12
20 18 26 52 14 2
11. x  6 cm, y  3.5 cm. ACE IKS, so AIKC  9
CE AE 7 8 12 CE 8 2
  . Then,     . Because     ,
KS IS y 4 x KS 4 1 6 Y (2, 5)
R (6, 6)
each side of ACE is twice as long as the O (18, 6)
3
corresponding side of IKS, so x  6 cm O (6, 2)
R (2, 2)
and y  3.5 cm. 3 6 9 12 15 18
x

12. z  1023 cm. RAM XAE, so corresponding ROY is a 3-4-5 triangle, so the perimeter of
sides are proportional. ROY is RO  OY  YR  4  5  3  12 units.
RA AM RM
     From the graph, you can see that the vertices of
XA AE XE
4 AM
    
z ROY are R(6, 6), O(18, 6), and Y(6, 15).
3 AE 8 ROY is a 9-12-15 triangle (a multiple of 3-4-5),
4

24 3  24 8
z so the perimeter of ROY is RO  OY 
YR  12  15  9  36 units.
32  3z perimeter of smaller triangle 12 units 1

perimeter of larger triangle    
32 2
z  3  103 36 units  3
16.
13. Yes, the corresponding angles are congruent. Yes,
the corresponding sides are proportional. Yes,
AED ABC. Because DE  BC, B  AED
and C  ADE (CA Conjecture). Also, A 
A (same angle), so all pairs of corresponding

206 CHAPTER 11 Discovering Geometry Solutions Manual


©2003 Key Curriculum Press
DG3SM586_182-228.qxd 8/2/06 6:07 PM Page 207

17. Possible answer: Assuming an arm is about three 22. a. b.


times as long as a face, each arm would be about
260 ft. The actual sculpture will be similar to the
60°
scale model, so the ratio between the length of the
60°
arm and the length of the face will be the same on
the actual sculpture as on the scale model. There-
fore the length of an arm on the sculpture will be 23. Approximately 92 gal. First
about 3(87.5)  262.5 ft, or about 260 ft. find the volume of the
portion of the cylinder in
18. a  36. Solve the proportion: cubic feet. The base of this
15 20 2 ft 2 ft
   solid is a segment of the base 60° 1 ft
a a  12
of the cylinder. Look at the 3 ft
15 20
a(a  12)  a  a(a  12)  
1 ft
 figure at right.
a  12
(a  12)(15)  20a The length of the hypotenuse of the small triangle
is twice the length of the shorter leg, so this is a
15a  180  20a 30°-60°-90° triangle, and the longer leg has length
180  5a  3 ft. Because the large triangle is isosceles, this
also means the arc measure of the segment is
a  36 2(60°)  120°. Now find the volume of the solid
19. ad  bc. Solve the proportion for ad. using the area of the segment as the base.

360°   r  2 H
a c a 1
   V  BH  bh
2
b d
Multiply both sides by bd, the product of the
  
360°   (2)  2 2
3 (1)5
120° 1
denominators.  2

a
 c
bd b  bd d   3  35
4
da  bc, or ad  bc
20
b  3  53 ft3
20. 
a  dc. From Exercise 19, ab  dc is equivalent to
ad  bc. Divide both sides by ac. Convert this volume from cubic feet to gallons.

20

ad  bc 7.5 gal
  53 ft3  
ad bc 3 1 ft3  92 gal
  
ac ac 24. Yes, because 182  242  302 (Converse of the
d b b d Pythagorean Theorem). The side lengths are a
  , or   
c a a c multiple of the 3-4-5 Pythagorean triple: 18  6(3),
1825
21. Possible answers: Jade might get  
4475 of the2650 24  6(4), 30  6(5).
profits, or $2,773.18, and Omar might get  
4475 of 25. a. Possible answer: Fold a pair of corresponding
the profits, or $4,026.82. The total amount spent
vertices (any vertex in the original figure and the
was $2,000  $825  $1,650  $4,475. Jade
corresponding vertex in the image) together and
contributed $1,000  $825  $1,825, and Omar
crease; repeat for another pair of corresponding
contributed $1,000  $1,650  $2,650.
vertices; the intersection of the two creases is the
1825
Jade’s share:  ($6,800)  $2,773.18
4475 center of rotation, P.
2650
Omar’s share:  
4475 ($6,800)  $4,026.82 Fold 1
Or, they could first take out their investments, and P
then divide the profit equally. If the profit was
$6,800  $2,000  $825  $1,650  $2,325, they
would each get ($2,325)2  $1,162.50. Then Jade’s Fold 2

share would be $1,825  $1,162.50  $2,987.50


and Omar’s share would be $2,650  $1,162.50 
$3,812.50. Either approach could be considered a
fair division.

Discovering Geometry Solutions Manual CHAPTER 11 207


©2003 Key Curriculum Press
DG3SM586_182-228.qxd 8/2/06 6:07 PM Page 208

b. Possible answer: Draw a segment (a chord) 3. 28 cm. It may help to rotate ARK so that you can
between a pair of corresponding vertices and see which sides correspond.
construct the perpendicular bisector; repeat for T
another pair of corresponding vertices; the inter-
A
section of the two perpendicular bisectors is the
36 42
center of rotation, P. 24 m

P R K D A

By the AA Similarity Conjecture, DAT RKA.


AT DT
  
KA RA
42 36
PROJECT   
m 24
Project should satisfy the following criteria: 36

24  32, so 42

m  32. 42  14(3), so m  14(2) 
● The grid on the original image contains enough 28 cm.
squares so that each square contains only a few lines.
4. n  54 cm, s  42 cm. The two triangles are
● The mural accurately reproduces the original. similar by the AA Similarity Conjecture (a pair of
Extra credit marked angles and a pair of vertical angles).
n 45 63
● A group of students produces a complex piece of art.     
36 30 s
EXTENSIONS 45
  32, so 3n6  32. 36  18(2), so n  18(3) 
30
A. Results will vary. 54 cm. Also, 6s3  32. 63  21(3), so s  21(2) 
B. If you dilate by a scale factor less than 0, the result 42 cm.
is a dilation plus a rotation of 180°.
5. No, 3370  3258 . Corresponding sides are not propor-
C. See the solutions to Take Another Look activities 1 tional, so the triangles are not similar. (If AUL
and 2 on page 228. was similar to MST, AU  and MS, and UL
 and ST ,
would be pairs of corresponding sides.)
LESSON 11.2 6. Yes, MOY NOT by the SAS Similarity Conjec-
EXERCISES ture. Compare the ratios of corresponding sides by
13  8
91  
104
writing each ratio in lowest terms.  
13  7
 87
1. 6 cm. By the AA Similarity Conjecture, LDI 12  8
CPA, so corresponding sides are proportional. and 9864   8
12  7  7. The angles included between
LD DI LI these two pairs of corresponding sides are marked
     as congruent, so the triangles are congruent by the
CP PA CA
4 g 7 SAS Similarity Conjecture.
    
6 9 1
102 7. Yes, PHY YHT because 2105  1162  192 (SSS
4 2 2 Similarity Conjecture). Yes, PTY is a right triangle
  , so each side of the smaller triangle is  as
6 3 3 because 202  152  252. In PHY, YH  12.
long as the corresponding side of the larger triangle.
(12-16-20 is a Pythagorean triple.) In YHT,
Therefore g  23(9)  6 cm.
HT  9. (9-12-15 is a Pythagorean triple.) In
2. h  40 cm, k  40 cm. By the AA Similarity PTY, PT  PH  HT  25 cm.
Conjecture, GAS JET, so corresponding sides
8. TMR THM MHR (AA Similarity
are proportional.
Conjecture). x  15.1 cm, y  52.9 cm, h 
GA AS GS 28.2 cm. Draw the three triangles separately and
     
JE ET JT orient them so that corresponding vertices and sides
h 24 32
     are in the same relative position.
50 30 k
24 R

30  45, so h

50  45. 50  10(5), so h  10(4)  M
68 60 R
40 cm. 32 h 32
x
4 32

5  ;
k 32  8(4), so k  8(5)  40 cm. T 60 M T y H M h H

208 CHAPTER 11 Discovering Geometry Solutions Manual


©2003 Key Curriculum Press
DG3SM586_182-228.qxd 8/2/06 6:07 PM Page 209

36 48
TMR THM by the AA Similarity Conjecture   
36 + f 48  32
because each triangle has a right angle and T is
common to both triangles. TMR MHR by the 36 48 3
    
36  f 80 5
AA Similarity Conjecture because each triangle has
36 3
a right angle and R is common to both triangles. 5(36  f )  
36  f  5(36  f )  5
 
Two triangles that are similar to the same triangle
are similar to each other, so TMR THM 5(36)  (36  f )(3)
MHR. 180  108  3f
Use a pair of similar triangles to find each of the 72  3f, so f  24 cm
unknown side lengths. From TMR MHR, CU CE
68 32 17 32 CUE CNT, so  CN  CT .
 
  , or   .
32 x 8 x 36 + f 48  32

(36 + f 
)  30   
17 32 (48  32)  g
8x  8  8x  x
36  24 80
17x  256   
66  24 80  g
256 60 80
x  1
7  15.1 cm   
90 80  g
From TMR THM, 6680  60
, or 17
  60
. 2 80
y 15 y   
17 60 3 80  g
15y  1
5  15y  y
 2 80
3(80  g)  3  3(80  g)   
80  g
17y  900
(80  g)(2)  3(80)
900
y  1
7  52.9 cm 160  2g  240
68 32 17 32
Also from TMR THM, 
60 or,
h  
15
.
h 2g  80, so g  40 cm
17 32
15h  1
5  15h  h
 11. Yes, THU  GDU and HTU  DGU.
17h  480 HUT DUG by the AA Similarity Conjecture.
480 p  52 cm, q  42 cm. THU  GDU and
h  1 7  28.2 cm HTU  DGU because in each case they are
9. Yes, QTA  TUR and QAT  ARU by two angles inscribed in the same arc, so HUT
the CA Conjecture. QTA QUR by the AA DUG by the AA Similarity Conjecture. (You could
Similarity Conjecture. e  623 cm. Use the similarity also use the pair of vertical angles as one pair of
statement to write a proportion that you can use to corresponding congruent angles.) Use the propor-
find e. tionality of corresponding sides of these similar
triangles to find p and q.
QT QA
 Q
R HU HT UT
QU     
3 4 DU DG UG
   p 60 56
4e 3
9  4 
5 q
8
3 4
8(4  e)  8  8(4  e)    p
4e 
39  43 and 5q6  43. Solve these two proportions
(4  e)(3)  8(4) separately.
p 4 56 4
12  3e  32 39  3
9  39  3 3q  q  3q  3
3e  20 p  52 cm 168  4q
20 2 q  42 cm
e  3  63 cm
10. f  24 cm, g  40 cm. Look at triangles COR, 12. SUN TAN by the AA Similarity Conjecture.
CUE, and CNT. Because OR  UE
 NT, COR  r  13 cm, s  20 cm. Use the same reasoning as in
CUE  CNT (CA Conjecture). All three trian- Exercise 11.
gles share C. Therefore, COR CUE SU UN SN
CNT by the AA Similarity Conjecture. Use a pair     
TA AN TN
of similar triangles to find each of the unknown s r 18
CO CR     
side lengths. COR CUE, so  CU  CE .
  40 26 36
s r 1
    
40 26 2

Discovering Geometry Solutions Manual CHAPTER 11 209


©2003 Key Curriculum Press
DG3SM586_182-228.qxd 8/2/06 6:07 PM Page 210

Each side of the smaller triangle is half as long as 21


16. r  R    1
, or r  R2  12. Look at a
the corresponding side of the larger triangle, so 2
portion of the figure and apply the Isosceles Right
r  13 cm and s  20 cm.
Triangle Conjecture to find the lengths of the
13. Yes, RGO  FRG and GOF  RFO. hypotenuses of the right triangles with legs of
GOS RFS by the AA Similarity Conjecture. length R and legs of length r.
t  28 cm, s  120 cm. RGO and FRG are
alternate interior angles formed when parallel lines
 and RF
OG  are cut by a transversal, RG
, so these
angles are congruent by the AIA Conjecture. GOF R 2
 RFO for a similar reason. Therefore GOS R

RFS by the AA Similarity Conjecture. R


r
GO OS GS
     r 2
RF FS RS r
48 t 36
     R
s 70 90
48 t 2
     2  r  R  R
r  2
s 70 5
48  2(24), so s  5(24)  120 cm. 2  r  R
r  2 R

70  5(14), so t  2(14)  28 cm. r 2  1  R2  1

14. w  20 cm, x  21 cm. Use the same reasoning as R2  1


r  
in Exercise 13 to show that ADS TOS. 2  1
AD DS AS
  
TO  OS  TS
2  1
rR 
2  1  
x 12 15
      2  1
35 w 25 The expression  can be rewritten by rational-
21
x 12 3
     izing the denominator:
35 w 5 2
2  1 2  1 2  1 2  1
35  7(5), so x  7(3)  21 cm.        2
2  1 2  1 2  1  2   12
12  4(3), so w  4(5)  20 cm. 2
2  1
15. x  50 cm, y  9 cm. Draw a perpendicular    2  12
21
segment from each of the three marked points in Therefore another way to express the radius of the
the first quadrant to the x-axis. Label the origin and small circles in terms of the radius of the large
the three points for reference. circle is r  R( 2  1)2.
y
E 17. She should order approximately 919 lb every three
months. Notice that the given information is about
two months, but you are asked for the amount
30 needed for three months. The amount of dog food
C
that would be needed for 8 dogs for three months is
  7  35  367.5 lb. Let x represent the number
5 A y 3
3 x
2
O 5B D F of pounds of dog food needed to feed 20 dogs for
15
x three months. Write a proportion and solve it to
find x.
OAB OCD OEF because all three trian- 8 dogs 20 dogs
  
gles share O and each has a right angle. Therefore 367.5 lb x lb
the ratio of the length of the shorter leg to the 8 20
  
length of the longer leg is the same in all three 367.5 x
triangles. 8 20
367.5x  367.5  367.5x  x
 
AB CD EF
     8x  7350
OB OD OF
3 y 30 7350
  15  x x  8  918.75
5
15  3(5), so y  3(3)  9. Phoung must buy a whole number of bags of dog
food, so she should order 919 bags.
30  10(3), so x  10(5)  50.

210 CHAPTER 11 Discovering Geometry Solutions Manual


©2003 Key Curriculum Press
DG3SM586_182-228.qxd 8/2/06 6:07 PM Page 211

18. 448 fish. Write a proportion. 21. The statue was about 40 ft, or 12 m, tall. To
tagged fish in sample fish previously tagged estimate, approximate the height of a person in the
 sample size   picture, measure a part of the statue in the picture,
estimated population
calculate the approximate height of that statue
Let x represent the estimate of the total population.
piece, and assume that the statue has the same
12 84 proportions as the average person.
  
64 x
12 84 IMPROVING YOUR VISUAL THINKING SKILLS
64x  6
4  64x  x

To create the tetrahedron, place the square faces together
12x  5376 and rotate one solid until it completes the tetrahedron.
x  448
19. 118 square units. Finding the slopes of the sides or
checking the side lengths in the Converse of the
Pythagorean Theorem will show that this is not a
right triangle, and the lengths of the altitudes are
not known, so you can’t use the usual area formula, EXTENSIONS
A  12bh. So graph the points on a coordinate grid A. See the solutions to Take Another Look activities 4
and draw a rectangle around the triangle. Label the and 5 on pages 228–229.
corners of the rectangle for reference and find their B. Draw the altitude to the hypotenuse.
coordinates.
y C
x
D (–9, 13) D
B (4, 13) c
a
d cx
A (–9, 5)
A b B
x
–10 10
ABC DBA DAC by the AA Similarity
F (4, –7)
E (–9, –7) C (1, –7) Conjecture. (Each pair of triangles has one right
–10
angle and one common angle.) Set up proportions
for the sides of the triangles. From the hypotenuse
Now you can find the area of the triangle by and longer legs, you get
subtracting the areas of the three right triangles b c
  
from the area of the rectangle. To find the length cx b
and width of the rectangle, find the distance b 2  c 2  cx
between the points.
From the hypotenuses and shorter legs, you get
Area of DEFB  lw  (13  (7))(4  (9)) a c
  
 260 square units x a
1 1 a2  cx
Area of AEC  2bh  2(1  (9))(5  (7))
Substitute a2 for cx in the equation above.
 60 square units
1 1 b 2  c 2  a2
Area of CFB  2bh  2(4  1)(13  (7))
a2  b 2  c 2
 30 square units
1 1 C. Not all isosceles triangles are similar. Possible
Area of DAB  2bh  2(13  5)(4  (9)) counterexample:
 52 square units
The area of ABC  260  60  30  52 
118 square units.
20. Yes, ABCD ABCD. The ratio of y
All isosceles triangles with base angles that measure
the perimeters is 21. The ratio of the
D' C' 50° are similar by the AA Similarity Conjecture
areas is 41.
because the three angles must measure 50°, 50°, and
x 80°. All isosceles right triangles are similar by the
A' B'
same reasoning.

Discovering Geometry Solutions Manual CHAPTER 11 211


©2003 Key Curriculum Press
DG3SM586_182-228.qxd 8/2/06 6:07 PM Page 212

8 h
LESSON 11.3   
6 4
8 h
EXERCISES 12  6  12  4
1. 16 m. Let h represent the height of the building. 16  3h
16 1
h  3  53

h The height of the flagpole is 513 m.


4. 10.92 m. Let h represent the height of the window.
4m
6m 24 m

The triangles are similar by the AA Similarity


Conjecture.
4 h
  
6 24
24  4(6), so h  4(4)  16.
h
The building is 16 m tall.
2. 4 ft 3 in. Let h represent the height of Melody’s
friend. 84 in.  7 ft. 1.82 m

1.22 m 7.32 m
5 ft h
The triangles are similar by AA, so corresponding
7 ft 6 ft
sides are proportional.
1.82 h
The triangles are similar by the AA Similarity   
1.22 7.32
Conjecture. 1.82 h
5 h (1.22)(7.32)  
1.22  (1.22)(7.32)  7.32
 
  
7 6 13.3224  1.22h
5 h
42  7  42  6 h
13.3224

1.22  10.92
30  7h
30 2 The window is 10.92 m high.
h  7  47ft
5. 5.46 m. Let h represent the height of the tree.
2 2
 ft  (12 in.)  3 in., so Melody’s friend is The two right triangles are similar by AA, so
7 7
approximately 4 ft 3 in. tall. corresponding sides are proportional.
3. 513 m. Let x represent the height of the flagpole and h
  
12.20  6.10
h represent the height of the goalpost. 1.82 6.10
h 18.3
(1.82)(6.1)  
1.82  (1.82)(6.1)  6.1
 
6.1h  33.306
10 m 33.306
x
h 
6.1  5.46
h
The tree is 5.46 m tall.
6m 4m 6. Thales used similar right triangles. The height of the
pyramid and 240 m are the lengths of the legs of
First find the height of the flagpole. The side one triangle; 6.2 m and 10 m are the lengths of the
lengths in the triangle on the left are doubles of the corresponding legs of the other triangle. The height
3-4-5 Pythagorean triple, so x  2(4)  8 m. The of the pyramid is 148.8 m.
triangles are similar by AA, so corresponding sides
6.2 H
are proportional.   
10 240
240  24(10), so H  24(6.2)  148.8.

212 CHAPTER 11 Discovering Geometry Solutions Manual


©2003 Key Curriculum Press
DG3SM586_182-228.qxd 8/2/06 6:07 PM Page 213

h 276 12
7. PR  90 m. R and O are both right angles and     
12 23 1
P is the same angle in both triangles, so PRE
2  12  1
h 12
POC by AA. Thus  PR RE 12  1 
PO  OC . Let x  PR. Then,
 
x 60 2
    . Solve this equation for x.
x + 45 90 3 h  12(12)  144
x 2
3(x  45)  
x + 45  3(x + 45)  3
  The height of the wall is 144 in., or 12 ft.
3x  2(x  45) 11. MUN MSA by AA. x  3123. Draw the
triangles separately to see the two triangles better.
3x  2x  90
Because UN SA
, MUN  MSA (CA Conjec-
x  90 ture), and M  M (same angle), so MUN
MSA by AA. Therefore corresponding sides of
The distance across the river is 90 m.
these triangles are proportional.
8. 300 cm. The diagram shows two similar rectangular MU UN
pyramids, each lying on a lateral face. The base of   
MS SA
the larger pyramid is the surface of the painting; its 9 15
dimensions are 45 cm by 30 cm. The base of the   x
19
smaller pyramid is an image of the painting; its 9 15
dimensions are 3 cm by 2 cm. The ratio of corre- 19x  19  19x  x
sponding sides of the bases (which are edges of the 9x  285
pyramid) is 435  320  115 . The ratio of heights of 285 2
the pyramids will be the same, so d  15(20)  x  9  313
300 cm. 12. BDC AEC by AA. y  63. DBC  EAC
45 cm because both angles are inscribed in the same arc,
3 cm and BDC  AEC for the same reason.
2 cm 30 cm
BD DC BC
    
AE EC AC
y 54
d 91  78
y 9
20 cm 91  13
9. Possible answer: Walk to the 91  7(13), so y  7(9)  63.
point where the guy wire t 13. GHF FHK GFK by AA. h  18163 ,
touches your head. Measure x  7193 , y  44143 . Each pair of triangles shares
h
your height, h; the distance one angle and each triangle has one right angle,
x
from you to the end of the y so the triangles are similar by AA. Draw the three
guy wire, x; and the distance triangles separately to see them more clearly.
from the point on the ground directly below the top F H
of the tower to the end of the guy wire, y. Solve a 20
48
h
y
H h
proportion to find the height of the tower, t: ht  xy. x
G 52 K F 48 K G 20 F
Finally, use the Pythagorean Theorem to find the
length of the guy wire:  t 2  y 2. To find h, use GFK FHK to write a proportion.
10. The triangles are similar by AA (because the ruler is 20 52
  
parallel to the wall), so Kristin can use the length of h 48
string to the ruler, the length of string to the wall, 20 13
  
and the length of the ruler to calculate the height of h 12
20 13
the wall. The height of the room is 144 in., or 12 ft. 12h  h  12h  1
2
Let h represent the height of the wall. 240  13h
240 6
h  1 
3  1813
h
12
23

276

Discovering Geometry Solutions Manual CHAPTER 11 213


©2003 Key Curriculum Press
DG3SM586_182-228.qxd 8/2/06 6:07 PM Page 214

To find x, use GFK GHF to write a proportion. b. 3.12.12, or 3.122. The polygons that surround
20 52 each vertex are an equilateral triangle and two
  2 regular dodecagons.
x 0
20 13 17. The golden ratio is approximately 1.618. To find the
  5
x golden ratio, solve the proportion AAX
B AX

XB .
20
5x  x  5x  153 AB AX
  
AX XB
100  13x
AX  XB AX
100 9   
x  1 
3  713
AX XB
AX  XB AX
To find y, use GFK FHK to write a proportion. AX  XB  AX  AX  XB  
XB
48 52
   XB  (AX  XB)  AX  AX
y 48
48 13 XB  AX  XB2  AX 2
  
y 12
AX 2  XB  AX  XB2  0
12y  y  12y  1
48 13
2 (XB)  (XB)
2  4(1)(X
B 2)
576  13y AX   2(1)
576 4 XB   5 • XB2
y  1 
3  4413 AX   2
14. XB  XB5
AX   2
1   5
8.2 cm AX  XB   2
3 cm 1.7 cm
Since we want AX XB, we must use
1 +  5
AX  XB   2 , so the golden ratio is
4.6 cm
1  
5
XB  
2 1  
  
AX 5
XB XB   1.618
2
Possible construction:

Possible answer: C
1 D
A  2(8.2)(1.7)  6.97 cm2
1 A
A  2(3)(4.6)  6.9 cm2 X B

The areas computed two different ways should be


equal, but here the results vary slightly due to Construct BC  12AB, with mB  90°. Then
inexact measurements. 5
AC   2 AB by the Pythagorean Theorem. Using
15. 523. Draw a radius of the circle circles, construct DC  BC and AX  AD. Then
to form a right triangle. 5 5 1
3 r AX  AD  AC  CD   2 AB  2 AB 

51
Apply the Pythagorean  AB. So
r3 2
Theorem to the right triangle. AB 5 1

  
AX
2

2

r2  52  (r  3)2 5  1 5  1 5 1

r 2  52  r 2  6r  9 21   5 21  5 1  
5
  51  4  
0  34  6r 2
Therefore X is the golden cut.
6r  34
34 17 2 18. a. Answers will vary.
r  6  3  53
b. Possible answer: The shape is an irregular curve.
16. a. 4.6.12. The polygons that surround each vertex You might describe it as a tear or a guitar pick.
are a square, a regular hexagon, and a regular
dodecagon.

214 CHAPTER 11 Discovering Geometry Solutions Manual


©2003 Key Curriculum Press
DG3SM586_182-228.qxd 8/2/06 6:07 PM Page 215

c. Possible answer: As the circular track becomes  of the similar triangles. CL  2(1.5)  3.0 cm
HA
smaller, the curve becomes more circular; as the and HA  2(2.4)  4.8 cm. Use triangle similarity
track becomes larger, the curve becomes more to write a proportion involving x.
pointed near the fixed point. As the rod becomes
DO CL
shorter, the curve becomes more pointed near the   
TE HA
fixed point; as the rod becomes longer, the curve x 3.0
becomes more like an oval. As the fixed point   
3.2 4.8
moves closer to the traced endpoint, the curve 3.2  23(4.8), so x  23(3.0)  2.0 cm.
becomes more pointed near the fixed point; as
the fixed point moves closer to the circular track, 6. Area of ARM  126 cm2, area of LEG 
the curve begins to look like a crescent moon. 504 cm2. First find the area of ARM.
1 1
IMPROVING YOUR VISUAL THINKING SKILLS Area of ARM  2bh  2(18)(14)  126 cm2
The three unshaded squares must be To find the area of LEG, you need to know the
arranged with one in each row and each . Let h represent this
length of the altitude to LE
column. height and apply the Proportional Parts Conjecture.
14 18 1
    
h 36 2
EXTENSIONS
h  14(2)  28 cm
A. Research results will vary.
1 1
Area of LEG  2bh  2(36)(28)  504 cm2
B. Results will vary.
7. 16 cm. Use the Angle Bisector/Opposite Side
LESSON 11.4 Conjecture.
v 20 4
EXERCISES     
36 45 9
1. 18 cm. Use the Proportional Parts Conjecture. 36  4(9), so v  4(4)  16 cm.
The segments with lengths h and 12 cm are
altitudes to corresponding sides AG and IC
 in the 8. 60 cm. Use the Angle Bisector/Opposite Side

h

33

3
similar triangles, so 12  22  2 . 12  6(2), so
 Conjecture.
h  6(3)  18 cm. 40 34 2
    
y 51 3
2. 12 cm. Use the Proportional Parts Conjecture. The
segments with lengths x and 16 cm are angle bisec- 40  20(2), so y  20(3)  60 cm.
tors to corresponding sides SK  and JM  in the 9. 449 cm. Use the Angle Bisector/Opposite Side
similar triangles, so 1x6  1284  34. 16  4(4), so Conjecture. The angle bisector divides the opposite
x  4(3)  12 cm. side into two segments with lengths x and 10  x.
3. CL  21 cm, CS  20 cm. From the similar triangles, x 12 4
    
IC CS IC 1 CS 1 CS 1 10  x 15 5
  . IC  EC,   , so   , or   , x 4
IE EP IE 2 EP 2 40 2
5(10  x)   10  x  5(10  x)  5
 
so CS  20 cm.
 is the median to PI , and in SIC, CL  5x  4(10  x)
In PIE, ES
is the median to SI , so by the Proportional Parts 5x  40  4x
Conjecture, IICE  CESL . Because IICE  12, CESL  12, or 9x  40
CL 1
  , so CL  21 cm. 40 4
42 2
x  9  49 cm
4. 15 cm. Use the Proportional Parts Conjecture. CL  p
, and DF
is the angle bisector to AP  is the angle 10. ab  q, pa  bq. By the Angle Bisector/Opposite
p
. AP
bisector to AY  and AY  are corresponding parts Side Conjecture, ab  q. Rewrite this equation with
a
of the similar triangles, so they are in the same ratio  on one side.
p

 
as any pair of corresponding sides. a p
bq b  bq q Multiply both sides by bq.
CL CA

DF DA qa  bp
25 35 5 qa bp
     qp  qp Divide both sides by qp.
FD 21 3
25  5(5), so FD  5(3)  15 cm. a b
   
p q

5. 2.0 cm. Use the Proportional Parts Conjecture. DO
 
and TE are medians to corresponding sides CL and
Discovering Geometry Solutions Manual CHAPTER 11 215
©2003 Key Curriculum Press
DG3SM586_182-228.qxd 8/2/06 6:07 PM Page 216

11. 63 cm. There are several ways to solve this divides this segment into two segments of lengths
problem. x cm and (12  x) cm. Apply the Angle Bisector/
Opposite Side Conjecture to find x.
First solution: The large right triangle is a
12  x 5
30°-60°-90° triangle with shorter leg (opposite   
x 13
the 30° angle) of length 9 cm, so by the 30°-60°-90° 12  x 5
Triangle Conjecture, the length of the hypotenuse 13x  x  13x  1 3
is 2(9)  18 cm. Use the Angle Bisector/Opposite 13(12  x)  5x
Side Conjecture.
156  13x  5x
k 18 2
  9  1, so k  63 cm. 156  18x
3 3
156 2
Second solution: The large right triangle is a x  1 
8  83 cm
30°-60°-90° triangle with shorter leg of length
9 cm, so by the 30°-60°-90° Triangle Conjecture, Now look at the small right triangle on the left.
the length of the longer leg is 9
3 cm. Then Its legs have lengths 5 cm and 12  x  12  823 
10
 cm.
k  3 3  9 3 , so k  63 cm. 3

Third solution: The large right triangle is a   10 2 100 325


y 2  52 + 3  25  9  9
30°-60°-90° triangle, so the obtuse triangle is 
13
y  

325 
325 • 25 513
 
isosceles with two 30° angles. Then the angle 9  9 3 3
cm
bisector (which is a leg of the obtuse isosceles
triangle and the hypotenuse of the small right 14. B  (3, 5), R  134, 7; hk  74. By the Dilation
triangle) has length k by the Converse of the Similarity Conjecture, PQR ABC. By
Isosceles Triangle Conjecture. Now apply the comparing the coordinates of A and P, you can see
Pythagorean Theorem to the small right that each coordinate of P is 74 of the corresponding
triangle. coordinate of A, so this is the scale factor of the
dilation. (The coordinates of each vertex of PQR
3 2  81  27  108
k2  (9)2  3 will be 74 times the coordinates of the corresponding
k  108
  36
 • 3  63
 cm vertex of ABC, so the coordinates of each vertex
in ABC will be 47 of the coordinates in the corre-
12. 6 cm. You don’t know the length of the third sponding vertex of PQR.) Use this ratio to find
(unmarked) side of the large triangle, but by the the coordinates of B and R. For B, the correspond-
Angle Bisector/Opposite Side Conjecture, you know ing vertex is Q.

   
that the ratio of its two parts (upper part to lower) 4 1 4 21
is 1150  32, so these lengths can be represented by x-coordinate of B  7 54  7 4  3
3a and 2a.
y-coordinate of B  784  74  5
4 3 4 35

The coordinates of B are (3, 5). For R, the


3a
corresponding vertex is C.
15
x 7 7 3
x-coordinate of R  4(1)  4  14
2a
7
y-coordinate of R  4(4)  7
10
The coordinates of R are 134, 7.
Using two pairs of corresponding angles formed
when the parallel segments are cut by transversals Because PQR is a dilation of ABC by a scale
(or using one of these pairs and the common angle factor of 74, the ratio of the lengths of the corre-
at the top of the figure), the triangle with base of sponding sides is 74. Therefore, by the Proportional
length x and the triangle with base of length 10 cm Parts Conjecture, the corresponding altitudes will
are similar. The ratio of the lengths of correspond- have the same ratio: hk  74.
3a 3a 3
3a  2a  5a  5 .
ing sides for theses two triangles is     2
15. 
x 3
Then 10  5, so x  6 cm. 1 . Draw an isosceles right triangle in which the
length of each side is 1. (Any side length will result
 
5 13
13. x  823 cm, y  3 cm. The larger right triangle in the same ratio for the areas.) Then by the
has hypotenuse of length 13 cm and shorter leg of Isosceles Right Triangle Conjecture, the length of
length 5 cm, so this is a 5-12-13 triangle, and the the hypotenuse is  2 . Let x represent the length of
length of the longer leg is 12 cm. The angle bisector one of the segments into which the angle bisector

216 CHAPTER 11 Discovering Geometry Solutions Manual


©2003 Key Curriculum Press
DG3SM586_182-228.qxd 8/2/06 6:07 PM Page 217

divides the leg to which it is drawn; then the length Possible proof: Consider similar triangles LVE and
of the other segment will be 1  x. MTH with corresponding angle bisectors EO  and
B
. Look at EOL and HAM. L  M
HA
because they are corresponding angles of LVE and
x MTH. mLEO  12mLEV and mMHA 
1
2 mMHT. mLEV  mMHA from correspond-
2
D
ing angles of the similar triangles, so 12mLEV 
1
1x mMHA, and therefore mLEO  mMHA by
2
A C substitution. Thus, EOL HAM by the AA
1
Similarity Conjecture. Corresponding sides of
EO EL
similar triangles are proportional, so HA  HM ; that
Use the Angle Bisector/Opposite Side Conjecture to  
write a proportion, and solve it to find x. is, corresponding angle bisectors are proportional to
x
   2 corresponding sides.
1x 1
x 2 18. H
(1  x)  
1  x  (1  x)  1
E
 
x   2 (1  x) L O V M A T

x  
2 x  
2 Possible answer: Consider similar triangles LVE
and MTH with corresponding altitudes EO  and
x 1  
2   
2

HA . You need to show that the corresponding
x 2
altitudes are proportional to corresponding sides,
1   2 EO EL
for example, HA  HM . If you show that LOE
1 1 1 1  
Area of ADC  2bh  2(1  x)  1  2  2x EO EL
MAH, then you can show that  HA  HM . You
 
1 1
 2  2  
1  2
2  know that L  M. Because EO  and MA are
altitudes, LOE and MAH are right angles, so
1 1   2 LOE  MAH, and LOE MAH by AA. You
 2    
2
1   2 21   2 EO EL
can then write the proportion  HA  HM , which
 
1   2   2 1
    shows that the corresponding altitudes are propor-
21   2 21   2 tional to corresponding sides.
Area of BDC  Area of ABC  Area of ADC
19. a. i. a  90°  50°  40°; b  90°  a  50°;
1 1
 2  1  1  2(1  x)  1 c  90°  b  40°
1 1 1 1 ii. a  90°  30°  60°; b  90°  a  30°;
 2  2  2x  2x
c  90°  b  60°
1
 2    2
  2
iii. a  90°  68°  22°; b  90°  a  68°;
1   2 21   2
c  90°  b  22°
 
2
  2 Conjecture: The altitude to the hypotenuse of a
Area of BDC 2 1
   right triangle divides the triangle into two right
Area of ADC
1 triangles that are similar to each other and to the
21  2
original right triangle.

 21  
2
 2
  b. i. h

r  hs. By the AA Similarity Conjecture, the
21  2 1 two smaller right triangles are similar to each
2 other and to the large right triangle. (See
1 Lesson 11.2, Exercise 8.) To keep correspond-
The ratio of the greater area to the lesser area ing sides straight, think in terms of shorter
2 leg, longer leg, and hypotenuse. The propor-
is 1 .
tion hr  hs says that the ratio of the length of
16. Label the third vertex C. Construct segment CD, the shorter leg to the length of the longer leg
AD 2x
which bisects C.  DB  3x .
  is the same in the two smaller right triangles.
y
17. E
H
ii. 
h  hx. This proportion says that the ratio of
the length of the longer leg to the length of
L O V M A T the shorter leg is the same in the two smaller
triangles.

Discovering Geometry Solutions Manual CHAPTER 11 217


©2003 Key Curriculum Press
DG3SM586_182-228.qxd 8/2/06 6:07 PM Page 218

n h
iii. 
h  m. This proportion says that the ratio of by the AA Similarity Conjecture. Therefore
the length of the longer leg to the length of corresponding sides are proportional.
the shorter leg is the same in the two smaller AB CB
  
right triangles. XB YB
AB CB
Conjecture: The altitude (length h) to the   1    1
XB YB
hypotenuse of a right triangle divides the AB XB CB YB
      
hypotenuse into two segments (lengths p and q) XB XB YB YB
p AX CY
such that h  hq.   
XB YB
a
20. If 
b  dc, then 24. a. 2a  b. If the rectangles are congruent, they
a c must have the same dimensions.
  1    1
b d
a b c d
      
b b d d
ab cd I a II 2a
  
b d b
21. The ratio will be the same as the
b
_
ratio for the original rectangle. 2
It will be 12 if the rectangle is
From this figure, you can see that the rectangles
divided like this:
will be congruent if 2a  b, or a  2b.
It might be any ratio if the rectangle
b. 2a  b. Perimeter of rectangle I  2a  2b
is divided like this:
and perimeter of rectangle II  2(2a)  22b 
4a  b. The perimeter of the new rectangle will
22. The four triangles are similar to the original by the
be equal to the perimeter of the original if 4a 
SSS Similarity Conjecture or the SAS Similarity
b  2a  2b, or 2a  b, the same relationship as
Conjecture. The sides of the inner small triangle are
in 24a.
midsegments of the original triangle, so each is half
as long as the opposite side. The figure below shows c. All values of a and b. Area of rectangle I  ab
sides that are congruent by the Triangle Midsegment and area of rectangle II  (2a)2b  ab. Thus,
Conjecture. From the figure, you can see that each the area of the new rectangle is equal to the area
of the four smaller congruent triangles is similar of the original for all values of a and b.
to the original triangle by the SSS Similarity
d. No values of a and b. If the rectangles are similar
Conjecture.
but not congruent, they cannot have the same
area. In 24c, you found that the rectangles always
have the same area, so there are no values of a
and b for which the new rectangle is similar, but
not congruent, to the original.

You can also use the SAS Similarity Conjecture 25. 9,120 m3, or approximately 28,651 m3. The
by using opposite angles of parallelograms volume of the truncated cone is the difference
(Parallelogram Opposite Angles Conjecture) as the between the volume of the large cone and the
included congruent angles to show that the four volume of the small cone. Recall that the
small triangles are congruent and common angles to volume of a cone is given by the formula
show that any of the three outer small triangles is V  BH  13r 2H.
similar to the original. Therefore all four small Vtruncated cone  Vlarge cone  Vsmall cone
triangles must be similar to the original.
1 1
AX CY  3(36)2(30)  3(24)2(20)
23. Possible answer: 
XB  
YB
B  9,120  28,651 m3
s s 2  s
26. 
22
 4s, or  4 . Connect the centers of two of
X Y
the larger circles and the center of the small circle
to form a 45°-45°-90° triangle. (This is a right
A C
triangle because two sides are parts of the diagonals
BXY  BAC by the CA Conjecture, and B of the square, and it is isosceles because the lengths
is common to both triangles, so BAC BXY

218 CHAPTER 11 Discovering Geometry Solutions Manual


©2003 Key Curriculum Press
DG3SM586_182-228.qxd 8/2/06 6:07 PM Page 219

of two sides are the sum of radii of the larger and LESSON 11.5
smaller circles.)
EXERCISES
1. 18 cm2. The ratio of the lengths of corresponding
sides of the similar triangles (smaller triangle to
s ME 6 1
CT  12  2 . Then by the
larger triangle) is    
Area of MSE
Area of CAT   2  ,
1 2
Proportional Areas Conjecture,  
Area of MSE
s
so  72  14. Therefore area of MSE 
1
(72)  18 cm2.
4
Each of the larger circles has diameter 2s and
Area of ANGL  16   LA  , so
Area of RECT 9 TR 2 TR
therefore radius 4s. In the isosceles right triangle, 2. 18.     
LA  34. Then
TR 3
the hypotenuse is formed from the radii of two   , so TR  6(3)  18.
24 4
larger triangles, so the length of this hypotenuse
Area of TRAP  25   5  , so the
Area of ZOID 16 4 2
3. a  5, b  10.    
is 24s  2s. Therefore, by the Isosceles Right
ratio of the lengths of corresponding sides of the
Triangle Conjecture, the length of each leg is
rectangles (smaller to larger) is 45. Use proportions
2s
s
 s
      1  s
2 involving corresponding sides to find a and b.
2  2 2 2 2 2 4 4 4 8
  , so a  5, and   , so b  10.
1 5 a 5 b

The radius of the smaller circle is the difference 4. 27 cm2. rs  35, so the ratio of the areas of the
between the length of a leg of the isosceles right semicircles (smaller to larger) is 352  295 . Let A
triangle and the radius of a larger circle, so the represent the area of semicircle R. Then
s
radius of the smaller circle is 
2 2
 4s. Area of semicircle R A 9
    . Solve the proportion
 Area of semicircle S 75 25
The expression for this radius can be written as a to find A.
single fraction if you first rationalize the denomi- A 9
  
nator of the first fraction and then subtract 75 25
A 9
fractions with a common denominator. 75   75  75  25
 
s  s s 2 s
  4    
s s 2
 4    4 A  27
22 2 2 2 22
s2 s s 2 s The area of semicircle R is 27 cm2.
 4 4
 
4 1
5. . The ratio of the lengths of corresponding
49
EXTENSIONS diagonals is the same as the ratio of the lengths
A. True. The ratio of the lengths of corresponding of corresponding sides, so by the Proportional
medians and the ratio of the lengths of correspond- Areas Conjecture, the ratio of the areas of the
ing altitudes both equal the ratio of the lengths of kites (smaller to larger) is 172  419 .
corresponding sides, so they equal each other.
6. 13. If the ratio of the areas is 19, by the Proportional
B. If the sides of the larger triangle are a, b, and c, Areas Conjecture, the ratio of the lengths of corre-
then the sides of the smaller triangle are 23a, 23b, and sponding sides will be 
1

1
9  3 . By the Proportional

2
c. Therefore the ratio of the perimeters is Parts Conjecture, the ratio of the lengths of alti-
3
2 2 2 tudes is equal to the ratio of the lengths of corre-
3a  3b  3c
2
   sponding sides, so the ratio of the lengths of
abc 3
corresponding altitudes is also 13.
If the altitude of the larger triangle (from say side a)
m2
or m 2
is h, then the corresponding altitude (from side 23a) n  . Draw cubes with sides of lengths m
7.  
n2 ,
is 23h. Therefore the ratio of areas is and n and find their surface areas.

  
12 23a 23h
  9  3
12ah
4

2 2
m
n

The ratio of the areas is less than the ratios of the Area  6m2 Area  6n2

 
length if the scale factor is less than one; it is 6m2 m2 m 2
greater if the scale factor is greater than one. Ratio of areas    
6n 2  n2  n
C. See the solution to Take Another Look activity 3 on
page 228.

Discovering Geometry Solutions Manual CHAPTER 11 219


©2003 Key Curriculum Press
DG3SM586_182-228.qxd 8/2/06 6:07 PM Page 220

8. The ratio of the volumes is 217 . The ratio of the 13. Volume of small cylinder  1944 ft3  6107.3 ft3.
3
surface areas is 19. 13  217 and 132  19. Volume of large cylinder 64

Volume of small cylinder  .
27 H  32 ft. Volume of
9. Possible answer: Assuming the ad is sold by area,
small cylinder  BH  r 2H  (9)2(24) 
Annie should charge $6,000. If the length and width
Volume of large cylinder
are both doubled, the area will be multiplied by 4, 1944 cm3  6107.3 cm3.  
Volume of small cylinder
so the bill will also be multiplied by 4: 4($1,500)  4608 72  64 H3
    , so  
64 H 3 64
  . Then 3 
$6,000. 1944 72  27 27 24 27 24
64 H3 64
, or   . 13,824  512(27), so H 3 
27 13,824 27
10. 1715 cm3. By the Proportional Parts Conjecture, 3
the ratio of the volume of the small pyramid to 512(64)  32,728. Then H     32 ft.
32,728
3
the volume of the large one is 47  64
.
343 Let V 14. 125
. By the Proportional Volumes Conjecture, the
27
represent the volume of the large pyramid. ratio of the volumes of the prisms (larger to
64
  
320 smaller) is 533  
125

27 .
343 V
64 320 15. 25. By the Proportional Volumes Conjecture,
343V  343  343V  V
 
the ratio of corresponding heights is 3 

8

2
125  5 .

64V  343  320
16. 23. For objects made of the same material (or any
343  320
V 64  343  5  1715 objects with the same density), weight is propor-
The volume of the large pyramid is 1715 cm3. tional to volume, so the ratio of volumes of the
two balls (smaller to larger) is 287 , and the ratio
11. H  16 cm, h  4 cm. Volume of large cone 
of radii is 3
8 2
  , so the ratio of diameters is
768 cm3  2412.7 cm3. Volume of small cone  2
27 3
Volume of large cone 64 also 3.
12 cm3  37.7 cm3. 
Volume of small cone  1 .

17. $1,953.13. The two warehouses are similar rectan-
By the Pythagorean Theorem, H  16 cm. Because
the ratio of the radius of the large cone to the gular prisms in which the ratio of the lengths of
radius of the small cone is 132  41, h  14(16)  the corresponding edges is 21.5, so the ratio of their
4 cm. volumes is 21.53. Therefore the volume of the large
1
Volume of large cone  BH  3r 2H warehouse is (2.5)3  15.625 times the volume of
1 the small warehouse, and the daily cost of cooling
 3(12)2  16
the large warehouse is 15.625 times the cost
 768 cm3  2412.7 cm3 of cooling the small warehouse: 15.625($125) 
1 1
Volume of small cone  3r 2H  3(3)2  4 $1,953.13.
 12 cm3  37.7 cm3 18. 2432 lb. The ratio of the weights of the two statues
Volume of large cone 768 cm3 768 64 is equal to the ratio of their volumes, 413  614 , so
Volume of small cone  
  
12 cm3  12  1 the weight of the large statue is 64 times the weight
h Volume of large prism
of the smaller statue: 64(38 lb)  2432 lb.
12. H  35.  125
Volume of small prism  27 . Volume of large

Area of base of small prism
19. Possible answer: No, 1448 3  7x, so a 4-foot chicken
9
prism  1500 cm3.  Area of base of large prism  25 
 that is similar to a 14-inch 7-pound chicken would
H , so H  

h 2 h 9 3 Volume of large prism weigh approximately 282 lb. It is unlikely that the
25  5 . Then, Volume of small prism 
  
legs of the giant chicken would be able to support
h  3  27 .
H 3 5 3 125
its weight.
Let V represent the volume of the large prism. 20. Possible answer: Assuming the body types of the
125 V goliath frog and the gold frog are similar, the gold
  
27 324 frog would weigh about 0.0001 kg, or 0.1 g. To find
125 V
(27)(324)  2
7  (27)(324)  324
 the scale factor, the lengths of both frogs must be
given in the same units. Convert 9.8 mm to meters:
(324)(125)  27V 9.8 mm  0.0098 m. Now compare the lengths of
324  125 the two frogs.
V 27  1500
Length of goliath frog 0.3 m 30.6

Length of gold frog    
The volume of the large prism is 1500 cm3. 0.0098 m  1

220 CHAPTER 11 Discovering Geometry Solutions Manual


©2003 Key Curriculum Press
DG3SM586_182-228.qxd 8/2/06 6:07 PM Page 221

Now find the ratio of the weights, which is the same 26. 60 ft2. This is an isosceles triangle.
as the ratio of the volumes of the frogs. Draw the altitude to the base.
Weight of goliath frog
  
30.6 3
Weight of gold frog  1  1
(30.6)3 The altitude to the base of an
isosceles triangle divides the triangle
13 ft
12 ft
13 ft

Let w represent the weight of the gold frog. into two congruent right triangles. 5 ft
10 ft
(30.6)3 3.2 Each of these triangles has a
   hypotenuse of length 12 ft and a
1 w
3.2
w 3  0.0001 kg, or 0.1 g
(30.6)
shorter leg of length 5 ft, so these are 5-12-13
triangles, and each of the longer legs has length
The gold frog would weigh about 0.1 g. 12 ft. This is the length of the altitude to the base,
so A  12bh  12(10)(12)  60 ft2.
21.
27. True (excluding the cross section through the vertex
of the pyramid)
22. 7.1 m 14.2 m 14.2 m 6.5 m 28. Two pairs of angles are congruent, so the triangles
are similar by the AA Similarity Conjecture.
However, the two sets of corresponding sides are
10 m
not proportional 6800  
135 , so the triangles are not
105

42 m
similar.
PROJECT
Project should satisfy the following criteria:
● The table of measured objects and the graph of data
42 m with the line of best fit are complete. The graph may
16 compare the shorter side to the longer side or
23. x  y  230 . Let h represent the length of the
,
3 compare the length-width ratio to one of the sides.
altitude to the hypotenuse of the large triangle.
First look at the small right triangle on the left. ● In the graph (shorter side, longer side), the slope of
the line of best fit represents the ratio of sides.
The hypotenuse of this triangle has length 5, and
the shorter leg has length 3, so this is a 3-4-5
triangle, and the longer leg has length 4. The con-
jecture developed in Lesson 11.4, Exercise 19b says
that the altitude to the hypotenuse of a right
triangle divides the hypotenuse into two segments
p
of lengths p and q such that h  hq. Applying that
conjecture to this figure gives 34  4x. Solve this
proportion to obtain x  136 . Now apply the
Pythagorean Theorem to the right triangles with
legs of lengths x and h and hypotenuse of length y. ● In the graph (shorter side, ratio of longer side to
shorter side), the slope may be nearly zero.
16 2
  256 144 400
y 2  x 2  h2  3  42  9  9  9

400
y  9  3
20

24. 8. By the Trapezoid Midsegment Conjecture,


EF  12(20  36)  12(56)  28. Now apply the
Triangle Midsegment Conjecture to BCD to
find YF  12(CD)  12(20)  10 and to ACD to
find EX  12(CD)  10. From the figure, EX 
XY  YF  EF, so XY  28  10  10  8.
25. 105.45 cm2. Use the Regular Polygon Area
Conjecture.
1 1
A  2aP  2(5.7)(37)  105.45 cm2

Discovering Geometry Solutions Manual CHAPTER 11 221


©2003 Key Curriculum Press
DG3SM586_182-228.qxd 8/2/06 6:07 PM Page 222

● Other representations of the data might include a LESSON 11.6


distribution of the ratios along with the plotted
mean value. EXERCISES
1. 5 cm. The small triangle at the top of the figure
is similar to TWE by the AA Similarity Conjec-
4 a 4 a 1 a
4  12  20 , or 16  20 . Then 4  20 , so
ture, so       
a  5 cm.
2. 3313 cm. Similar triangles are formed in the same
way as in Exercise 1, so corresponding sides are
proportional.
20 15
  
● The equation of the line of best fit is used to make b 15  10
predictions about other rectangles. 20 15
  
● It is unlikely that students will find the golden ratio, b 25
but their explanation might mention it. 20 3
  
b 5
Extra credit
20 3
5b  b  5b  5
● Predictions are checked with the measurements of
other rectangles. 100  3b
Other representations of the data are included. 100 1
● b  3  333 cm
● Students gather more data or combine data. 3. 45 cm. Similar triangles are formed in the same way
Golden Rectangle Extensions as in Exercises 1 and 2.
40 60
A. See the solution for Lesson 11.6, Exercise 26 on   
70 c  60
page 225.
4 60
  
B. The ratio of consecutive Fibonacci numbers 7 c  60
approaches the golden ratio. 4 60
7(c  60)  7  7(c  60)   
c  60
EXTENSIONS
4(c  60)  420
A. These laws seem to be analogous to the Propor-
tional Areas Conjecture. 4c  240  420
B. You can use the relationship (ratio of sizes of 4c  180
n-dimensional parts)  (scale factor)n with the 180
c  4  45 cm
understanding that the ratio of the fractal to the
next-smaller similar piece (which must have the You can also find c by writing the proportion
60 c  60
same dimension) is 3, whereas the corresponding   , which compares corresponding side
40 70
scale factor is 2, so the equation becomes 3  2n. lengths within each triangle. This will also lead to
By approximations or logarithms, the dimension n c  45 cm.
can be found to be about 1.58. 4. 21 cm. Use the Parallel/Proportionality Conjecture.
C. If the base and height of one triangle are B and H, 24 36
  
then abB and abH are the base and height of the other 14 d
12 36
2
triangle. Its area, then, is 12abBabH  ab 12BH,   
7 d
which is b times the area of the first triangle.
a 2
36  3(12), so d  3(7)  21 cm.

EXPLORATION • WHY ELEPHANTS HAVE BIG EARS


IMPROVING YOUR VISUAL THINKING SKILLS
The only way to paint on all but Three painted faces
exactly 60 small cubes is to paint
three faces of a 5-by-5-by-5 cube.

222 CHAPTER 11 Discovering Geometry Solutions Manual


©2003 Key Curriculum Press
DG3SM586_182-228.qxd 8/2/06 6:07 PM Page 223

5. 28 cm. Use the Parallel/Proportionality Conjecture. Because XY GO and XY


 FR
, GO
 FR
, so FROG
36 e
   is a trapezoid.
45 35
4 e
   12. a  32 cm, b  62 cm. There are several ways
5 35 to find a and b. Two approaches are shown here.
35  7(5), so e  7(4)  28 cm.
First solution: By the distance formula, a  b 
6. No. By the converse part of the Parallel/ (12  3) 2
 (0  9)2  9 2  92 

Proportionality Conjecture, r AN  if 15  25 . But  2 • 9  9


2 2 . Using corresponding angles
36 55
  
15 3  5
 
5
, and 25
 
5
  5
 
5
 , so the sides
36 3  12 12 55 5  11 11 formed when the parallel lines are cut by the
are not divided proportionally. Therefore, r is not transversals that are two sides of the large triangle,
.
parallel to AN the small and large triangles are similar by the
 if 48  54 . To compare these two ratios, AA Similarity Conjecture. From the similar
7. Yes. m FL 56 63 triangles,
rewrite each of them in lowest terms.
4 a a
48 86 6     
12 a  b
    7 9 2
56 87 1 a
54 96 6 
    7 3 9 2
63 97
92  32(3), so a  32(1)  32. Then
The sides are divided proportionally, so m FL .
b  92  a  92  32  62.
8. m  6 cm, n  4.5 cm. Apply the Extended Second solution: First use the distance formula to
3
Parallel/Proportionality Conjecture. 24  m 1
, or  
2 find a  b  92, as shown in the first solution.
3 2 3 9
, so m  6 cm.   , so 2n  9, and n   cm, Then b  9 2  a. Apply the Parallel/Proportion-
m 3 n 2
or 4.5 cm. ality Conjecture.
9. w  1313 cm, x  21.6 cm. Use the Extended 4 a
  
8 92 a
Parallel/Proportionality Conjecture.
1 a
12
  
x 18 10
     
10 18 24 w 2 92 a
6 x 3 10 1
2  a  2  2(92  a)  
      a
5 18 4 w 29
9 2 a
6 x 10 3
90  5  90  18 4w  w  4w  4 92  a  2a
108  5x 40  3w 2  3a
9
108 40
x  5 w  3 a  3
2 cm
1
 21.6 cm  133 cm b  9
2  3
2  6
2 cm
. No, n is not parallel to EA
10. Yes, m EA . No, m is 13. This construction follows the method used
not parallel to n. m EA  if   12 5 5 in Example C on page 606 of your book. The
40  35 25  20 , or
25 15 25 1
  . This is true because    and   .
15 1 Extended Parallel/Proportionality Conjecture guar-
75 45 75 3 45 3 antees that the sides of the triangle are divided
 if  25  40 15 65 15
n EA   , or   . This is false
35 25  20 35 45 proportionally, and therefore EF is divided into five
because 6355  173 and 1455  13. equal parts.
 but n is not parallel to
Because m is parallel to EA
, m and n can’t be parallel to each other.
EA
 GO
11. Yes, XY . Yes, XY
 FR
. Yes, FROG is a trapezoid.
 GO
First look at ZXY. XY  if 24  36 , or 3  3,
8 12
which is true. Now look at ZFR. XY  FR  if
24  8 36  12 32 48
  , or   , or 2  2, which is true.
16 24 16 24
E F

Discovering Geometry Solutions Manual CHAPTER 11 223


©2003 Key Curriculum Press
DG3SM586_182-228.qxd 8/2/06 6:07 PM Page 224

14. I J 18. Possible proof:


a b
  
c d
a b
cd  c  cd  d
ad  cb
ad  ab  cb  ab
a(d  b) b(c  a)
  
ab ab
 into six equal parts using the method
First divide IJ db ca
  
of Example C and Exercise 13. Then draw a circle b a
 as radius. Mark off
with one of the equal parts of IJ So two pairs of corresponding sides of XYZ
six radii around the circle and connect the endpoints and XAB are proportional. X  X, so
of the arcs to form a regular hexagon. The perimeter XYZ XAB by the SAS Similarity Conjecture.
of the hexagon is six times the length of one of the Because XYZ XAB, XAB  XYZ. Hence,
, so the perimeter is IJ
equal parts of IJ .  YZ
AB  by the Converse of the Parallel Lines
Conjecture.
15. Extended Parallel/Proportionality Conjecture
19. Set the screw so that the shorter lengths of the
styluses are three-fourths as long as the longer
lengths. The two isosceles triangles will always be
similar because their vertex angles are congruent
vertical angles.
20. x  4.6 cm, y  3.4 cm. The two transversals to the
16. You should connect the two 75-marks. By the parallel lines can be extended until they intersect,
Extended Parallel/Proportionality Conjecture, forming a triangle, so you can apply the Extended
drawing a segment between the 75-marks will Parallel/Proportionality Conjecture.
75
form a segment that has length  
100 , or 75%, of y
3 7 8
the original.      
7 8 4 x
17. 2064 cm3  6484 cm3. Let x 7y  24 7x  32
represent the height of the small x 24 32
10 cm y  7  3.4 cm x  7  4.6 cm
missing cone. Use the similar
triangles to write a proportion; 12 cm 21. x  9.8 yd, y  8.7 yd, z  6.5 yd. Apply the
then solve the proportion to find x. 16 cm Extended Parallel/Proportionality Conjecture.
x x  12
   Road frontage  9  8  6  23 yd
10 16
x x  12 x 25
80  10  80  16
   
9 23
8x  5(x  12) 23x  225 yd
225
8x  5x  60 x  3  9.8
3x  60 y 25
  
8 23
x  20 23y  200
The height of the small cone is 20 cm. 200
y  2
3  8.7 yd
1 1
Vlarge cone  BH  3r 2H  3(16)2(32)
z  15  x  y  25  9.8  8.7  6.5 yd
8192
 3 cm3 22. 343
. Let m
729 n represent the ratio of side lengths of the
1 1 two cubes (smaller to larger). Then 4891  m , so
2
Vsmall cone  BH  3r 2H  3(10)2(20) n
m 7
   (Proportional Areas Conjecture), and there-
2000 n 9
 3 cm3 fore the ratio of the volumes of the two cubes is
9  729 (Proportional Volumes Conjecture).
7 3 343
8192 2000 6192
Vtruncated cone  3  3  3
 2064 cm3  6484 cm3

224 CHAPTER 11 Discovering Geometry Solutions Manual


©2003 Key Curriculum Press
DG3SM586_182-228.qxd 8/2/06 6:07 PM Page 225

23. She is incorrect. She can make only nine 8 cm 27. Possible proof:
diameter spheres. Given: Circumscribed quadrilateral ABCD, with
Small cookies: d  4 cm, r  2 cm points of tangency P, Q, R, and S
Show: AB  DC  AD  BC
Large cookies: d  8 cm, r  4 cm
B
Radius of large cookie 4 cm 2
    
Radius of small cookie 2 cm 1
By the Proportional Volumes Conjecture, the ratio P
Q
of corresponding volumes is 213  81. Therefore
A
it will take 8 times as much dough to make 36
cannonballs with 8 cm diameter as with 4 cm diam- C
S
eter, but doubling the recipe gives only twice as
R
much dough. Thus, she can make only 14 as many
large cannonballs as small ones: 14(36)  9. D

24. 6x 2; 24x 2; 54x 2. Each face of a cube is a square If you use segment addition, you can show that
whose area is the square of the edge length, and each sum of the lengths of opposite sides is com-
every cube has six congruent square faces. posed of four lengths, AB  CD  (AP  BP) 
(DR  CR) and AD  BC  (AS  DS)  (BQ 
Edge x: Surface area  6x 2
CQ). By the Tangent Segment Conjecture you know
Edge 2x: Surface area  6(2x)2  6(4x 2)  24x 2 that AP  AS, BP  BQ, CR  CQ, and DR  DS,
and the four lengths in each sum are equivalent.
Edge 3x: Surface area  6(3x)2  6(9x 2)  54x 2
Here are the algebraic steps to show that the whole
25. 13r. The chord and the two radii r
sums are equivalent.
that meet the circle at the
r 60° r
AB  DC  (AP  BP) Segment addition.
endpoints of the minor arc
 (DR  CR)
form an equilateral triangle,
so the central angle is 60°.  (AS  BQ) Substitute AS for AP,
 (DS  CQ) BQ for BP, DS for
Let C represent the circumference DR, and CQ for CR.
of the circle.
60 1 1  (AS  DS) Regroup the measure-
Arc length   C (2r)  r
360  6 3  (BQ  CQ) ments by common
26. a. Possible construction method: Use the triangle- points of tangency.
and-circle construction from Lesson 11.3, Exer- AB  CD  AD  BC Use segment addition
. Then
cise 17 to locate the golden cut, X, of AB to rewrite the right
use perpendicular lines and circles to create a side as the other sum
rectangle with length AB and width AX. of opposite sides.

b. Possible construction method: Construct golden 28.


rectangle ABCD following the method from 26a.
For square AEFD, locate EF by constructing circle
A and circle D each with radius AD. Repeat the
process of cutting off squares as often as desired.
For the golden spiral from point D to point E, IMPROVING YOUR VISUAL THINKING SKILLS
construct circle F with radius EF; select point D, A. B. C.
point E, and circle F and choose Arc On Circle
from the Construct menu.

D. This shape cannot be made.

Discovering Geometry Solutions Manual CHAPTER 11 225


©2003 Key Curriculum Press
DG3SM586_182-228.qxd 8/2/06 6:07 PM Page 226

EXTENSIONS 5. w  6 cm, x  4.5 cm, y  7.5 cm, z  3 cm.


A. See the solutions to Take Another Look activities 6 ABCDE FGHIJ, so corresponding sides are
and 7 on page 229. proportional.
AB BC CD DE AE
B. V         
c FG GH HI IJ FJ
a 6 4 3 5 2
M T
ᐉ         
e
d
9 w x y z
b 2 4 3 5 2
        
L f U 3 w x y z
4 2
From   , w  6 cm.
, then LUV MTV by the AA
If  LU
w 3

Similarity Conjecture. From x  3, x  92  4.5 cm.


3 2

If LUV MTV, then ab



cd
 From 5y  23, y  125  7.5 cm.
a c .
If ab

cd

a
  c
. From 2z  23, z  3 cm.
a c , then b d
Possible completion of proof: 6. x  416 cm, y  712 cm. ABC DBA, so corre-
ab cd sponding sides are proportional.
   AB BC AC
a c     
a b c d DB BA DA
       5 6 9
a a c c     
b d x 5 y
1  a  1  c From 5x  65, 6x  25, and x  265  416 cm.
b d a c 6
 9y, 6y  45, and y  465  712 cm.
  , so    From 
5
a c b d
7. 13 ft 2 in. First change 5 ft 8 in. to 523 ft, 11 ft 3 in.
EXPLORATION • TWO MORE FORMS OF VALID REASONING to 1114 ft, and 8 ft 6 in. to 812 ft. Let h represent the
height of the tree. Use the similar right triangles to
EXTENSION write a proportion; then solve the proportion to
Investigation results will vary. find h.
2
53
h  
CHAPTER 11 REVIEW  1 1 1
114  82 82
EXERCISES
2
1. x  24. 15  3(5), so x  3(8)  24. 53
h 
 3 1
2. x  66. Multiply both sides of the given proportion 194 82
by the product of the denominators, 11x.
17
4
11x  1
24 3
1  11x  x
 17 2 2

h   3  1
73

7 9 17
4x  264 4 
2
x  66 h   2

7 9 3
3. x  6. Multiply both sides of the given proportion 4
79 2 158 1
by the product of the denominators, 9x. h  4  3  1 
2  136 ft  13 ft 2 in.
4 x
9x  x  9x  9 8. It would still be a 20° angle. A dilation doesn’t
36  x 2 change angle measures.
x  6 9. The method from Lesson 11.6,
Example C can be used to
4. x  17. Writing 3440 in lowest terms as 17

20 allows you  into seven equal
divide KL
to work with smaller numbers. lengths. However, once you
x 17
   have constructed the segment
x  3 20
that connects the point where
x 17
20(x  3)  
x  3  20(x  3)  20
  the seventh arc intersects the
20x  340, so x  17 ray to L, it is necessary only
to connect the point where
K P L
the third of the seven arcs

226 CHAPTER 11 Discovering Geometry Solutions Manual


©2003 Key Curriculum Press
DG3SM586_182-228.qxd 8/2/06 6:07 PM Page 227

 by constructing a parallel
intersects the ray with KL 14. 54. By the Proportional Areas Conjecture, the ratio
line. Let P be the point where this line intersects of the radii is 

25 5
16  4 .
 
. Then KP  3(KL) and PL  4(KL), so KP  3.
KL 7 7 KL 4 15. $266.67. The ratio of corresponding sides of the
10. 15 m. Make a sketch. Smith deck to the Jones deck is 1162  2105  43.
A Therefore the ratio of the areas of the decks is
2m
3  9. Let S represent the cost of staining the
B 4 2 16
E 4m
Smith family’s deck.
S 16
  
20 m
150 9
S 16
450  150  450  9
 
r 3S  800
C D
800
S  3  266.67
ABE ADC by the AA Similarity Conjecture.
(A is a common angle, and each triangle has a It will cost the Smith family $266.67 to have their
right angle.) deck stained.
AB AD 16. 640 cm3. Use the Proportional Volumes Conjec-
  
BE CD ture. The ratio of the dimensions of the small
2 3 26 cylinder to the large cylinder is 23.
  
2 r Volume of small cylinder
   287
2 3
26   
3  r Volume of large cylinder 3
Let V represent the volume of the small cylinder.
r 3  26
8 V
26   
r    15 m 27 2160
3 2160  80(27), so V  80(8)  640 cm2.
11. 4 gal; 8 times. The ratio of corresponding edges of
the boxes (larger to smaller) is 21, so the ratio of 17. w  32, x  24, y  40, z  126. Apply the
Extended Parallel/Proportionality Conjecture to
surface areas is 212  41 (Proportional Areas
find w, x, and y.
Conjecture), and the ratio of volumes is 213  81.
36 48
Therefore Lucy’s box will have 4 times the surface   
24 w
area of Charlie’s box, so she will need 4 gal of paint, 3 48
  
and her box has 8 times the volume of his box. 2 w
48  16(3), so w  16(2)  32.
12. Possible answer: You would measure the height and
36 48
weight of the real clothespin and the height of the   
18 x
sculpture.
2 48
Wsculpture Hsculpture 3   
 
1 x
   x  24
Wclothespin Hclothespin
If you don’t know the height of the sculpture, you 36 48
  
could estimate it from this photo by setting up a 30 y
6 48
ratio, for example   
5 y
Hperson Hsculpture 48  8(6), so y  8(5)  40.
  
Hperson’s photo Hsculpture’s photo
The small triangle at the top of the figure (with
13. 9
. The ratio of the lengths of the corresponding base of length 42) is similar to the large triangle
49 (with base of length z). Use a proportion based on
sides is equal to the ratio of the perimeters because
the perimeter is the sum of the lengths of the sides. this similarity to find z.
Therefore the ratio of the lengths of corresponding 36  24  18  30 36
  
sides is 37, so by the Proportional Areas Conjecture, z 42
the ratio of the areas of the two parallelograms 108 6
  
(smaller to larger) is 372  499 . z 7
108  18(6), so z  18(7)  126.

Discovering Geometry Solutions Manual CHAPTER 11 227


©2003 Key Curriculum Press
DG3SM586_182-228.qxd 8/2/06 6:07 PM Page 228

18. 841 coconuts. Apply the Proportional Areas 20. Possible answer: If food is proportional to body
1
Conjecture. volume, then  
8000 of the usual amount of food is
Height of statue required. If clothing is proportional to surface area,
58 29
     then 1

400 of the usual amount of clothing is
Height of statuette 2 1
required. It would take 20 times longer to walk a
Surface area of statue
 
29 2 841
 given distance.
Surface area of statuette  1  1
21. The ice cubes would melt faster because they have
Because it will take the milk of one coconut to
greater surface area.
cover the surface of the statuette, it will take
841 coconuts to cover the surface of the full-size TAKE ANOTHER LOOK
statue. 1. This rule describes a dilation by a factor of k
19. Because you are concerned with ratios in 19a and followed by a translation by (b, c).
19b, it doesn’t make any difference what lengths you 2. For each vertex, draw a ray from point A through
choose. For convenience, assign the square a side the vertex. Mark off a point on the ray whose
length of 2 before calculating areas and volumes. distance from point A is the scale factor times the
Then the radius of the circle is 1, and the base and distance of the vertex from point A. The coordinates
height of the triangle are each 2. of the vertices are not multiplied by the scale factor,
a. 1 to 4 to 12, or 4 to  to 2. Find the areas of the but the lengths of the sides are.
square, the circle, and the triangle, using 2 as the y y
side length of the square.
Area of square  2  2  4 square units (4, 15)
(–14, 9)
Area of circle    (1)2   square units
( _83 , 7)
10 , 5)
(– __ A (2, 3)
1 A (2, 3) 3
Area of triangle  2(2)(2)  2 square units
x x
Area of square : Area of circle : Area of triangle is (– _43 , 1) 14 , 1)
(__
3
(–8, –3) (10, –3)
4 :  : 2, or 1 : 4 : 12.
b. 3 : 2 : 1. The cylinder has radius 1 and height 2, 3. This is true by the Angle Bisector/Opposite Side
the sphere has radius 1, and the cone has radius Conjecture.
1 and height 2.
4. Figure A shows the positions of the Moon and the
Volume of cylinder  r 2H  (1)2(2) Sun during a total eclipse, relative to an observer at
 2 cubic units point A. The Moon blocks all of the Sun except the
Sun’s corona from Earth’s view. ABC ADE by
4 4
Volume of sphere  3r 3  3(1)3 the AA Similarity Conjecture. AB is the approximate
4 distance from Earth to the Moon. BC is the approx-
 3 cubic units imate diameter of the Moon. AD is the approximate
1 1 distance from Earth to the Sun. DE is the approxi-
Volume of cone  3r 2H  3(1)2(2) mate diameter of the Sun. (Note: The figures on the
2 next page are not drawn to scale. Because the actual
 3 cubic units
distances are so great, relative to the diameters,
Compare these volumes.  and AD
don’t worry that AB  are not precisely
2 3 3 tangent to the spheres of the Moon and the Sun;
    1  4  2
2 2
4 4 nor will we worry about what points on the Moon
 
3 3 and the Sun we use in measuring the distance from
2 3 3 Earth.)
    1  2  1
2 2
2 2 By similar triangles, BAC
B AD

  
DE . If the Moon were
3 3
smaller (or farther away), these ratios wouldn’t be
Therefore the ratio of the volume of the cylinder equal—the Moon wouldn’t block all of the Sun’s
to the volume of the sphere to the volume of the light (Figure B). If the Moon were larger (or closer),
cone is 3 : 2 : 1. it would block an area larger than that of the Sun—
c. Possible answer: It represents the 3 : 2 : 1 ratio of we wouldn’t be able to see the Sun’s corona during
the volumes of the cylinder, sphere, and cone. an eclipse (Figure C).

228 CHAPTER 11 Discovering Geometry Solutions Manual


©2003 Key Curriculum Press
DG3SM586_229-251.qxd 8/2/06 6:11 PM Page 229

The combination of the Moon’s distance and diam-


eter makes it fit “just right” in front of the Sun CHAPTER 12
during a total eclipse.
D
LESSON 12.1
B
A
EXERCISES
C E
1. 0.6018
Figure A
2. 0.8746
B 3. 0.1405
A
x
C 4. 11.57. sin 40°  , so x  18(sin 40°)  11.57.
18
Figure B 5. 30.86. cos 52°  1x9 , so x(cos 52°)  19, and x 
19
  30.86.
B cos 52°
x
A 6. 62.08. tan 29°  , so x  112(tan 29°)  62.08.
112
C
7. sin A  st; cos A  rt; tan A  rs. The length of the
Figure C side opposite A is s, the length of the side adja-
cent to A is r, and the length of the hypotenuse
5. Possible answers include any two triangles, one of is t.
whose sides have lengths a, b, and c, with the sides
8. sin   45; cos   35; tan   43. The right triangle
2
of the other having lengths b, c, and cb. If one
similar triangle has side lengths a, b, and c, then the has horizontal leg of length 6, vertical leg of length
other triangle has side lengths ar, br, and cr, where 8, and hypotenuse of length 10, so sin   180  45,
r is the scale factor. Because two sides must be cos   160  35, and tan   86  43.
congruent without the triangles being congruent, let 7 24 7 24
9. sin A  ; cos A  ; tan A  ; sin B  ;
b  ar and c  rb. Then c  ar  r  ar 2. To find 25 25 24 25
7 24
values of r for which this relationship holds, use the cos B  ;
25 tan B  .
7 To find that the length of
Triangle Inequality Conjecture. a + b  c, the hypotenuse is 25, use the Pythagorean Theorem,
a + ar  ar 2. (Similar equations that reduce to this
or recall that 7-24-25 is a Pythagorean triple.
occur for other pairs of sides.) Solve the inequality:
1  
5 10. 30°. A  sin1(0.5)  30°.
r  
2 11. 53°. B  cos1(0.6)  53°.
In other words, r must be less than the golden ratio. 12. 30°. C  tan1(0.5773)  30°.
6. The converse is not true. One counterexample: 13. 24°. x  tan1
106   24°.
48

4 6
14. 35 cm. tan 30°  2a0, so a(tan 30°)  20, and
20
8 12 a tan 30°  35 cm.

b
6 9 15. 15 cm. sin 65°  , so b  17(sin 65°)  15
17 cm.
16. 105 yd. cos 70°  3c6, so c(sin 70°)  36, and
7. Conjecture: If three sides of one triangle are 36
c cos 70°  105 yd.

parallel to the three sides of another triangle,
then the triangles are similar. 17. 40°. tan d  107
,
128 so d  tan1
128   40°.
107

Proof: Extend all sides so that they intersect. Then 18. 50 cm. cos 15°  4e8 , so e(cos 15°)  48, and
for each angle, apply the Corresponding Angles e 48
cos 15°  50 cm.

Conjecture twice.
19. 33°. sin f  36
,
66 so f  sin13666   33°.
20. 18 in. The radius of the circle is 21 in., so the diam-
eter is 42 in. The angle opposite the diameter shown
in the figure is a right angle because every angle
inscribed in a semicircle is a right angle. Therefore
the triangle is a right triangle with hypotenuse
g
of length 42 in. Then sin 25°  42 , so g 
42(sin 25°)  18 in.

Discovering Geometry Solutions Manual CHAPTER 12 229


©2003 Key Curriculum Press
DG3SM586_229-251.qxd 8/2/06 6:11 PM Page 230

21. Approximately 237 m. Let b represent the length of 26. Box of ice cream. To find the better buy, you first
the base of the quadrilateral (which is a rectangle) need to find the volumes of the two containers. The
and h represent the height. Find the length of the radius of the cylinder is 3 in.
base and then the height: sin 35°  8b5 , so b 
Volume of cylinder  BH  r 2H  (3)2(8)
85(sin 35°)  48.75 m, and cos 35°  8h5, so h 
85(cos 35°)  69.63 cm. Then the perimeter of  72 in.2
the rectangle is 2b  2h  2(48.75)  2(69.63) 
237 m.
Volume of box (square prism)  BH  (6  6)(8)
 288 in.2
22. x  121 ft. x is not the length of a side of a right
Volume of box 288
triangle, but you can find x by subtracting length     1.27
Volume of cylinder 72
a from length b in this figure.
If the box of ice cream were exactly as good a buy
as the cylinder, the box would cost 1.27($3.98) 
$5.05. Because the actual cost of the box is only
$4.98, the box of ice cream is the better buy.
280 ft 27. 83 cm. Make a sketch. Draw radii to the
endpoints of the chord and label the figure for
reference.
55° 75°
x a A
b

280
From the large right triangle, tan 55°   
b , so
280
b(tan 55°)  280, and b   tan 55°  196 ft. From

280 12 cm 4 cm
the small right triangle, tan 75°   
a , so
D
O E
C
280
a(tan 75°)  280, and a   tan 75°  75 ft. Therefore

x  196  75  121 ft.
23. 6.375. To solve this proportion, multiply both sides B
by 24, which is the least common denominator.
x 17 DE  EC  12  4  16 cm. The diameter of
    is a
3 8 the circle is 16 cm, so the radius is 8 cm. OA
radius, so OA  8 cm. Also, OC  is a radius, so

x 17
24 3  24 8   OE  OC  EC  8  4  4 cm. Look at OAE.
8x  51 This is a right triangle with shorter leg of length
51 4 cm and hypotenuse of length 8 cm, so use the
x  8  6.375 Pythagorean Theorem to find the length of the
24. 2.2. To solve this proportion, multiply both sides by longer leg, AE. 42  (AE)2  82, so (AE)2 
11x, the product of the two denominators. 64  16  48, and AE  48   43  cm. By
5 25 the same reasoning (or because OAE  OBE),
   BE  43 cm. Therefore AB  2(43) 
x 11
83 cm; that is, the length of the chord is 83 cm.

5
11x x  11x 1
25
1   28. V  288 ft3, S  144 ft2. Use the formulas for
55  25x the volume and surface area of a sphere.
55 11 4 4 4
x  2 
5  5  2.2 V  3r 3  3(6)3  3  216  288 ft3
25. 16-inch pizza. The ratio of the diameters of the two S  4r 2  4(6)2  4(36)  144 ft2
pizzas is 2106  54, so the ratio of their radii is 54.
IMPROVING YOUR VISUAL THINKING SKILLS
Therefore, by the Proportional Areas Conjecture, the
2
ratio of their areas is 54  2156 . If the 20-inch pizza
were exactly as good a buy as the 16-inch pizza, its
price would be 2156 ($12.50)  $19.53. But the price
of the 20-inch pizza is $20, so the 16-inch pizza is
the better buy.

230 CHAPTER 12 Discovering Geometry Solutions Manual


©2003 Key Curriculum Press
DG3SM586_229-251.qxd 8/2/06 6:11 PM Page 231

EXTENSIONS 3. Approximately 65 m. Let d represent the distance


A. Refer to the diagram at the top of page 622 in your between the ship and the shore.
42
book. tan 33°  d
length of side adjacent to A
cotangent of A   length of side opposite A d(tan 33°)  42
b 42
or cot A  a d tan 33°  65
length of hypotenuse The ship is approximately 65 m from the shore.
secant of A  
length of side adjacent to A
4. Approximately 188 m. Let d represent the distance
c
or sec A  b that the diver needs to walk along the ocean floor to
length of hypotenuse the wreckage. Make a sketch.
cosecant of A   length of side opposite A 12°
40
c
or csc A  a 12°
d
B. The completed table should show the sine, cosine,
Because the angle of depression from the ship to
and tangent for angles measuring from 5° to 85° in
the wreckage is 12°, the angle of elevation from
5° increments, with all values that are not exact
the wreckage to the ship is also 12°. (The angle of
rounded to the nearest thousandth. Here are the
depression is equal to the angle of elevation because
actual values: (See table at bottom of page.)
these are alternate interior angles formed when two
parallel lines are cut by a transversal. See the
LESSON 12.2 diagram at the top of page 627 in your book.)
40
EXERCISES tan 12°  d
1. Approximately 655 m. Let x represent the distance d(tan 12°)  40
between General Han Xin’s position and the palace. 40
Make a sketch. d tan 12°  188
The diver has to walk approximately 188 m along
the ocean floor to the wreckage.
800 m 5. Approximately 227 m. Let h represent the height of
the clouds. Make a sketch.

35°
x

x
cos 35°   
800
x  800(cos 35°)  655 h

The palace was approximately 655 m from General


Han Xin’s position.
2. Approximately 101 m. Let
h represent the height of the 74°
kite. Make a sketch. h 65 m
h
tan 39°    h
125 39° tan 74°  65
125 m
h  125(tan 39°)  101
h  65(tan 74°)  227 m
The height of Benny’s kite is approximately 101 m.
The clouds are approximately 227 m high.

Lesson 12.1, Extension B


Angle 5° 10° 15° 20° 25° 30° 35° 40° 45° 50° 55° 60° 65° 70° 75° 80° 85°
sin 0.087 0.174 0.259 0.342 0.423 0.500 0.574 0.643 0.707 0.766 0.819 0.866 0.906 0.940 0.966 0.985 0.996
cos 0.996 0.985 0.966 0.940 0.906 0.866 0.819 0.766 0.707 0.643 0.574 0.500 0.423 0.342 0.259 0.174 0.087
tan 0.087 0.176 0.268 0.364 0.466 0.577 0.700 0.839 1.00 1.19 1.43 1.73 2.14 2.75 3.73 5.67 11.4

Discovering Geometry Solutions Manual CHAPTER 12 231


©2003 Key Curriculum Press
DG3SM586_229-251.qxd 8/2/06 6:11 PM Page 232

6. Let h represent the height of the balloon above the 11. 2 m. Draw a segment perpendicular to the two
level of the tripod (where the theodolite is placed) parallel lines, as shown below. Let d represent the
and d represent the horizontal distance along the length of this segment.
ground from a point directly below the tripod to a
point directly below the balloon. Make a sketch.

2.7 m d e
1400 m h 62°

44°
d
1m d 1m From the right triangle, sin 62°  ,
2.7 so d 
2.7(sin 62°)  2 m.
a. Approximately 974 m
h Look at the quadrilateral to the right of the triangle.
sin 44°   
1400 One pair of opposite sides is marked as parallel. The
h  1400(sin 44°)  973 other sides are also parallel by the Converse of the
Because the tripod is 1 m tall, the balloon is Parallel Lines Conjecture, using the right angles as
h  1  974 m high. congruent corresponding angles. Because of the
right angles, this parallelogram is a rectangle. Oppo-
b. Approximately 1007 m site sides of a rectangle are congruent by the Paral-
d lelogram Opposite Sides Conjecture, so e  d. Thus
cos 44°   
1400 e  2 m.
d  1400(cos 44°)  1007
12. 22 in. The figure is a kite because it has exactly two
She is approximately 1007 m from a point distinct pairs of congruent consecutive sides. The
directly below the balloon. Kite Diagonals Conjecture says that the diagonals of
c. Yes, the height of the balloon would be 1 m less a kite are perpendicular, and the Kite Diagonal
because you don’t have to account for the tripod. Bisector Conjecture says that the diagonal connecting
The distance to a point under the balloon would the vertex angles of a kite is the perpendicular
not change. bisector of the other diagonal. Therefore, in this
figure, the diagonals divide the kite into two pairs
7. Approximately 1621 m. Let d represent the distance of congruent right triangles, and the shorter
between the ship and the lighthouse at the second diagonal (of length d1) is bisected by the longer
sighting. one. Look at the triangle in the upper right. In
this right triangle, the length of the hypotenuse is
20 in. and the length of the shorter leg is 12d1.
d
1
First Second d1
sighting 42° sighting 90° cos 56°   2
1800 m
20
1
d d1  20(cos 56°)
tan 42°    2
1800 d1  40(cos 56°)  22 in.
d  1800(tan 42°)  1621 m
a 13. 49°. The large triangle is isosceles, and the segment
8. 9 cm. sin 32°  ,
17 so a  17(sin 32°)  9 cm. of length 16 cm is the altitude to the base. The alti-
18
9. 64°. sin x  
20  0.9, so x  sin1(0.9)  64°. tude to the base of any isosceles triangle bisects the
base, so this altitude divides the isosceles triangle
10. 7 cm. Every angle inscribed in a semicircle is a right into two congruent right triangles each with legs of
angle, so the triangle in this figure is a right triangle lengths 16 cm and 14 cm, and with the angle of
whose hypotenuse is a diameter of the circle. The measure f opposite the longer leg.
diameter of the circle is 2r. 16 8
12 6 tan f  14  7

cos 32°  2r  r
r(cos 32°)  6 
8
f  tan1 7  49°
6
r cos 32°  7 cm

232 CHAPTER 12 Discovering Geometry Solutions Manual


©2003 Key Curriculum Press
DG3SM586_229-251.qxd 8/2/06 6:11 PM Page 233

14. 127°. Look at the right triangle inside the cone. 22. The block has a volume of 90 cm3, but it displaces
Let  represent the measure of the larger acute only 62.8 cm3 of water. So not all of the block is
angle of the right triangle (the angle opposite under water, which means it floats.
the 16-meter side). tan   1162  43, so  
Volume of block (rectangular prism)  (3)(5)(6)
tan143  53°. The angles with measures  and
 form a linear pair, so   180°  53°  127°.  90 cm3
Another way to find  is to first find the measure of Volume of displaced water  volume of cylinder
the smaller acute angle of the right triangle. Let  with radius 5 cm and height 0.8 cm  r 2H 
represent this angle measure. Then tan   1126  (5)2(0.8)  20  62.8 cm3
3
  0.75, so   tan1(0.75)  37°. The angle of
4 23. CZ  AX  BY. AX  is the altitude to BC
, BY is
measure  is an exterior angle of the right triangle,
, and CZ
the altitude to AC  is the altitude to AB
.
so, by the Triangle Exterior Angle Conjecture,  
For any choice of base, the area of the triangle
  90°  37°  90°  127°.
must be the same. Therefore the area of ABC is
1 1 1
15. 30°. First look at the base of the rectangular prism. (AB)(CZ)  (BC)(AX)  (AC)(BY ). Multiply
2 2 2
The sides of this rectangular base have lengths 8 ft by 2 to get (AB)(CZ)  (BC)(AX)  (AC)(BY).
and 15 ft, so the length of the diagonal is 17 ft by Because the product of base and height must be the
the Pythagorean Theorem. (You may recall that same for all three base-height pairs, the longest base
8-15-17 is a Pythagorean triple.) Now look at the must go with the shortest altitude, the middle-
right triangle inside the prism, with legs of lengths length base with the middle-length altitude, and
10 ft and 17 ft. In this triangle, the angle of measure the shortest base with the longest altitude. Because
 is opposite the 10-foot side, so tan   1107 and AB  BC  AC, the lengths of the altitudes are
  tan11107   30°. related by the inequality CZ  AX  BY.
h
16. 64 cm. tan 58°  ,
40 so h  40(tan 58°)  64 cm. 24. a. decreases, approaching 0
17. 45°. D  sin1(0.7071)  45° b. decreases, approaching 0
18. 60°. E  tan1(1.7321)  60° c. remains 90°
19. 60°. F  cos1(0.5)  60° d. increases, approaching (AO)2
x
20. a. 15.04. 4.7  , so x  4.7(3.2)  15.04.
3.2 e. decreases, approaching 1
16.4 16.4
b. 2.05. 8   x , so 8x  16.4, and x  8  2.05.
 f. decreases, approaching 1
c. 5.2304. 0.3736  1x4 , so x  14(0.3736)  5.2304. 25. R(8, 7) and E(3, 9), or R(4, 3) and E(1, 1).
One way to find the two possible squares SQRE is
d. Approximately 2.644. 0.9455  2x.5, so 0.9455x 
2.5 to make a careful drawing on graph paper. Draw
2.5, and x   0.9455  2.644.

 and then draw the two squares that have SQ
SQ 
1
21. x  3.5; y  97  9.14. Label B as a side. (You can use a ruler and protractor to
the points in the figure for draw the squares, using the protractor to draw
reference. y
. You can
right angles and the ruler to copy SQ
8
also construct the squares with compass and
A  D by the Inscribed
straightedge.)
Angles Intercepting Arcs A E 7
4
Conjecture. B  C for x C y

the same reason. Therefore 8 E (3, 9)


D
ABE  DCE by the AA
R (8, 7)
Similarity Conjecture. (You could also use the
vertical angles as a pair of congruent angles.) S (1, 4)

Because the triangles are similar, corresponding Q (6, 2)


sides are proportional. x
E (–1, –1)
AB BE AE
     R (4, –3)
DC CE DE
y 8 4
8    
7 x
8 From this diagram, you can see that one of the
From 
7  4x, 8x  28, so x  28

8  3.5. possible squares with consecutive vertices S(1, 4)
y
From 
8  87, 7y  64, so y  64

7  9 17  9.14. and Q(6, 2) has vertices R(8, 7) and E(3, 9), and the
other has vertices R(4, 3) and E(1, 1).

Discovering Geometry Solutions Manual CHAPTER 12 233


©2003 Key Curriculum Press
DG3SM586_229-251.qxd 8/2/06 6:11 PM Page 234

Another approach is to use the facts that the slopes 2. 4 cm2. Use the SAS Triangle Area Conjecture with
of perpendicular lines are negative reciprocals of a  3.1 cm, b  3.1 cm, and C  50°.
each other and that all sides of a square have the 1 1
A  2ab sin C  2(3.1)(3.1)sin 50°  4 cm2
same length to compute the possible coordinates for
R and E. 3. 11,839 cm2. Divide the quadrilateral into two trian-
gles by drawing the diagonal connecting the two
To go from S to E, move right 5 units and down
angles whose measures are not given. Use the SAS
2 units.
Triangle Area Conjecture to find the area of each
Upper square: From S(1, 4), move right 2 units triangle and then add the results.
and up 5 units to E(3, 9). From Q(6, 2), move right
Area of triangle with sides of lengths 124 cm and
2 units and up 5 units to R(8, 7).
115 cm and included angle of 78°:
Lower square: From S(1, 4), move left 2 units and 1 1
A  2ab sin C  2(124)(115)sin 78°  6974 cm2
down 5 units to E(1, 1). From Q(6, 2), move
left 2 units and down 5 units to R(4, 3). Area of triangle with sides of lengths 95 cm and
104 cm and included angle of 100°:
PROJECT
1 1
Project should satisfy the following criteria: A  2ab sin C  2(95)(104)sin 100°  4865 cm2
● Project includes research, narrative, drawings, and a Area of quadrilateral  6,974  4,865  11,839 cm2
model appropriate for the latitude.
4. 407 cm2. The figure is a regular octagon inscribed
● In climates with greater temperature extremes: The in a circle. Draw radii from the center to each
overhang should keep sunshine off the windows in the vertex of the octagon to form eight congruent
summer but allow it to hit the windows in the winter. isosceles triangles. Each of these triangles has vertex
360°
angle of  8  45° and legs of length 12 cm.

● In a tropical climate: Windows on the south and west
The area of each triangle is 12(12)(12)sin 45° 
sides of the model should be smaller and higher, or a
72(sin 45°), so the area of the octagon is
covered patio could be attached to the house so that
8  72(sin 45°)  576(sin 45°)  407 cm2.
the sun’s rays never hit the windows.
5. 35 cm. Use the Law of Sines with a  28 cm,
EXTENSION
b  w, A  52°, and B  79°. Start with the Law
Possible solution: Climb or fly 3 Measured
of Sines and solve for b. (This was also done in
angle
to a point that’s, say, 3 miles high, Example B on page 636 in your book.)
and measure the angle between r
sin A sin B
the horizon and the vertical. r   
a b
Because your line of sight to the b sin A  a sin B
horizon is a tangent line perpendi-
a sin B
cular to a radius, the sine of that b 
sin A
r
angle is  
r  3. 28(sin 79°)
w   sin 52°  35 cm
EXPLORATION • INDIRECT MEASUREMENT 6. 17 cm. Use the Law of Sines with a  12 cm, b  x,
IMPROVING YOUR VISUAL THINKING SKILLS A  37°, and B  58°. Use the expression for b
found in Exercise 5.
a sin B
b 
sin A
12(sin 58°)
x   sin 37°  17 cm
7. 30 cm. Use the Law of Sines with a  41 cm, b  y,
A  87°, and B  46°. Use the expression for b
LESSON 12.3 found in Exercise 5.
a sin B
EXERCISES b 
sin A
1. 329 cm2. Use the SAS Triangle Area Conjecture with 41(sin 46°)
a  29 cm, b  25 cm, and C  65°. x   sin 87°  30 cm
1 1
A  2ab sin C  2(29)(25)sin 65°  329 cm2

234 CHAPTER 12 Discovering Geometry Solutions Manual


©2003 Key Curriculum Press
DG3SM586_229-251.qxd 8/2/06 6:11 PM Page 235

sin A sin C
8. 56°. Use the Law of Sines with a  36 cm, c    
a c
29 cm, and C  42°. Start with the Law of Sines
c sin A  a sin C
and solve for sin A; then find A.
a sin C
sin A sin C c 
   sin A
a c
a sin C 2500(sin 62°)
sin A  c c   sin 79°  2200 m
36(sin 42°) b. Approximately 1600 m. The distance between
sin A   29  is the
Alphonse and Collette is AC. Because AC
side of ABC opposite B, its length can also be

36(sin 42°)
A  sin1 29  represented by b.
A  56° b
a sin B

sin A
The measure of A is approximately 56°. 2500 sin 39°
b   sin 79°  1600 m
9. 45°. Use the Law of Sines with a  445 m,
b  325 m, and A  77°. c. Approximately 1400 m. Draw the altitude from A
 to form two right triangles. Let h repre-
to DC
sin A sin B
   sent the length of this altitude. You can use either
a b
b sin A of these two right triangles to find h, which
sin B  a
represents Alphonse’s height in the balloon.
325(sin 77°) From the triangle on the left, sin 39°  hc, so
sin B   445 h  c(sin 39°)  2200(sin 39°)  1400 m.


325(sin 77°)
B  sin1  445  45  12. The other two walls were 300 ft and approximately
413 ft. The area was approximately 45,000 ft2.
B  45°
First by the Triangle Sum Conjecture, mC 
The measure of B is approximately 45°. 180°  87°  46.5°  46.5°. C  A, so ABC
  AB
is isosceles with BC  by the Converse of the
10. 66°. You can’t find mC directly with the Law of
, is Isosceles Triangle Conjecture. Thus BC  AB 
Sines because the length of its opposite side, AB
300 ft. Now use the Law of Sines to find AC.
not given. Instead, use the Law of Sines to find
sin B sin C
mA, and then use the result to find mC.   
AC AB
sin A sin B (AC)(sin C)  (AB)(sin B)
  
a b
a sin B (AB)sin B 300 sin 87°
sin A  b AC    
sin C  sin 46.5°  413 ft
362(sin 63°) Now use the SAS Triangle Area Conjecture to find
sin A   
415 the area of the triangle.

 
362(sin 63°) 1 1
A  sin1  A  2(AB)(BC)sin 87°  2(300)(300)sin 87°
415
A  51°  45,000 ft2

By the Triangle Sum Conjecture, mA  mB  13. Approximately 48 m. Label the figure for reference.
mC  180°, so mC  180°  mA  mB 
180°  51°  63°  66°.
11. a. Approximately 2200 m. The distance between c
Alphonse and Beatrice is AB. Because AB is the
side of ABC opposite C, its length can also be
 is the side oppo-
represented by c. Similarly, BC
site A, so its length can be represented by a. b 68°
By the Triangle Sum Conjecture, mA  a 16°
180°  39°  62°  79°. 16°
18 m

Discovering Geometry Solutions Manual CHAPTER 12 235


©2003 Key Curriculum Press
DG3SM586_229-251.qxd 8/2/06 6:11 PM Page 236

In this figure, the height of the tree is represented Triangle Conjecture, the length of the shorter leg
by c  a. Look at the small right triangle with legs is 12(3)  32 cm, and the length of the longer leg is
of lengths a and b and hypotenuse of length 18 m. 3

2 3 cm. The shorter leg of the triangle is a radius
Because the leg of length b is parallel to the ground, of the cone, and the longer leg is the height.
the smaller acute angle of this triangle (opposite the
side of length a) measures 16° by the CA Conjec-
ture. Use trigonometric ratios in this triangle to find
1 1 1 3 2 3
 
V  3BH  3r 2H  3 2 23 
9 3
a and b.  8 cm3  6 cm3
a  CD , A  D by the AIA Conjec-
sin 16°  18, so a  18(sin 16°). 17. Because AB
b ture. Because D and B intercept the same arc,
cos 16°  1,
8 so b  18(cos 16°). D  B. Therefore A  B by the transitive
Now look at the large right triangle with legs of property. So, ABE is isosceles by the Converse of
lengths b and c. the Isosceles Triangle Conjecture.

tan 68°  bc, so c  b(tan 68°)  18(cos 16°)(tan 68°). 18. Fold the paper so that the two Fold 1 Fold 2
points coincide. Draw a line along
c  a  18(cos 16°)(tan 68°)  18(sin 16°)  48 m the fold. Draw another line through
14. 2366 cm; approximately 41°. Each side of the the two points. These two lines
pyramid has 91 steps, each of height 26 cm, so contain the diagonals of the square.
the height of the pyramid is 91(26)  2366 cm Now fold the paper so that the two
(or 23.66 m). The depth of the pyramid from the lines coincide. Mark the vertices on the other line.
outer edge to the center is 91(30)  2730 cm 19. AC  60 cm, AE  93.75 cm, AF  117 cm. Apply
(or 27.30 m). To find the angle of ascent, make the Extended Parallel/Proportionality Conjecture.
a sketch. AB BI 20
From ABI  ACH, 
AC  ,
CH so 
AC  1468  13.
In this figure,  is the angle AC  20(3)  60 cm.
of ascent. 2366 cm
2366 From ABI  AEF, AA
B BI 20 16
E  EF , so AE  75 .
  
tan     ␪ 1500
2730 2730 cm 16(AE)  20(75)  1500, so AF   16 


  tan1  
2366
 
2730  41°
93.75 cm.
Also from ABI  AEF, EBFI  AAFI , so 1765  A25F .
15. Approximately 5°. Make a sketch. 1875
Then 16(AF)  75(25)  1875, so AE   16 

117 cm.
20. The diagonal rod in Box 1 is longer. For each box,
you need to find the length of the space diagonal,
which can be found by extending the Pythagorean
Theorem to three dimensions.
55

Box 1: Let d represent the length of the space
diagonal.
d  
52  92
 42  
122  11.05 in.
Box 2: Let e represent the length of the space
diagonal.
4.8
e  5
2  62  62 
  97  9.85 in.
4.8
sin   5,
5 so   sin
1  
4.8
55  5°. Therefore the diagonal rod in Box 1 is longer.
16. Approximately 6 cm3. Look at the right triangle IMPROVING YOUR VISUAL THINKING SKILLS
inside the cone. The angle of measure 120° is an
Make a table or use recursive formulas to discover the
exterior angle of the triangle. Its supplement is a 60°
patterns.
angle, so the triangle is a 30°-60°-90° right triangle,
with hypotenuse of length 3 cm. By the 30°-60°-90° 1. 151 2. 100

236 CHAPTER 12 Discovering Geometry Solutions Manual


©2003 Key Curriculum Press
DG3SM586_229-251.qxd 8/2/06 6:11 PM Page 237

EXTENSION 7. Approximately 43 cm. The


See the solutions to Take Another Look activities 1 and two radii and the chord form x

2 on page 251. an isosceles triangle with legs 24 in. 126° 24 in.


of length 24 cm and vertex
angle of measure 126°. The
LESSON 12.4 vertex angle is the included
EXERCISES angle between the two legs of an
isosceles triangle, so you can use the Law of Cosines
1. 32 cm. The lengths of two sides and the measure
to find the base of the isosceles triangle, which is
of the included angle are given. Use the Law of
the chord of the circle. Let x represent the length of
Cosines to find w.
the chord.
c 2  a 2  b 2  2ab cos C
x 2  242  242  2(24)(24)cos 126°
w2  412  362  2(41)(36)cos 49°
x  
242  
242  2(24)(
24)cos 126°  43 cm

w  2(41)(3

412 362  32 cm
6)cos 49°
8. Approximately 30°. By the Side-Angle Inequality
2. 47 cm. Use the Law of Cosines to find y. Conjecture, the smallest angle in the triangle is
opposite the shortest side, so use the Law of
c 2  a 2  b 2  2ab cos C
Cosines to find the measure of the angle opposite
y 2  322  422  2(32)(42)cos 78° the 4-meter side. Let A be the smallest angle of
the triangle.
y  
322  42
 2 2(32)(42)cos 78°  47 cm

42  72  82  2(7)(8)cos A
3. 341 cm. Use the Law of Cosines to find x.
42  72  82
cos A  2(7)(8)
x 2  2352  2822  2(235)(282)cos 82°
x  
2352 
2822  s
2(235)(282)co 82° 
A  cos1 
42  72  82
2(7)(8) 
 30°
 341 cm 9. Approximately 116° and B C

4. 74°. The lengths of all three sides of the triangle are 64°. Sketch parallelogram
ABCD with diagonal BD . 15 cm 19 cm
given. Use the Law of Cosines to find mA. Notice
 and KR  42 cm.
that A is opposite KR From the figure, you can A 20 cm D
422  342  362  2(34)(36)cos A see that the 19-centimeter
. (19 cm 
diagonal must be the shorter one, BD
422  342  362  2(24)(36)cos A , A, and C
20 cm, so the angles opposite BD
422  342  362 must be acute.) Use the Law of Cosines to find
cos A  2(34)(36) mA.
A  cos1  
422  342  362
2(34)(36)  74°  192  152  202  2(15)(20)cos A
192  152  202
5. 64°. Use the Law of Cosines to find mB. Notice cos A  2(15)(20)
 and TE  390 cm.
that B is opposite TE
3902  3502  3802  2(350)(380)cos B 
192  152  202
A  cos1 2(15)(20)  64°
3902  3502  3802  2(350)(380)cos B By the Parallelogram Consecutive Angles Conjec-
ture, A and B are supplementary, so mB 
3902  350 2  380 2
cos B  2(350)(380) 180°  mA  180°  64°  116°. By the Parallel-
ogram Opposite Angles Conjecture, C  A and
B  cos1  
3902  3502  3802
2(350)(380)  64°  D  B. Therefore the measures of the angles of
the parallelogram are approximately 64° and 116°.
6. 85°. Use the Law of Cosines to find mC. Notice
 and DL  508 cm.
that C is opposite DL 10. Approximately 6 min. 60 minutes 80 minutes
20°
Make a sketch. Divide the h
5082  3282  4182  2(328)(418)cos C large triangle into right a b
x
5082  3282  4182  2(328)(418)cos C triangles by drawing an
5082  3282  4182 altitude. Because distances and times are propor-
cos C  2(328)(418) tional, the lengths on the diagram can represent
times as well as distances, so we can ignore

5082  3282  4182
C  cos1 2(328)(418)  85°  distances entirely. First find the time that Captain
Malloy actually flew (following the detour). He flew

Discovering Geometry Solutions Manual CHAPTER 12 237


©2003 Key Curriculum Press
DG3SM586_229-251.qxd 8/2/06 6:11 PM Page 238

60 min on the first leg (1 hr) and 80 min on the 14. The ladder is at approximately a 76°
second leg. Therefore he flew a total time of angle and is not safe. The ladder should
60  80  140 min. be between 6.5 ft and approximately
14.3 ft from the base of the wall.
To find the straight-line time, x, find a and b from
the two right triangles. Make a sketch to show the position of
a Regina’s ladder leaning against the wall.
cos 20°  60, so a  60(cos 20°)  56.4 min. 25 ft
6
h cos   25  0.24
sin 20°  60, so h  60(sin 20°).
  cos1(0.24)  76°, so the ladder is
By the Pythagorean Theorem, b   802  
h2  not safe.
 80  
2 (60(sin 20°))  77.3 min. Then x 
2
To find the range of safe distances, let d
a  b  56.4  77.3  133.7 min. ␪
represent the distance between the base 6 ft
Therefore the extra time that the captain flew by of the ladder and the base of the wall.
taking the detour was approximately 140  133.7  cos   2d5 , so d  25(cos ). If   55°, d 
6.3  6 min. 25(cos 55°)  14.3 ft. If   75°, d  25(cos 75°) 
6.5 ft.
11. Approximately 87.8 ft.
x
Make a sketch. 30 ft The ladder should be between approximately 6.5 ft
20°
30 and 14.3 ft from the base of the wall.
sin 20°  x
a
15. sin A  c
x sin 20°  30
30 b
x cos A  c
sin 20°  87.71
c a

a
If the angle is 20°, the ramp will be approximately tan A  b A b
87.71 ft long. If the angle is less than 20°, the ramp a
will be longer than 87.71 ft. Because 20° is the 
sin A c a c a
maximum angle permitted, round up to 87.8 ft. 
cos A b  c  b  b  tan A

c
12. Approximately 139 m. Let h
16. a. PR  x  .
3 . Let M be the midpoint of base TR
represent the height of the
    
Draw PM and FM . PA  RM and PA RM , 
original pyramid. Make a
so PAM  RMA by AIA, and AM   MA.
sketch. h
65° So PAM  RMA by SAS, and similarly
Look at the right triangle in TMP  APM by SAS. So, by CPCTC, AM 
the diagram. Notice that the 130 65 PM  x, and the three triangles are equilateral.
length of the shorter leg is Also draw PC, the altitude from P to TM
 in
one-half the length of a side of the square, or PTM.
1
(130)  65 cm.
2 P
x
A
h
  tan 65°
65 x x
h  65(tan 65°)  139 m
M
T R
13. Approximately 143 m. Let L C 2x
21°
L represent the top of the 55 m Any altitude of an equilateral triangle is also a
lighthouse, B the base of 21°
S median, so TC  2x. Then PTC is a right
the lighthouse, and S the B
triangle with one leg of length 2x and hypotenuse
position of the sailboat.
of length x, so this is a 30°-60°-90° triangle and
Make a sketch. x 3
PC   2 . Now look at PCR. In this triangle,
x 3
From the given information, LB  55 m and the the lengths of the legs are PC   2 and CR 
angle of depression is 21°. By the AIA Conjecture, TR  TC  2x  2x  32x . Use the Pythagorean
mLSB  21°. Use SLB to find SB, the distance Theorem in PCR to find PR.
between the sailboat and the base of the lighthouse. x 
     34x  94x
3 2 3x 2 2 2
55 (PR)2   2  2
tan 21°   
SB
12x 2
(SB)(tan 21°)  55  4  3x 2
55 PR  
3x 2  x 3
SB  tan 21°  143 m

238 CHAPTER 12 Discovering Geometry Solutions Manual


©2003 Key Curriculum Press
DG3SM586_229-251.qxd 8/2/06 6:11 PM Page 239

b. Because x 2  (x 3)2  (2x)2, TPR is a right S  rl  r 2  (5)


89   (5)2
triangle by the Converse of the Pythagorean
 5
89   25  5
89  25
Theorem. Therefore mTPR  90°.
 227 cm2
17. a. Decreases. PAB will go from an obtuse angle to
a right angle to an acute angle, and then to To find the slant height of the pyramid, draw a
smaller and smaller acute angles. right triangle inside the pyramid with right-angle
vertex at the center of the square base, the shorter
b. Increases then decreases. APB will always be an
leg an apothem of the square, and the longer
acute angle. It will increase until it reaches its
leg the height of the pyramid. Then the slant
maximum measure when P is directly above the
 so that APB is isosceles with height will be the length of the hypotenuse. Use
midpoint of AB
 the Pythagorean Theorem to find the slant
base AB . After that, as P continues to move to
height: l  (1.25
 )2  82  8.9 cm.
the right, APB will be an acute angle that gets
smaller and smaller. To find the surface area, use one of the formulas
for the surface area of a regular pyramid. (See
18. a. The base perimeters are equal. The base of the
page 448 of your book.) If S represents the
cone is a circle with diameter 10 cm. The base of
surface area, a is the length of the apothem of
the pyramid is a square with side length 2.5 cm.
the regular base, b is the base length, and n is
The base perimeter of the cone is the circumfer- the number of sides in the polygon, the surface
ence of a circle with d  10 cm: C  d  area of a pyramid can be found from the
(10)  10 cm. formula S  12nb(l  a).
1 1
The base perimeter of the pyramid is the S  2nb(l  a)  2  4  2.5(8.9  1.25)
perimeter of a square with side length 2.5 cm:
 201 cm2
P  4s  4(2.5)  10 cm.
The surface area of the cone is approximately
Both solids have the same base perimeter, 10 cm.
227 cm2, while the surface area of the pyramid
b. The cone has the greater volume. The cone has is approximately 201 cm2, so the cone has the
radius 5 cm and height 8 cm, and the pyramid greater surface area.
also has height 8 cm.
19. a. A translation to the right 10 units; (x, y) →
1 1 1
Cone: V  3BH  3r 2H  3(5)2  8 (x  10, y). The lines x  2 and x  3 are
200 both vertical lines, so they are parallel, and you
 3 cm
3
can apply the Reflections over Parallel Lines
1 1 50
Pyramid: V  3BH  3(2.5)2  8   3 cm
3 Conjecture. This conjecture tells you that the
composition of these two reflections will be a
Thus the volume of the cone is greater than the translation and that the second image of each
volume of the pyramid; in fact, the volume of the point will be twice the distance between the
cone is four times the volume of the pyramid. parallel lines. Because the lines are 3  (2) 
c. The cone has the greater surface area. In order to 5 units apart, and because the second reflection is
find their surface areas, you need to find the slant over the line farther to the right, the composition
heights of the cone and the pyramid. Recall that l is equivalent to a translation to the right 10 units.
is used to represent slant height. Every point is moved 10 units to the right, but
not moved up or down, so its x-coordinate
To find the slant height of the cone, look at the increases by 10 while its y-coordinate does not
right triangle inside the cone. The shorter leg is a change. Thus the rule is (x, y) → (x  10, y).
radius of the circular base, the longer leg is the
height of the cone, and the slant height is the b. A rotation 180° about the origin; (x, y) →
length of the hypotenuse. Use the Pythagorean (x, y). This is a composition of two reflec-
Theorem to find the slant height: l  tions over intersecting lines, so you can apply the
 52  82   89 . Use the formula for the Reflections over Intersecting Lines Conjecture.
surface area of a cone using slant height. (See The x-axis and y-axis are perpendicular, so the
page 449 of your book.) If S represents the smaller angle between them is 90°. Therefore the
surface area and l represents the slant height, composition of the two reflections is a single
S  rl  r 2. rotation, and the angle of rotation is
2(90°)  180°.

Discovering Geometry Solutions Manual CHAPTER 12 239


©2003 Key Curriculum Press
DG3SM586_229-251.qxd 8/2/06 6:11 PM Page 240

20. Possible answer: PROJECT


Project should satisfy the following criteria:
● The project problem is solved.

x
y hyp  r 1  r 2 r2

leg  r 1  r 2 r1
x AB
2y AB  2
r1r2
The first rectangle is a square, while the second
rectangle has a length that is twice its width. The
sides of these two rectangles are not proportional, r3
so the rectangles are not similar. hyp  r 1  r 3
21. Possible answer: leg  r 1  r 3 r1
x
x x  2
r1r3
120° 120° y
x x 120° 120°
y y

60° 60° 60° 60°


hyp  r 2  r 3
2x 2y r2
leg  r 2  r 3
22. Results can vary from the simple (circles, segments) r3 y
y  2
r2r3
to the complex (cardioids, roses, epicycloids). Begin
with the simplest case where all factors are equal.
With circles of the same radius and with endpoints
of the segment traveling in the same direction, at The three legs, AB, x, and y, are related by
the same rotational speed, and starting from the AB  x  y, or 2
r1r2  2r
1r3  2r
2r3
same relative position, the midpoint will trace a
circle equal in size to the constructed circles. ● A sangaku tablet is replicated.
Changing the size of both circles will change the ● At least two other sangaku are given, and their
size of the circle traced, as will changing the starting solutions are attempted.
positions of the endpoints. The maximum radius of
the traced circle is limited by the radius of the Extra credit
constructed circles; the smallest observable trace is a ● Additional cultural information is included (for
single point. example, that sangaku were hung in both Shinto
With all other factors equal and the endpoints of shrines and Buddhist temples).
the segment traveling in opposite directions around ● The project includes some history of Japanese mathe-
the circles, the midpoint will trace an ellipse. The matics (wasan) or the origins of sangaku.
major axis of the ellipse is equal to the sum of the
radii of the two constructed circles. The minor axis ● r1r2  2
The relationship 2 r1r3  2r2r3 is
1 1 1
is equal to their difference. Varying the starting rewritten as      and is related to the
r3 r1 r2
positions of the endpoints changes the orientation Pythagorean Theorem.
of the ellipse. Changing the rotational speed of the
endpoints will produce more complex polar curves, EXTENSION
which you might describe as flowers, roses, or See the solutions to Take Another Look activities 3 and
“spirograph-like.” 4 on page 251.
Changing the distance between the centers has no
effect on the shape of the trace, only its position.
No matter how you vary the factors, you should
find the midpoint trace will always trace over itself.

240 CHAPTER 12 Discovering Geometry Solutions Manual


©2003 Key Curriculum Press
DG3SM586_229-251.qxd 8/2/06 6:11 PM Page 241

LESSON 12.5 4. Approximately 5.9 m. Let l represent the length of


the cylinder.
EXERCISES 2.5
sin 25°  l
1. Approximately 12.7 m. Make a sketch.
4.5° l(sin 25°)  2.5
1m 2.5
d l sin 25°  5.9
1 The cylinder should be approximately 5.9 m long.
tan 4.5°  d
d(tan 4.5°)  1 5. a. Approximately 9.1 km. N

1 Make a careful drawing.


d 
tan 4.5°  12.7 m 137°
First find . From the
The maximum permitted angle is 4.5°. If the angle E
diagram,   180°  N 5 km
is smaller than 4.5°, the distance between the 137°  43°. ␪
building entrance and the bottom of the ramp will 8 km ␤
42°
be greater than 12.7 m, so the minimum distance is The north lines in the ␣ x
approximately 12.7 m. diagram are parallel, so E
the measure of the
2. Approximately 142 mi/hr. 20
angle between the sides
The resultant vector can be 124°
of lengths 8 km and 5 km is 42°    85°. In
represented by the longer the diagram, x represents the distance between
diagonal of the parallelogram. Annie and Sashi’s position and the base camp.
By the Parallelogram Consecu- 130
Use the Law of Cosines to find x.
tive Angles Conjecture, the
measure of the obtuse angles x 2  82  52  2(8)(5)cos 85°
in the parallelogram is x  
82  52
 2(8)(5)co
s 85°  9.1
180°  56°  124°. The
longer diagonal is the third They are approximately 9.1 km away from their
side of a triangle with sides of lengths 130 and 20 base camp.
(representing speeds of 130 mi/hr and 20 mi/hr) (Note: The value of x will be used in the calcula-
and included angle of 124°. tion of the angle in 5b. In order to get a more
Let v represent the length of the resultant vector. accurate result, either store the actual value of x
Use the Law of Cosines to find v. in your calculator or use a more accurate
approximation, such as 9.057, in the calculation
v 2  1302  202  2(130)(20)(cos 124°) of  in 5b.)
v  130
 2  202  2(130)(20)(cos 124°)  142
 b. Approximately 255°. N N

The resulting speed is approximately 142 mi/hr. The angle that you need
to find is shown by the ␪ 5 km
3. Approximately 51 km. Let P represent the position arrow in the figure. ␪
of the patrol boat, L the position of the lighthouse, (Recall that bearing is ␤
and S the position of the station. measured clockwise x

S from the north.)

20 km Use the Law of Sines to find .


23° sin  sin 85°
P L 8  
x
20
sin 23°  
PS
 sin   
8 sin 85°
9.057
(PS)(sin 23°)  20
PS  
20
sin 23°  51

  sin1 
8 sin 85°

9.057  62°
By the AIA Conjecture, the angle adjacent to the
The boat is approximately 51 km from the station. angle with measure  has measure   43°. The
bearing for the return trip is approximately
360°  62°  43°, or approximately 255°.

Discovering Geometry Solutions Manual CHAPTER 12 241


©2003 Key Curriculum Press
DG3SM586_229-251.qxd 8/2/06 6:11 PM Page 242

6. Approximately 240 m. Use the right triangle to The distance that Olivia traveled is approximately
find AC. 5.48  2.94  8.42 km.
AC
tan 58°  
150 To find her flying speed, first change 3 minutes to
AC  150(tan 58°)  240 hours: 3 min  630 hr  210 hr  0.05 hr.
d 8.42 km
The distance between A and C is approximately d  rt, so r  t   0.05 hr  168 km/hr.
240 m.
10. Approximately 12.8 m at x
7. Approximately 42 ft. Let x an angle of approximately
represent the height of the y
48°. Let l represent the Hill
portion of the tree still x length of pipe needed to
standing and y represent the go through the hill. 14.5 m 11.2 m
length of the bent-over 38° 58°
Use the Law of Cosines
portion. Then x  y 20.5 ft
to find l.
represents the original height
of the tree. Change 20 ft 6 in. to 20.5 ft because this l 2  (14.5)2  (11.2)2  2(14.5)(11.2)cos 58°
is an easier measurement to use in calculations.
l    (11
(14.5)2 .2)2  cos
2(14.5)(11.2) 58°
x
tan 38°   
20.5  12.8 m
x  20.5(tan 38°)  16 ft
Let A be the vertex of the angle between the
20.5 14.5-meter side and the side of length l. Use the
cos 38°  y
Law of Sines to find mA.
20.5
y cos 38°  26 ft
sin A sin 58°
  
11.2 12.8
x  y  16  26  42 ft
11.2(sin 58°)
The original height of the tree was approximately sin A   
12.8
42 ft.
8. They must dig at an angle of approximately 71.6° 
11.2(sin 58°)
A  sin1    48°
12.8 
 to form
and dig for approximately 25.3 m. Draw AB 11. Approximately 108 cm3. In order
right triangle ABC. Then B is the angle that you to find the volume of the prism,
need to find. first find the area of the pentag-
24
tan B  8  3 onal base. Divide the pentagon
into five congruent isosceles
B  tan1(3)  71.6° triangles by drawing segments
Use the Pythagorean Theorem to find BC. from the center of the pentagon
to its vertices.
BC    (AB
(AC)2  )2  
242  82  25.3 m
To find the area of the regular
9. Approximately 168 km/hr. Make a sketch. pentagon, you need to find 72°
Distance traveled the apothem, which is the 36°
a
in 3 min
height of one of the five
triangles shown in the figure
above. Look at one of these 1.5 cm
6.3 km 6.3 km 3 cm
triangles. Let a represent the
49° 65° height of the triangle, which
x y is the apothem of the pentagon.
6.3 The vertex angle of this triangle is 15(360°)  72°
tan 49°  x
because this is one of five congruent angles around
x(tan 49°)  6.3 the center of the pentagon. Because the altitude
6.3
x tan 49°  5.48 km
from the vertex angle to the base of an isosceles
triangle is also an angle bisector and a median, each
6.3
tan 65°  y of the congruent right triangles into which the
altitude divides the isosceles triangle has a shorter
y(tan 65°)  6.3 leg of length 1.5 cm and a smallest angle of 36°.
6.3
y tan 65°  2.94 km

242 CHAPTER 12 Discovering Geometry Solutions Manual


©2003 Key Curriculum Press
DG3SM586_229-251.qxd 8/2/06 6:11 PM Page 243

1.5
tan 36°  a 2 2  242
x 2  x 
a(tan 36°)  1.5 x 2  2x 2  576
1.5
a 
tan 36° 3x 2  576
Recall that the formula A  12asn gives the area of a x 2  192
regular polygon with n sides, apothem a, and side
length s. In this case, use B to represent the area of x  192
  
64 • 3  8
3
the polygon because this is the base area of a prism. Now find the volume of the cube.
1

1 1.5
B  2asn  2   
tan 36°  3  5  15.48 cm
2 3 3  512
V  x 3  8  33  15363 cm3
Finally, find the volume of the prism.  2660 cm3
V  BH  (15.48)(7)  108 cm3 14. The area increases by a factor of 9. The ratio of the
lengths of corresponding sides is 31; by the Propor-
12. Sample answer: Any regular tional Areas Conjecture, the ratio of corresponding
360°
___
polygon can be divided into n areas is 312  91.
n congruent isosceles
y
triangles, each with a
360°
vertex angle of  
n .
(0, 2)
(–3, 0)  21
A  _1 1
2
The altitude from the vertex angle of an isosceles x
–6 (1, 0) 6
triangle bisects the angle and the opposite side.
This altitude is also an apothem of the regular 2  69
A  _1 3

polygon.
–6 (0, –6)
360°
___ 360°
___
2n 2n
a 15. The large and small triangles are similar by the
AA Similarity Conjecture because each triangle has
_s _s
2 2 a right angle and the triangles share an acute angle.
360°
For simplicity, let    
2n . Now use the tangent
Because the triangles are similar, corresponding
ratio to find the length of the apothem. sides must be proportional, which would give
s 15  5
 5 5 20
  , or   . But this situation is
tan    2 4 12 4 12
a impossible because 2102  53 and 54  53, so this
s
a 
2 tan  is not true.
Now use the area formula for a regular polygon. 16. a. Decreases then increases. PA decreases until P is
1
A  2aP directly above A so that PA  AB
 and PAB is a
right angle; then PA increases as P moves farther
1 s
A  2  2 tan   ns
   AB
to the right. (PA is at a minimum when PA 
ns2 360° because the shortest distance between two lines is
A ,
4 tan  where   2n
 the perpendicular distance.)
13. 15363 cm3  2660 cm3. A space diagonal of the b. Does not change. The area of APB does not
cube will be a diameter of the sphere, and the change because AB is always a base of the
diameter of the sphere is 24 cm. Let x represent the triangle, and the corresponding height is always
edge length of the cube. The diagonal of a square the (perpendicular) distance between the two
base of the cube is the hypotenuse of an isosceles lines.
right triangle with legs of length x, so, by the
17. a. A reflection over the x-axis; (x, y) → (x, y).
Isosceles Right Triangle Conjecture, the length of
Choose a point (x, y) in any quadrant. By the
this diagonal is x 
2 . Apply the Pythagorean
Coordinate Transformations Conjecture or by
Theorem to a right triangle with the diagonal of
understanding that this transformation reverses
the base as the longer leg, an edge of the cube as
the two coordinates of any point, the image of
the shorter leg, and the space diagonal as the
(x, y) under this transformation is (y, x). Now
hypotenuse.
rotate that first image point (y, x) counterclock-
wise by 270° about the origin to obtain the second

Discovering Geometry Solutions Manual CHAPTER 12 243


©2003 Key Curriculum Press
DG3SM586_229-251.qxd 8/2/06 6:11 PM Page 244

image, which is the point (x, y). The result of Use the tangent ratio to find the radius.
the composition is to transform (x, y) to (x, y), r
tan 0.26°   
r  233,900
so the rule for the composition is (x, y) →
(x, y). By the Coordinate Transformations (r  233,900)tan 0.26°  r
Conjecture, this is a reflection over the x-axis.
r(tan 0.26°)  233,900(tan 0.26°)  r
y
r(tan 0.26°)  233,900(tan 0.26°)  r  0
(y, x) yx
r(tan 0.26°  1)  233,900(tan 0.26°)
270°
(x, y) 233,900 tan 0.26°
r   tan 0.26°  1  1,070 mi
x

(x, –y)
B. The probability of an exact isosceles or equilateral
triangle is theoretically 0. Because there are
infinitely many points along the straw, the proba-
bility of choosing the same point more than once
1
is  
infinity . The probability that one angle will
b. A reflection over the y-axis; (x, y) → (x, y). measure exactly 90° is similarly 0. But, because
Choose a point (x, y) in any quadrant. By the measurements are rounded, you will occasionally
Coordinate Transformations Conjecture, the get a special triangle. To sort the data for acute and
image of this point under a 180° rotation about obtuse triangles, use the inequality based on the
the origin is the point (x, y). Now reflect Pythagorean Theorem.
(x, y) over the x-axis. This transformation
reverses the sign of the y-coordinate, but keeps EXPLORATION • TRIGONOMETRIC RATIOS AND
the x-coordinate the same, so the second image is THE UNIT CIRCLE
the point (x, y). The result of the composition
is to transform (x, y) to (x, y), so the rule for PROJECT
the composition is (x, y) → (x, y). By the Project should satisfy the following criteria:
Coordinate Transformations Conjecture, this is a
● Sketches and descriptions show important character-
reflection over the y-axis.
istics of each function. (Sample: The graph of y 
y
sin(x) has a minimum y-value of 1 and a maximum
y-value of 1. The period is 360°.)
180°
● Descriptions show relationships between functions.
(–x, y) (x, y)
(Sample: Sine and cosine have the same shape, but the
x
cosine curve is translated left 90°. Both graphs have
(–x, –y) points in common when x equals 315°, 135°, 45°,
and 225°, or every 180° from 45°.)
Extra credit
● Symmetry in the graphs is recognized.
18. Software creations will vary.
● Observations are generalized. (Sample: cos x 
IMPROVING YOUR ALGEBRA SKILLS sin(x  90°) or sin x  cos x for x  45°  180n,
1. x  5z0 . 2x  3y, so x  32y  32(5w)  where n is any integer value.)
3

2  5  230z  5z0 . ● Characteristics are explained in terms of the
18 13 13 trigonometric definitions. (Sample: As x approaches
2. x  z7x  13y, so x 
. y
7  (28w)
7 
13

7   296z  1z8 .
28 90°, the tangent function approaches infinity, because
as the measure of one of the acute angles in a right
EXTENSIONS triangle approaches 90°, the opposite side gets longer
A. Make a sketch. and longer and the adjacent side gets shorter
0.26° and shorter, approaching 0; therefore the ratio
length of opposite side
r  gets infinitely large.)
E r M length of adjacent side
233,900 mi

244 CHAPTER 12 Discovering Geometry Solutions Manual


©2003 Key Curriculum Press
DG3SM586_229-251.qxd 8/2/06 6:11 PM Page 245

CHAPTER 12 REVIEW cone. The angle with measure 112° is an exterior


angle of the triangle. Its supplement, which is
EXERCISES the larger acute angle in the triangle, measures
1. 0.8387 180°  112°  68°. Use the triangle to find h.
h
2. 0.9877 tan 68°  18
3. 28.6363 h  18(tan 68°)
4. sin A  ab; cos A  bc; tan A  ac. (Store this value in your calculator rather than
rounding this intermediate result.)
length of side opposite A a
sin A  length of hypotenuse  b
Now find the volume of the cone.
length of side adjacent of A c 1 1 1
V  3BH  3r 2H  3(18)2(18 tan 68°)
cos A  length of hypotenuse  b
 15,116 cm3
length of side opposite A a
tan A  
length of side adjacent to A  c 12. Yes, the plan meets the act’s requirements. The angle
5. sin B  187 ; cos B  1157 ; tan B  185 . The side lengths of ascent is approximately 4.3°. Use the side view to
in this triangle are a multiple of the 8-15-17 find the slope of the ramp.
Pythagorean triple: 16  2(8), 30  2(15), so OB  rise 1.5
slope   
2(17)  34. The trigonometric ratios are the same run  20  0.075
in this triangle as in an 8-15-17 triangle. The slope must be less than 112  0.083. Because
0.075  0.083, the ramp is not too steep. By looking
6. sin   s; cos   t; tan   st. Draw a perpen- at the top and front views, you can see that there
dicular segment from the point (t, s) to the x-axis is a 5-by-5 ft landing for every 1.5 ft of rise, so
to form a right triangle. In this triangle, the length the landing requirement is exceeded. Thus, all
of the horizontal leg will be t, the length of the requirements of the Americans with Disabilities
vertical length will be s, and the length of the Act are met.
hypotenuse will be 1, the radius of the circle. Then
sin   1s  s, cos   1t  t, and tan   st. The angle of ascent is tan112.
0   4.3°.
5

7. 33°. A  sin1(0.5447)  33°. 13. Approximately 52 km. Let d represent the distance
between the sailboat and the dock. In the figure
8. 86°. B  cos1(0.0696)  86°. below, S represents the position of the sailboat,
9. 71°. C  tan1(2.9043)  71°. L represents the position of the lighthouse, and
D represents the position of the dock.
10. 1823 cm2. The shaded region is a semicircle; you D
need to know the radius of the circle in order to
find its area. By the Angles Inscribed in a Semicircle
d 30 km
Conjecture, the triangle in the figure is a right
triangle whose hypotenuse is the diameter of the
35°
circle that is drawn in the figure. Let c equal the S L
length of the hypotenuse. 30
41 sin 35°  d
sin 37°  c
d(sin 35°)  30
c(sin 37°)  41 30
41 d 
sin 35°  52
c 
sin 37° The distance between the lighthouse and the dock is
(Store this value in your calculator rather than approximately 52 km.
rounding this intermediate result.)
14. Approximately 7.3°. Let  be the angle of descent.
Let r represent the radius of the circle. Then r  12c, Recall that the angle of descent (or angle of depres-
and the area of the semicircle is given by A  sion) is measured with respect to the horizontal.
r 2       1823 cm2.
1 1 1 41 2
2 2 2 sin 37° Make a sketch.
11. 15,116 cm3. Look at the right triangle inside the ␪
cone. The shorter leg of this triangle is a radius of 5.6 km
the circle, and the longer leg is the height of the 44 km

Discovering Geometry Solutions Manual CHAPTER 12 245


©2003 Key Curriculum Press
DG3SM586_229-251.qxd 8/2/06 6:11 PM Page 246

By the AIA Conjecture, the smaller acute angle in accurate final answer, it is important not to round
the right triangle is congruent to the angle of intermediate results.)
descent. Use this triangle to find . 6.3
tan 42°  x
5.6
tan   44 x(tan 42°)  6.3
5.6
  tan1 4 4  7.3°   x
6.3

tan 42°
The angle of descent is approximately 7.3°. 6.3
tan 15°  y
15. Approximately 22 ft. Let l represent the length of
one rafter, not including the overhang. The center y(tan 15°)  6.3
line of the house is the altitude from the vertex 6.3
y 
tan 15°
angle of the isosceles triangle, so it is also a median.
Look at the right triangle on the left, which has Then JK  y  x   tan 15°  tan 42°  km.
6.3
 
6.3

longer leg of length 12(32)  16 ft and hypotenuse Convert 20 seconds to hours:


of length l. 1 min 1 hr 1
16 20 sec   60 sec  60 min  180 hr
  
cos 36°  l
To find the speed of the jet, use the formula d  rt,
l(cos 36°)  16 or r  dt:
16
l 
cos 36°  20 ft
d   6.3  
tan 
15°
6.3
tan  
42° km
Add 2 ft for the overhang. The carpenter should r  t  
1
make each rafter approximately 22 ft long.  hr
180

 
16. Approximately 6568 m. In the figure below, P repre- 6.3 6.3
sents the position of the patrol boat, H represents    
tan 15°  tan 42° (180) km/hr  2973 km/hr
the position of the helicopter, and L represents the 18. 393 cm2. Use the SAS Triangle Area Conjecture.
position of the landing spot.
1 1
H
A  2ab sin C  2(24)(40) sin 55°  393 cm2
6800 m
19. 30 cm. Use the Law of Sines.
15° sin A sin B
P L   
a b
PL
cos 15°    sin 76° sin 53°
6800   
w 25
PL  6800(cos 15°)  6568
w(sin 53°)  25(sin 76°)
The patrol boat is approximately 6568 m from the 25(sin 76°)
point where the package will land. w   sin 53°  30 cm
17. Approximately 2973 km/hr. Let A represent 20. 78°. Use the Law of Sines.
Amelia’s position, J represent the jet’s position sin A sin C
  
the first time she sees it, and K represent the jet’s a c
sin A sin 50°
position 20 seconds later.   
37 29
J K
29(sin A)  37(sin 50°)
15°
42° 6.3 km 37 sin 50°
A
sin A  2 9
x

 
y 37 sin 50°
A  sin1 2 9  78°
In order to find the speed of the jet, you will
21. 105 cm. Use the Law of Cosines.
need to find the distance JK that the jet flew in
20 seconds. There are two right triangles in the c 2  a 2  b2  2ab cos C
 is not a side of either of them.
figure, but JK
However, notice that JK  y  x, and that x and y
x 2  (101)2  (65)2  2(101)(65)  cos 75°
are lengths of legs of the right triangles, which can x    (65
(101)2 )2  2(101)(
 
65) • cos 75°
be found from the given information. (To get an
 105 cm

246 CHAPTER 12 Discovering Geometry Solutions Manual


©2003 Key Curriculum Press
DG3SM586_229-251.qxd 8/2/06 6:11 PM Page 247

22. 51°. Use the Law of Cosines. The length of the 36°. Because this altitude is also a median, the
side opposite B is 27 cm, so b  27 cm. length of the shorter leg (opposite the 36° angle) is
Let a  27 cm and c  34 cm to identify the other 18 cm.
two sides. 18
tan 36°  a
b 2  a 2  c 2  2ac cos B
a(tan 36°)  18
272  272  342  2(27)(34)cos B 18
a tan 36°  25 cm
272  272  342  2(27)(34)cos B
25. 72 cm2. There are several possible approaches to this
342
cos B   
2(27)(34) problem. Three different possible solutions are
shown here.
B  cos1  
342

2(27)(34)  51°  First solution: Make a sketch. Draw the triangle and
23. 759 cm2. The figure is a rhombus, and the diagonal the altitude from the vertex angle to the base of this
divides the rhombus into two congruent isosceles isosceles triangle.
triangles. Let x represent the side length of the 8 cm 8 cm
rhombus. Apply the Law of Cosines.
262  x 2  x 2  2  x  x  cos 48° 12 cm h 12 cm

676  2x 2  2x 2  cos 48°


676
2x 2   
1  cos 48°
1 676 338
x 2  2   
1  cos 48°  1  cos 48°
Although you can solve for x at this point, it is not
necessary, because the next formula you’ll use Let h represent the height of the isosceles triangle
involves x 2 rather than x. (the length of the altitude). This altitude divides the
Apply the SAS Triangle Angle Conjecture. isosceles triangle into two congruent right triangles.
Use the Pythagorean Theorem to find h.
1 1
Atriangle  2ab sin C  2  x  x  sin 48°
122  
h   82  
80  
16 • 5
1 1

 2x 2  sin 48°  2 
338
1  cos 48°   sin 48° 16  5  4
  5 cm

 22 
1  cos 48°   sin 48°
1 338
Arhombus  2  Atriangle Now find the area of the triangle.
1 1
A  2bh  2  16  4 5  32 5 cm2  72 cm2
338
 
1  cos 48°  sin 48°  759 cm2 Second solution: Use Hero’s formula,
24. Approximately 25 cm. Draw segments from the A   s(s  a
)(s  b
)(s  c), where s is the
center of the pentagon to its five vertices to form semiperimeter of the triangle. (See pages 453 and
five congruent isosceles triangles. The apothem of 454 in your book.)
the pentagon is the height of each of these five
triangles and divides each of them into two For this triangle, let a  12 cm, b  12 cm, and
congruent right triangles. c  16 cm.
1 1 1
s  2(a  b  c)  2(12  12  16)  2(40)  20
A   s(s  a
)(s  b)(s  c)
 20(20
  12)(
20  1
2)(20 
16)
36° a  20(8)(
8)(4)   
5120  72 cm2

18 cm  can be simplified: 5120


Note that 5120 
36 cm    5
1024 • 5  1024   325 . This shows
that the answers obtained in the first two solutions
Use one of the small right triangles to find a, the are equal. It is not necessary to simplify the radicals
apothem of the pentagon. Each of the five central if you use your calculator to obtain a decimal
360°
angles in the figure has measure  5  72°. Because

approximation rather than giving an exact answer.
the altitude from the vertex angle to the base of an
isosceles triangle is also an angle bisector, the Third solution: Notice that the first two solutions
smaller acute angle in the right triangle has measure do not involve trigonometry. It is also possible to

Discovering Geometry Solutions Manual CHAPTER 12 247


©2003 Key Curriculum Press
DG3SM586_229-251.qxd 8/2/06 6:11 PM Page 248

solve this problem by using a combination of the 27. Approximately 33.5 cm2. The diameter of the circle
Law of Cosines and the SAS Triangle Area Conjec- is 24 cm, so the radius is 12 cm. Thus you can use
ture. Look at the sketch for the first solution above, the same basic diagram as in Exercise 25. The
but ignore the altitude and look at the whole shaded region in the figure below is the segment
isosceles triangle. Let  be the central angle in the of the circle whose area you need to find.
circle, which is the vertex angle of the isosceles
16 cm
triangle. Use the Law of Cosines to find an expres-
sion for the measure of this angle. ␪_
12 cm 2 12 cm
162  122   12  12  cos 
122 2 ␪

16  12  12  2  12  12  cos 
2 2 2

162  122  122 32


cos     
1
 
2  12  12 288  9
1
  cos1 9 
You can now find the area of the triangle by using Asegment  Asector  Atriangle
the SAS Triangle Area Conjecture. Because this area
If you solved Exercise 25, you already have found
formula involves sin , it is better to substitute the
the area of the triangle in this diagram. (If not, refer
exact value of  obtained above rather than a
to the solution for Exercise 25, which shows three
decimal approximation.
methods for finding this area.) Therefore you only
1 1
  
1
A  2ab sin C  2(12)(12)  sin cos1 9 need to find the area of the sector and then subtract
the area of the triangle.
 72 cm2
sin 2  182  32, so 2  sin132, and   2 sin132.
26. Approximately 15.7 cm. Make a sketch.
Find the area of the sector.


 
2 sin1 23

  r      122 cm2
2
6 cm 360° 360°
45°
From the first or second method of solution for
Exercise 25, the area of the triangle is 325 cm2.
Asegment  Asector  Atriangle


 
2 sin1 23
Draw radii to the eight vertices of the octagon,  
360°    122  32
5
forming eight congruent isosceles triangles. The
vertex angle of each of these triangles measures
360°
  45°. In one of the isosceles triangles, draw  33.5 cm2
8
the altitude to the base, which is an apothem of the 28. Approximately 10.1 km/hr N
octagon. Let r represent the radius of the circle, at an approximate bearing of 5
which is the length of a leg of the triangle. 24.5°. Place the figure in your
45°

The altitude divides the book with respect to the r


13
isosceles triangle into two 3 cm north and east lines.
congruent right triangles, r
Recall that bearing is always 45°
each with shorter leg of length 22.5° measured clockwise from
E
3 cm and smallest angle of the north line. Because
45°
measure  2  22.5°. Use this

90°  45°  45°, the angle between the east line
triangle to find r. and the vector representing the velocity in still water
3
sin 22.5°  r is also 45°. The vector representing the current is
parallel to the east line, so, by the AIA Conjecture,
r(sin 22.5°)  3 the angle of the triangle opposite the side of length
3
r 
sin 22.5°  7.84
r is also 45°. Use the Law of Cosines to find r.
r 2  52  132 – 2(5)(13)cos 45°
d  2r  2  
3

sin 22.5°  15.7 cm r  
52  1 
32 – 2(5)(13) cos 45°  10.1
The diameter of the circle is approximately 15.7 cm.
248 CHAPTER 12 Discovering Geometry Solutions Manual
©2003 Key Curriculum Press
DG3SM586_229-251.qxd 8/2/06 6:11 PM Page 249

The speed of Leslie’s kayak is approximately 50. C. Apply the Extended Parallel/Proportionality
10.1 km/hr. Now find the direction (bearing) of Conjecture.
Leslie’s kayak. Let  be the angle of the triangle 6 4
  
opposite the side of length 5. Use the Law of Sines 963 m
to find . 6
  
4
sin  sin 45° 18 m
  
5 r m  12
5 sin 45°
sin    r 51. D. The displaced volume is (4)(8)(9)  288 cm3.

  sin1 5 sin 45°


r 
 20.5° 52. C

The bearing is 45°    24.5°. 53. A. Use proportional sides of similar triangles. Let
h equal the height of the boy.
29. False. An octahedron is a polyhedron with eight h 32 8
faces.     
1.75 12 3
30. False 3h  8(1.75)  14
14 2
h  3 ft  43 ft  4 ft 8 in.
100
54.  3
3 cm . The central angle of the base of the
remaining portion of the cone is 360°  240° 
120 1
120°, so the solid is  360  3 of the cone.
 
1 1
Vcone    2
3 BH  3 r H

31. True 32. True


Vportion of cone  
1
 1 1
  2 
3 Vcone  3 3 r H  9
1
r 2H
m2 1 300 100
33. False. The ratio of their areas is n2 .  9(5)2(12)  9  3 cm3
34. True 55. 28 cm3. The solid is a truncated cone. To find its
length of side opposite T volume, subtract the volume of the missing cone
35. False. Tangent of T   .
length of side adjacent to T (portion removed) from the volume of the complete
36. True 37. True cone. The height of the missing cone is 6  3 
3 cm. The radius of the missing cone is not shown,
38. True
but can be found using proportional sides of similar
39. True. The area of this triangle is 12  6  8  right triangles. Let x represent the radius of the
sin 60°  12  6  8  
3
2  123  cm3. (You can missing cone.
find sin 60°    3 3 x
2 from the relationship between   
6 4
the sides of a 30°-60°-90° triangle.) x  2 cm
1
40. False. The slope of line 2 is m. 1 2 1
Vcomplete cone   3 r H  3 (4)  6  32 cm
 2 3

41. False 1 2 1
Vmissing cone   3 r H  3 (2)  3  4 cm
 2 3

Vtruncated cone  32  4  28 cm3


56. 30.5 cm3, or 621  cm3. The volume of the solid,
which is a shell of a cone, is the difference between
the volume of the complete cone and the volume
of the missing cone (the portion of the original
cone that has been removed).
42. B 43. C 1 2 1
Vcomplete cone   
3 r H  3 (5) (7.5)  62.5 cm
2 3

44. C. (cos )2  (sin )2  1 for any angle . 1 2 1


Vmissing cone   
3 r H  3 (4) (6)  32 cm
2 3
45. D 46. B
61
Vshell  62.5  32  30.5 cm3  2 cm3
47. B 48. A
49. B

Discovering Geometry Solutions Manual CHAPTER 12 249


©2003 Key Curriculum Press
DG3SM586_229-251.qxd 8/2/06 6:11 PM Page 250

57. 33. Use inductive reasoning. opposite side into two segments whose lengths are
in the same ratio as the lengths of the two sides
Number of people 2 3 4 5
forming the angle. This conjecture can be used to
Number of hugs 1 3 6 10 find x.
14 28
From the table, you can find a rule: If there are n   
n(n  1)
x x
people, there will be 2 hugs. You can also find 14 28 7
    
this pattern by the following reasoning: Each person x 48 12
hugs each other person exactly once. If there are 14  2(7), so x  2(12)  24 cm.
n people, each of them must hug each of the other Then LN  14  x  14  24  38 cm.
people exactly once, and there are (n  1) other
people. However, if you count n(n  1) hugs, you Go back to the similar triangles to find y.
ML LN
would be counting each hug twice, once of each   
QP PR
of the two people hugging. Therefore, if there are
n(n  1) 28 38
n people, there will be 2 hugs. Use algebra to   
n(n  1) 21 y
find the value of n for which 2  528.
4 38
n(n  1)   
2  528 3 y
4y  114
n(n  1)  1056
y  28.5 cm
n 2  n  1056
60. Approximately 18 cm. By the Triangle Sum Conjec-
n 2  n  1056  0 ture, the measure of the third angle of the triangle
(n  32)(n  33)  0 is 180°  48°  26°  106°. Use the Law of Sines
to find x.
n  32  0 or n  33  0 sin 26° sin 106°
  
n  32 or n  33 x 39
x(sin 106°)  39(sin 26°)
Reject 32 as a possible solution because the
39 sin 26°
number of people must be a positive integer. There x 
sin 106°  18 cm
were 33 people at the reunion.
61. (x  5)2  (y  1)2  9. To write the equation of
58. (x, y) → (x  1, y  3). The first translation moves the circle, you need to find its center and radius.
each point of the triangle right 2 units and down The center of the circle is the midpoint of any
1 unit, while the second translation moves each diameter. Let C represent the center of the circle.
point of the image triangle left 1 unit and down
2 units. Thus, the composition moves each point 
5  5 2  4
C  2,  2 
 (5, 1)
of the original triangle right 1 unit and down
The radius is the distance between the center and
3 units; the rule for the single translation is
any point on the circle. The distance between (5, 1)
(x, y) → (x  1, y  3). (Notice that you don’t
and (5, 4) is 3 units. (You can use the distance
need to know the coordinates of the vertices to
formula to find this distance, but because both
find the rule.)
points have x-coordinate 5, it is easier to just find
59. w  48 cm, x  24 cm, y  28.5 cm. First use the the difference of their y-coordinates: 4  1  3.)
similar triangles to find w. Because LMN  Substitute h  5, k  1, and r  3 in the equation
PQR, corresponding sides are proportional. of a circle.
LM MN
   (x  h)2  (y  k)2  r 2
PQ QR
28 w (x  5)2  (y  1)2  32
  
21 36
(x  5)2  (y  1)2  9
4 w
   62. Sample answer: Each interior angle in a regular
3 36
36  12(3), so w  12(4)  48 cm. pentagon is 108°. Three angles would have a sum of
324°, 36° short of 360°, which would leave a gap.
Now recall the Angle Bisector/Opposite Side Conjec- Four angles would have a sum exceeding 360° and
ture: A bisector of an angle of a triangle divides the hence create an overlap.

250 CHAPTER 12 Discovering Geometry Solutions Manual


©2003 Key Curriculum Press
DG3SM586_229-251.qxd 8/2/06 6:11 PM Page 251

63. Approximately 99.5 m. Let h represent the height of Conjecture, and thus ABD  CBD by
the temple and x represent the height of the temple CPCTC. This is equivalent to saying that
above Ertha’s eye level. Make a sketch. mABC  2  mABD. (This proves that the
altitude from the vertex angle of any isosceles
triangle is also the angle bisector.)
x
h
 is the perpendicular
b. Possible answers: BD
37° . BD
bisector of AC  is the angle bisector of
1.5 m 130 m
ABC. BD  is the median to AC . From 66a, you
tan 37°  
x know that BD is the angle bisector. The other
130 possible answers follow from using more corre-
h  x  1.5  130(tan 137°)  1.5  99.5 m sponding parts of the right triangles or by
64. 30 ft. By the Parallel Lines Intercepted Arcs applying the Vertex Angle Bisector Conjecture:
Conjecture, mHK  mSJ . Therefore mHK In an isosceles triangle, the bisector of the vertex
1 1 angle is also the altitude and the median.
(360°  80°  40°)  (240°)  120°. Now use the
2 2
Arc Length Conjecture to find the circumference, TAKE ANOTHER LOOK
and then use the circumference to find the radius.
1. Several approaches are possible. Here is one of the
Here, r represents the radius of the circle and C
simplest.
represents the circumference.
a 1 1 b
 120° 1       
360°  C  3
Length of HK  C sin A sin A sin B sin B
 
a b
1 c
20  3C A similar argument can be used for  .
sin C
1
3(20)  3 3C   2. You might want to use geometry software to help
you see patterns. Measurements indicate that
C  60 ft CB2A and CB1A are supplementary; for example,
60 mCB2A  57.7° and mCB1A  123.3°. The Law
C  2r, so 60  2r, and r   
2  30 ft. of Sines, then, says that sin   sin(180°  ).
65. 4 cm. The shaded region is a sector of the circle 3. a 2  b 2  c 2  2bc cos A; b 2  a 2  c 2  2ac cos B
with central angle of measure 360°  135°  225°.
Use the given area of the sector to find the radius. 4. You can make a right triangle (by drawing an

 
225° 5 altitude) and then apply the Pythagorean Theorem
Asector   
360° r  8
2 r 2 to the two triangles formed. For example, let AD
5 , let x  DC, and let h  AD.
be the altitude to BC
10  8r 2 Then c 2  (a  x)2  h 2  a 2  2ax  x 2  h 2.
Because b 2  x 2  h 2, you have c 2  a 2  2ax
8 8 5

(10)   r 2
5 5 8   b 2  a 2  b 2  2ax. But bx  cos C, so x 
r 2  16 b cos C. Therefore c 2  a 2  b 2  2ab cos C.
A
r  16
4
(Because r represents the radius of a circle, only the b c
h
positive square root makes sense in this situation.)
The radius of the circle is 4 cm. C x D B
a
66. B
5. Both x and y can be related to the circumference of
the cone’s base, which is the length of the sector’s
arc. The radius of the arc is L, the slant height of
360°  2L 
x
the cone, so the arc’s length is  
L  x
. The radius of the cone, then, is this
180°
C
circumference divided by 2, or L  360° . The sine of
x
A D 
y
 is this radius over L, or , so y  2 sin1.
x x
a. mABC  2  mABD. Look at the two right 2 360° 360°
x
  CB
triangles in the figure. AB  and BD
  BD
. Or use the Law of Cosines with c   180° , a  L,

  CD
Then, by the Pythagorean Theorem, AD . and b  L to arrive at the equivalent answer:
x2
Thus ABD  CBD by the SSS Congruence y  cos11   64,800L .

Discovering Geometry Solutions Manual CHAPTER 12 251


©2003 Key Curriculum Press
DG3SM586_252-296.qxd 8/2/06 6:15 PM Page 252

16. True; Angle Addition Postulate


CHAPTER 13
17. True; Segment Addition Postulate
LESSON 13.1 18. ● That all men are created equal.
EXERCISES ● That they are endowed by their creator with
1. A postulate is a statement accepted as true without certain inalienable rights, that among these are
proof. A theorem is deduced from other theorems life, liberty, and the pursuit of happiness.
or postulates. ● That to secure these rights, governments are insti-
2. Subtraction: Equals minus equals are equal. Multi- tuted among men, deriving their just powers from
plication: Equals times equals are equal. Division: the consent of the governed.
Equals divided by nonzero equals are equal. ● That whenever any form of government becomes
3. Reflexive: Any figure is congruent to itself. destructive to these ends, it is the right of the
people to alter or to abolish it, and to institute
C new government, laying its foundations on such
B
principles and organizing its powers in such form
A ABC  ABC as to them shall seem most likely to effect their
safety and happiness.
Transitive: If Figure A is congruent to Figure B and
Figure B is congruent to Figure C, then Figure A is  and BO
19. 1. AO  are radii; 3. AOB is isosceles, Defi-
congruent to Figure C. nition of isosceles
If   , then  20. 1. 1  2, Given; 2. m  n; 3. 3  4,
P Q R S X Y P Q X Y
Corresponding Angles Postulate
  RS
If PQ  and RS
  XY
, then PQ
  XY
.   BC
, Given; 4. ABC  BAD,
21. 1. Given; 2. AD
Symmetric: If Figure A is congruent to Figure B, SSS Congruence Postulate; 5. D  C
then Figure B is congruent to Figure A.   CB
22. 1. Angle Bisector Postulate; 2. AB ; 3. ABD
X L  CBD, Definition of angle bisector; 4. Identity
If 
property; 5. BAD  BCD, SAS Congruence
Y Z M N Postulate; 6. A  C, CPCTC
L X 23. Add the two integers: (2n  1)  (2m  1)  2n 
then 
2m  2  2(n  m  1), which is always even.
M N Y Z
24. Multiply the two integers: (2n  1)(2m  1) 
If XYZ  LMN, then LMN  XYZ. 4nm  2n  2m  1  4nm  2n  2m  2  1 
4. Reflexive property of equality; reflexive property of 2(2nm  n  m  1)  1, which is always odd.
congruence 25. Let n be any integer. Then the next two consecutive
5. Transitive property of congruence integers are n  1 and n  2. The sum of these
three integers is (n)  (n  1)  (n  2) 
6. Subtraction property of equality n  n  1  n  2. Combining like terms gives
7. Division property of equality 3n  3  3(n  1), which is divisible by 3.

8. Distributive; Subtraction; Addition; Division 26. 299 m. Draw the perpendicular segment from the
top of the mountain to the ground. Labels have
9. Given; Addition property of equality; Multiplication been added to the resulting figure for reference
property of equality; Commutative property of below. In this figure, x represents the distance from
addition B to the mountain peak, D.
10. True; definition of midpoint D

11. True; Midpoint Postulate x h

12. True; definition of angle bisector 22° 38°


A B y C
13. True; Angle Bisector Postulate 220 m

14. False; Line Intersection Postulate Notice that the figure contains two right triangles.
h
15. False; Line Postulate 220  y . From BCD,
From ACD, tan 22°   
h
tan 38°  y.

252 CHAPTER 13 Discovering Geometry Solutions Manual


©2003 Key Curriculum Press
DG3SM586_252-296.qxd 8/2/06 6:15 PM Page 253

From the first equation, h  (220  y)(tan 22°), and Use similar triangles. From the two triangles that
from the second equation, h  y(tan 38°). Therefore share the angle with measure , hx  4684  0.75, so
(220  y)(tan 22°)  y(tan 38°). Solve this equation. h  0.75x. From the two triangles that share the
h 80
angle with measure ,  64  x  64  1.25, so h 
 
(220  y)(tan 22°)  y(tan 38°)
1.25(64  x). Therefore 0.75x  1.25(64  x). Solve
220(tan 22°)  y(tan 22°)  y(tan 38°) this equation for x.
220(tan 22°)  y(tan 38°)  y(tan 22°) 0.75x  1.25(64  x)
220(tan 22°)  y(tan 38°  tan 22°) 0.75x  80  1.25x
220 tan 22° 2x  80
y 
tan 38°  tan 22°
x  40
h  y(tan 38°)   
220 tan 22°

tan 38°  tan 22° (tan 38°)  h  x(tan )  40(0.75)  30 ft
By the Pythagorean Theorem, x 2  h2  y 2, so
29. FG  26 and DG  23 because ABGF and
x  h 2  y 2  299 m.
 BCDG are parallelograms. Triangle FGD is right
27. Volume: sphere, cylinder, cone (mFGD  90°) by the Converse of the
Pythagorean Theorem because (2 6 )2  (2
3 )2 
Cone: V  13r 2H  13(6)2(12)  144 
6 . But mFGB  128° by the Parallelogram
2
452.4 cm3
Consecutive Angles Conjecture and mDGB  140°
Sphere: V  43r 3  43(5)3  500

3  523.6 cm3 by the Parallelogram Opposite Angles Conjecture.
So mFGD  92° because the sum of the angles
Cylinder: V  r 2H  (5.5)2(5.5)  522.7 cm3
around G is 360°. So mFGD is both 90° and 92°.
Surface area: cylinder, cone, sphere 30. x  54°, y  126°, a  7.3. First look at the right
Cone: l (slant height)  12
 2  62 

 180 triangle that contains the 27° angle. The measure of
the third angle in this triangle is 90°  27°  63°.

36 • 5  65  cm
This angle and the angle marked as congruent to it
S  r 2  rl  (6)2  (6)65  form a linear pair with the angle of measure x. Thus
36  365  366.0 cm2 x  2(63°)  180°, so x  54°. Now look at the right
triangle in which the hypotenuse has length 7.3. In
Sphere: S  4r 2  4(5)2  100  314.2 cm2
this triangle, the measure of the unmarked angle is
Cylinder: S  2r 2  2rH  2(5.5)2 90°  54°  36°. Next look at the triangle that
 2(5.5)(5.5)  121  380.1 cm2 contains the side of length 6.8. By the AIA Conjec-
ture, the angle opposite this side is a right angle. The
Length of longest rod that will fit inside: cone, angle whose measure is the sum of this right angle
cylinder, sphere and the adjacent 36° angle forms a vertical angle
Cone: length of longest rod  slant height  with the angle of measure y, so y  90°  36° 
65 cm  13.4 cm 126°. Now look at the triangle in the center of the
figure that contains a 63° angle and an angle that
Sphere: length of longest rod  diameter  10 cm forms a linear pair with the angle of measure y. The
Cylinder: length of longest rod  hypotenuse of measure of that angle is 180°  y  180°  126° 
right triangle with one leg of length 11 cm (diam- 54°. (You can also find this angle measure by AIA
using x  54°.) The measure of the third angle of
eter of base) and other leg of length 5.5 cm (height
this triangle is 180°  63°  54°  63°, so the
of cylinder)   112  
5.52  151.25
  12.3 cm
triangle is isosceles. The side opposite one of the 63°
28. 30 ft. Let h represent the height at which the guy angles is 7.3, so the side opposite the other 63° angle
wires cross. Add labels to the figure for reference. is also 7.3 (Converse of the Isosceles Triangle
Conjecture). Finally, look at the obtuse triangle with
angles of measure y and 27°. Because y  126°, the
measure of the third angle in this triangle is 180° 
126°  27°  27°, so this triangle is also isosceles.
80 ft From the 63°-63°-54° triangle, you know that the
48 ft length of the side opposite the 27° angle on the far
h right is 7.3, so the length of the side opposite the
␣ x y ␤ other 27° angle must also be 7.3, that is, a  7.3.
64 ft

Discovering Geometry Solutions Manual CHAPTER 13 253


©2003 Key Curriculum Press
DG3SM586_252-296.qxd 8/2/06 6:15 PM Page 254


31. a. .The shaded region is a quarter-circle Paragraph Proof: By the definition of supplemen-
4
with radius 1, so A  14r 2  14(1)2  4. tary angles, m1  m2  180°. By the definition
of congruence, m1  m2. Therefore, by substitu-
b. 1  4. The area of the shaded region is the
tion, m1  m1  180°, or 2m1  180°. By the
difference between the area of the square of side
division property, m1  90°. Then, by the transi-
length 1 and the area of the shaded region in
tive property, m2  90°. Thus, by the definition of
31a, so A  12  4  1  4.
a right angle, 1 and 2 are both right angles.

c. 
2  1. By looking at the figures from 31a and
6. Given: Two angles are congruent
31b and using the symmetry of these figures, you
can see that the area of the shaded region is the Show: The supplements of the two angles are
difference between the area you found in 31a and congruent
the area you found in 31b. Therefore (The figure below shows supplementary angles as




  
A  4  1  4  4  1  4  2  1.
linear pairs, but they do not have to be for the
proof to work.)
IMPROVING YOUR REASONING SKILLS
Because only four moves are allowed and the letter in
each place changes, each letter must change only once. 1 2 3 4
There are several possibilities for each string and for
Given: 2  3, 1 and 2 are supplementary,
original creations. Here’s one of many possible sets:
4 and 3 are supplementary
1. MATH ⇒ MATE ⇒ RATE ⇒ ROTE ⇒ ROSE Show: 1  4
2. MATH ⇒ MATE ⇒ MARE ⇒ MORE ⇒ CORE Paragraph Proof: By the definition of supplemen-
tary angles, m1  m2  180° and m4 
3. MATH ⇒ MASH ⇒ MAST ⇒ MOST ⇒ HOST
m3  180°. Then, by the transitive property,
4. MATH ⇒ MASH ⇒ MASS ⇒ MESS ⇒ LESS m1  m2  m4  m3. By the definition of
congruence, m2  m3, so, by substitution,
5. MATH ⇒ LATH ⇒ LATE ⇒ LAVE ⇒ LIVE
m1  m2  m4  m2. By the subtraction
Sample answer for original creation: MATH ⇒ PATH ⇒ property, m1  m4. Therefore, by the definition
PITH ⇒ PITA ⇒ PICA of congruence, 1  4.
7. Given: Two angles are right angles
LESSON 13.2
Show: The two angles are congruent
EXERCISES
1. Linear Pair Postulate 1 2
2. Parallel Postulate, Angle Addition Postulate, Linear
Pair Postulate, CA Postulate (through the AIA Given: 1 and 2 are right angles
Theorem) Show: 1  2
3. Parallel Postulate Paragraph Proof: By the definition of a right
angle, m1  90° and m2  90°. Then, by the
4. Perpendicular Postulate transitive property, m1  m2. Therefore, by the
5. Given: Two angles are both congruent and supple- definition of congruence, 1  2.
mentary 8. Given: Two lines cut by a transversal to form
Show: Each angle is a right angle congruent alternate interior angles
(These angles may form a linear pair, as in the Show: The two lines are parallel
figure below, but they do not have to for the proof 3
to work.) 1

1 2 2

21
Given: Lines 1 and 2 cut by transversal 3;
Given: 1  2 and 1 and 2 are supple- 1  2
mentary
Show: 1  2
Show: 1 and 2 are right angles

254 CHAPTER 13 Discovering Geometry Solutions Manual


©2003 Key Curriculum Press
DG3SM586_252-296.qxd 8/2/06 6:15 PM Page 255

Flowchart Proof 11. Given: Two parallel lines cut by a transversal to


form interior angles on the same side of the
1  2
transversal
Given
1  3 1   2 Show: The two angles are supplementary
Transitive CA Postulate 3
2  3
property of 3 1
VA Theorem congruence 1
2
3 2
9. Given: Two parallel lines
cut by a transversal to form 1 1

alternate exterior angles 3


2 Given: Lines 1 and 2 cut by transversal 3; 1  2
Show: The alternate exte-
rior angles are congruent
2 Show: 1 and 2 are supplementary
Given: Lines 1 and 2 cut Flowchart Proof (See proof at bottom of page)
by transversal 3; 1  2 12. Given: Two lines cut by a transversal to form
Show: 1  2 supplementary interior angles on the same side of
Flowchart Proof the transversal
Show: The two lines are parallel
1  3
3
VA Theorem 1
1   2 1  2
31
Given Transitive 2
3  2 2
property of
CA Postulate congruence

10. Given: Two lines cut by a transversal to form Given: Lines 1 and 2 cut by transversal 3;
congruent alternate exterior angles 1 and 2 are supplementary
Show: The lines are parallel Show: 1  2
3
Paragraph Proof: By the definition of supplemen-
1
1
tary angles, m1  m2  180°. By the Linear Pair
3
2 Postulate, 1 and 3 are supplementary. Then, by
4
2
the definition of supplementary angles, m1 
m3  180°. Therefore, by the substitution prop-
erty, m1  m3  m1  m2. By the subtrac-
Given: Lines 1 and 2 cut by transversal 3;
tion property, m3  m2. Angles 3 and 2 are
1 2
alternate interior angles, so, by the Converse of the
Show: 1  2 AIA Theorem, 1  2.
Flowchart Proof
13. Given: Two lines in the 1
A
1 and 2 cut 1  3 same plane, each parallel 1
2
by transversal 3 4  2 to a third line B 2
3
Given VA Theorem 3  4 1   2 Show: The two lines are 3

Transitive Converse parallel to each other


1  2
property of AIA
Given Theorem
Given: Coplanar lines 1,
2, and 3 with 1  2 and 3  2
Show: 1  3

Lesson 13.2, Exercise 11.


1 and 3 are m1  m3 
supplementary 180°

1 and 2 cut Linear Pair Definition of m1  m2  1 and 2 are


by transversal 3 Postulate supplementary 180° supplementary
Substitution Definition of
2  3 m2  m3 property supplementary
CA Postulate Definition of
congruence

Discovering Geometry Solutions Manual CHAPTER 13 255


©2003 Key Curriculum Press
DG3SM586_252-296.qxd 8/2/06 6:15 PM Page 256

Paragraph Proof: By the Line Postulate, construct 16. 1066 cm3. The volume of the truncated pyramid is
 with A on 1 and B on 2. This transversal will
AB the difference between the volume of the complete
intersect 3 by the Parallel Postulate. By the Interior pyramid (before the top was sliced off) and the
Supplements Theorem, 1 and 2 are supplemen- volume of the top that was sliced off, which is a
tary, so, by the definition of supplementary angles, smaller pyramid. These two pyramids will be
m1  m2  180°. By the CA Postulate, 2  referred to as large pyramid and small pyramid in
3, so, by the definition of congruence, m2  this solution.
m3. Then, by the substitution property, m1 
The cut is two-thirds of the distance from the base
m3  180° and 1 and 3 are supplementary by
to the vertex, or one-third of the distance from the
definition. Therefore 1  3 by the Converse of the
vertex to the base. This means that the height of the
Interior Supplements Theorem.
small pyramid is one-third the height of the large
14. Given: Two lines in the same plane are each one. The two pyramids are similar, so each side
perpendicular to a third line of the base of the small triangle is one-third the
Show: The two lines are parallel to each other length of the corresponding side of the large
3
triangle. By the Proportional Volumes Conjecture,
Vsmall pyramid
1 1
    
Vlarge pyramid 3
1 3 1
27
2
2 Then the volume of the truncated pyramid is
1  217  2267 of the volume of the large pyramid, or
26
Given: Coplanar lines 1, 2, and 3 with 1  2 (1107)  1066 cm3.
27
and 3  2 17. No. Make a table showing the pieces needed.
Show: 1  2
Area (ft)2
Flowchart Proof
Bottom (2-by-3 rectangle) 6
1  2, 3  2
Given
2 sides 
3-by-112 rectangle 9
Back and front 2-by-112 rectangle 6
1 is a right angle;
2 is a right angle 2 rooftops 3-by-2 rectangle  812
Definition of 2 gable ends (2-by-1 triangle) 2
perpendicular
Total 3112 ft2
1  2 Plywood (4-by-8 rectangle) 32 ft2
Right Angles Are The area is less than that of one sheet of plywood.
Congruent Theorem
However, it is impossible to cut the correct size
pieces from one piece. Two sheets would be enough.
1   2
Converse of
18. x  92°. In the left isosceles triangle, one base angle
AIA Theorem forms a linear pair with the 112° angle, so each base
angle measures 180°  112°  68°, and the vertex
15. Linear Pair angle measures 180°  2(68°)  44°. Now look at
Postulate the middle isosceles triangle. The base angle that
has one side along the top parallel line has measure
VA Theorem CA Postulate 68° by the AIA Theorem. Therefore this is also a
68°-68°-44° triangle. Finally, look at the isosceles
triangle on the right. The base angle in the lower
Converse of
AIA Theorem left of this triangle has measure 180°  68°  68° 
44°. The measure of the other base angle must also
be 44°, so x  180°  2(44°)  92°.
Converse of
AEA Theorem 19. a. Always. Because its diagonals bisect each other,
ABCD is a parallelogram (converse of
Parallelogram Diagonals Conjecture). Then

256 CHAPTER 13 Discovering Geometry Solutions Manual


©2003 Key Curriculum Press
DG3SM586_252-296.qxd 8/2/06 6:15 PM Page 257

BAD and ADC are supplementary because Case 1: A, B, and P are collinear.
consecutive angles of a parallelogram are supple- Paragraph Proof: P is on AB  and CD (given), so
mentary (Parallelogram Consecutive Angles P  E by the Line Intersection Postulate. P is the
Conjecture). midpoint of AB by the definition of perpendicular
b. Sometimes. ADM and MAD are complemen- bisector, so AP  BP by the definition of midpoint.
tary if mAMD  90°, that is, if AC  BD. But Case 2: A, B, and P are not collinear.
the diagonals of a parallelogram are perpendic- Plan: Use the SAS Congruence Postulate to get
ular only if the parallelogram is a rhombus. This AEP  BEP. Then use CPCTC to get AP   BP .
parallelogram may or may not be a rhombus;
you don’t have enough information to tell. Proof:
Statement Reason
c. Sometimes. A diagonal of a parallelogram could
 is the perpen-
1. CD 1. Given
be longer than the sum of the lengths of two
opposite sides, or it could be shorter or the same dicular bisector of

AB
length. There is not enough information to tell.
2. E is the midpoint 2. Definition of perpen-
d. Never. By the Triangle Inequality Conjecture, 
of AB dicular bisector
AD  CD  AC.

3. AE  BE 3. Definition of
IMPROVING YOUR REASONING SKILLS midpoint
321   AB
4. PE  4. Definition of perpen-
EXTENSION dicular bisector
Given: 1  2, 1 and 2 cut by transversal 3 5. PEA and PEB 5. Definition of
are right angles perpendicular
Show: 2  6
6. PEA  PEB 6. Right Angles Are
3
Congruent Theorem
1 2 1
4 3   PE
7. PE  7. Identity property of
5 6 2 congruence
7 8
8. AEP  BEP 8. SAS Congruence
Postulate
Paragraph Proof: 2  7 by AEA. 7  6 by the
  BP
9. AP  9. CPCTC
VA Theorem, so 2  6 by the transitive property of
congruence. 10. AP  BP 10. Definition of
congruence
LESSON 13.3 2. Given: A segment with a point that is equally
distant from the endpoints of the segment
EXERCISES
Show: The point is on the perpendicular bisector of
1. Given: A point on the perpendicular bisector of a the segment
segment
 be the segment and P be the point. In
Let AB
Show: The point is equidistant from the endpoints order to prove the theorem, two cases need to be
of the segment considered.

Case 1: A, P, and B are collinear. (P is on AB
C
between A and B.)
A P B
P
Given: A, P, and B are collinear; PA  PB

Show: P is on the perpendicular bisector of AB
A E B
Paragraph Proof: PA  PB is given, so PA   PB
D by the definition of congruence. Because P is on
, this means that P is the midpoint of AB
AB  by
 with perpendicular bisector CD
Given: AB ; the definition of midpoint. Then P is on the
 intersects AB
CD 
 at E; P is a point on CD  by the definition of
perpendicular bisector of AB
Show: AP  BP perpendicular bisector.

Discovering Geometry Solutions Manual CHAPTER 13 257


©2003 Key Curriculum Press
DG3SM586_252-296.qxd 8/2/06 6:15 PM Page 258

Case 2: A, P, and B are not collinear. (P is not Plan: Use the identity property and the ASA
.)
on AB Congruence Postulate to get ABC  BAC. Then
P use CPCTC and the definition of isosceles triangle.
Proof:
A B
Statement Reason
E
1. A  B 1. Given
Given: A, P, and B are not collinear; PA  PB
2. B  A 2. Symmetric property of

Show: P is on the perpendicular bisector of AB congruence
Paragraph Proof: Let E be the midpoint of AB   
3. AB  BA 3. Identity property of
(Midpoint Postulate). Use the Line Postulate to congruence
. PA  PB is given, so PA
draw PE   PB  by the
 
definition of congruence. EA  EB by the defini- 4. ABC  BAC 4. ASA Congruence
  PE
tion of midpoint. PE  by the identity property Postulate
of congruence. Therefore AEP  BEP by the   BC
5. AC  5. CPCTC
SSS Congruence Postulate. Then AEP BEP by 6. ABC is isosceles 6. Definition of isosceles
CPCTC and AEP and BEP are a linear pair by triangle
construction and are thus supplementary by the 5. Given: An angle with a point that is equally distant
Linear Pair Postulate. So AEP and BEP are both from the sides of the angle
right angles by the Congruent and Supplementary
Theorem. Therefore PE is the perpendicular bisector Show: The point is on the bisector of the angle

of AB by the definition of perpendicular bisector. B

3. Given: Isosceles triangle P

Show: The base angles of the triangle are congruent C A


C
Given: BCA with P in the interior of the angle;
 and CB
P is equidistant from CA .
Show: P is on the angle bisector of BCA
A B Paragraph Proof: Use the Line Postulate and
Perpendicular Postulate to draw perpendicular
Given: Isosceles triangle ABC
segments PB and PA from P to the sides of the
Show: A  B angle and also to draw BA. By the definition of
Plan: Use the identity property and the SSS distance from a point to a line and the given
Congruence Postulate to get ABC  BAC. equidistance, PB  PA, so PB  PA  by the defini-
Therefore A  B by CPCTC. tion of congruence. Thus PAB is isosceles with
Proof:  and PA
legs PB . Therefore, by the Isosceles Triangle
Theorem, PAB  PBA. By the Angle Addition
Statement Reason
Postulate, mCAP  mBAC  mPAB, and
 
1. AC  BC 1. Definition of isosceles mCBP  mCBA  mPBA. mCAP 
  AC
2. BC  2. Symmetric property of mCBP  90° by the definition of perpendicular.
congruence Thus, by the subtraction property, BAC  ABC.
  AB
3. AB  3. Identity property of Then, by the Converse of the Isosceles Triangle
congruence Theorem, CB  CA . By the identity property of

congruence, CP  CP . Therefore ACP  BCP
4. ABC  BAC 4. SSS Congruence
by the SSS Congruence Postulate, and thus
Postulate
ACP  BCP by CPCTC. Therefore CP  is the
5. A  B 5. CPCTC angle bisector of BCA by the definition of angle
4. Given: Triangle with two C bisector, and P is on the angle bisector.
congruent angles
Show: The triangle is isosceles
Given: ABC with A  B
A B
Show: ABC is isosceles

258 CHAPTER 13 Discovering Geometry Solutions Manual


©2003 Key Curriculum Press
DG3SM586_252-296.qxd 8/2/06 6:15 PM Page 259

6. Given: A triangle with the three perpendicular 7. Given: A triangle with the three angle bisectors
bisectors of the sides Show: The three angle bisectors are concurrent
Show: The three perpendicular bisectors are C
concurrent
C m m n 
n
Q
P
A B
A B

Given: ABC with lines , m, and n as the angle
bisectors of A, B, and C, respectively
Given: ABC with lines , m, and n as perpen- Show: , m, and n are concurrent
, BC
dicular bisectors of sides AB , and AC,
respectively Paragraph Proof: By the Line Intersection Postu-
late, lines  and m intersect in exactly one point;
Show: , m, and n are concurrent
call that point Q. Because Q is on the angle bisec-
Paragraph Proof: By the Line Intersection Postu- tors of both A and B, by the Angle Bisector
late, lines  and m intersect in exactly one point; Theorem, Q is equally distant from AB  and AC and
call that point P. Because P is on the perpendicular  
also from AB and BC . Then, by the transitive prop-
 and BC
bisectors of both AB , by the Perpendicular  and BC
erty, Q is equally distant from AC . Then, by
Bisector Theorem, AP  BP and BP  CP. Then, the Converse of the Angle Bisector Theorem, point
by the transitive property, AP  CP. Then, by the Q is on n. Therefore , m, and n are concurrent by
Converse of the Perpendicular Bisector Theorem, the definition of concurrent.
point P is on line n, the perpendicular bisector of
. Therefore , m, and n are concurrent by the
AC
definition of concurrent.

Lesson 13.3 (continued)

8. Given: A triangle with one exterior angle drawn B


2
Show: The measure of the exterior angle is equal to the
sum of the measures of its two remote interior angles 1 3 4
A C
Given: ABC with interior angles 1, 2, and 3
at vertices A, B, and C, respectively; 4 an exterior angle at vertex C
Show: m4  m1  m2
Plan: Use the Linear Pair Postulate and the definition of supplementary angles to get
m3  m4  180°. Then use the Triangle Sum Theorem and the transitive
property to get m1  m2  m3  m3  m4. Therefore m1  m2 
m4 by the subtraction property.
Proof: Statement Reason
1. 4 is an exterior angle at vertex C 1. Given
2. 3 and 4 are supplementary 2. Linear Pair Postulate
3. m3  m4  180° 3. Definition of supplementary
4. m1  m2  m3  180° 4. Triangle Sum Theorem
5. m1  m2  m3  m3  m4 5. Transitive property
6. m1  m2  m4 6. Subtraction property
9. Given: Any quadrilateral D
3 4
Show: The sum of the measures of the four angles is 360°
A
Given: Quadrilateral ABCD C
1
Show: mA  mABC  mC  mCDA  360° 2
B
Plan: Use the Triangle Sum Theorem and the addition property
to get mA  m1  m3  mC  m4  m2  360°. Then use the Angle
Addition Postulate and the substitution property to get mA  mABC  mC 
mCDA  360°.

Discovering Geometry Solutions Manual CHAPTER 13 259


©2003 Key Curriculum Press
DG3SM586_252-296.qxd 8/2/06 6:15 PM Page 260

Proof: Statement Reason



1. Construct DB 1. Line Postulate
2. mA  m1  m3  180° 2. Triangle Sum Theorem
3. mC  m4  m2  180° 3. Triangle Sum Theorem
4. mA  m1  m3  mC  4. Addition property
m4  m2  360°
5. mA  (m1  m2)  mC  5. Commutative and associative
(m3  m4)  360° properties of addition
6. m1  m2  mABC 6. Angle Addition Postulate
7. m3  m4  mADC 7. Angle Addition Postulate
8. mA  mABC  mC  8. Substitution property
mADC  360°
10. Given: Isosceles triangle C

Show: The medians to the congruent sides are congruent


N M
Given: Isosceles triangle ABC with AC  BC ; BN
 is the median
 and AM
to AC  is the median to BC
A B

Show: BN  AM 
Plan: Use the definitions of median and midpoint to get BM  12BC and AN  12AC.
Then use the multiplication property and the substitution property to get AN  BM.
By the identity property, the Isosceles Triangle Theorem, and the SAS Congruence
Postulate, ABN  BAM. Therefore BN   AM  by CPCTC.
Proof: Statement Reason
 is the median to AC
1. BN ; 1. Given
 is the median to BC
AM 
;
2. N is the midpoint of AC 2. Definition of median
M is the midpoint of BC
  BC
3. AC  3. Given
4. AC  BC 4. Definition of congruence
1 1
5. 2AC  2BC 5. Multiplication property
1 1
6. AN  2AC; BM  2BC 6. Definition of midpoint
7. AN  BM 7. Substitution property
8.   BM
AN  8. Definition of congruence
9.   BA
AB  9. Identity property of congruence
10. A  B 10. Isosceles Triangle Theorem
11. ABN  BAM 11. SAS Congruence Postulate
12.   AM
BN  12. CPCTC
11. Given: Isosceles triangle C

Show: The angle bisectors to congruent sides are congruent


Q P
  BC
Given: Isosceles triangle ABC with AC ; BQ is the angle
 and AP
bisector to AC  is the angle bisector to BC

A B
Show: AP  BQ 
Plan: Use the definition of angle bisector to get mPAB  12mCAB and mQBA 
1
mCBA. Then use the Isosceles Triangle Theorem, the multiplication property, and
2
the substitution property to get PAB QBA. By the identity property and the
ASA Congruence Postulate, ABP BAQ. Therefore AP  BQ  by CPCTC.

260 CHAPTER 13 Discovering Geometry Solutions Manual


©2003 Key Curriculum Press
DG3SM586_252-296.qxd 8/2/06 6:15 PM Page 261

Proof: Statement Reason


 is the angle bisector to AC
1. BQ ; 1. Given
 
AP is the angle bisector to BC
 is the angle bisector of CBA;
2. BQ  is opposite ABC; BC
2. AC  is opposite CAB
 is the angle bisector of CAB
AP
1
3. mPAB  2mCAB; mQBA  3. Definition of angle bisector
1
mCBA
2
  BC
4. AC  4. Given
5. CAB CBA 5. Isosceles Triangle Theorem
6. mCAB  mCBA 6. Definition of congruence
1 1
7. 2mCAB  2mCBA 7. Multiplication property
8. mPAB  mQBA 8. Substitution
9. PAB QBA 9. Definition of congruence
 BA
10. AB  10. Identity property of congruence
11. ABP BAQ 11. ASA Congruence Postulate
 BQ
12. AP  12. CPCTC
12. Given: Isosceles triangle C

Show: The altitudes to congruent sides are congruent


T S
  BC
Given: Isosceles triangle ABC with AC ; AS
 is the altitude to
, and BT
BC  is the altitude to AC

A B
 BT
Show: AS 
Plan: Use the Isosceles Triangle Theorem, the Right Angles Are Congruent Theorem,
and the SAA Theorem to get ASB  BTA. Then AS   BT  by CPCTC.
Proof: Statement Reason
 is the altitude to BC
1. AS ; 1. Given
 is the altitude to AC
BT 
  BC
2. AS ; BT  AC 2. Definition of altitude
3. ASB is a right angle; 3. Definition of perpendicular
BTA is a right angle
4. ASB BTA 4. Right Angles Are Congruent Theorem
 BC
5. AC  5. Given
6. ABC CAB 6. Isosceles Triangle Theorem
7.  BA
AB  7. Identity property of congruence
8. ASB BTA 8. SAA Theorem
9.  BT
AS  9. CPCTC

Lesson 13.3 (continued)


Paragraph Proof: By the definition of median,
13. Theorem A: median → angle bisector  BD
AD . AC
 BC is given. CD
 CD by the
C
identity property of congruence. Therefore
ADC BDC by the SSS Congruence Postulate.
ACD BCD by CPCTC, and CD  is the angle
bisector of ACB by the definition of angle bisector.
A D B Theorem B: angle bisector → altitude
 BC
Given: Isosceles triangle ACB with AC ; CD
 is C


the median to AB
 is the angle bisector of ACB
Show: CD
A D B

Discovering Geometry Solutions Manual CHAPTER 13 261


©2003 Key Curriculum Press
DG3SM586_252-296.qxd 8/2/06 6:15 PM Page 262

Given: Isosceles triangle ACB with AC BC ; CD


 is Reject 15 because y represents a side length and
the angle bisector of ACB therefore must be positive, so y  3.
Show: CD is the altitude from C to AB
 Now apply the Pythagorean Theorem to ABD to
Paragraph Proof: By the definition of angle find x:
bisector, ACD BCD. AC  BC is given.
5 2
x 2  y 2  3
 CD
CD  by the identity property of congruence.
Therefore ADC  BDC by the SAS Congruence x 2  9  45
Postulate. CDA CDB by CPCTC. By the x 2  36
Linear Pair Postulate and the Congruent and
Supplementary Theorem, ADC and BDC are x6
 is the altitude by
both right angles. Therefore CD 15. 21.9 m. Let S be the location of the nest on the top
the definitions of perpendicular and altitude. of the shorter tree and T be the location of the nest
Theorem C: altitude → median on the taller tree.
C T

U
6.1 m
A B d
D

 BC
Given: Isosceles triangle ACB with AC ; CD
 is

the altitude from C to AB S Q
Show: CD  is the median to AB

3.6 m
Paragraph Proof: By the definitions of altitude and
perpendicular, CDA and CDB are right angles.

m
.4
27
Then CDA  CDB by the Right Angles Are P
 BC
Congruent Theorem. AC  is given, and 16
.7
therefore A  B by the Isosceles Triangle m

Theorem. Thus ADC  BDC by the SAA


Theorem, and AD BD
 by CPCTC. Therefore CD  R

 by the definition of median.


is the median to AB Triangle PQR is a right triangle with hypotenuse
. Apply the Pythagorean Theorem to this triangle
RQ
14. x  6, y  3. Label points in the figure for
reference. to find PQ.
A y 16.7 2  (PQ)2  27.42
D
3 5 12
PQ  
27.42 
16.72  
471.87
x
(This length is approximately 21.72 m, but do not
C
B round this intermediate result.)
ABD ACB by the AA Similarity Conjecture.   TQ
Because both trees are vertical, SP . ST and PQ 
(A is common to both triangles; ADB and are not parallel. Thus PSTQ is a trapezoid. Draw
ABC are both right angles.) Corresponding sides , the perpendicular segment from S to TQ
SU , to
of similar triangles are proportional, so AAC
B AD
 
AB . form parallelogram PSUQ (which is also a
Use this proportion to find y. rectangle). Opposite sides of a parallelogram are
3 5 y congruent, so SU  PQ   471.87 . Now look at
 STU. Because SU   TU , this is a right triangle
y  12 35
with hypotenuse ST . Because SP and UQ  are oppo-
5 2
y(y  12)  3
site sides of a parallelogram, UQ  SP  3.6, so
y 2  12y  45 TU  TQ  UQ  6.1  3.6  2.5. Apply the
Pythagorean Theorem to STU to find d  ST.
y 2  12y  45  0
2
d 2  471.87
  (2.5)2
(y  15)(y  3)  0
y  15  0 or y  3  0 d   471
 .87 2 
(2.5)2  
478.12
21.9

y  15 or y  3 The distance between the two nests is about 21.9 m.

262 CHAPTER 13 Discovering Geometry Solutions Manual


©2003 Key Curriculum Press
DG3SM586_252-296.qxd 8/2/06 6:15 PM Page 263

16. First image: (0, 3); second image: (6, 1). Let B repre- find the area of the shaded region, add twice this
sent the first image of A under the glide reflection difference to the area of the triangle from 18a:
and let C represent the second image. Use a graph  3  3  3   3 
A   2     
to find the images. 4 6 4  4 3 2
y
  3
A (–2, 9) 10
 
3 4
19. One possible sequence:
(0, 7)
(–4, 7)
(2, 5) 1. Fold A onto B and crease. Label the crease 1.
5 (6, 5)
Label the midpoint of the arc M.
B (0, 3)
2. Fold line 1 onto itself so that M is on the crease.
C (6, 1) Label this crease 2.
x
–5 5 10
1
M
2
–5 xy5 A B

To find the images of A and B under the reflection, , and  is the desired
draw the line x  y  5, or y  5  x. M is the midpoint of AB 2
tangent.
First glide reflection: The image of A(2, 9) under
the reflection across the line x  y  5 is (4, 7); 20. mBAC
13°. AB   BC , so ABC is a right
the image of (4, 7) under the translation (x, y) → triangle with hypotenuse AC . By the Pythagorean
(x  4, y  4) is (0, 3), so the coordinates of B Theorem, AB  5 2  122  13. Thus

are (0, 3).
tan BAC  BAC B  3
, and mBAC 
13
Second glide reflection: Start with (0, 3), the image tan1133 
13°.
of A under the first glide reflection. The image of
 and WX
21. In each figure, draw YW  to form WXY,
(0, 3) under the reflection is (2, 5); the image of
(2, 5) under the translation is (6, 1), so the XOW, and YOW.
coordinates of C are (6, 1).  is a radius and YX
a. B. In each figure, OW  is a
17. BC  FC makes ABCF a rhombus, so the diagonals, diameter.
 and BF
AC , are perpendicular. mFGC  90°, so In Figure A, OW  10 cm, so YX  20 cm.
mCFG  mFCG  90°. FD   GE, so m2  Because mWOX  40° and WOX is a central
  40°. Because WYX
angle of the circle, mWX
mCFG  90°. By substitution, m2  mFCG. ,
1  FCG by AIA, so 1  2 by transitivity. is an inscribed angle that intercepts WX
1
mWYX  2(40°)  20°. Because it is inscribed
3
18. a. .
4 The arcs show that the shaded region is an in a semicircle, YWX is a right angle and
equilateral triangle with side length 1. Any alti- WYX is a right triangle with hypotenuse YX.
WX WX
tude of this triangle divides the triangle into two Therefore sin 20°  YX  20 , so
 
YW YW
congruent 30°-60°-90° triangles with shorter leg WX  20 sin 20°, and cos 20°  YX  20 , so
 
of length 12, so the length of the longer leg, which YW  20 cos 20°. Thus perimeter of WXY 
3
is the height of the triangle, is 12(
3)   2 . YX  YW  WX  20  20 sin 20° + 20 cos 20°

Therefore A  12bh  12(1)   3 3
45.6 cm.
2  4 .

In Figure B, OW  20 cm, so YX  40 cm. Use
b. 6. The shaded region is a sector of a circle with
the same reasoning as above to find the other
radius 1. The central angle of this sector is the
two side lengths in WXY. First mWOX  10°,
same angle as the angle in the lower left of the   10°, and mWYX  1(10°)  5°.
so mWX
shaded triangle in 18a. Because that triangle is 2
Then WX  40 sin 5° and YW  40 cos 5°. Thus
equilateral, the measure of each of its angles is
perimeter of WXY  YX  YW  WX  40
60°, so the sector has a 60° central angle. There-
60° 1   40 sin 5° + 40 cos 5°
83.3 cm.
fore A   360° r  6 (1)  6 .
 2  2 
Therefore the perimeter of WXY is greater in
  3
c. 
3  4 . The difference between the areas of the Figure B.
shaded regions in 18b and 18a is 6  
3
4 . To

Discovering Geometry Solutions Manual CHAPTER 13 263


©2003 Key Curriculum Press
DG3SM586_252-296.qxd 8/2/06 6:15 PM Page 264

1 1 1
b. B. In both figures, XOW is an isosceles triangle 22. a. x  2(a  c); y  2(a  b); z  2(b  c)
whose legs are radii of the circle. Apply the SAS 1 1 1
b. w  2(a  b); x  2(b  c); y  2(c  d);
Triangle Area Conjecture to XOW in both
figures. 1
z  2(d  a)
1
Figure A: A  2(OW)(OX)sin WOX IMPROVING YOUR VISUAL THINKING SKILLS
1
 2(10)(10)sin 40°
32.1 cm2
1
Figure B: A  2(OW)(OX)sin WOX
1
 2(20)(20)sin 10°
34.7 cm2
Therefore the area of XOW is greater in
Figure B.

LESSON 13.4
EXERCISES
1. Given: A quadrilateral with two pairs of congruent opposite D C
angles
Show: The quadrilateral is a parallelogram
Given: Quadrilateral ABCD with A  C and B  D A B

Show: ABCD is a parallelogram


Plan: Use x to represent the measures of one pair of congruent angles and y to repre-
sent the other pair. Use the Quadrilateral Sum Theorem and the division property to
get x  y  180°. Therefore the opposite sides are parallel by the Converse of the
Interior Supplements Theorem.
Proof: Statement Reason
1. A  C; B  D 1. Given
2. mA  mC  x; mB  mD  y 2. Definition of congruence
3. x  x  y  y  360° 3. Quadrilateral Sum Theorem
4. 2x  2y  360° 4. Combine like terms
5. 2(x  y)  360° 5. Distributive property
6. x  y  180° 6. Division property
7. A and D are supplementary 7. Definition of supplementary
  AB
8. DC  8. Converse of the Interior Supplements Theorem
9. A and B are supplementary 9. Definition of supplementary

10. AD  BC 10. Converse of the Interior Supplements Theorem
11. ABCD is a parallelogram 11. Definition of parallelogram
2. Given: A quadrilateral with one pair of opposite sides that D C
3 2
are both parallel and congruent
Show: The quadrilateral is a parallelogram 1
4
Given: Quadrilateral ABCD with AD  BC and AD  BC  A B

Show: ABCD is a parallelogram


Plan: Use the AIA Theorem, the identity property, and the SAS Congruence Postulate
to get ADC  CBA. Then use CPCTC and the Converse of the AIA Theorem to
  DC
get AB .

264 CHAPTER 13 Discovering Geometry Solutions Manual


©2003 Key Curriculum Press
DG3SM586_252-296.qxd 8/2/06 6:15 PM Page 265

Proof: Statement Reason



1. Construct AC 1. Line Postulate
  BC
2. AD  2. Given
3. 1  2 3. AIA Theorem
 BC
4. AD  4. Given
 
5. AC  CA 5. Identity property of congruence
6. ADC  CBA 6. SAS Congruence Postulate
7. 3  4 7. CPCTC
8.   AB
DC  8. Converse of the AIA Theorem
9. ABCD is a parallelogram 9. Definition of parallelogram
3. Given: Rhombus D C
7 6
8 5
Show: Each diagonal of the rhombus bisects two opposite angles
Given: Rhombus ABCD with diagonals AC  and BD  1
2 3
4
A B
Show: AC bisects DAB and BCD; BD  bisects ADC and CBA
Plan: Use the definition of rhombus, the identity property, and the SSS Congruence
Postulate to get ABC  ADC. Then use CPCTC and the definition of angle
 bisects DAB and BCD. Repeat using diagonal BD
bisector to prove that AC .
Proof: Statement Reason
1. ABCD is a rhombus 1. Given
 
2. AB  AD 2. Definition of rhombus
  DC
3. BC  3. Definition of rhombus
  AC
4. AC  4. Identity property of congruence
5. ABC  ADC 5. SSS Congruence Postulate
6. 1  2; 6  5 6. CPCTC
 bisects DAB and BCD
7. AC 7. Definition of angle bisector
  CD
8. AD  8. Definition of rhombus
 
9. AB  CB 9. Definition of rhombus
 
10. BD  BD 10. Identity property of congruence
11. ADB  CDB 11. SSS Congruence Postulate
12. 3  4; 8  7 12. CPCTC

13. BD bisects ABC and ADC 13. Definition of angle bisector
4. Given: Parallelogram D C

Show: The consecutive angles are supplementary


Given: Parallelogram ABCD
A B
Show: A and B are supplementary; B and C are
supplementary; C and D are supplementary; D and A are supplementary
  BC
Plan: Use the definition of a parallelogram to get AD  and AB
  DC
. Then use
the Interior Supplements Theorem.
Flowchart Proof
A and B are supplementary
AD  BC C and D are supplementary
Definition of Interior Supplements Theorem
ABCD is a parallelogram
parallelogram
Given B and C are supplementary
AB  DC D and A are supplementary
Definition of Interior Supplements Theorem
parallelogram

Discovering Geometry Solutions Manual CHAPTER 13 265


©2003 Key Curriculum Press
DG3SM586_252-296.qxd 8/2/06 6:15 PM Page 266

5. Given: A quadrilateral with four congruent sides D C


3 2
Show: The quadrilateral is a rhombus
Given: Quadrilateral ABCD with AB  BC CD  DA
 1
4
A B
Show: ABCD is a rhombus
Plan: Use the identity property and the SSS Congruence Postulate to get ABD
CDB. Then use CPCTC and the Converse of the AIA Theorem to get AB   CD and
  CB
AD . Therefore ABCD is a rhombus by the definitions of parallelogram and
rhombus.
Proof: Statement Reason

1. Construct BD 1. Line Postulate
 CD
2. AB  2. Given
 CB
3. AD  3. Given

4. BD DB 4. Identity property of congruence
5. ABD CDB 5. SSS Congruence Postulate
6. 1 2 6. CPCTC
  CD
7. AB  7. Converse of the AIA Theorem
8. 3 4 8. CPCTC
 
9. AD  CB 9. Converse of the AIA Theorem
10. ABCD is a parallelogram 10. Definition of parallelogram
11. ABCD is a rhombus 11. Definition of rhombus
6. Given: A quadrilateral with four congruent angles D C

Show: The quadrilateral is a rectangle


Given: Quadrilateral ABCD with A  B  C  D
A B
Show: ABCD is a rectangle
Plan: Use the Converse of the Opposites Angles Theorem to prove that ABCD is a
parallelogram. Then use the definition of rectangle.
Proof: Statement Reason
1. A  C; B  D 1. Given
2. ABCD is a parallelogram 2. Converse of Opposite Angles Theorem
3. A  B  C  D 3. Given
4. mA  mB  mC  mD 4. Definition of congruence
5. mA  mB  mC  mD  360° 5. Quadrilateral Sum Theorem
6. 4mA  360° 6. Substitution property
7. mA  90° 7. Division property
8. mA  mB  mC  mD  90° 8. Substitution property
9. ABCD has four right angles 9. Definition of right angle
10. ABCD is a rectangle 10. Definition of rectangle
7. Given: Rectangle D C

Show: The diagonals of the rectangle are congruent


Given: Rectangle ABCD with diagonals BD  and AC 
  AC
Show: BD  A B

Plan: Use the definition of a rectangle to prove that ABCD is a parallelogram and
DAB  CBA. Then use the Opposite Sides Theorem, the identity property, and
the SAS Congruence Postulate to get DAB  CBA. Finish with CPCTC.

266 CHAPTER 13 Discovering Geometry Solutions Manual


©2003 Key Curriculum Press
DG3SM586_252-296.qxd 8/2/06 6:15 PM Page 267

Proof: Statement Reason


1. ABCD is a rectangle 1. Given
2. ABCD is a parallelogram 2. Definition of rectangle
 CB
3. DA  3. Opposite Sides Theorem
4. DAB and CBA are right angles 4. Definition of rectangle
5. DAB  CBA 5. Right Angles Are Congruent Theorem
6.  BA
AB  6. Identity property of congruence
7. DAB  CBA 7. SAS Congruence Postulate
8.  AC
BD  8. CPCTC
8. Given: A parallelogram with congruent diagonals D C

Show: The parallelogram is a rectangle


Given: Parallelogram ABCD with DB AC 
A B
Show: ABCD is a rectangle
Plan: Use the Opposite Sides Theorem, the identity property of congruence, and the
SSS Congruence Postulate to get DAB CBA. Repeat the above steps to get
ADC CBA and DAB  BCD. Then use CPCTC and the transitive property
to get DAB CBA BCD ADC. Finish with the Four Congruent Angles
Rectangle Theorem.
Flowchart Proof
DB  AC
Given

DAB  CBA DAB  CBA


SSS Congruence CPCTC
Postulate
AB  BA
Identity
property

DAB  CBA 
AC  CA ADC  CBA ADC  CBA
ADC  BCD
Identity SSS Congruence CPCTC
Transitive property
property Postulate

ABCD is a AD  BC ABCD is a
parallelogram DC  AB rectangle
Given Opposite Sides Four Congruent
Theorem Angles Rectangle
Theorem
DB  BD DAB  BCD DAB  BCD
Identity SSS Congruence CPCTC
property Postulate

9. Given: Isosceles trapezoid D C

Show: The base angles are congruent


Given: Isosceles trapezoid ABCD with DC   AB
, AD
  BC

A E B
Show: A  B
  CB
Plan: Use the Parallel Postulate to construct DE . Then use the Opposite Sides
Theorem and the transitive property to prove that AED is isosceles. Therefore
A  B by the Isosceles Triangle Theorem and the CA Postulate.

Discovering Geometry Solutions Manual CHAPTER 13 267


©2003 Key Curriculum Press
DG3SM586_252-296.qxd 8/2/06 6:15 PM Page 268

Proof: Statement Reason


  AB
1. DC  1. Given
  CB
2. Construct DE  2. Parallel Postulate
3. DCBE is a parallelogram 3. Definition of parallelogram
 BC
4. DE  4. Opposite Sides Theorem
 
5. AD BC 5. Given
 DE
6. AD  6. Transitive property of congruence
7. ADE is isosceles 7. Definition of isosceles triangle
8. A DEA 8. Isosceles Triangle Theorem
9. DEA B 9. CA Postulate
10. A  B 10. Transitive property of congruence
10. Given: Isosceles trapezoid D C

Show: The diagonals are congruent


  AB
Given: Isosceles trapezoid ABCD with DC , AD
 BC

A B
 BD
Show: AC 
Plan: Use the Isosceles Trapezoid Theorem, the identity property, and the SAS
Congruence Postulate to get DAB CBA. Then AC  BD  by CPCTC.
Flowchart Proof
Isosceles trapezoid
DAB  CBA
ABCD with DC  AB
Isosceles Trapezoid
Theorem
AD  BC DAB  CBA BD  AC
Given SAS Congruence CPCTC
Postulate
AB  BA
Identity property

11. Given: Parallelogram with a diagonal that bisects two opposite D


4
C
3
angles
2
Show: The parallelogram is a rhombus 1
A B
Given: Parallelogram ABCD; AC bisects DAB and BCD
Show: ABCD is a rhombus
Plan: Use the Opposite Angles Theorem, the multiplication property, and the
definition of angle bisector to get 1  3. Then use the Converse of the
Isosceles Triangle Theorem, the definition of isosceles triangle, and the Opposite
Sides Theorem to get AB BC  DC  AD
.
Proof: Statement Reason

1. AC bisects DAB and BCD 1. Given
2. 1 2; 3 4 2. Definition of angle bisector
3. m1  m2; m3  m4 3. Definition of congruence
4. m1  m2  mDAB; 4. Angle Addition Postulate
m3  m4  mBCD
5. 2m1  mDAB; 2m3  mBCD 5. Substitution property
1 1
6. m1  2(mDAB); m3  2(mBCD) 6. Multiplication property
7. ABCD is a parallelogram 7. Given
8. DAB  BCD 8. Opposite Angles Theorem

(Proof continued)

268 CHAPTER 13 Discovering Geometry Solutions Manual


©2003 Key Curriculum Press
DG3SM586_252-296.qxd 8/2/06 6:15 PM Page 269

(Exercise 11 Proof continued)

Statement Reason
9. mDAB  mBCD 9. Definition of congruence
1 1
10. 2(mDAB)  2(mBCD) 10. Multiplication property
11. m1  m3 11. Substitution property
  BC
12. AB  12. Converse of the Isosceles Triangle Theorem
   BC
13. AB DC ; AD  13. Opposite Sides Theorem
 BC
14. AB  DC
 AD 14. Transitive property of congruence
15. ABCD is a rhombus 15. Definition of rhombus
12. Given: A pair of parallel lines intersected by a second W Z
pair of parallel lines; the distance between each pair of
lines is the same
Q
Show: The parallelogram formed is a rhombus
Given: WZ   XY , XW
  YZ
, distance between WZ
 and X P Y

 equals distance between XW


XY  and YZ 
Show: XWZY is a rhombus
Plan: Use the Converse of the Angle Bisector Theorem to prove that WY  is the angle
bisector of Y and of W. Therefore WXYZ is a rhombus by the Converse of the
Rhombus Angles Theorem.
Proof: Statement Reason
  XY
1. WZ , XW
  YZ 1. Given
2. WXYZ is a parallelogram 2. Definition of parallelogram
3. Distance from W to XY   3. Given
distance from W to YZ 
 is the angle bisector of Y
4. WY 4. Converse of the Angle Bisector Theorem
5. Distance from Y to XW   5. Given
distance from Y to WZ 
 is the angle bisector of W
6. WY 6. Converse of the Angle Bisector Theorem
7. XWZY is a rhombus 7. Converse of the Rhombus Angles Theorem
13. Linear Pair Postulate CA Postulate

Interior Supplements Theorem

Consecutive Angles Theorem

14. ASA Congruence Line Postulate Angle Addition


Postulate Postulate

Parallelogram Converse of the


Diagonal Lemma IT Theorem SSS Congruence
Postulate

Opposite Sides Opposite Angles


Theorem Theorem IT Theorem

Converse of the Rhombus Converse of the Angle


Angles Theorem Bisector Theorem

Double-Edged
Straightedge Theorem

Discovering Geometry Solutions Manual CHAPTER 13 269


©2003 Key Curriculum Press
DG3SM586_252-296.qxd 8/2/06 6:15 PM Page 270

Lesson 13.4 (continued) Because the center of dilation is (8, 2) rather than
(0, 0) and the scale factor is 2, the mapping rule

15. V 1 V 2 has length 12.8 and bearing 72.6°. Label the is (x, y) → (8  2(x  8), 2  2(y  2)) 
vertices in the figure, and use the given bearings
(2x  8, 2y  2). You can verify that this rule
and vector lengths to find angle measures and side
gives the same results as the graph for the image
lengths in parallelogram ABCD and ACD.
of each point of ABC.
N B C
17. 2386 ft2. Divide the

V1 A
grazing area into five 8 E8 60°
40° 50° V1  V2 D 25 15
sections and find the 10
12
10

A 
V2 D area of each. 18 12
B
Notice that each C
Because the bearing of V 1 is 40° and the bearing section is a sector of
30

2 is 90°, mBAD  90°  40°  50°. Because


of V a circle, but the radii
consecutive angles of a parallelogram are supple- of the circles differ
mentary, mADC  180°  50°  130°. Because depending on the positions of the sectors in the
opposite sides of a parallelogram are congruent, diagram.
CD  5. Use the Law of Cosines to find AC, the
2. Sector A: r  15 ft; central angle  60°, so sector is
length of the resultant vector, V 1 V 1
 of circle
6
(AC)2  (AD)2  (CD)2  2(AD)(CD)(cos 130°) 1 1 1
Asector A  6r 2  6(15)2  6  225  37.5 ft2
(AC)2  92  52  2(9)(5)(cos 130°)
Sector B: r  40 ft; central angle  90°, so sector is
AC  
92  52
 2(9)(5)(c
os  
12.8
130°) 1
 of circle
4
1 1 1
(Keep all digits of the value of AC in your calculator Asector B  4r 2  4(40)2  4  1600  400 ft2
for use in the next calculation.)
Sector C: r  30 ft; central angle  90°, so sector is
Let   mCAD. Use the Law of Sines to find . 1
 of circle
4
sin  sin 130° 1 1 1
  
5 AC Asector C  4r 2  4(30)2  4  900  225 ft2
5 sin 130°
sin    Sector D: r  18 ft; central angle  90°, so sector is
AC 1
 of circle
4
5 sin 130°
  sin1  AC 
17.4°  1 1 1
Asector D  4r 2  4(18)2  4  324  81 ft2

The bearing of the resultant vector, V 1 V 2, is Sector E: r  8 ft; central angle  90°, so sector is
1
90°  17.4°  72.6°.  of circle
4
1 1 1
16. A(6, 10), B(2, 6), C(0, 0); mapping rule: Asector E  4r 2  4(8)2  4  64  16 ft2
(x, y) → (2x  8, 2y  2). Draw ABC on graph
To find the total grazing area, add the areas of the
paper and locate the center of the dilation, (8, 2).
five sectors:
Label (8, 2) as P. To find the vertices of the image
triangle, ABC, draw PA, PB, and PC . Extend 37.5 + 400 + 225 + 81 + 16  759.5 
each of these segments to locate A, B, and C 2386 ft2
such that AA  PA, BB  PB, and CC  PC.
18.
y
P (8, 2) Lines of Rotational
C (4, 1) Name symmetry symmetry
C (0, 0)
x
parallelogram none 2-fold
trapezoid none none
B (3, –2) A (7, –4) kite 1 diagonal none
square 2 diagonals 4-fold
B (–2, –6) 2  bisectors of sides
rectangle 2  bisectors of sides 2-fold
A (6, –10) rhombus 2 diagonals 2-fold

Use the figure to find the coordinates of the image isosceles trapezoid 1  bisector of sides none
triangle: A(6, 10), B(2, 6), and C(0, 0).

270 CHAPTER 13 Discovering Geometry Solutions Manual


©2003 Key Curriculum Press
DG3SM586_252-296.qxd 8/2/06 6:15 PM Page 271

19. a. B. Look at XYZ in both figures and find the find the corresponding distances for Figure B
unknown side lengths in each of these triangles. without drawing.
In both figures, let   mXYZ. 52  (
6  6)2  
169  13

Figure A: Apply the Pythagorean Theorem to
62  (
 5  6)2  
157
12.5
obtain XZ   42  52  41 . XYZ is a right
triangle, with right angle Z. Use the tangent ratio 62  (
 6  5)2  
157
12.5
to find . 
12.5, for
The shortest distance is 157
XZ  41 Figure B.
tan     
YZ  9
20. a. 19°. EOD is a central angle of circle O, so

41 mED  38°. EAD is an inscribed angle that
 tan1 
9
35.4° , so mA  1(38°)  19°. OA
also intercepts ED 
2
Figure B: Follow the steps applied to Figure A. 
and OE are both radii of the circle, so AOE is
XZ   62  52  61
 an isosceles triangle with OA  OE. Therefore, by
the Isosceles Triangle Theorem, mAEO 
XZ
tan       61
YZ  6 mA  19°.
61 b. 52°. Look at DGO. Because GF is tangent
  tan1  6
52.5°  by the Tangent
  GF
to circle O at D, OD
Thus mXYZ is greater in Figure B. Conjecture. Thus mODG  90°. GOD and
EOD are the same angle, so mGOD  38°.
b. B. Remember that the shortest distance between
Therefore mDGO  180°  90°  38°  52°.
two points is a straight line. To find the straight
line from X to Y on the surface of Figure A, draw   EC
c. 52°. Because GF , HEO  DGO by the
a net of the prism. The path has to cross two CA Postulate. OEC and OEH are the same
faces, and there are six ways to do so, so there are angle, so mOEC  52°. OEC is isosceles
six paths to consider. But each path is the mirror  and OC
because OE  are both radii of the circle.
image of the path opposite it, so there are only Therefore OCE  OEC by the Isosceles
three different paths. Triangle Theorem, so mOCE  52°. Finally,
X
  EC
because OB , BOC  OCE by AIA, so
mBOC  52°.
4
  360°  mED
d. 232°. mEAB   mDC  mCB . In
5 Z 9 Y 
20a, you found that mED  38°. Look at the
, DC, and CB
. Because GH 
42  (
 5  9)2  
212
14.6 central angles for ED
X
is the altitude to the base of isosceles triangle
EOC, it is also the angle bisector, so mCOH 
4
38°. Then, because COH and COD are the
5
Z
same angle and COD intercepts DC , mDC 

38°. From 20c, mBOC  52°, so mCB  52°.
9 Therefore mEAB  360°  38°  38°  52° 
232°.
Y
e. 19°. Notice that HED is the same angle as
(4  
   
194
13.9 CED, which is an inscribed angle that
 9)2 52 . From 20d, mDC  38°, so
intercepts DC
1 
X
mHED  mCED  2mDC  12(38°)  19°.
5
IMPROVING YOUR VISUAL THINKING SKILLS
K

4
G

F
J

Z 9 Y
B

(5  4
 )2  92  
162
12.7
Notice that the expressions for Figure A are the EXTENSION
three possible ways to group the three dimen-
Answers will vary.
sions into a pair and a “singleton,” so you can

Discovering Geometry Solutions Manual CHAPTER 13 271


©2003 Key Curriculum Press
DG3SM586_252-296.qxd 8/2/06 6:15 PM Page 272

EXPLORATION • PROOF AS CHALLENGE AND DISCOVERY creates a contradiction because ZOID is a trapezoid,
and a trapezoid cannot be a parallelogram. Thus
IMPROVING YOUR ALGEBRA SKILLS the assumption that ZO  ID is false, and the
There is a flaw in the proof. If h  n  p, then h  n  conjecture is true.
p  0. The step from h(h  n  p)  n(h  n  p) to
9. Given: A chord of a circle
h  n is not valid, because when you divide both sides
by h  n  p, you are dividing by zero. Show: The perpendicular bisector of the chord
passes through the center of the circle
LESSON 13.5 B

C D
EXERCISES
O E
1. D. Paris is in France; Tucson is in the United States;
London is in England. Therefore Bamako must be A
the capital of Mali.
Given: Circle O with chord AB  and perpendicular
2. C. The “Sir” in 2A shows that Halley is English; 
bisector CD
Julius Caesar was an emperor, not a scientist;
Madonna is a singer. Therefore Galileo Galilei must 
Show: CD passes through O
be the answer. Paragraph Proof: It is given that CD  is the
perpendicular bisector of AB . Then, by the defini-
3. No, the proof is claiming only that if two particular
tion of perpendicular bisector, D is the midpoint of
angles are not congruent, then the two particular . Assume CD  does not pass through O. Use the
AB
sides opposite them are not congruent. A different  and OA  and the
Line Postulate to construct OB
pair of sides might still be congruent. , where OE
Perpendicular Postulate to construct OE
4. Yes, this statement is the contrapositive of is the perpendicular segment from O to AB .
Example B and they are logically equivalent. Because OE  AB , OEB and OEA are right
angles, so OEB  OEA by the Right Angles Are
5. 1. Assume the opposite of the conclusion;  OA because they are
Congruent Theorem. OB
2. Triangle Sum Theorem; 3. Substitution property;
radii of the same circle, so B  A by the
4. mC  0, Subtraction property  OE by the
Isosceles Triangle Theorem. OE
6. Paragraph Proof: Assume ZOID is equiangular. identity property of congruence. Therefore
Then, by the definition of equiangular, Z  OEB OEA by the SAA Congruence Theorem,
O  I  D. Then, by the Four Congruent and EB EA by CPCTC. Then E is the midpoint
Angles Rectangle Theorem, ZOID is a rectangle. , and OE
of AB  is the perpendicular bisector of AB
.
Therefore ZOID is a parallelogram, which creates a This contradicts the assumption that the perpen-
contradiction because, by definition, a trapezoid has dicular bisector of AB does not pass through O.
exactly one pair of parallel sides and thus cannot be (A segment can have only one midpoint and one
a parallelogram. Therefore the assumption that ZOID perpendicular bisector.) Thus the assumption is
is equiangular is false and the conjecture is true. false and the conjecture is true.
7. Paragraph Proof: Assume CD  is the altitude to 10. a  75°, b  47°, c  58°. By the Interior Supple-
. Then, by the definition of altitude, CD
AB   AB , ments Theorem, a  180°  105°  75°. b  a 
and by the definition of perpendicular, CDA and 58°  180°, or b  75°  58°  180°, so b  180° 
CDB are right angles. Thus CDA  CDB by 75°  58°  47°. By the CA Postulate, c  58°.
the Right Angles Are Congruent Theorem. CD 
 by the identity property of congruence. It is 11. 42 ft3. The volume of the container is the sum of the
CD
 is a median, so D is the midpoint of volumes of a cylinder with radius 3 ft and height 3 ft
given that CD
, and AD  BD by the definition of midpoint. and a cone with radius 3 ft and height 5 ft.
AB
Then ADC  BDC by the SAS Congruence Vcylinder = r 2H  (3)2(3)  27 ft3
 BC
Postulate. Therefore AC  by CPCTC, so 1 1
Vcone = 3r 2H  3(3)2(5)  15 ft3
ABC is isosceles. It is given that ABC is scalene.
Therefore the assumption that CD  is the altitude to Vcontainer = 27 + 15 = 42 ft3
 is false and the conjecture is true.
AB 
 12 . The shaded region is the difference
3
12. a. 
4
8. Paragraph Proof: Assume ZO  ID. Then ZO  between the shaded area in Lesson 13.1, Exer-
 by the definition of congruence. ZO
ID   ID
 is cise 31a and the shaded area in Lesson 13.3,
given. Therefore, by the Opposite Sides Parallel and Exercise 18c, so subtract the areas you found in
Congruent Theorem, ZOID is a parallelogram. This those exercises:

272 CHAPTER 13 Discovering Geometry Solutions Manual


©2003 Key Curriculum Press
DG3SM586_252-296.qxd 8/2/06 6:15 PM Page 273

Because the two radii are congruent and the


 
 3
A  4  3   
 
  
3
4  4  3  4 two tangent segments are congruent (Tangent
Segments Conjecture), the quadrilateral is a
3 
 
4  12
kite. By the Tangent Conjecture, the two
nonvertex angles are right angles. Therefore the
b. 1  3  3. To find the area of the shaded two vertex angles must be supplementary. These
region, subtract four times the shaded area from two angles are the angle formed by the tangents
12a from the area of the square. and the central angle of the minor intercepted arc.
A14  
3 
 
4  12  1  

3  3 IMPROVING YOUR REASONING SKILLS
13. a. Always. This is the Angles Inscribed in a Semi-
1

1 1
 
1 1

V  3BH  3 2h(a  b) H  3 2(2x)(2b  b) H
circle Conjecture.
1 12
 3(x(3b))H  bxH  x 2H  x 2  x  12x
b. Never. An angle inscribed in a major arc inter-
cepts a minor arc, where the sum of the major EXTENSIONS
arc and the minor arc is the complete circle. By A. Conjecture: The largest angle of a triangle must
the Inscribed Angle Conjecture, the measure of have a measure of at least 60°.
an angle that intercepts a minor arc will be half
Given: Triangle ABC with A the largest angle
of an arc measure that is greater than 0° and less
than 180°, so the angle measure will be greater Show: mA 60°
than 0° and less than 90°. Thus the angle will Paragraph Proof: Assume mA
60°. Because
always be acute. A is the largest angle of the triangle, mB

mA and mC


mA. Therefore mB
60°
c. Sometimes. Because your book defines angles to
and mC
60°. Then mA  mB  mC

have measures that are greater than 0° and less


60°  60°  60°  180°, or mA  mB 
than 180°, this statement is true only for minor
mC
180°. This creates a contradiction because,
arcs. If the arc is a semicircle, its measure is 180°.
by the Triangle Sum Theorem, mA  mB 
The measure of a major arc is found by
mC  180°. Thus the assumption that mA

subtracting the measure of the corresponding


60° is false, and the conjecture is true. (If the
minor arc from 360°.
triangle is isosceles or equilateral so that mB 
d. Sometimes. The measure of an angle formed by mA and/or mC  mA, the proof is still valid.)
two intersecting chords equals the average of the B. Conjecture: The largest side of a triangle must be
measures of the two intercepted arcs, so this opposite an angle with a measure greater than or
statement is true only when the two intercepted equal to 60°.
arcs are congruent. That happens when the  the longest side
Given: ABC with BC
chords are diameters.
Show: mA  60°
e. Always. Make a sketch. , and
Paragraph Proof: Because A is opposite BC
Draw radii to the two  is the longest side of ABC, A is the largest
BC
points of tangency.
angle of the triangle by the Side-Angle Inequality
Conjecture. By the theorem that was proved in
Extension A, mA  60°.

LESSON 13.6
EXERCISES
1. Given: Two inscribed angles in a circle that intercept the
A
same or congruent arcs X
Show: The inscribed angles are congruent
Case 1: Two angles intercepting the same arc
Given: Circle with inscribed angles XAW and XBW B

intercepting XW W

Show: XAW  XBW


Paragraph Proof: By the Inscribed Angle Theorem, mXAW  12(mXW) and
1 
mXBW  2(mXW ). Therefore, by the transitive property, mXAW  mXBW, so
XAW  XBW by the definition of congruence.

Discovering Geometry Solutions Manual CHAPTER 13 273


©2003 Key Curriculum Press
DG3SM586_252-296.qxd 8/2/06 6:15 PM Page 274

Case 2: Two angles intercepting congruent arcs A


Z

Given: Circle with inscribed angle ZAY intercepting ZY and
inscribed angle XBW intercepting XW; ZY
  XW 
Y
Show: ZAY  XBW
B X
Paragraph Proof: By the Inscribed Angle Theorem,
mZAY  12(mZY ) and mXBW  1(mXW ). Because W
2
  1  1 
ZY  XW is given, 2(mZY )  2(mXW ) by the definition of
congruence and the multiplication property. Therefore mZAY  mXBW by the
transitive property, so ZAY XBW by the definition of congruence.
2. Given: Quadrilateral inscribed in a circle D
Show: The opposite angles of the quadrilateral are C
supplementary
Given: Circle with inscribed quadrilateral ABCD
Show: A and C are supplementary; B and D are B
supplementary A

Plan: Use the Inscribed Angle Theorem, the addition property, and
the distributive property to get mA  mC  12(mBCD   mDAB ). Then use the
definition of degrees in a circle, the substitution property, and the definition of
supplementary to get A and C are supplementary. Follow the above steps to get
B and D are supplementary.
Proof: Statement Reason
1. ABCD is inscribed in the circle 1. Given
1  1 
2. mA  2(mBCD ); mC  2(mDAB ) 2. Inscribed Angle Theorem
1  1 
mB  2(mACD ); mD  2(mABC )
1  1 
3. mA  mC  2(mBCD )  2(mDAB ) 3. Addition property
1  1 
mB  mD  2(mACD )  2(mABC )
1  )
4. mA  mC  2(mBCD  mDAB 4. Distributive property
1  )
mB  mD  2(mACD  mABC
1
5. mA  mC  2(arc measure of circle) 5. Arc Addition Postulate
1
mB  mD  2(arc measure of circle)
1
6. mA  mC  2(360°) 6. Arc measure of full circle  360°
1
mB  mD  2(360°)
7. mA  mC  180° 7. Multiplication property
mB  mD  180°

8. A and C are supplementary 8. Definition of supplementary


B and D are supplementary
3. Given: Circle with parallel secants B
Show: The secants intercept congruent arcs
 and CD
Given: Circle with secants AB ; AB
  CD
 A
D

Show: AC BD 
. Then use
Plan: Use the Line Postulate to construct AD C

the AIA Theorem and the Inscribed Angle Theorem to


 BD
get AC .

274 CHAPTER 13 Discovering Geometry Solutions Manual


©2003 Key Curriculum Press
DG3SM586_252-296.qxd 8/2/06 6:15 PM Page 275

Proof: Statement Reason



1. Construct AD 1. Line Postulate
2. CDA BAD 2. AIA Theorem
3. mCDA  mBAD 3. Definition of congruence
1 
4. mCDA  2(mAC ); 4. Inscribed Angle Theorem
1 
mBAD  2(mBD )
1  1 
5. 2(mAC )  2(mBD ) 5. Transitive property

6. mAC  mBD  6. Multiplication property
 BD
7. AC  7. Definition of congruence
4. Given: Parallelogram inscribed in a circle
A C
Show: The parallelogram is a rectangle
Given: Parallelogram ACER inscribed in a circle
Show: ACER is a rectangle
Plan: Use the Cyclic Quadrilateral Theorem, the Opposite R E

Angles Theorem, and the Congruent and Supplementary


Theorem to show that all four angles are right angles.
Proof: Statement Reason
1. ACER is inscribed in a circle 1. Given
2. A and E are supplementary; 2. Cyclic Quadrilateral Theorem
C and R are supplementary
3. ACER is a parallelogram 3. Given
4. A E; C R 4. Opposite Angles Theorem
5. A and E are right angles; 5. Congruent and Supplementary Theorem
C and R are right angles
6. ACER is a rectangle 6. Definition of rectangle
5. Given: A circle and a point outside the circle; two segments P
tangent to the circle with the point as one endpoint and 1
S 2
the point of tangency as the other endpoint
Show: The segments are congruent
Given: Circle O with point P outside the circle; PS 
 tangent to the circle at S and T, respectively
and PT O
T

Show: PS  PT 
Plan: Use the Line Postulate to construct OS , OT
, and OP
.
Then use the Tangent Theorem, the Converse of the Angle
Bisector Theorem, and the SAA Congruence Postulate to get
OSP OTP. Use CPCTC to show PS  PT .
Proof: Statement Reason
1. Construct OS, OT, and OP 1. Line Postulate

2. OS OT 2. Definition of circle
  
3. OS  PS ; OT  PT  3. Tangent Theorem
4. OSP is a right angle; 4. Definition of perpendicular
OTP is a right angle
5. OSP OTP 5. Right Angles Are Congruent Theorem
6. O is on the angle bisector of SPT 6. Converse of the Angle Bisector Theorem
7. 1 2 7. Definition of angle bisector
8. OSP OTP 8. SAA Theorem
 PT
9. PS  9. CPCTC

Discovering Geometry Solutions Manual CHAPTER 13 275


©2003 Key Curriculum Press
DG3SM586_252-296.qxd 8/2/06 6:15 PM Page 276

6. Given: Circle with two intersecting chords A


Show: The measure of the angle formed by the chords is D 3
2
1
half the sum of the measures of the two intercepted arcs E C
 and CD
Given: Circle with chords AB  intersecting at E
1  
Show: m1  2(mAC  mBD )
B
Paragraph Proof: Use the Line Postulate to construct AD,
forming 2 and 3. By the Inscribed Angle Theorem,
) and m3  1(mBD
m2  12(mAC ). Then, by the addition and distributive
2
 )  1(mAC
properties, m2  m3  2(mAC )  12(mBD
1   mBD
). By the
2
Triangle Exterior Angle Theorem, m1  m2  m3, so, by the transitive
  mBD
property, m1  12(mAC ).

7. Intersecting Secants Theorem: The measure of an angle formed by two P


secants intersecting outside a circle is half the difference between the x
A
measure of the larger intercepted arc and the measure of the smaller
b
intercepted arc. C
Given: Circle O with point P outside the circle; secant PB  intersects O
B

the circle at A and B; secant PD intersects the circle at C and D a
1  )
Show: x  2(mBD  mAC
Paragraph Proof: Using the Line Postulate, construct AD . By the Triangle D

Exterior Angle Theorem, b  a  x, so, by the subtraction property, x  b  a.


By the Inscribed Angle Theorem, b  12(mBD ) and a  1(mAC ). Then, by the
2
    mAC
substitution and distributive properties, x  2(mBD )  2(mAC )  12(mBD
1 1 ).

Lesson 13.6 (continued)


10. A(4.0, 2.9), P(1.1, 2.8). To find the coordinates
8. Angle Addition of A, draw the perpendicular segment from A to the
Postulate
x-axis, forming a right triangle with horizontal leg
Line ASA Congruence SSS Congruence of length a and vertical leg of length b.
Postulate Postulate Postulate
A

Converse of the SAA IT


IT Theorem Theorem Theorem
SAS Congruence 5 b
Postulate
Right Angles Are
Congruent Theorem
Midpoint 36°
Converse of the Angle Postulate a
Bisector Theorem Tangent
Theorem a
Perpendicular
Postulate 
5  cos 36°, so a  5 cos 36°  4.0.
Tangent Segments
b
Theorem

5  sin 36°, so b  5 sin 36°  2.9.
9. Linear Pair CA Parallel Angle Addition Arc Addition To find the coordinates of P, draw a
Postulate Postulate Postulate Postulate Postulate
perpendicular segment from P to the
SSS y-axis, forming a right triangle with 3 q
Congruence
Postulate
VA
Theorem
horizontal leg of length p and vertical
leg of length q. (Absolute-value bars P p

IT
AIA are used here because the side lengths of
Theorem
Theorem the triangle are positive, but p and q, the coordinates
Triangle Sum of P, are negative because P is in Quadrant III.)
Theorem
ASA Congruence p
Postulate   sin 22°, so p  3 sin 22°, p  1.1, and
Exterior Angle 3
Sum Theorem p  1.1.
Parallelogram Congruent and
q
Inscribed Angle Diagonal Lemma Supplementary Theorem   cos 22°, so q  3 cos 22°, q  2.8, and
Theorem 3
Opposite Angles Right Angles q  2.8.
Cyclic Quadrilateral Theorem Are Congruent
Theorem Theorem

Parallelogram Inscribed
in a Circle Theorem

276 CHAPTER 13 Discovering Geometry Solutions Manual


©2003 Key Curriculum Press
DG3SM586_252-296.qxd 8/2/06 6:15 PM Page 277

11. BY  3, YC  1, AZ  9. First look at ABC with


  BC
XZ . By the Parallel/Proportionality Conjecture,
AX AZ 6 AZ
  . Then   , so 2(AZ)  18 and AZ  9. x
XB ZC 2 3 h
Now look at ABC again, this time with ZY   AB
. a
c

Apply the Parallel/Proportionality Conjecture again P


b
YC ZC
to get the proportion  BY  AZ . BC  4, BY 

YC 3
4  YC, so 4  YC  9 . Then 9(YC)  3(4  YC),
  15. a. 27°. P is formed by two secants intersecting
or 9(YC)  12  3(YC), so 12(YC)  12, and outside the circle, so apply the Intersecting
Secants Theorem: mP  12(mNQ   mOM ) 
YC  1. Therefore BY  4  YC  3. 1 1
(94°  40°)  (54°)  27°.
, AB
12. BC , AC, CD
, AD. First use the Triangle Sum 2 2

Theorem to find the angle measures that are not b. 47°. QON is an inscribed angle that intercepts
, so, by the Inscribed Angle Theorem,
NQ
given. In ABC, mABC  180°  60°  59° 
61°. In ADC, mACD  180°  57°  61°  62°. mQON  12(94°)  47°.
Now apply the Side-Angle Inequality Conjecture to c. 67°. QRN is formed by two intersecting chords,
each triangle to find the relative lengths of the sides.  and ON
QM , so, by the Intersecting Chords
Theorem, mQRN  12(mNQ   mOM ) 
In ABC, BC  is opposite the smallest angle, AB
 is
1 1

opposite the middle-size angle, and AC is opposite (94°  40°)  (134°)  67°.
2 2
the largest angle, so BC  AB  AC. d. 133°. Look at QMP. From 15a, mP  27°.
In ADC, AC  is opposite the smallest angle, CD
 is PQM is the same angle as OQM, which is an
, so, by the
 is opposite
opposite the middle-size angle, and AD inscribed angle that intercepts MO
the largest angle, so AC  CD  AD. Inscribed Angle Theorem, the measure of this
angle is 12(40°)  20°. Therefore, by the Triangle
 is common to both triangles and is the
Because CD Sum Theorem, mQMP  180°  27°  20° 
longest side of one triangle but the shortest side of 133°. Another way to find mQMP is to notice
the other, you can arrange all five segments in order that QMN is an inscribed angle that intercepts
, AB
from shortest to longest: BC , AC, CD
, AD
. , so mQMN  47°. Then, because QMN
NQ
13. 32.5. Let P and Q be the centers of the smaller and and QMP form a linear pair, mQMP 
 and QB
larger circles, respectively. Draw radii PA . 180°  47°  133°.
Also draw a segment AR  so that AR
  PQ and R is e. Cannot be determined. mONF depends on
.
on QB , or mMO
mOMN   mMN . You know that
B
 
mOQ  mMN  360°  94°  40°  226°,
10 but there is not enough information to
A
R
determine mOQ  and mMN individually.
34 10
10 34
10 P Q f. Cannot be determined. As in 15e, there is not
20
enough information to determine mMN.

, then bisect it. Label the


16. First construct OP
  AB and QB  AB
, midpoint M. Then construct a circle with center M
By the Tangent Theorem, PA
  QB
. AR  PQ
 by construction. Therefore and radius PM. Label the intersection of the two
so PA  and PB
, which
circles A and B. Finally, construct PA
PARQ is a parallelogram. Because opposite sides of
are the required tangents.
a rectangle are congruent, QR  PA, so QR  10,
and AR  PQ, so AR  34. Apply the Pythagorean
Theorem to right triangle ABR to find AB: (AB)2 
(BR)2  (AR)2, so AB  (AR) 2  (BR)2 
 A
2  102 

 
34 1056 32.5.
14. As long as P is inside the triangle, a  b  c  h.
O M P
Proof: Let x be the length of a side. The areas of
the three small triangles are 12xa, 12xb, and 12xc.
The area of the large triangle is 12xh. So 12xa  B
1 1 1 1
xb  xc  xh. Divide by x on both sides to
2 2 2 2
get a  b  c  h.

Discovering Geometry Solutions Manual CHAPTER 13 277


©2003 Key Curriculum Press
DG3SM586_252-296.qxd 8/2/06 6:15 PM Page 278

IMPROVING YOUR REASONING SKILLS based on this starting pattern does not include an
The color of a spot is determined by the colors of the all-green row. However, some other starting pattern
two spots above it. If they are the same, the spot is can lead to an all-green row. In fact, starting with an
yellow; otherwise, the spot is green. (The last spot in a all-green row gives an eight-row cycle, which after the
row is always green because there can’t be two same- ninth row is all green and any row in the cycle would
colored spots above it.) A row could never be all yellow lead to all green. The figure below shows rows 7 through
because the last spot is always green. Because there are 11. The shaded spots represent yellow.
only 25  32 possible ways to color a row (two choices
for each of the first five spots), some row must be
repeated after at most 32 rows and then the whole cycle
will repeat. For example, the first row is repeated as the
ninth row, so the cycle is eight rows long. The cycle

LESSON 13.7
EXERCISES
1. Given: Two similar triangles with V
T
corresponding altitudes
Show: The altitudes are proportional to
the corresponding sides B H A G E L
Given: BTA GVL; TH  is the
; VE
altitude to BA  is the altitude to GL

BT TH
Show:  GV  VE

Plan: Use the Right Angles Are Congruent Theorem, CASTC, and the AA Similarity
BT TH
Postulate to get BHT GEV. Therefore  GV  VE by CSSTP.
 
Proof: Statement Reason
1. BTA GVL 1. Given
2. B G 2. CASTC
 is the altitude to BA
3. TH ; 3. Given
 is the altitude to GL
VE 
  BA
4. TH ; VE  GL  4. Definition of altitude
5. THB is a right angle; 5. Definition of perpendicular
VEG is a right angle
6. THB VEG 6. Right Angles Are Congruent Theorem
7. BHT GEV 7. AA Similarity Postulate
BT TH
8.  
GV  VE 8. CSSTP

AA Similarity Postulate

Right Angles Are


Congruent Theorem

Corresponding Altitudes
Theorem

2. Given: Two similar triangles with G


corresponding medians A

Show: The medians are proportional to the


corresponding sides B Y I
S L M
Given: BGI SAM; GY  is the median to
; AL
BI  is the median to SM

BG GY
Show:   
SA  AL

278 CHAPTER 13 Discovering Geometry Solutions Manual


©2003 Key Curriculum Press
DG3SM586_252-296.qxd 8/2/06 6:15 PM Page 279

Paragraph Proof: By the definitions of median and midpoint, BY  YI and


SL  SM. Then, by the Segment Addition Postulate and the substitution property,
BI  2BY and SM  2SL, or BY  12BI and SL  12SM. BGI SAM is given,
so, by CSSTP,
2BI 1
BG
  B
I
 
1  B
Y
SA SM 2SM SL
Also, B S by CASTC. Therefore BYG SLA by the SAS Similarity
BG GY
SA  AL by CSSTP.
Theorem. Therefore  

Segment Segment SAS AA


Parallel CA
Addition Duplication Congruence Similarity
Postulate Postulate
Postulate Postulate Postulate Postulate

SAS Similarity
Theorem

Corresponding
SAS Similarity Medians
Theorem

3. Given: Two similar triangles with corresponding C F


angle bisectors 13 2 4

Show: The angle bisectors are proportional to the


corresponding sides D Q E

Given: ACB DFE; CP  is the angle bisector A P B



of ACB; FQ is the angle bisector of DFE
AC CP
Show:   
DF  FQ
Plan: Use the definition of angle bisector, the Angle Addition Postulate, and the substi-
tution property to get mACB  2m1 and mDFE  2m2. Then use CASTC and
AC CP
the AA Similarity Postulate to get APC DQF. Therefore  DF  FQ by CSSTP.
 
Proof: Statement Reason

1. CP is the angle bisector of ACB; 1. Given
 is the angle bisector of DFE
FQ
2. 1 3; 2 4 2. Definition of angle bisector
3. m1  m3; m2  m4 3. Definition of congruence
4. mACB  m1  m3; 4. Angle Addition Postulate
mDFB  m2  m4
5. mACB  2m1; mDFB  2m2 5. Substitution property
6. ACB DFE 6. Given
7. A D 7. CASTC
8. ACB DFE 8. CASTC
9. mACB  mDFE 9. Definition of congruence
10. 2m1  2m2 10. Transitive property
11. m1  m2 11. Division property
12. 1 2 12. Definition of congruence
13. APC DQF 13. AA Similarity Postulate
AC CP
14.   
DF  FQ 14. CSSTP

Discovering Geometry Solutions Manual CHAPTER 13 279


©2003 Key Curriculum Press
DG3SM586_252-296.qxd 8/2/06 6:15 PM Page 280

4. Given: Triangle cut by a line passing through two sides C


and parallel to the third side D 1 E
Show: The line divides the two sides proportionally
Given: ACB with DE   AB

2
DA EB A B
Show:  CD
  
CE
Plan: Use the CA Postulate and the AA Similarity Postulate to get CDE CAB.
CD  DA CE  EB
Then use CSSTP and the Segment Addition Postulate to get    .
CD CE
DA EB
Therefore CD  CE by algebra and the subtraction property.
 
Proof: Statement Reason
 
1. DE  AB 1. Given
2. 1 2 2. CA Postulate
3. DCE ACB 3. Identity property
4. CDE CAB 4. AA Similarity Postulate
CA CB
5.   
CD  CE 5. CSSTP
6. CA  CD  DA; CB  CE  EB 6. Segment Addition Postulate
CD  DA CE  EB
7.  CD
  
CE 7. Substitution property
CD DA CE EB
8.      
CD  CD  CE  CE 8. Algebra
DA EB
9. 1   
CD  1  CE
 9. Algebra
DA EB
10.   
CD  CE 10. Subtraction property
5. Given: Triangle cut by a line passing through two sides, C
dividing them proportionally D 1 E
Show: The line is parallel to the third side
Given: ACB with DE  intersecting CA
 and CB
 such that
2
DA EB A B
  
CD CE
Show: DE  AB

DA EB
CD  1 CE  1. Then use algebra and
Plan: Use the addition property to get   
CA CB
CD  CE . Therefore ABC DEC by the
the Segment Addition Postulate to get   
SAS Similarity Theorem, 1 2 by CASTC, and DE   AB
 by the CA Postulate.
Proof: Statement Reason
DA EB
1.   
CD  CE 1. Given
DA EB
2. 1    
CD  1  CE 2. Addition property
CD DA CE EB
3.      
CD  CD  CE  CE 3. Algebra
CD  DA CE  EB
4.  CD
  
CE 4. Algebra
5. CA  CD  DA; CB  CE  EB 5. Addition property
CA CB
6.  
CD  CE 6. Substitution property
7. ACB DCE 7. Identity property
8. ABC DEC 8. SAS Similarity Theorem
9. 1 2 9. CASTC
  AB
10. DE  10. CA Postulate

280 CHAPTER 13 Discovering Geometry Solutions Manual


©2003 Key Curriculum Press
DG3SM586_252-296.qxd 8/2/06 6:15 PM Page 281

6. Given: Right triangle with altitude to the hypotenuse C

Show: The two smaller triangles formed are similar to


each other and to the original triangle
Given: Right triangle ABC with right angle ACB; CD  is A
1 2
B
D

the altitude to hypotenuse AB
Show: ADC ACB CDB
Plan: Use the Right Angles Are Congruent Theorem, the identity property, and
the AA Similarity Postulate to get ADC ACB and ACB CDB.
Therefore ADC ACB CDB by the transitive property of similarity.
Proof: Statement Reason
1. ACB is a right angle 1. Given
 is the altitude to hypotenuse AB
2. CD  2. Given
  AB
3. CD  3. Definition of altitude
4. CDA is a right angle 4. Definition of perpendicular
5. CDA ACB 5. Right Angles Are Congruent Theorem
6. 1 1 6. Identity property
7. ADC ACB 7. AA Similarity Postulate
8. CDB is a right angle 8. Definition of perpendicular
9. ACB CDB 9. Right Angles Are Congruent Theorem
10. 2 2 10. Identity property
11. ACB CDB 11. AA Similarity Postulate
12. ADC ACB CDB 12. Transitive property of similarity
7. Given: Right triangle with altitude to the hypotenuse C

Show: The length of the altitude is the geometric


mean of the lengths of the two segments on h

the hypotenuse A B
x D y
Given: Right triangle ABC with right angle ACB;
 is the altitude to hypotenuse AB
CD 
Show: CD is the geometric mean of AD and DB
Paragraph Proof: By the Three Similar Right Triangles, ADC CDB. Then, by
AD CD x h
CSSTP, CD  DB , or h  y . This is equivalent to h  xy, or h  xy
    2
 (h must be
positive). Thus, by the definition of geometric mean, h is the geometric mean of
x and y, or CD is the geometric mean of AD and DB.
8. Given: Right triangle
b a
Show: The square of the length of the hypotenuse is equal
to the sum of the squares of the lengths of the two legs
Given: Right triangle with legs of lengths a and b and d c–d
c
hypotenuse of length c
Show: a 2  b 2  c 2
Paragraph Proof: Use the Perpendicular Postulate to draw the altitude to the
hypotenuse. By the Three Similar Right Triangles Theorem, each of the small right
triangles is similar to the large right triangle. Because the small triangle on the right
cd
is similar to the large triangle, ac  
a by CSSTC. This proportion is equivalent to
a c
  . (The ratio of the length of the shorter leg to the length of the hypotenuse
cd a

Discovering Geometry Solutions Manual CHAPTER 13 281


©2003 Key Curriculum Press
DG3SM586_252-296.qxd 8/2/06 6:15 PM Page 282

is the same in both triangles.) By algebra, either of these proportions is equivalent to


a 2  c(c  d). By the distributive property, a 2  c 2  cd. Now look at the other
small triangle, which is also similar to the large triangle by the Three Similar Right
Triangles Theorem, so bc  db by CSSTP. This proportion is equivalent to db  bc. (The
ratio of the length of the hypotenuse to the length of the longer leg is the same in
both triangles.) By algebra, either of these proportions is equivalent to b2  cd.
Substitute cd for b 2 in the equation a 2  c 2  cd to obtain a 2  c 2  b 2. Then, by
the addition property, a 2  b 2  c 2.
9. Given: Triangle with the square of one C E
side equal to the sum of the squares of
a b a b
the other two sides
Show: The triangle is a right triangle B
c
A D x
F

Given: ABC with side lengths a, b,


and c such that a 2  b 2  c 2
Show: ABC is a right triangle with right angle C
Paragraph Proof: Construct right triangle DEF with E a right angle, and with legs
of lengths a and b. Let x represent the length of the hypotenuse of this triangle. By
the Pythagorean Theorem, a 2  b 2  x 2. Because a 2  b 2  c 2 is given, x 2  c 2
by the transitive property of equality. Because x and c both represent side lengths
and therefore must be positive numbers, x   c 2  c. Therefore DEF ABC
by the SSS Congruence Postulate. Then E C by CPCTC. E is a right angle by
construction and E C, so C must also be a right angle. Therefore ABC is a
right triangle with right angle C.
10. Given: Two right triangles with the hypotenuse H L
and one leg of one triangle congruent to the
hypotenuse and one leg of the other triangle
Show: The triangles are congruent Y P E G

Given: Right triangles HYP and LEG with right


 LG
angles Y and E; HP ; YP
 EG
Show: HYP LEG
Plan: Use the Pythagorean Theorem to write expressions for the lengths of the
unknown legs. Show that the expressions are equivalent. The triangles are congruent
by SSS or SAS.
Proof: Statement Reason
1. HYP is a right triangle with right 1. Given
angle Y; LEG is a right triangle with
right angle E
 LG
2. HP ; YP
 EG  2. Given
3. HP  LG; YP  EG 3. Definition of congruence
4. (HY)  (YP)  (HP) ;
2 2 2 4. Pythagorean Theorem
(LE)2  (EG)2  (LG)2
5. HY   (HP)2  (YP)2 ; 5. Algebra
LE   (LG) 
2  (EG)2
6. HY  LE 6. Substitution and transitive properties

7. HY LE  7. Definition of congruence
8. HYP LEG 8. SSS Congruence Postulate

282 CHAPTER 13 Discovering Geometry Solutions Manual


©2003 Key Curriculum Press
DG3SM586_252-296.qxd 8/2/06 6:15 PM Page 283

Lesson 13.7 (continued) legs are radii and therefore have length 12(9.6) 
11. 4.8 cm. These altitudes divide each of the isosceles
CA Segment Addition AA Similarity
Postulate Postulate Postulate triangles into two congruent right triangles in which
the length of one leg is half the length of the chord,
and the hypotenuse is a radius of the circle.
Parallel/Proportionality
Theorem 2.6 cm
4.1 cm
12. Segment Duplication
Postulate x 4.8 cm y
4.8 cm

Parallel
Postulate

CA
Apply the Pythagorean Theorem to each of the right
Postulate triangles shown to find x and y.

AA Similarity x    (2.6
(4.8)2  )2
Postulate
y    (4.1
(4.8)2  )2
SAS
Congruence The distance between the two chords is x  y 
Postulate
  (2.6)2  (4.8)
(4.8)2   2  (4.1)2
1.5 cm.

SSS
Congruence
Postulate
Now look at the case where the chords are on
SAS Similarity opposite sides of the diameter.
Theorem
5.2 cm
SSS Similarity
Theorem
x
13. Perpendicular AA Similarity
Postulate Postulate

y
Right Angles
Are Congruent
8.2 cm
Theorem

Three Similar
Right Triangles
Theorem
Notice that the distances of the two chords from the
Pythagorean center, x and y, are the same as before, but now the
Theorem
distance between the chords is x  y 
14. Approximately 1.5 cm or 6.5 cm. There are two   (2.6)2  (4.8)
(4.8)2   2  (4.1)2
6.5 cm.

possible answers because the chords can be 15. a. C. The perimeter of the hexagon is 6(10) 
either on the same side or on opposite sides of the 60 cm, and the perimeter of the pentagon is
diameter they are parallel to. First look at the case 5(12)  60 cm, so the perimeters are equal.
where the chords are on the same side of the
diameter. The longer chord will be closer to the b. A. Divide each polygon into congruent isosceles
center of the circle. triangles by drawing congruent segments from the
centers of the polygons to each of their vertices.
5.2 cm In each case, the apothem is an altitude of one of
x these triangles, so this segment divides the isosceles
8.2 cm triangle into two congruent right triangles.
y
Regular hexagon:
9.6 cm
mRST  16(360°)  60°, S

so RST is an equilateral
triangle and RSU is a 30°
30°-60°-90° triangle. In a
 is the
this triangle, RU
 is the
shorter leg and SU 60°
In each case, the distance from the center to the R U T
longer leg. Because RU 
chord is the altitude of an isosceles triangle whose 1
(10)  5 cm, a  SU  53 cm
8.66 cm.
2

Discovering Geometry Solutions Manual CHAPTER 13 283


©2003 Key Curriculum Press
DG3SM586_252-296.qxd 8/2/06 6:15 PM Page 284

Regular pentagon: L

X 118° S

36° 57°
G W Q
A R N
P
54°
W Z Y
Z
mWXY  15(360°)  72°. Because WXY is T

isosceles, mXWY  12(180°  72°)  54°.


Now look at right triangle WXZ. Here, WZ 
1 Look at PSQ. Because PS  is tangent to circle Q,
(WY)  6 cm. Because XWZ and XWY
2  
PS  SQ by the Tangent Conjecture, so
are the same angle, mXWZ  54°. Then 6a 
mPSQ  90°. Therefore mSQP  180° 
tan 54°, so a  6 tan 54°
8.26 cm.
57°  90°  33°. Because SQZ is a central
Thus the apothem is greater in the hexagon. angle in circle Q and SQZ and SQP are the
same angle, mSZ   33°. Because PSQ
c. A. Recall that you can find the area of a regular
polygon by using the formula A  12aP, where PTQ by the SSS Congruence Postulate (using
a is the length of the apothem and P is the the two pairs of congruent radii and the
perimeter (Regular Polygon Area Conjecture). common side), mTQZ  mSQZ  33°,
Because both of the regular polygons have the so mSQT  66°. In circle Q, SLT is an
, so mSLT 
inscribed angle that intercepts SZT
same perimeter (from 15a), the polygon with the 1  1
(mSZT )  (66°)  33°. GLT is the same
greater apothem will have the greater area. From 2 2
15b, you know that this is the hexagon. angle as SLT, so mGLT  mSLT  33°.

d. A. Recall that the Polygon Sum Conjecture states b. 66°. mSQT was found in the solution for 18a.
that the sum of the measures of the n interior  PT
c. 57°. PS  because they are both radii of
angles of an n-gon is 180°(n  2). It follows that  QT
circle P, and QS  because they are both
as the number of sides of an n-gon increases, the radii of circle Q. Therefore PSQT is a kite.
sum of the measures of the interior angles will Then PQ  ST  because the diagonals of a kite
increase. Therefore, because the hexagon has are perpendicular, so SRQ is a right triangle.
more sides than the pentagon, the sum of the Look at the angles in this triangle. From 18a,
measures of the interior angles will be greater in mSQR  33°, so mRSQ  90°  33° 
the hexagon. 57°. RSQ is the same angle as TSQ, so
e. C. Recall that the Exterior Angle Sum Conjecture mTSQ  57°.
states that for any polygon, the sum of the mea- d. 62°. Look at GTL. SGT is an inscribed angle
sures of a set of exterior angles (one exterior ). mSWT
in circle P, so mSGT  12(mSWT 
angle at each vertex) is 360°. Therefore the sum mSPT by the definition of arc measure, and
is the same for both polygons. mSPT  2(57°)  114° by the Kite Diagonal
16. a. The vectors are diagonals of your quadrilateral. Conjecture. So mSGT  12(114°)  57°. This
gives one angle measure in GTL because SGT
b. A 180° rotation about the midpoint of the and LGT are the same angle. From 18a,
common side; the entire tessellation maps onto mGLT  33° (GLT is the same angle as
itself. SLT ). Then, by the Triangle Sum Conjecture,
17. a. A 180° rotation about the midpoint of any side mGTL  180°  57°  33°  90°. By the
Angle Addition Postulate, mGTS  mSTL 
b. Possible answer: A vector running from each mGTL. Because GTS is an inscribed angle in
vertex of the quadrilateral to the opposite vertex   1(118°)  59°.
circle P, mGTS  12mGS 2
(or any multiple of it) Then mSTL  mGTL  mGTS  90° 
 and ST
18. a. 33°. Label the intersection point of PQ  as 59°  31°. Because STL is an inscribed angle
), or mSL
in circle Q, mSTL  12(mSL 
R and the intersection points of the two circles
 as Z and W, as shown in the figure
with PQ 2(mSTL)  2(31°)  62°.
below. Also construct GT.
e. PSQ and PTQ are both right angles by the
Tangent Theorem and thus are supplementary.
Therefore SPT and SQT must also be

284 CHAPTER 13 Discovering Geometry Solutions Manual


©2003 Key Curriculum Press
DG3SM586_252-296.qxd 8/2/06 6:15 PM Page 285

supplementary by the Quadrilateral Sum 32 cm


Theorem. Therefore PSQT is cyclic by the
32 cm 32 cm
Converse of the Cyclic Quadrilateral Theorem. 27 cm

 is tangent to circle Q, mPSQ  90°


f. Because PS 13.5 cm
r
(Tangent Theorem). Because mPSQ  90°, SQ  x

must be tangent to circle P (Converse of Tangent
Theorem). x 27
   
3 x  32 32
19. 1
2 1
2 32x  27(x  32)

 32x  27x  864


5x  864
2
x  172.8
Substitute the area expressions you found in
Lesson 13.1, Exercise 31c and in Lesson 13.5, Then, by the Pythagorean Theorem, r 
2  (13.5)2
172.3
172. The rise, which
Exercise 12b. )
(172.8 

 2  1  1   3
is the radius of the semicircular arch, is approxi-
 3   mately 172 cm. The span is the diameter of the
A   2 arch, so it is twice the rise. Therefore the span is
  approximately 344.6
344 cm. Use the inverse sine
  1  1  3  3
2 to find the central angle measure:  = sin1 172.8 
13.5
  2


 4.5°, so the central angle is approximately 2(4.5°) 


   3 2 
 6  1 

3 9°. Divide into 180° to find the number of
2 2  2 1 180°
voussoirs:  9°  20.

 3
20. 1  6   EXTENSIONS
4
A. B
 E
sh
h
Substitute the area expressions you found in Lesson D Q F A D C
13.5, Exercise 12a and in Exercise 19 above. b sb

A 4  12 12  
3      
3
2 1  Given: ABC DEF with 
AB BC AC
  
DE  EF  DF  s;
 is the altitude to AC
BD ; EQ
 is the altitude to DF
3      
 3  
 
3
Area of ABC
2 11 6  4 Show: Area of DEF  s
4 12 12 2

21. a. CDG CFG by SAA; GEA GEB by Paragraph Proof: Choose AC  as the base of ABC
SAS; DGA FGB by the Hypotenuse Leg 
and DF as the base of DEF. Let b  DF and
Theorem. AC AC 
h  EQ.  DE  s, so b  s, and AC  sb. BD is the
 
 CF
b. CD  and DA
 FB  by CPCTC; CD    
altitude to AC and EQ is the altitude to DF , so, by
DA  CF  FB (addition property). Therefore BD AB
the Corresponding Altitudes Theorem,  EQ  DE .
 
 CB
CA , and ABC is isosceles. BD BD
Thus, by the transitive property,  EQ  s, or h  s,
 
c. The figure is inaccurate. so BD  hs. Now find the area of each triangle:
1 1
d. The angle bisector does not intersect the perpen- Area of DEF  2(DF)(EQ)  2bh
dicular bisector inside the triangle as shown,
1 1
except in the special case of an isosceles triangle, Area of ABC  2(AC)(BD)  2(sb)(sh)
when they coincide.
22. 173 cm, 345 cm, 20 stones. Draw the trapezoid, and
1 1
 2s 2bh = s 2 2bh  
extend the legs until they meet to form a triangle.
Let r represent the rise. Use proportional sides in Area of ABC
    s 2
1
s 2 2bh  
similar triangles to find x. Area of DEF 1
bh
2

Discovering Geometry Solutions Manual CHAPTER 13 285


©2003 Key Curriculum Press
DG3SM586_252-296.qxd 8/2/06 6:15 PM Page 286

B. G H y
A B
D (b, ?) C (?, ?)
F C L I

a 2  b2
E D
K J x
A (0, 0) B (a, 0)
Given: Two similar polygons with scale factor s
Show: The area of one polygon is s 2 times the area
The shorter leg of the right triangle has length b.
of the other polygon
, the hypotenuse of this triangle, is a
Because AD
Paragraph Proof: Divide each polygon into (n  2) side of the rhombus, its length is a. Therefore, by
triangles by drawing the diagonals from one vertex.
the Pythagorean Theorem, the length of the longer
(The process is shown for hexagons.) Show that
leg is a 2  b 2. This is the y-coordinate of D and
AFE GLK and ABC GHI by the SAS
Similarity Theorem. Then use CSSTP and CASTC therefore also the y-coordinate of C. Thus the
and the subtraction property to show that coordinates of C are a  b, a 
2  b 2 and

AED GKJ and ACD GIJ. the coordinates of D are b, a 2  b 2.
(Although this process is shown only for hexagons, 4. Possible answer:
it can be generalized to any polygons.) Then, y
because the polygons are made up of similar trian-
gles, you can use the result from Extension A to
show that their areas are similar, with ratio s 2. T (0, b) C (a , b )

IMPROVING YOUR ALGEBRA SKILLS


1. Add; 5x  y  29 R (0, 0) E (a , 0)
x

2. Add; 10x  2y  50
 00 0
 
3. Subtract; x  y  4 Slope RE a0  a 0
 b0 b
 
USING YOUR ALGEBRA SKILLS 10
Slope CE a  a  0 (undefined)
 bb 0
 
EXERCISES Slope TC a0  a 0
1. B(a, 0). The y-axis passes through the midpoint of  b0 b
 
, the base of the isosceles triangle. B lies on the
AB
Slope TR 0  0  0 (undefined)
, B must
x-axis. Because (0, 0) is the midpoint of AB Opposite sides have the same slope and are there-
be the same distance from the origin as A, so the fore parallel by the parallel slope property. (In this
coordinates of B are (a, 0). case, two sides have undefined slope.) Two sides are
horizontal and two sides are vertical, so the angles
2. C(a  b, c). Because ABCD is a parallelogram, are all right angles. Because RECT is a parallelogram
  AB
DC . AB lies along the x-axis, so DC
 must also with four right angles, it is a rectangle.
be a horizontal segment. Therefore the y-coordinate
of C must be the same as the y-coordinate of D, 5. Possible answer:
which is c. Because opposite sides of a parallelogram y
are congruent, DC  AB. Because AB  a  0  a,
the x-coordinate of C must be a  b to get CD  I (b, c)
(a  b)  b  a.
Z b, c
( ) Y a+b, c
( )
3. C a  b,  a 2  b 2, D b, 
a 2  b 2. The 2 2 2 2
 is a  0  a. All sides of a rhombus
length of AB
are congruent, so the length of each of the other x
three sides must also be a. A rhombus is a T (0, 0) X a,0 R (a , 0)
2 ( )
  AB
parallelogram, so DC . Therefore DC  must be
a horizontal segment, and C and D will have the same
y-coordinate. The x-coordinate of C must be a  b
a0 00 a
Coordinates of X  2, 2  2, 0    
to get CD  (a  b)  b  a. Draw a perpen-
dicular segment from D to the x-axis to form a
ab 0c ab c
Coordinates of Y  2, 2  2, 2    
right triangle. b0 c0 b c
Coordinates of Z  2, 2  2, 2    
286 CHAPTER 13 Discovering Geometry Solutions Manual
©2003 Key Curriculum Press
DG3SM586_252-296.qxd 8/2/06 6:15 PM Page 287

, RI
X, Y, and Z are the midpoints of TR , and TI
, Therefore EQ  EU  UQ, so EQ  EU  UQ
,
respectively, by the coordinate midpoint property. and EQU is equilateral by definition.
Therefore XY, YZ
, and ZX are midsegments by
8. Given: A rectangle with both diagonals
definition.
Show: The diagonals are congruent
6. y
y

P (c , d ) A(a – c , d )

T (0, b) C (a , b )

x
T (0, 0) R (a , 0)
x
R (0, 0) E (a , 0)
 00 0
 
Slope TR a0  a 0
d0 d Given: Rectangle RECT with diagonals RC  and TE
  
Slope RA a  c  a  c 
Show: RC TE 
 dd 0
 
a – c – c  a  2c  0
Slope AP Proof: To show that two segments are congruent,
use the distance formula to show that they have the
 d0 d
 
Slope PT c0  c same length.
 and AP
TR  have the same slope and are parallel TE  
(a  0
) 2  (0
 b)2  a
2  b2

 and RA
by the parallel slope property. PT  have
RC  
(0  a
)2  (0
 b)2  a
2  b2

unequal slopes, so they are not parallel. Thus TRAP
has only one pair of parallel sides and is a trapezoid  TE
RC  because both segments have the same
by definition. length. Therefore the diagonals of a rectangle are
congruent.
Use the distance formula to find the lengths of the
nonparallel sides. 9. Given: A triangle with one midsegment
PT  
(c  
 (d
  
  0)2 0)2 c2 d2 Show: The midsegment is parallel to the third side
and half the length of the third side
RA  
(a  c
 a)2
 (d 
0)2
y
   d 2  
(c)2  c 2  d 2
The nonparallel sides of the trapezoid have the I (c , d )
same length, so trapezoid TRAP is isosceles by
definition. Z c,d ( ) Y a+c, d
( )
2 2 2 2

7. y

x
U (0, a 3) T (0, 0) R (a , 0)

Given: TRI and midsegment ZY 


E Q 1
(–a , 0) (a , 0)
x
Show: ZY   TR and ZY  TR
2
Proof: To show that two segments are parallel, use
the parallel slope property. To compare lengths, use
the distance formula.
The y-axis splits EQU into two 30°-60°-90° trian-
 00 0
 
gles, and the altitude of the equilateral triangle is Slope TR a0  a 0
the longer leg of each of the 30°-60°-90° triangles,
d d
so the y-coordinate of U will be a3.   
   2 2 0
Slope ZY 0
EQ  2a ac c a
   
2 2 2
EU  
(a  0
)2  0 
a32  
a 2  3
a2 The slopes are the same, so the segments are
 and ZY
parallel by the parallel slope property. TR 
 
4a 2  2a
UQ  
(a  0)2
 0 
a32  
a 2  3
a2
 
4a 2  2a

Discovering Geometry Solutions Manual CHAPTER 13 287


©2003 Key Curriculum Press
DG3SM586_252-296.qxd 8/2/06 6:15 PM Page 288

are horizontal lines, so to find their lengths, subtract Proof: To show that a quadrilateral is a kite, use the
their x-coordinates. distance formula to show that only two pairs of
adjacent sides have the same length.
TR  a  0  a
ac c a 1 1 KI  
(0  a)
2  (b  0)2 
 a 2  b2

ZY  2  2  2  2a  2TR
IT  
(0  (
a))2 
 (b 
0)2  a
2  b2

Thus the midsegment is half the length of the third
side. Therefore the midsegment of a triangle is TE  
(0  (
a))2 
 (c 
0)2  a
2  c2

parallel to the third side and half the length of the
EK  
(a  0
)2  (0
 c)2  a
2  c2

third side.
 and IT
Adjacent sides KI  have the same length and
10. Given: Trapezoid  
adjacent sides TE and EK have the same length, and
Show: The midsegment is parallel to the bases because b  c, the pairs are not equal in
y length to each other. Therefore KITE is a kite by
definition. Therefore, if only one diagonal of a
quadrilateral is the perpendicular bisector of the
P (b , c ) A (d , c) other diagonal, then the quadrilateral is a kite.
12. Given: A quadrilateral with midpoints connected to
M N form a second quadrilateral
x Show: The second quadrilateral is a parallelogram
T (0, 0) R (a , 0)
y


Given: Trapezoid TRAP with midsegment MN

Show: MN  TR D (d , e) L b+d, c+e
( )
2 2
d, e
Proof: By the coordinate midpoint property, the ( 2 2 ) A (b , c )

coordinates of M are 2b, 2c and the coordinates of G


ad c R a+b, c
( )
N are  2 , 2 . To show that the midsegment and
  2 2

bases are parallel, find their slopes. x


Q (0, 0) P a , 0 U (a , 0)
c c 2 ( )
  
   2 2 0
Slope MN    0
ad b adb Given: Quadrilateral QUAD with midpoints P, R, L,
   
2 2 2 and G
0  0
      0 0 Show: PRLG is a parallelogram
Slope TR a0 a
cc 0 Proof: To show that a quadrilateral is a parallelo-
  
Slope PA db  db 0 gram, show that opposite sides have the same slope.
c c
The slopes are equal. Therefore the lines are parallel.   0  c
 2 2
Slope PR      b
ab a b
11. Given: A quadrilateral in which only one diagonal    
2 2 2
is the perpendicular bisector of the other ce c e
   
Show: The quadrilateral is a kite       e
Slope RL
2 2 2
bd ab da da
y    
2 2 2
e ce c
I (0, b)     c
   2 2 2
Slope LG    b
d bd b
T (– a, 0) K(a, 0)     
M(0, 0)
x 2 2 2
e e
  0  e
 2 2
Slope GP  da

d a da
   
E (0 , c) 2 2 2
 and LG
PR  have the same slope, and RL  and GP

Given: Quadrilateral KITE with diagonals IE and have the same slope, so the opposite sides are
. IE
TK  is the perpendicular bisector of TK
, but TK
 parallel by the parallel slope property, and PRLG is
 b  c
is not the perpendicular bisector of IE a parallelogram by definition. Therefore the figure
Show: KITE is a kite formed by connecting the midpoints of the sides of
a quadrilateral is a parallelogram.

288 CHAPTER 13 Discovering Geometry Solutions Manual


©2003 Key Curriculum Press
DG3SM586_252-296.qxd 8/2/06 6:15 PM Page 289

13. Given: An isosceles triangle with the midpoint of ● To illustrate proof 4, draw rays that start at the point
the base connected to the midpoint of each leg to of intersection of the two intersecting lines and that
form a quadrilateral pass through corresponding points on the original
Show: The quadrilateral is a rhombus figure, and then draw its first and second images.
y ● In proof 5, find the ordered pairs of the three
midpoints and the equations of the three medians.
D a2 , h2
( ) Then solve the equations simultaneously.
A (a , h)
Extra credit
F 32a, h2 ● You create or find unusual proofs or proofs for several
( )
of the theorems.

B (0, 0) E (a, 0) C (2a , 0)


x Sample proofs:
1. Three, four, or five triangles fitted about a point
Given: Isosceles triangle ABC with midpoint of leave a gap that can be closed by folding the
base, E, and midpoints of legs, D and F, connected triangles to start a tetrahedron, an octahedron, or
to form quadrilateral ADEF an icosahedron, respectively. Likewise, three squares
or three pentagons leave a gap and can be folded
Show: ADEF is a rhombus
to start a cube or a dodecahedron. Three hexagons
Proof: Use the distance formula to show that all the leave no gap, and three of any polygon with more
sides of ADEF have the same length. than six sides overlap. (See the Platonic Solids


a  2a
  
h  2 
2a 
h2
h 2 2 2 2 video.)
AD 
2. If each vertex is an endpoint of an even number of
a 2  h2
 edges, then whenever a traveler enters that vertex on
  2 one edge, at least one unused edge is available for
departure. (The vertex at which the traveler begins

32a 
a 
h2 
h 
2a 
h2
2 2 2 2
AF  will have one edge left for ending there.) If all but
2  h2
two vertices have an even number of edges, then the
a
   same argument applies to the nonspecial vertices;
2
moreover, one of those special vertices can be used


a  2a
  
0  2  
 
h2
h 2 a 2 2 2 as a beginning and the other as an ending, so the
DE  2 network can be traveled. If more than two vertices
a 2  h2 have an odd number of edges, the network cannot

  2 be traveled.
3. The n-gon can be divided into n triangles with a

 
a 
h2  0   
 
h2
3a 2 a 2 2 2
EF  2 2 common vertex by drawing a segment from an
interior point to each of the n vertices. (Each
a 2  h2

  2
segment drawn to one of the n vertices is the side
of two triangles, and each side of the n-gon is the
AD  AF  DE  EF by the transitive property. side of one triangle; this gives 2n  n  3n sides
Therefore ADEF is a rhombus by the definition of a and therefore n triangles.) Each triangle has interior
rhombus. angles whose measures sum to 180°. Because the
PROJECT sum of the measures of the interior angles of the
Project should satisfy the following criteria: n-gon is the sum of the measures of the triangles
minus the 360° around the central point, it equals
● Proofs, which might not match the sample proofs, are
n  180°  360°  (n  2)180°.
logically valid and clearly written. You need not create
the proofs yourself; you might research them and  reflected over two lines, OM
4. To consider PQ  and
write them up to show your understanding. , you must consider three cases: PQ
OL  is outside
the angle and is reflected over the nearer line first;
● Use the Triangle Sum Theorem for proof 3. To prove  is inside MOL; and PQ
PQ  is outside the angle
it formally for any n-gon requires mathematical
induction.

Discovering Geometry Solutions Manual CHAPTER 13 289


©2003 Key Curriculum Press
DG3SM586_252-296.qxd 8/2/06 6:15 PM Page 290

and is reflected over the farther line first. Consider when arranged around a point, must have measures
the second case: that add exactly to 360°. An Archimedean tiling can
Q' be constructed using only equilateral triangles,
squares, or regular hexagons. Any other regular
P' L
polygon would create either a gap or an overlap
P in the single shape tiling. Hence, there are three
Q monohedral tessellations, which are Archimedean
O
tilings.
M
To find other possible edge-to-edge tilings, use the
P'' fact that the measure of an interior angle of a
180°(n  2)
regular n-gon is n degrees. If the n1-gon,
Q'' n2-gon, . . . , and nk-gon meet at a vertex, you
 PO
 and know that
Because reflection is an isometry, PO
 P O
PO , so PO P O. Likewise, QO
 180°(n1  2) 180°(n2  2) 180°(nk  2)
 and QO Q O, so QO  Q O. By angle     · · ·  
QO n1 n2 nk
addition, mPOP  (mP OM  mMOP)   360°
(mPOL  mLOP). Also, because reflection is an n1  2 n2  2 nk  2
isometry, P OM MOP and POL LOP.  n1   n2 + · · ·   nk  2
So, by substitution, mPOP  2(mMOP 
mPOL). But mMOP  mPOL  mMOL. There are 17 groups of positive integers that satisfy
Therefore mPOP  2mMOL. The same proce- this equation. Therefore there are 17 choices of
dure applies to QOQ . Because PQ  is the same polygons that can be fitted around a vertex without
distance from point O as is P Q and because both gaps or overlaps. In four cases, these polygons can
images move the same number of degrees, therefore, be arranged in two ways (such as 32.4.12 and
by the definition of rotation, the image P
Q is 3.4.3.12). Thus there are 21 possible types of
 about point O a distance of
a rotation of PQ vertices: 36, 34.6, 33.42, 32.4.3.4, 32.4.12, 3.4.3.12,
2mMOL. Similar proofs can be given for the 3.6.3.6, 32.62, 3.4.2.6, 3.4.6.4, 3.7.42, 3.8.24, 3.9.18,
other cases. 3.10.15, 3.122, 44, 4.5.20, 4.6.12, 4.82, 52.10, and 63.
After checking each of these types, you will find 11
5. The coordinates of the midpoints are that are edge-to-edge tilings by regular polygons
ce df ae bf ac bd
D 2 , 2 , E  2 , 2 , and F  2 , 2 .
      with all vertices of the same type: 36, 34.6, 33.42,
The equations of the medians are 32.4.3.4, 3.6.3.6, 3.4.6.4, 3.122, 44, 4.6.12, 4.82, and
63. (See Tilings and Patterns by Gruenbaum and
d  f  2b b(c  e)  a(d  f )
: y  x  
AD Shephard.)
c  e  2a c  e  2a
b  f  2d d(a  e)  c(b  f ) EXTENSION
: y   x  
BE a  e  2c a  e  2c
b  d  2f f(a  c)  e(b  d)
: y   x  
CF a  c  2e a  c  2e b c
h
Solving any two of these equations simultaneously
yields the coordinates of the centroid, (x, y) 
ace bdf
  3 , 3 . a

Given: Triangle with side lengths a, b, and c;


6. Here is one of many proofs that 2 is irrational:
altitude with length h to side of length a
Assume that 2 is rational and then arrive at a
Show: Area of triangle   s(s  a)(s  b
)(s  c),
contradiction. If 2 is rational, then it can be where s   abc

2
written as a fraction, ab, in reduced form. Because
a 2 Paragraph Proof: The altitude divides the triangle
  
b 2 , ba2  2. Because a and b have no common into two right triangles and divides the side of
2 2
factors, ba2 cannot reduce to an integer, so ba2  2. length a into two parts. By the Pythagorean
Theorem, the lengths of these parts are b 2  h2
This is a contradiction, so  2 is irrational. 
and c 2  h. Therefore a  b
2 2  h 
2
7. An Archimedean tiling is a distinct edge-to-edge
 c 2  h2. To eliminate radicals from this equation,
tiling of regular polygons with all vertices of the
square both sides twice.
same type. For these shapes to fill the plane from
edge to edge without gaps or overlaps, their angles,

290 CHAPTER 13 Discovering Geometry Solutions Manual


©2003 Key Curriculum Press
DG3SM586_252-296.qxd 8/2/06 6:15 PM Page 291

a  b
2  h2 
 c 2  h2 16A2  (a b  c)(a  b  c)
 (b  c  a)(b  a  c)
a 2  b
2  h2 

2  h2 2
 c
(a  b  c)(a  b  c)(b  c  a)(b  a  c)
a 2  b 2  h2  2b
2  h2 2  h2 + c 2  h2
c  A2   16
a 2  b 2  c 2  2h2  2b
2  h2 2  h2
c 
(a 2  b2  c2  2h2)2  2
b2  
 
h2 c2 h2 2

A2  2 2 2  
a b  c a  b  c b  c  a

((a 2  b 2  c 2)  2h2)2  2b
2  h2
c 
2  h2 2   
bac
2
 
(a 2  b 2  c 2)2  4h2(a 2  b 2  c 2)  4h4 In Hero’s formula, s represents the semiperimeter of
abc abc
 4(b 2  h2)(c 2  h2) the triangle: s   . Then s  a   
2 2
2a bca abc 2b abc
  , s  b      ,
(a 2  b 2  c 2)2  4h2a2  4h2b 2  4h2c 2  4h4 2 2 2 2 2
abc 2c abc
and s  c   2
    .
2 2
 4(b 2c 2  b 2h2  c 2h2  h4)
abc abc bca
(a 2  b 2  c 2)2  4h2a2  4h2b 2  4h2c 2  4h4
222
Because A2 
 2  
bac abc bca abc
2

2 2
 4b 2c 2  4b 2h2  4c 2h2  4h2
 2, A  s(s  a)(s  b)(s  c).
abc 2
(a 2  b 2  c 2)2  4h2a2  4b 2c 2
Therefore A  s(s
  a)(s
  b)(s
  c).
4h2a2  4b 2c 2  (a 2  b 2  c 2)2
Let A represent the area of the triangle. Using a as CHAPTER 13 REVIEW
the length of the base, the standard formula for the
area of a triangle gives A  12ah. The last equation EXERCISES
above contains 4h2a2 on one side, so rewrite 1. False. The quadrilateral could be an isosceles
A  12ah to get an expression for 4h2a 2. trapezoid.
1
A  2ah 2. True
2A  ah 3. False. The figure could be an isosceles trapezoid
or a kite.
4A2  a 2h2
4. True
16A2  4a 2h2  4h2a 2
Because 16A2  4h2a2 and 4h2a2 = 4b 2c 2  5. False. The angles are supplementary, but not neces-
(a 2  b 2  c 2)2, by the transitive property 16A2  sarily congruent.
4b 2c 2  (a 2  b 2  c 2)2. Rearrange terms in this 6. False. See Lesson 13.5, Example B.
equation to write A2 as a product of four fractions.
Some of the steps below involve factoring the differ- 7. True
ence of two squares using the factoring pattern 8. perpendicular
x2  y 2  (x  y)(x  y).
9. congruent
16A2  4b 2c 2  (a 2  b 2  c 2)2
10. the center of the circle
16A2  (2bc)2  (a 2  b 2  c 2)2
11. four congruent triangles that are similar to the
16A2  ((2bc)  (a 2  b 2  c 2)) original triangle
 ((2bc)  (a 2  b2  c 2)) 12. an auxiliary theorem proven specifically to help
16A2  (2bc  a 2  b 2  c 2)(2bc  a 2  b 2  c 2) prove other theorems

16A2  (a2 (b 2  2bc  c 2)) 13. If a segment joins the midpoints of the diagonals of
a trapezoid, then it is parallel to the bases.
 ((b2  2bc  c 2)  a 2)
14. Angle Bisector Postulate
16A2  (a 2 (b  c)2)((b  c)2  a 2)
15. Perpendicular Postulate
16A2  (a (b  c))(a  (b  c))
16. Assume the opposite of what you want to prove;
 ((b  c)  a)((b  c)  a) then use valid reasoning to derive a contradiction or
an absurdity.

Discovering Geometry Solutions Manual CHAPTER 13 291


©2003 Key Curriculum Press
DG3SM586_252-296.qxd_k1 1/3/07 4:38 PM Page 292

17. a. That smoking is not glamorous Given: Isosceles ABC with AC  BC ; altitudes
 
AD and BE ; ED
b. If smoking were glamorous, then this smoker
would look glamorous. This smoker does not Show: ED   AB

look glamorous, therefore smoking is not Paragraph Proof: AC   BC  is given, so EAB 
glamorous. DBA by the Isosceles Triangle Theorem. AEB 
18. False. The parallelogram is a rhombus. (It could be BDA by the definition of altitude and the Right
Angles Are Congruent Theorem. AB   AB by the
a square, but it might not be.)
reflexive property of congruence. AEB  BDA
19. False. In fact, the measure of the angle between by SAA. Therefore AE  BD by CPCTC. AXE 
them equals the measure of one of the other base BXD by the VA Theorem, so AXE  BXD by
angles. The figure below shows an example of this SAA. Therefore EX  DX  and AX
  BX  by
situation. CPCTC, so AXB and EXD are both isosceles.
80° 80° XAB  XBA and XED  XDE by the
Isosceles Triangle Theorem. EXD  AXB by the
80° VA Theorem. Then, by the Triangle Sum Theorem,
100° 100° mXAB  mXBA  mAXB  180° 
80° mXED  mXDE  mEXD. Then mXAB 
50° 50° mXBA  mXED  mXDE, so mXBA 
50° 50° mXBA  mXED  mXED, so 2mXBA 
2mXED. Therefore XBA  XED, so ED   AB

If the angle bisectors were perpendicular, then the by the AIA Theorem.
base angles would be right angles, and the shape
would be a rectangle. Therefore it could not be a 22. True
trapezoid. E M

20. False. The sum of the measures of the interior 4 B


3
angles of the pentagon formed by the perpendicular 2
1
bisectors is (5  2)(180°)  540°, so x  540°  R O
(180°  2b)  360°  2b. The perpendicular bisec-
 and
Given: Rhombus ROME with diagonals RM
tors of the sides of congruent sides of the isosceles

EO intersecting at B
trapezoid will be perpendicular if 360°  2b  90°.
Solving this equation gives b  135°. Therefore the   EO
Show: RM 
statement is true only if the measures of one pair of Proof:
base angles of the trapezoid are 135°. Statement Reason
x 1. 1  2 1. Rhombus Angles
a a
Theorem
2. RO  RE 2. Definition of rhombus
b b   RE
3. RO  3. Definition of
x ⫽ 540° ⫺ (180° ⫹ 2b)
congruence
x ⫽ 360° ⫺ 2b   RB
4. RB  4. Identity property of
congruence
21. True (except in the special case of an isosceles right
triangle, in which the segment is not defined 5. ROB  REB 5. SAS Congruence
because the feet coincide). Postulate
C
6. 3  4 6. CPCTC
7. 3 and 4 are 7. Linear Pair Postulate
supplementary
8. 3 and 4 are right 8. Congruent and
angles Supplementary
E D Theorem
X   EO
9. RM  9. Definition of
A B
perpendicular

292 CHAPTER 13 Discovering Geometry Solutions Manual


©2003 Key Curriculum Press
DG3SM586_252-296.qxd 8/2/06 6:15 PM Page 293

23. True BAD DCB by the Opposite Angles Theorem.


D E C Therefore 12mBAD  12mDCB by the multiplica-
2
tion property, so m1  m2 by the transitive
  DC
property. AB  by the definition of parallelo-
1 3
A F B gram, so 2 and 3 are supplementary by the Inte-
 bisecting
Given: Parallelogram ABCD with AE rior Supplements Theorem, that is, m2  m3 

DAB and CF bisecting DCB 180°. Then, by substitution, m1  m3  180°,
so, by definition and the substitution property, 2
  CF
Show: AE    CF
 by
and 3 are supplementary. Therefore AE
Paragraph Proof: m1  12mBAD and m2  the Converse of the Interior Supplements Theorem.
1
mDCB by the definition of angle bisector.
2

Lesson 13 Review (continued)

24. Plan: Use the Inscribed Angle Theorem, the addition property, and the distributive
property to get mP  mE  mN  mT  mA  12(mTN   mAT 
  mEP
mPA   mNE  ). Because there are 360° in a circle, mP  mE  mN 
mT  mA  180°.
Proof: Statement Reason
1  1 
1. mP  2mTN ; mE  2mAT ; 1. Inscribed Angle Theorem
1  1 
mN  2mPA ; mT  2mEP ;
1 
mA  2mNE
2. mP  mE  mN  mT  2. Addition property
1  1  1 
mA  2mTN  2mAT  2mPA 
1  1 
mEP  2mNE
2
3. mP  mE  mN  mT  3. Distributive property
1   
mA  2(mTN  mAT  mPA 
  mNE
mEP )

4. mP  mE  mN  mT  4. There are 360° in a circle,


1 substitution property
mA  2(360°)  180°
25. Given: Right triangle RTH T

Show: At least one nonright angle (H or T) has measure


less than or equal to 45°
Indirect Paragraph Proof: Assume mH  45° and mT  45°. R H
Then mH  mT  45°  45°, so mH  mT  90°.
Because mR  90°, this would give mH  mT  mR  90°  90°, or
mH  mT  mR  180°. But this creates a contradiction because mH 
mT  mR  180° by the Triangle Sum Theorem. Therefore the assumption that
mH  45° and mT  45° is false, and the conclusion that either mH 45° or
mT 45° (or possibly both) is true.

26. Given: TYR with midsegment MS Y

  TR 1
 and MS  (TR)
Show: MS 2 M S
Plan: Use the definition of midpoint, the Segment Addition
Postulate, the substitution property, and the division property
MY 1 SY 1 T R
TY  2 and RY  2 . Then use the identity property and
to get     
the SAS Similarity Theorem to get MSY TRY. Therefore
MS  12(TR) by CSSTP and the multiplication property, and MS   TR
 by CASTC and
the CA Postulate.

Discovering Geometry Solutions Manual CHAPTER 13 293


©2003 Key Curriculum Press
DG3SM586_252-296.qxd 8/2/06 6:15 PM Page 294

Proof: Statement Reason


 is a midsegment of TYR
1. MS 1. Given
; S is the
2. M is the midpoint of YT 2. Definition of midsegment

midpoint of YR
3. MY  MT; YS  SR 3. Definition of midpoint
4. TY  2(MY ); RY  2(SY ) 4. Segment Addition Postulate
MY 1 SY 1
5.   ;  
TY  2 RY  2 5. Division property
MY SY
6.  
TY  RY
 6. Transitive property
7. MYS TYR 7. Identity property
8. MSY TRY 8. SAS Similarity Theorem
MS MY
9.   
TR  TY 9. CSSTP
MS 1
10.   
TR  2 10. Transitive property
1
11. MS  2(TR) 11. Multiplication property
12. YMS YTR 12. CASTC
  TR
13. MS  13. CA Postulate
27. Given: Trapezoid ZDYO with midsegment TR  D Y
13
  1
Show: TR  ZO ; TR  2(ZO  DY )
R
Plan: Use the Line Postulate to extend ZO and DR. T

Then use the Line Intersection Postulate to label P 42


 
as the intersection of ZO and DR . DYR POR Z O P

by the SAA Congruence Theorem; thus DY OP by 
CPCTC. Use the Triangle Midsegment Theorem and the substitution property to get
TR  12(ZO  DY ). Also, apply the Triangle Midsegment Theorem to get TR  ZO.
Proof: Statement Reason
1. Extend ZO and DR 1. Line Postulate
 
2. ZO and DR intersect at point P 2. Line Intersection Postulate

3. TR is the midsegment of trapezoid 3. Given
ZDYO

4. R is the midpoint of YO 4. Definition of midsegment
5. YR  OR 5. Definition of midpoint

6. YR OR 6. Definition of congruence
7. DRY PRO (3 4) 7. VA Theorem
  ZO
8. DY  8. Definition of trapezoid
  ZP
9. DY  9. Z, O, and P are collinear
10. YDR OPR 10. AIA Theorem
11. DYR POR 11. SAA Theorem
 PO
12. DY  12. CPCTC
13. DY  OP 13. Definition of congruence
1
14. TR  2(ZP) 14. Triangle Midsegment Theorem (ZDP)
15. ZO  OP  ZP 15. Segment Addition Postulate
1
16. TR  2(ZO  OP) 16. Transitive property
1
17. TR  2(ZO  DY) 17. Substitution property
  ZP
18. TR  18. Triangle Midsegment Theorem (ZDP)
 
19. TR  ZO 19. Z, O, and P are collinear

294 CHAPTER 13 Discovering Geometry Solutions Manual


©2003 Key Curriculum Press
DG3SM586_252-296.qxd 8/2/06 6:15 PM Page 295

28. a. Conjecture: The quadrilateral formed when the midpoints of the sides of a
rectangle are connected is a rhombus.
b. D G C

H F

A E B

, BC
Given: Rectangle ABCD with E, F, G, and H the midpoints of AB , CD , and DA
,
respectively
Show: EFGH is a rhombus
Plan: Use the Right Angles Are Congruent Theorem and the SAS Congruence Postu-
late to get AEH BEF DGH CGF. Then use CPCTC.
Proof: Statement Reason
1. ABCD is a rectangle 1. Given
2. ABCD is a parallelogram 2. Definition of rectangle
 DA
3. BC ; AB
 CD  3. Opposite Sides Theorem
4. BC  DA; AB  CD 4. Definition of congruence

5. F is the midpoint of BC ; H is the 5. Given
; E is the midpoint
midpoint of DA
; G is the midpoint of CD
of AB 
6. BF  FC; DH  HA; 6. Definition of midpoint
AE  EB; CG  GD
7. BF  FC  BC; DH  HA  DA; 7. Segment Addition Postulate
AE  EB  AB; CG  DG  CD
8. 2(BF)  2(FC)  BC; 2(DH)  8. Substitution property
2(HA)  DA; 2(AE)  2(EB)  AB;
2(GC)  2(GD)  CD
1 1
9. BF  FC  2(BC); DH  HA  2(DA); 9. Multiplication property
1 1
AE  BE  2(AB); CG  DG  2(CD)
10. BF  CF  DH  AH; AE  BE  10. Transitive property
CG  DG
 CF
11. BF  DH AH ; AE
 BE  11. Definition of congruence
 DG
CG 
12. A, B, C, and D are right angles 12. Definition of rectangle
13. A B C D 13. Right Triangles Are Congruent Theorem
14. AEH BEF DGH CGF 14. SAS Congruence Postulate
15.  EF
EH  GH GF
 15. CPCTC
16. EFGH is a rhombus 16. Four Congruent Sides Rhombus Theorem
29. a. Conjecture: The quadrilateral formed when the midpoints of the sides of a
rhombus are connected is a rectangle.
b. I
P M
Y X
L J

O N
K

, JK
Given: Rhombus IJKL with M, N, O, and P the midpoints of IJ , KL
, and LI
,
respectively
Show: MNOP is a rectangle

Discovering Geometry Solutions Manual CHAPTER 13 295


©2003 Key Curriculum Press
DG3SM586_252-296.qxd 8/2/06 6:15 PM Page 296

Paragraph Proof: Apply the Triangle Midsegment Theorem to LIJ to get PM   LJ



   
and to LKJ to get ON  LJ . Then, by the Parallel Transitivity Theorem, PM  ON .
Similarly, apply the Triangle Midsegment Theorem to ILK to get PO   IK and to
   
IJK to get MN  IK . Then, by the Parallel Transitivity Theorem, PO  MN . Thus
MNOP has two pairs of opposite parallel sides and is by definition a parallelogram.
Because IJKL is a rhombus, LJ   IK  by the Rhombus Diagonals Conjecture. Then
IYL, KYL, IYJ, and KYJ are right angles by the definition of perpendicular.
Using the parallel sides found earlier from the Triangle Midsegment Theorem, MNOP
is made up of four small parallelograms, so IYL MPO, KYL PON,
IYJ PMN, and KYJ MNO by the Opposite Angles Theorem. Thus the
four angles of MNOP are all right angles. Thus MNOP is a parallelogram with four
right angles, so, by definition, it is a rectangle.
30. a. Conjecture: The quadrilateral formed when the midpoints of the sides of a kite
are connected is a rectangle.
b. I
P M
L J

O N

Given: Kite IJKL with M, N, O, and P the midpoints of IJ , JK


, KL
, and LI
,
respectively
Show: MNOP is a rectangle
Paragraph Proof: Apply the Triangle Midsegment Theorem to LIJ to get PM   LJ

  
and to LKJ to get ON  LJ . Then, by the Parallel Transitivity Theorem, PM  ON .
Also, by the Triangle Midsegment Theorem, PM  12(LJ) and ON  12(LJ), so PM 
 ON
ON by the transitive property. Then PM  by the definition of congruence.
Therefore MNOP is a parallelogram by the Opposite Sides Parallel and Congruent
  IK
Theorem. Because IJKL is a kite, LJ  by the Kite Diagonals Conjecture. Then, by
the CA Postulate and Interior Supplements Theorem, the four angles of MNOP are
all right angles. Thus MNOP is a parallelogram with four right angles, so it is a
rectangle by definition.
31. Given: Circle O with chords AB and CD intersecting at P C

Show: AP  PB  DP  PC
 and
Plan: Use the Line Postulate to construct chords DB B

AC . Then use the Inscribed Angles Intercepting Arcs P
Theorem and the AA Similarity Postulate to get APC A
O

DPB. Therefore, by CSSTP and the multiplication property,


AP  PB  DP  PC.
D

Proof: Statement Reason


 and AC
1. Construct DB  1. Line Postulate
2. CAB CDB; ACD DBA 2. Inscribed Angles Intercepting Arcs Theorem
3. APC DPB 3. AA Similarity Postulate
AP PC
4.  
DP  PB 4. CSSTP
AP PC
5. (DP)(PB)  
DP  (DP)(PB)  PB
  5. Multiplication property
6. AP  PB  DP  PC 6. Algebra

296 CHAPTER 13 Discovering Geometry Solutions Manual


©2003 Key Curriculum Press

Das könnte Ihnen auch gefallen